w FOR FIRST AID THE® USMLE STEP 2 CS Fifth Edition

TAO LE, MD, MHS Associate Clinical Professor of Medicine and Pediatrics Chief, Section of Allergy and Immunology Department of Medicine University of Louisville

VIKAS BHUSHAN, MD Diagnostic Radiologist

MAE SHEIKH-ALI, MD Associate Professor of Medicine Associate Program Director, Endocrinology Fellowship Program Division of Endocrinology, and University of Florida College of Medicine–Jacksonville

KACHIU CECILIA LEE, MD, MPH Clinical and Research Fellow Wellman Center for Photomedicine Department of Dermatology Massachusetts General Hospital, Harvard Medical School

New York / Chicago / San Francisco / Lisbon / London / Madrid / Mexico City Milan / New Delhi / San Juan / Seoul / Singapore / Sydney / Toronto Copyright © 2014, 2012, 2010, 2007, 2004 by Tao Le. All rights reserved. Except as permitted under the United States Copyright Act of 1976, no part of this publication may be reproduced or distributed in any form or by any means, or stored in a database or retrieval system, without the prior written permission of the publisher, with the exception that the program listings may be entered, stored, and executed in a computer system, but they may not be reproduced for publication.

ISBN: 978-0-07-180933-7

MHID: 0-07-180933-3

The material in this eBook also appears in the print version of this title: ISBN: 978-0-07-180426-4, MHID: 0-07-180426-9. eBook conversion by codeMantra Version 1.0

All trademarks are trademarks of their respective owners. Rather than put a trademark symbol after every occurrence of a trademarked name, we use names in an editorial fashion only, and to the benefit of the trademark owner, with no intention of infringement of the trademark. Where such designations appear in this book, they have been printed with initial caps.

McGraw-Hill Education eBooks are available at special quantity discounts to use as premiums and sales promotions or for use in corporate training programs. To contact a representative, please visit the Contact Us page at www.mhprofessional.com.

First Aid for the® is a registered trademark of McGraw-Hill Education.

NOTICE

Medicine is an ever-changing science. As new research and clinical experience broaden our knowledge, changes in treatment and drug therapy are required. The authors and the publisher of this work have checked with sources believed to be reliable in their efforts to provide information that is complete and generally in accord with the standards accepted at the time of publication. However, in view of the possibility of human error or changes in medical sciences, neither the authors nor the publisher nor any other party who has been involved in the preparation or publication of this work warrants that the information contained herein is in every respect accurate or complete, and they disclaim all responsibility for any errors or omissions or for the results obtained from use of the information contained in this work. Readers are encouraged to confirm the information contained herein with other sources. For example and in particular, readers are advised to check the product information sheet included in the package of each drug they plan to administer to be certain that the information contained in this work is accurate and that changes have not been made in the recommended dose or in the contraindications for administration. This recommendation is of particular importance in connection with new or infrequently used drugs.

TERMS OF USE

This is a copyrighted work and McGraw-Hill Education and its licensors reserve all rights in and to the work. Use of this work is subject to these terms. Except as permitted under the Copyright Act of 1976 and the right to store and retrieve one copy of the work, you may not decompile, disassemble, reverse engineer, reproduce, modify, create derivative works based upon, transmit, distribute, disseminate, sell, publish or sublicense the work or any part of it without McGraw-Hill Education’s prior consent. You may use the work for your own noncommercial and personal use; any other use of the work is strictly prohibited. Your right to use the work may be terminated if you fail to comply with these terms.

THE WORK IS PROVIDED “AS IS.” McGRAW-HILL EDUCATION AND ITS LICENSORS MAKE NO GUARANTEES OR WARRANTIES AS TO THE ACCURACY, ADEQUACY OR COMPLETENESS OF OR RESULTS TO BE OBTAINED FROM USING THE WORK, INCLUDING ANY INFORMATION THAT CAN BE ACCESSED THROUGH THE WORK VIA HYPERLINK OR OTHERWISE, AND EXPRESSLY DISCLAIM ANY WARRANTY, EXPRESS OR IMPLIED, INCLUD- ING BUT NOT LIMITED TO IMPLIED WARRANTIES OF MERCHANTABILITY OR FITNESS FOR A PARTICULAR PURPOSE. McGraw-Hill Education and its licensors do not warrant or guarantee that the functions contained in the work will meet your requirements or that its operation will be uninterrupted or error free. Neither McGraw-Hill Education nor its licensors shall be liable to you or anyone else for any inaccuracy, error or omission, regardless of cause, in the work or for any damages resulting therefrom. McGraw-Hill Education has no responsibility for the content of any information accessed through the work. Under no circumstances shall McGraw-Hill Education and/or its licensors be liable for any indirect, incidental, special, punitive, consequential or similar damages that result from the use of or inability to use the work, even if any of them has been advised of the possibility of such damages. This limitation of liability shall apply to any claim or cause whatsoever whether such claim or cause arises in contract, tort or otherwise. DEDICATION

To the contributors of this and past editions, who took time to share their experience, advice, and humor for the benefit of future physicians.

and

To our families, friends, and loved ones, who supported us in the task of assembling this guide. This page intentionally left blank CONTENTS

Contributors ...... ix Faculty Reviewers ...... xi Preface...... xiii Acknowledgments...... xv How to Contribute...... xvii

SECTION 1 GUIDE TO THE USMLE STEP 2 CS 1

Introduction ...... 2 USMLE Step 2 CS—The Basics...... 2 Preparing for the Step 2 CS...... 9 Test-Day Tips ...... 11 First Aid for the IMG...... 13 Supplement—The USMLE Step 2 CS Travel Guide...... 21

SECTION 2 THE PATIENT ENCOUNTER 39

Introduction ...... 40 Doorway Information...... 42 Taking the History...... 44 The Physical Exam ...... 54 Closure ...... 63 How to Interact with Special Patients...... 65 Challenging Questions and Situations ...... 67 Counseling ...... 77 The Patient Note ...... 79

SECTION 3 MINICASES 85

Headache ...... 87 Confusion/Memory Loss...... 89 Loss of Vision...... 91 Depressed Mood...... 92 Psychosis ...... 93

v Dizziness ...... 94 Loss of Consciousness...... 95 Numbness/Weakness ...... 96 Fatigue and Sleepiness ...... 98 Night Sweats ...... 100 Insomnia ...... 100 Sore Throat...... 101 Cough/Shortness of Breath ...... 102 Chest Pain ...... 105 Palpitations...... 107 Weight Loss...... 108 Weight Gain ...... 109 Dysphagia...... 110 Neck Mass ...... 111 Nausea/Vomiting...... 111 Abdominal Pain ...... 112 Constipation/Diarrhea ...... 116 Upper GI Bleeding ...... 118 Blood in Stool...... 119 Hematuria...... 119 Other Urinary Symptoms...... 120 Erectile Dysfunction...... 122 Amenorrhea...... 123 Vaginal Bleeding ...... 124 Vaginal Discharge...... 126 Dyspareunia...... 126 Abuse ...... 127 Joint/Limb Pain...... 128 Low Back Pain...... 132 Child with Fever ...... 133 Child with GI Symptoms...... 134 Child with Red Eye ...... 136 Child with Short Stature...... 136 Behavioral Problems in Childhood...... 137

SECTION 4 PRACTICE CASES 139

Case 1 46-Year-Old Man with Chest Pain...... 142 Case 2 57-Year-Old Man with Bloody Urine...... 152 vi Case 3 51-Year-Old Man with Back Pain ...... 161 Case 4 25-Year-Old Man Presents Following Motor Vehicle Accident.... 170 Case 5 28-Year-Old Woman Presents with Positive Pregnancy Test...... 180 Case 6 10-Year-Old Girl with New-Onset Diabetes...... 189 Case 7 74-Year-Old Man with Right Arm Pain...... 197 Case 8 56-Year-Old Man Presents for Diabetes Follow-up ...... 206 Case 9 25-Year-Old Woman Presents Following Sexual Assault ...... 216 Case 10 35-Year-Old Woman with Calf Pain ...... 225 Case 11 62-Year-Old Man with Hoarseness ...... 235 Case 12 67-Year-Old Woman with Neck Pain ...... 243 Case 13 48-Year-Old Woman with Abdominal Pain ...... 251 Case 14 35-Year-Old Woman with Headaches...... 260 Case 15 36-Year-Old Woman with Menstrual Problems...... 269 Case 16 28-Year-Old Woman with Pain During Sex ...... 278 Case 17 75-Year-Old Man with Hearing Loss ...... 287 Case 18 5-Day-Old Boy with Jaundice...... 296 Case 19 7-Month-Old Boy with Fever...... 305 Case 20 26-Year-Old Man with Cough...... 314 Case 21 52-Year-Old Woman with Jaundice ...... 323 Case 22 53-Year-Old Man with Dizziness ...... 332 Case 23 33-Year-Old Woman with Knee Pain ...... 340 Case 24 31-Year-Old Man with Heel Pain...... 350 Case 25 18-Month-Old Girl with Fever ...... 360 Case 26 54-Year-Old Woman with Cough...... 369 Case 27 61-Year-Old Man with Fatigue...... 379 Case 28 54-Year-Old Man Presents for Hypertension Follow-up ...... 388 Case 29 20-Year-Old Woman with Sleeping Problems ...... 397 Case 30 2-Year-Old Girl with Noisy Breathing...... 407 Case 31 21-Year-Old Woman with Abdominal Pain ...... 416 Case 32 65-Year-Old Woman with Forgetfulness and Confusion...... 426 Case 33 46-Year-Old Man with Fatigue...... 436 Case 34 32-Year-Old Woman with Fatigue ...... 445 Case 35 27-Year-Old Man with Visual Hallucinations ...... 454 Case 36 32-Year-Old Man Presents for Preemployment Physical...... 463 Case 37 55-Year-Old Man with Bloody Stool ...... 472 Case 38 66-Year-Old Man with Tremor...... 481 Case 39 30-Year-Old Woman with Weight Gain ...... 490

vii Case 40 6-Month-Old Girl with Diarrhea...... 499 Case 41 8-Year-Old Boy with Bed-Wetting ...... 507 Case 42 11-Month-Old Girl with Seizures ...... 515 Case 43 21-Year-Old Man with Sore Throat ...... 523 Case 44 49-Year-Old Man with Loss of Consciousness ...... 532

SECTION 5 TOP-RATED REVIEW RESOURCES 541

How to Use the Database ...... 542

Appendix...... 547 Index...... 551 About the Authors...... 573

viii CONTRIBUTORS

CONTRIBUTING AUTHORS Raeda Alshantti, MD Hospitalist, Alshifa Hospital University of Damascus School of Medicine

Melissa Marie Cranford, MD Resident, Department of Psychiatry Yale-New Haven Hospital

Kevin Day, MD Resident, Department of Medical Imaging University of Arizona Medical Center

Michael King, MD Resident, Department of Anesthesiology Massachusetts General Hospital

Jasmine Rassiwala, MD, MPH Resident, Department of Internal Medicine University of California, San Francisco

Ruba Sheikh-Ali, MD Clinical Researcher University of Florida College of Medicine–Jacksonville

Jody Tversky, MD Assistant Professor Clinical Director Division of Allergy and Clinical Immunology Johns Hopkins University School of Medicine

Jinyu (Jane) Zhang, MD Resident, Department of Internal Medicine Thomas Jefferson University

ASSOCIATE CONTRIBUTING AUTHOR Mohammad Samer Agha, MD Internal Medicine Consultant Clinical Director, Internal Medicine Al-Kalamoon University, Damascus

ix This page intentionally left blank FACULTY REVIEWERS

Kelly A. Best, MD, FACOG Associate Professor, Division of General Obstetrics and Gynecology Department of Obstetrics and Gynecology University of Florida College of Medicine–Jacksonville

Arkadiy Finn, MD Clinical Instructor Department of Medicine Warren Alpert Medical School, Brown University

Nilmarie Guzman, MD Assistant Professor, Division of Infectious Disease Department of Medicine University of Florida College of Medicine–Jacksonville

Jeffrey G. House, DO Associate Professor, Division of General Medicine Department of Medicine University of Florida College of Medicine–Jacksonville

Nizar F. Maraqa, MD Assistant Professor, Division of Pediatric Infectious Diseases and Immunology Department of Pediatrics University of Florida College of Medicine–Jacksonville

Carlos Palacio, MD, MPH Associate Professor, Division of General Internal Medicine Department of Medicine University of Florida College of Medicine–Jacksonville

Jigme M. Sethi, MD, FCCP Associate Professor Department of Medicine Warren Alpert Medical School, Brown University Chief, Pulmonary, Critical Care, and Sleep Medicine Memorial Hospital of Rhode Island

xi This page intentionally left blank PREFACE

The USMLE Step 2 CS can be a source of stress and anxiety, especially among in- ternational medical graduates (IMGs), who often find themselves at a disadvantage because of their non-U.S. training background. First Aid for the USMLE Step 2 CS is our “cure” for this exam. This book represents a virtual medicine bag of high-yield tools for students and IMGs, including:

An updated exam preparation guide for the new USMLE Step 2 CS, including proven study and exam strategies for clinical encounters based on the patient- centered interview. Expanded guidelines on how to deal with challenging situations, including a range of situations that pose ethical and confidentiality issues. Detailed descriptions of high-yield physical exam maneuvers that will win you points without costing time. Forty-four full-length practice cases that allow you to simulate the actual Step 2 CS exam, updated to reflect recent exam changes that test your ability to docu- ment the patient’s most likely diagnoses and how they are supported by the history and physical exam findings. A revised and expanded set of minicases representing common complaints de- signed to help you rapidly develop a working set of differential diagnoses.

This book would not have been possible without the suggestions and feedback of medical students, IMGs, and faculty members. We invite you to share your thoughts and ideas to help us improve First Aid for the USMLE Step 2 CS. See How to Con- tribute, p. xvii.

Louisville Tao Le Los Angeles Vikas Bhushan Jacksonville Mae Sheikh-Ali Boston Kachiu Cecilia Lee

xiii This page intentionally left blank ACKNOWLEDGMENTS

This has been a collaborative effort from the start. We gratefully acknowledge the thoughtful comments, corrections, and advice of the many medical students, resi- dents, international medical graduates, and faculty who have supported the authors in the continuing development of First Aid for the USMLE Step 2 CS.

For support and encouragement throughout the process, we are grateful to Thao Pham, Isabel Nogueira, Louise Petersen, and Jonathan Kirsch.

Thanks to our publisher, McGraw-Hill, for the valuable assistance of its staff. For enthusiasm, support, and commitment to the First Aid series, thanks to our editor, Catherine Johnson. For outstanding editorial work, we thank Andrea Fellows, our developmental editor. Finally, a special thanks to Rainbow Graphics, especially David Hommel, Tina Castle, and Susan Cooper, for remarkable editorial and production support.

Louisville Tao Le Los Angeles Vikas Bhushan Jacksonville Mae Sheikh-Ali Boston Kachiu Cecilia Lee

xv This page intentionally left blank HOW TO CONTRIBUTE

First Aid for the USMLE Step 2 CS incorporates many contributions from students and faculty. We invite you to participate in this process. Please send us:

Study and test-taking strategies for the Step 2 CS exam High-yield case topics that may appear on future Step 2 CS exams Personal comments on review books that you have examined

For each entry incorporated into the next edition, you will receive up to a $20 Ama- zon.com gift certificate and a personal acknowledgment in the next edition. Sig- nificant contributions will be compensated at the discretion of the authors. The pre- ferred way to submit entries, suggestions, or corrections is via our blog:

www.firstaidteam.com

Otherwise, you can e-mail us directly at:

[email protected]

Contributions sent earlier will receive priority consideration for the next edition of First Aid for the USMLE Step 2 CS.

NOTE TO CONTRIBUTORS All entries are subject to editing and reviewing. Please verify all data and spellings carefully. In the event that similar or duplicate entries are received, only the first entry received will be used. Please follow the style, punctuation, and format of this edition as much as possible. All contributions become property of the authors.

INTERNSHIP OPPORTUNITIES The author team of Le and Bhushan is pleased to offer part-time and full-time paid internships in medical education and publishing to motivated medical students and physicians. Internships may range from two to three months (eg, a summer) up to a full year. Participants will have an opportunity to author, edit, and earn academic credit on a wide variety of projects, including the popular First Aid series. Writing/ed- iting experience, familiarity with Microsoft Word, and Internet access are required. For more information, e-mail a résumé or a short description of your experience along with a cover letter and writing sample to fi[email protected].

xvii This page intentionally left blank SECTION 1

Guide to the USMLE Step 2 CS

Introduction USMLE Step 2 CS—The Basics Preparing for the Step 2 CS Test-Day Tips First Aid for the IMG Supplement—The USMLE Step 2 CS Travel Guide GUIDE TO THE USMLE STEP 2 CS 2 and IMGsmakeinpreparingfortheStep2CSincludefollowing: patient actorstosimulateclinicalencounters.Commonmistakesmedicalstudents USMLE Step2CS,whichdistinguishesitselffromotherexamsbyusinglive Step 2ClinicalKnowledge(CK),youmayfinditchallengingtoprepareforthe Even ifyouareaproattakingstandardizedexamssuchastheUSMLEStep1and encounters with“standardizedpatients.” ing Examination(USMLE)Step2ClinicalSkills(CS)—atestinvolvingclinical required topassaclinicalskillsexamknownastheUnitedStatesMedicalLicens- Canadian medicalstudentsaswellinternationalgraduates(IMGs)are As aprerequisitetoenteringresidencytrainingintheUnitedStates,allU.S.and skills thatprovidethefoundation forthesafeandeffectivepracticeofmedicine.” attention isdevotedtotheprinciples ofclinicalsciencesandbasicpatient-centered cludes emphasisonhealthpromotion anddiseaseprevention.Step2ensuresthatdue clinical scienceessentialfortheprovisionofpatientcare undersupervision,andin- the abilityofexamineestoapplymedicalknowledge,skills, andunderstandingof According totheUSMLEWeb site(www.usmle.org), “Step2ofthe USMLE assesses of MedicalExaminers(NBME)andtheFederationState MedicalBoards(FSMB). otherUSMLEexams,theStep2CS issponsoredbytheNationalBoard Like Introduction the Step2CSwithfiveorganizedsections: This bookwillguideyouthroughtheprocessofefficientlypreparingforandtaking INTRODUCTION USMLE STEP2CS—THEBASICS Section 5 Section 4 to helpyourapidlydevelopfocuseddifferentialsduringtheexam. Section 3 Section 2 Section 1 Neglecting tocarryouteasybutrequiredpatientinteractions Failing tounderstandhowinteractwithapatientappropriately Taking unfocusedhistoriesandphysicalexams Becoming flusteredbychallengingquestionsorsituations Managing timepoorlyduringpatientencounters Failing tounderstandtherequiredobjectivesforeachpatientencounter Not developingalogicalplanofattackbasedonpatient“doorwayinformation” exam Not practicingenoughwithmockpatientscenariosbeforetakingtheactual Panicking becauseoftheunfamiliarformattest ratesotherresourcesthathelpyoupreparefortheStep2CS. offersfull-lengthpracticecasestohelpyousimulatetherealthing. reviewscriticalhigh-yieldstepstotakeduringthepatientencounter. youtotheStep2CS. introduces provideshigh-yieldminicasesforcommondoorwaychiefcomplaints GUIDE TO THE USMLE STEP 2 CS 3 on a solid level to communicate to evaluate your while maintaining a comfortable and Test designers aim Test application of clinical professional rapport. professional knowledge and ability Roughly This alludes to the minimum level This alludes to the This refers to anything and everything you This refers to anything This signifies that as an intern, you’ll typically have a This signifies that Here again, emphasis is placed on the bare-bones clinical science Here again, emphasis is placed on the knowledge and communication skills needed to help reduce morbidity and mor- knowledge and communication skills tality. stated, this means that it’s not all about acute MIs, trauma, or sepsis, but also acute MIs, trauma, or sepsis, but also not all about that it’s stated, this means of their own health. about enabling patients to take control of clinical sciences and basic patient- “Attention is devoted to the principles for the safe and effective practice centered skills that provide the foundation of medicine”: have learned in medical school so far. have learned in medical of patient care”: “Essential for the provision needed to provide patient care. of knowledge and skills “Under supervision”: watching over you. resident and an attending prevention”: on health promotion and disease “Includes emphasis “Assesses the ability of examinees to apply medical knowledge, skills, and of examinees to apply medical “Assesses the ability science”: understanding of clinical exam is become familiar with its format, practice focused history taking and patient exam is become familiar with its format, practice focused history taking and interactions, and present cases in a logical and well-rehearsed fashion. The underlying philosophy of the Step 2 CS, therefore, is not to cover the same The underlying philosophy of the Step its primary objective is or Step 2 CK. Rather, factual knowledge tested on the Step 1 knowledge base by communicating with to test your ability to apply a fundamental a basic mock patients toward the goal of extracting enough information to generate for the differential diagnosis and workup plan. So the best one can do to prepare But precisely how does one demonstrate the ability to manage disease and promote But precisely how does one demonstrate good Do this by examin- health by communicating? The answer is simple: practice. you can. Then logically synthesize what you ing as many patients and colleagues as For IMGs, we must emphasize that this uncovered by communicating your findings. with native English speakers. practice should be done in English, ideally In summary, the test designers want to evaluate your application of clinical knowl- the test designers want to evaluate your application In summary, well enough to work with other house staff on edge and your ability to communicate and healthy. a joint mission to help keep patients alive What Is the USMLE Step 2 CS? also required to exhibit competence in written English and to demonstrate critical also required to exhibit competence in written English and to demonstrate clinical skills by writing a brief patient note (PN), follow-up orders, and a differential diagnosis. and Canadian medical students seeking to obtain their medical licenses—as well as and Canadian medical students seeking to obtain their medical licenses—as commu- all IMGs seeking to start their residencies in the United States—have the pass To nication, interpersonal, and clinical skills necessary to achieve these goals. the test, all examinees must show that they can speak, understand, and communicate are in English as well as take a history and perform a brief physical exam. Examinees The USMLE Step 2 CS is a one-day exam whose objective is to ensure that all U.S. The USMLE Step 2 CS is a one-day exam whose objective is to ensure that An impressive statement, but what does it mean? Let’s dissect the statement so that statement so that dissect the Let’s what does it mean? statement, but An impressive CS and anticipate the Step 2 philosophy underlying understand the you can better on test day. you may encounter of questions and scenarios the types GUIDE TO THE USMLE STEP 2 CS 4 Many studentschoose exam inpediatricor There isnophysical Instead, youshould phone encounters. focus onobtaining a thorough history style formatwhen effective closure. to useabullet- typing thePN. and delivering following: of eachroom(andhenceisoftenreferredtoas“doorwayinformation”),includesthe review somepreliminaryinformation.Thisinformation,whichispostedonthedoor Before enteringaroomtointeractwithanSP, youwillbegivenanopportunityto How IstheStep2CSStructured? in thecurriculaofaccreditedU.S.medicalschools.Theseclerkshipsareasfollows: represent thetypesofpatientswhoaretypicallyencounteredduringcoreclerkships the resultingvideotapeswillnotbeusedforscoring.ThecasesinStep2CS qualityassurancepurposes,avideocamerawillrecordallclinicalencounters,but For mixed intermsofage,gender, ethnicity, organsystem,anddiscipline. Half ofthecasesareperformedbeforelunchbreakandhalfafterward.SPswillbe over thecourseofaboutaneight-hourday, includinga30-minute breakforlunch. participating intheassessment.WhenyoutakeStep2CS,willsee12SPs simulate variousclinicalproblems.TheSPsgivethesameresponsestoallcandidates patients” (SPs),allofwhomarelaypersonswhohavebeenextensivelytrainedto physicians’ offices,andemergencydepartments.Thetestmakesuseof“standardized The Step2CSsimulatesclinicalencountersthatarecommonlyfoundinclinics, ing the room, introducing yourself,obtaininganappropriatehistory,ing theroom,introducing conductinga form theclinicalencounter, whichincludesreadingthe doorwayinformation,enter- You willbegiven15minutes(withawarningbellsoundedafter10minutes)toper- do Examinees you findyourselfrunningahead ofschedule,youmightconsidertellingthepatient out thedoor. Onceyouleavetheexamination room,youmaynotreenterit.Soif If youhappentofinishaclinical encounterearly, thereisnoneedforyoutorush make thetypingprogramunavailable. ees willnotbepermittedtohandwritethePNunlesstechnical difficultiesontestday might have.Afterleavingtheroom,youwillhave10minutes totypeaPN.Examin- the diagnosesbeingconsidered,andadviseSPabout anyfollow-upplansyou workup. You willalsobeexpectedtoansweranyquestionstheSPmightask,discuss focused physicalexam,formulatingadifferentialdiagnosis, andplanningadiagnostic physical examisrequiredundersuchcircumstances. assuming theroleofpediatricpatients’parentsrecounthistories,andno Chief complaint and vitals (temperature, respiratory rate, pulse, blood pressure) Chief complaintandvitals(temperature,respiratoryrate,pulse,blood Patient characteristics(name,age,gender) Emergency medicine Family medicine Psychiatry Pediatrics Obstetrics andgynecology Surgery Internal medicine not interactwithchildrenduringpediatricencounters.Instead,SPs GUIDE TO THE USMLE STEP 2 CS 5 disorders in your disorders Do not list unlikely important this may that are most likely. that are differential, however differential, be in actual practice. Instead, focus on the differential diagnoses differential (discussed at In addition to assessing . The skills you demonstrate in the The skills you demonstrate patient-centered interview To demonstrate your data interpretation skills, you will To SPs will evaluate your data-gathering skills by documenting SPs will evaluate your data-gathering Your final score will represent the average of your individual PN scores over final score will Your all 10 scored clinical encounters. actual practice physicians must develop the ability to recognize and rule out actual practice physicians must develop record only the most likely diagnoses a range of disorders, you will be asked to findings that support each. Physicians along with the positive and negative who score the PN make a global assessment based on documentation and or- ganization of the history and physical exam; the relevance, justification, and order of the differential diagnosis; and the initial testing modalities proposed. credit—for having done so. Data interpretation. possible your analysis of a patient’s be asked to document, as part of the PN, such diagnoses are supported or refuted diagnoses and your assessment of how and physical exam. Although in by the evidence obtained from the history they will note whether you asked the questions listed on their checklists, suc- they will note whether you asked the questions and correctly conducted the physical cessfully obtained relevant information, of the procedures on their check- exam (as indicated by your performance procedures that are not on an SP’s lists). If you asked questions or performed at the same time will not lose checklist, you will not receive credit—but Data gathering. Specifically, to the clinical encounter. your ability to collect data pertinent clinical encounter are reflected in your ICE score. This score will reflect your clinical encounter are reflected in your skills data-gathering and data interpretation length in Section 2) in which you identify and respond to the broader scope of thelength in Section 2) in which you identify and respond to the broader scope of the patient, gathering and providing information, helping the patient make deci- will be evaluated on your ability emotions. You sions, and supporting the patient’s to tailor your questions and responses to the specific needs of the case presented concerns. Overall, the CIS subcom- and on your capacity to react to the patient’s ponent focuses on your ability to conduct a your data-gathering abilities, SPs will evaluate your communication and interper- withsonal skills. According to the USMLE, these include fostering a relationship 1. Integrated Clinical Encounter (ICE) score. ter: the SP and a physician. The SP will evaluate you at the end of each encounter The SP will evaluate you at ter: the SP and a physician. and a second for the physical exam, checklists: one for the history, by filling out three the PN you write after skills. The physician will evaluate a third for communication clinical encounter as a score, which will be based on the overall Your each encounter. by the following overall communication skills, will be determined whole and on your three components: that you are organizing your notes, as one or two last-minute questions might pop last-minute questions notes, as one or two are organizing your that you into mind. Scored? the Step 2 CS How Is Of will score each encoun- people encounters, 10 will be scored. Two your 12 patient 2. Communication and Interpersonal Skills (CIS) score. GUIDE TO THE USMLE STEP 2 CS 6 components oftheStep 2 CStopasstheexam. You mustpassallthree likely tofailbecauseof fail theStep2CS,U.S. Among studentswho ICE scores, and IMGs are mostlikelytofail because oftheCIS. months inadvance. Register asearly students are most test centersfillup possible, assome date andcenter. application deadline,youshouldapplyearlytoensurethatgetyourpreferredtest and mailittotheECFMGwithregistrationfee.Althoughthereisnospecific org/gradoverview.asp ordownloadthepaperapplicationfromECFMGWeb site line usingtheECFMG’s InteractiveWeb Application(IWA) athttps://iwa2.ecfmg. examinees (clicktheappropriatelinkatwww.nbme.org). IMGscaneitherapplyon- U.S. studentsmustregisterusingtheNBME’s interactiveWeb site forapplicantsand www.ecfmg.org. site oftheEducationalCommissionforForeignMedicalGraduates(ECFMG)at www.usmle.org orcheckwithyourdean’s office.IMGsshouldalsorefertotheWeb evolving. ForthemostcurrentinformationonregisteringforStep2CS,goto USMLE Step.However, registrationinformationandproceduresareconstantly Applicants canregisterdirectlyfortheStep2CSwithouthavingpassedanyother How DoIRegistertoTake theUSMLEStep2CS? www.usmle.org orcanbeobtainedonCDwhenyouregister. Thevideo isanexcel- by e-mail.Orientationmanualsandvideosofsample encounters areavailableat After yourapplicationhasbeenprocessed,youwillreceive aschedulingpermit .SpokenEnglishProficiency(SEP)score. 3. to causefailure.FewIMGsfailallthreesubcomponents. challenge forIMGscomparedtoU.S.studentsbutisstilltheleastlikelycomponent IMGs, theCISismostlikelycomponenttocausefailure.TheSEPmoreofa U.S. studentsfailtheexamasawhole,itismostlikelyduetopoorICEscores.For Relatively fewU.S.studentsfailtheCIS,andevenfewerSEPcomponent.If one infourexamineesfailing. (see Table 1-1).However, thefailurerateishigheramongIMGs,withapproximately newsisthatmostU.S.andCanadianmedical studentspass components. Thegood cal score.To passtheStep2CSoverall,candidatesmustallthreeindividual components oftheexam.UnlikeStep1or2CK,youwillnotreceiveanumeri- will includeagraphicrepresentationofyourstrengthsandweaknessesonallthree The gradeyoureceiveontheStep2CSwillbeeithera“pass”or“fail.”Your report IMGs U.S./Canadian TABLE 1-1. by SPs with checklists. by SPswithchecklists. patient’s concernsbeyondjustthediagnosis.TheCISperformanceisdocumented scales. spoken English.TheSEPscoreisbasedonSPevaluationsthatmakeuseofrating ciation, wordchoice,andthedegreeofeffortSPmustmaketounderstandyour Step2CSPassRates o etdPsigN.Tse Passing No.Tested Passing No. Tested 5027%1,8 77% 97% 13,780 17,164 77% 98% 15,042 18,361 0021 2011–2012 2010–2011 This componentscoresyouonpronun- GUIDE TO THE USMLE STEP 2 CS 7 tem (OASIS) account on the ECFMG/NBME Web site. An e-mail is sent to you tem (OASIS) account on the ECFMG/NBME Web of once your score report has been uploaded onto your account page. A fixed schedule USMLE Web site (www.usmle.org), the ECFMG Web site (www.ecfmg.org), and the and the site (www.ecfmg.org), the ECFMG Web site (www.usmle.org), USMLE Web Section 1 Supplement to this text. to Get My Scores? How Long Will I Wait Step 2 CS results are posted to your On-line Applicant Status and Information Sys- Where Can I Take the Exam? Can I Take Where Evalu- The Step 2 CS will be administered at five regional sites called Clinical Skills under ation Collaboration centers (see Figure 1-1). Additional centers are currently consideration. to the For detailed information about cities, hotels, and transportation, please refer therefore advised to avoid such exchanges and instead to reschedule test dates only therefore advised to avoid such exchanges registered late and your only options are within the formal protocols. If you have check back frequently for openings closer to later than you would like, be sure to your desired date. with another applicant. The Step 2 CS scheduling system does not allow anyone to with another applicant. The Step 2 CS on behalf of another applicant. In addition, schedule or reschedule an appointment basis—so if you cancel your appoint- the system works on a first-come, first-served your test date might be claimed by some- ment in anticipation of such an exchange, the system at the same time. Applicants are one else who happens to be logged onto date. You will need to pay $400 if you miss an appointment without canceling or will need to pay $400 if you miss an appointment date. You site change, so please check the USMLE Web rescheduling. These fees are subject to for the current fee schedule. (www.usmle.org) test dates. Some appli- a word of caution regarding the exchange of scheduled Finally, on online forums to swap their appointment cants have been known to post requests Although you cannot extend your eligibility periodAlthough you cannot can- for the Step 2 CS, you can charged a fee if you cancel will not be You cel or reschedule your examination date. your scheduled test date, not including the or reschedule 14 calendar days before will be levied if you cancel or reschedule at a fee of $150 day of the test. However, any time during the 14-day period your scheduled test before (but not including) or by telephone. Access information will be included with your registration materials. information will be included with or by telephone. Access offered may be sessions. You offer both morning and afternoon Note that test centers which morning sessions are if you select a date and center for an afternoon session when and center that offer you a morning session, to select a date already filled. Try an inveterate night owl). fresher and more relaxed (unless you are you are likely to be lent preparation resource that shows exactly how the Step 2 CS is administered as the Step 2 CS shows exactly how resource that lent preparation you have received the exam. Once yourself during you should conduct well as how starting one year, the Step 2 CS for are eligible to take permit, you your scheduling will list your scheduling permit Your was processed. date your application from the eligibility period, requirements for admis- scheduling instructions, and identification site Web test through the NBME or ECFMG can schedule the sion to the exam. You GUIDE TO THE USMLE STEP 2 CS 8 gained fromyourfirstattempttooptimizepreparation andimproveyourperfor- it maybebesttobeginpreparationretest.Usetheknowledgeandexperienceyou exactly howorwhyyoumayhavefailed.Eveniffeelyourresultsareunjustified, to changeyouroverallexamresults,andlittleinformationisprovidedexplain able toappealandrequestarescoringofyourexam.However, doingsoisunlikely If forsomereasonyouthinkthatreceivedafailingscoreunfairly, youmaybe application andanappropriatefee. Inaddition,eachtimeyoutaketheexammustsubmitanew 12-month period. youfailtheStep2CS,canretakeit,butnotmorethanthreetimeswithinany If What IfIFail? residency training. pass theStep2CS,yourpassingscoreremainsvalidforpurposeofapplying score isnotdisclosedtoyouoranyoftheprogramswhichapply. Onceyou you receiveindicatesonlywhetherpassedorfailedtheexam.Your numerical request foraduplicatereporttotheNBMEorECFMG.Again,score the test.Ifyoudonotreceiveyourresultswithinthattime,mustsendawritten whichisusually1–3monthsfromthedateof of thecorrespondingreportingperiod, test date.MostexamineeswhotaketheStep2CSreceivetheirscoresonfirstday ispublishedontheUSMLEWebscore-reporting periods sitewellinadvanceofyour the latestreexaminationandappealpolicies. shot. CheckyourorientationmanualortheUSMLEand/or ECFMGWeb sitesfor curate assessmentofyourtrueclinicalskillsistopractice moreandgiveitanother fees acknowledged,themosteffectiveresponsetowhatyou perceivemaybeaninac- that isfairandaccurate,theexamwillalwayshaveasubjective component.Costly mance. ItisworthrecognizingthateventhoughtheNBME trieshardtodesignatest FIGURE 1-1. Step 2CSTest Centers GUIDE TO THE USMLE STEP 2 CS 9 During look the the clinical encounter, When you first meet a patient, be sure to When you first meet a patient, be sure Allow the patient to express his or her concerns Allow the patient to express his or her In your encounters, you should appear confident, Always keep the patient well draped. You can cover You Always keep the patient well draped. It is probably best to wash your hands just before the physical It is probably best to wash your hands not on the leg or hand. Do not exaggerate your facial expressions in an effort to not on the leg or hand. Do not exaggerate your facial expressions in an effort Never talk to a patient convince the patient that you empathize with him or her. Men should wear slacks, a shirt, and a tie. Women should consider slacks and Men should wear slacks, a shirt, and a tie. Women low-heeled shoes and should avoid wearing skirts above the knee. Maintain appropriate body language. patient in the eye, smile when appropriate, and show compassion. When trying but to console a patient, you may place your hand on his or her shoulder or arm only one at a time. Be sure to ask permission before you uncover any part of the only one at a time. Be sure to ask permission before you uncover any part of should also ask permission to untie body you are doing so. You and explain why gown and should tie the gown again when you are done. the patient’s Be mindful of appearance. a clean white lab coat calm, and friendly as well as serious and professional. Wear over professional-looking but comfortable clothes. Do not wear shorts or jeans. tive. Use “draping manners.” exam, but it is better to do so at the the patient at any time before the physical body Do not expose large portions of the patient’s beginning of the encounter. and at the same time; instead, uncover only the parts that need to be examined, without interrupting or interjecting your own thoughts. Your demeanor should own thoughts. Your without interrupting or interjecting your be curious, nonjudgmental, and compassionate. your hands. Wash opportunity to briefly reflect and perhaps exam. Hand washing also gives you an is acceptable to use gloves as an alterna- ask a confirmatory question or two. It Introduce yourself to the patient. Jones”), introduce last name (eg, “Mr. smile, address the patient by his or her and establish good eye contact. shake hands firmly, yourself clearly, Actively listen to the patient. Interacting with patients in a professional and empathetic manner in a professional and empathetic Interacting with patients a good history medical Taking and focused physical exam Performing an appropriate information Counseling and delivering differential diagnosis organized PN that includes a reasoned a logical and Typing PREPARING FOR THE STEP 2 CS THE STEP FOR PREPARING In this section, we will briefly explore a few of these skills. Section 2 reviews these will briefly explore a few of these skills. In this section, we mechanics of the clinical encounter and PN. skills in greater detail in addition to the each encounter. These are simple and easy to learn but require practice. These each encounter. Ability to Interact with Patients in a Professional Way in a Professional Ability to Interact with Patients Thereelements of the CIS component that you must incorporate into are several In cer- need to demonstrate mind that you will Step 2 CS, keep in preparing for the These skills include the critical clinical skills in order to pass. tain fundamental but following: GUIDE TO THE USMLE STEP 2 CS 10 should becareful notto use complexlanguage IMGs shouldfocuson U.S. medicalstudents communication and interpersonal skills. or medicaljargon. may indicatethatyouaredriftingawayfromthecorrect diagnosis.You shouldalso If youfeelthatapatientisnotfollowingyourlineofquestioning,becareful,asthis tant thancoveringeverysingledetail. goal ofuncoveringandacknowledgingsalientpositivesnegativesismoreimpor- surveyofthechiefcomplaintwitha any hiddencomplaints.Rememberthatagood goal istodirecteachinterviewtowardexploringthechiefcomplaintanduncovering fore, youshouldaskonlythosequestionsthatarerelevanttothespecificcase;your complaint. However, beawarethatyouwillnotabletocovereverything. There- history. Itistruethatyoucanpreparealistofquestionstouseforeverysystemor interviewingtechniquesyouuseshouldallowtocollectathoroughmedical The Ability toTake aGoodMedicalHistory is thenameofthatdrugagain?” prescribed, donothesitateto ask,“Canyoupleaserepeatwhatsaid?”or“What If youdonotunderstandwhatapatienthassaidorrecognize adrugthathasbeen address itappropriately. Donotbedefensiveorhostile. calm, firm,andfriendly. Askaboutthereasonforapatient’s angerorcomplaint,and Do notbeintimidatedbyangrypatients.Rememberthat SPsareonlyactors,sostay stances, youshouldpretendthatthefindingsarereal. as realones(eg,simulationofwheezesduringchestauscultation). Insuchcircum- bear inmindthatphysicalfindingsmaybesimulatedand maynotlookthesame Focus yourconcentrationonthepatient. and closure. while standingsomewhereheorshecannotseeyou,especiallyduringthehistory following: nuances ofyourphysicalexamtechnique.You canshowconcernbydoingthe rapportthanto perfectthe purposeful. Itismoreimportanttomaintaingood tion toeverythingthepatientsaysanddoes,becausebehaviorismostlikely any partofthepatient’s body, andexplainwhatyouintendtodo.Payatten- may beonepossibility, butthereareothersthatweneedtoconsideraswell.” is duetocoloncancer.” You mayrespondtothiswithsomethinglike“That sound incorrecttoyou.Apatientmaytellyou,“IamsurethatthepainIhave Respect thepatient’s beliefs. you liketotellmeaboutit?” shoulder orarm.You maythensaysomethinglike“You mustfeelsad.Would take abriefmomentofsilenceandplaceyourhandlightlyonthepatient’s Show compassionforapatient’s sadness. of yourpainandgiveyoutherighttreatment.” pain, andIwanttohelpyou,butneedexamineyoulocatethesource touch hisorherabdomenbecauseofseverepain,say, “Iknow thatyouarein Show compassionforthepatient’s pain. and offtheexaminationtable.Donotrepeatpainfulprocedures. Keep thepatientcomfortable. Donotrejectapatient’s beliefsevenifthey Helpthepatientsitup,liedown,andgetonto Ask permissionbeforeyouexamine Ifthepatientdoesnotallowyouto To demonstrateempathy, youmay GUIDE TO THE USMLE STEP 2 CS 11 strategy. scratch paper. is an excellent is an excellent No watches of any are pens/pencils or are digital, are allowed in digital, are kind, either analog or the test area. Neither the test area. patient communication patient communication The summary technique The summary . Test . Test M . P . Nonetheless, you should plan M . P at least once during the interview. during the interview. at least once and the afternoon session at 3 . . or 3:30 M M . . A A summary technique summary proctors will generally wait up to 30 minutes for latecomers, so the actual exam proctors will generally wait up to 30 minutes for latecomers, so the actual exam usually does not begin until 8:30 in the exam room. license or a passport) Bring a government-issued photo ID (eg, a U.S. driver’s that carries your signature. Do not come to an afternoon session early in an attempt to meet candidates from Do not come to an afternoon session early in an attempt to meet candidates the morning session, as they are not allowed to leave until you are safely secured to arrive 30 minutes before your session is scheduled to begin. You may not use watches (analog or digital), cell phones, or beepers at any time may not use watches You during the exam. A locker will be provided to secure your items. The morning session starts at 8 TEST-DAY TIPS TEST-DAY false reassurances. Trendelenburg position. Trendelenburg the patient what you really think open. Tell When counseling a patient, always be can be made only after some tests have is wrong, and explain that the final diagnosis are planning to conduct. should also explain some of the tests you been ordered. You and never offer have in a realistic manner, Address any concerns the patient may to render a final diagnosis. to render a final diagnosis. or she still has any questions. After you re- Before you leave, ask the patient if he answer your question?” Make sure the patient spond, follow up by asking, “Did that avoid the use of complex medical jargon. It is understands what you are saying, and lie back than to tell them to assume a reverse much simpler to ask patients to gently ings, offer your medical opinion (including a concise differential diagnosis), describe opinion (including a concise differential ings, offer your medical In doing so, you should and outline possible treatments. the next step in diagnosis, not try and do patient only the things you know, the honest. Tell always be clear and may not leave that area until the exam has been completed. During this time, the may not leave that area until the exam has been completed. During this time, following conventions should be observed: The a stethoscope and a white coat. A limited Step 2 CS is a one-day exam. Bring ham- if you happen to forget yours. Tendon number of stethoscopes will be provided and pen lights are provided in the rooms. You mers, tongue depressors, tuning forks, or the afternoon session. The duration of the will be scheduled for either the morning hours. Step 2 CS, including orientation, testing, and breaks, is approximately eight you Once you have entered the secured area of the assessment center for orientation, Ability to Counsel and Deliver InformationAbility to Counsel At thetell the patient about your find- you will be expected to encounter, end of each Finally, remember to use the remember Finally, has just told what the patient briefly summarizing which involves This technique, you finish taking be used either after own words, may using the patient’s you, often that you remem- will help ensure exam. Summarizing or after the physical the history write the PN. history before you leave the room to ber the details of the GUIDE TO THE USMLE STEP 2 CS 12 desk ifyouneedthem. snacks tokeepatyour Bring waterorenergy center. Checkitwith the hotelfront desk. luggage tothetest Don’t bringyour The followinggeneralprinciples willhelpyouexcelontheStep2CS: Some FinalWords After the 30-minute waiting period hasended,thestaffwillgiveyouanametag, After the30-minutewaitingperiod irregular andmaybequestioned. during theencounters),speakonlyinEnglish;doingotherwisewillbeconsidered breaks, donottalkaboutthecasesyouencountered.Duringbreaks(and,ofcourse, The Step2CSisnotasocialevent,sowhenyoumeetwithothercandidatesduring your assigneddesk,notinthestoragearea. withyou,keepiton to youuntiltheendofexam—soifdoplanbringfood for yourbreaks.Alsorememberthatpersonalbelongingswillnotbeaccessible staff willprovideyouwithonemeal,maywanttobringsomehigh-energysnacks drink onthisdesksothatitwillbeaccessibleduringbreaktime.Althoughthetesting In thebreakroom,youwillbeassignedaseatanddesk.You or cankeepyourfood exam butalsoduringbreaks.You cannotleavethecenterduringbreakperiods. encounters. Finally, rememberthatsmokingisstrictlyprohibitednotonlyduringthe Use thebathroomduringthesebreaks,asyouwillnothavetimetodoso meal. Thesecondbreaklasts15minutesandtakesplaceaftertheeighthencounter. takes placeafterthefourthencounter. Duringthisbreak,thestaff will serveyoua You willbegiventwobreaksduringtheexam.Thefirstbreaklasts30minutesand try eachpieceofequipmentmadeavailabletoyouduringthissession. equipment, especiallythebed,footextension,andheadelevation.Donothesitateto that youwillfindintheexaminationrooms.Examineandbecomefamiliarwiththis youtotheequipment ment. Anorientationsessionwillthenbeheldtointroduce thebeginningofyoursession,youwillbeaskedtosignaconfidentialityagree- At after theexam,youcankeepyourluggageatfrontdeskofhotel. watches, cellphones,purses,andhandbags.Ifyouareplanningtotravelimmediately locker forbelongingsthatyouarenotallowedtocarryduringtheencounter, suchas not storeitforyou.You willbeprovidedonlywithacoat rackandasmallstorage youaretravelingwithluggage,donotbringittothetestsite,asstaffcan- If than thepenprovidedatexamsite. need tobringapenofyourown;infact,youarenotalloweduseanythingother numbered badgetobewornaroundyourarm,apen,andclipboard.Thereisno clinic day. era, anddon’t trytolookforitduringtheencounters.Actasyouwouldonaregular safety oftheSPsandcandidatestoensurequality. Sodon’t worryaboutthecam- are notusedforscoringpurposes.To thecontrary, theyareusedonlytoensurethe Finally, rememberthateventhoughallyourencountersarevideotaped,thesetapes ter withoutit. Be suretobringyouradmissionpermit!You willnotbeadmitted tothetestcen- jet lag,eatwell,andgetexercise. Asluggishaffectandacloudymindcanleadto Remember torestbeforethe exam. Try togiveyourself afewdaystoovercome GUIDE TO THE USMLE STEP 2 CS 13 not perfection. Go for efficiency, Go for efficiency, . M . P If a patient is wearing a sombrero, If a patient is wearing a Clear your head before proceeding to head before proceeding Clear your graduation. You must be a graduate of a medical school You that was listed in the IMED at the time of your You need not be perfect. In fact, given the need not be perfect. In fact, given You (pay the fee) for the Match regardless of your ECFMG (pay the fee) for the Match regardless in the Match, however, the National Residency Matching Pro- in the Match, however, register Medical Students Medical School Graduates IMG Eligibility for the USMLE Step 2 CS participate placed tattoo might suggest certain risk behaviors, not just a keen appreciation suggest certain risk behaviors, not just placed tattoo might of body art. time constraints involved, the Step 2 CS rewards efficiency and relative com- the Step 2 CS rewards efficiency time constraints involved, pleteness over perfection. for everything you see. There is a reason and the diarrhea the case. He might have been in Mexico, inquire why this is a prominently Similarly, diarrhea. be a simple traveler’s he presents with may inefficiency and poor rapport. This is especially important if you are scheduled important if you This is especially and poor rapport. inefficiency as 11 can run as late session, which for an afternoon the past. the present, not Think about or should have should have done about what you Thinking encounter. your next you from your current encounter. asked will only distract perfection. Passing does not require You are not required to have passed the English-language proficiency test or the Test of English as a Foreign of English as a Foreign test or the Test to have passed the English-language proficiency not required are You FIRST AID FOR THE IMG 12 months of graduation when you take the exam. You must be enrolled in a foreign medical in a foreign be enrolled must You school listed in the International Medical Edu- (IMED, http://imed.ecfmg.org) cation Directory both at the time you apply and at the time you must also be within take the assessment. You a TABLE 1-2. TABLE Language to be eligible for the Step 2 CS. requirements for certification even if you have not received your certificate) by the requirements for certification even if you have not received your certificate) deadline (typically in February of each year). Applicants who do not rank-order-list residency in the United States, your timetable should reflect that and should be care- residency in the United States, your timetable fully planned at least one year in advance. are allowed to You status. To ECFMG gram (NRMP) requires that you be ECFMG certified (or that you meet Before the ECFMG for a Step 2 CS application, you must first take several contacting 1-2). whether you are eligible (see Table preliminary steps. Begin by ascertaining site for the latest eligibility criteria. Check the ECFMG Web to take the exam, you will need to factor Once you have established your eligibility “Match”). If you are planning to apply for a in the residency matching process (the If you are an IMG candidate seeking to pass the Step 2 CS, you must take a number If you are an IMG candidate seeking to a timetable to mastering logistical details to of variables into account, from plotting formulating a solid test preparation strategy. Determining Eligibility GUIDE TO THE USMLE STEP 2 CS 14 All theUSMLEexams within aseven-year need tobepassed period forECFMG certification. FIGURE 1-2. whether youarereadyforit,keepthefollowingpointsinmind: IndecidingwhentoapplyfortheStep2CS,takeit,and a seven-yearperiod. ECFMG certification,youneedtopasstheStep1,2CK,andCSwithin but notbeforeyouareconfidentthatfullyprepared.Remembertoget H-1B visa.Insummary, taketheStep2CSassoonyou areeligible(seeTable 1-2), perfect last-minuteboosttoyourapplicationandmayalsomakeyoueligibleforthe results backbeforetherank-order-list scoreonStep3canprovidea deadline.Agood applications. Inaddition,ifyouarecertifiedearly, youcan takeStep3andgetyour who haveyettotaketheStep2CS—evenifsuchcandidatesmoreimpressive are likelytoconsideryouareadyapplicantandmayfavoroverothercandidates submit yourapplicationforresidencyinthefall.Shouldyoudoso,programs isasignificant advantagetoobtainingECFMGcertificationbythetimeyou There target MatchDay(seeFigure1-2). fore, youshouldtaketheStep2CSnolaterthanOctoberinyearbeforeyour meet theserequirementswillautomaticallybewithdrawnfromtheMatch.There- knowledge andskills. weeks, factoringinyourlevelofEnglishproficiencyaswellmedical IMGs, preparationfortheexamtypicallyrequiresanywherebetween1and12 Schedule yourexamonthedatethatyouexpecttobefullypreparedforit.For residency. take theStep2CSinJuneorJulyordertobecertifiedwhenyouapplyfor three monthsbeforeyourdesiredexaminationdate.Ideally, youshouldaimto yourtestdatecanbedifficultduringbusyseasons.Applyatleast Scheduling Typical Step2CSTimelineforIMGs Match Prior to Year Match Year of June July May Dec Sept Mar Nov Oct Aug Mar Jan Apr Feb U.S. MatchDay Rank-order-list deadline certification ECFMG Receive to takeexam Typical period location Schedule testdateand Step2CS forUSMLE Register GUIDE TO THE USMLE STEP 2 CS 15 IWA to minimize to minimize IWA Use the ECFMG’s Use the ECFMG’s delays and errors. delays and An application that contains a photograph of the applicant that was taken more An application that contains a photograph of the applicant that was taken more than six months before the date the application was submitted no- An application in which the signature of the medical school official or the tary public is more than four months old does An application in which the medical school or notary public seal or stamp not cover a portion of your photograph An application that is not written in ink or is illegible An incomplete application An application that is not the original document (ie, faxed or photocopied) An application that contains a nonoriginal signature or photograph Determine how long it will take to get an appointment at the embassy. Determine how long it will take to get visa and whether a clearance periodFind out how long it will take to get the is required. in which the exam centers are located. Check travel availability to the cities Check with the U.S. embassy in your country to determine whether you need a Check with the U.S. embassy in your visa. If you choose to apply for the Step 2 CS using a paper application, it will take up paper application, Step 2 CS using a to apply for the If you choose take as few but it may notification of registration, to receive your to four weeks IWA. the ECFMG’s if you use to receive this information as 10 days a letter to present to the U.S. consulate in your country. This letter will be sent to in your country. a letter to present to the U.S. consulate 2 CS (ie, after you have paid the fee) and you only after you apply to take the Step a visa. For this reason, it is wise not to will not guarantee that you will be granted have arrived in the United States or have at schedule your actual exam day until you least obtained your visa. As proof of the reason for your visit to the United States, the ECFMG will send you As proof of the reason for your visit to visa and, having done so, for obtaining that visa (regardless of how time-consuming so, for obtaining that visa (regardless visa and, having done the Step 2 CS, you should may be). Before you apply to take and difficult this process therefore complete the following tasks: If also factor in the time outside the United States, you must you are an IMG living States if need a visa to come to the United do not a visa. You it may take to obtain Citizens of countries par- citizen or a permanent resident. you are a U.S. or Canadian not as European Union countries) may Program (such Waiver ticipating in the Visa whether you need a are responsible for determining You either. need to obtain a visa Some residency programs use the Step 2 CS as a screening tool to select IMG appli- use the Step 2 CS as a screening Some residency programs the Match. so it is ideal to meet the deadline for cants for interviews, mistake. Applications that contain the following common errors will be returned: Application Tips When application to take the Step 2 CS, be sure to read it carefully you receive your do not want your application returned to you—thus wasting before filling it out. You careless valuable time—simply because you forgot to answer a question or made a GUIDE TO THE USMLE STEP 2 CS 16 proficiency isbasedonthefollowingcomponents: ing theStep2CSliesinorganizingtheseskillsandpracticing.Your spokenEnglish language skillsneededtopasstheStep2CS.Forsuchcandidates,keypass- that mostIMGswhohavealreadypassedtheUSMLEStep1basicEnglish newsis the mainobstaclefacingIMGsatotherendofspectrum.Thegood of yourlifeinanEnglish-speakingcountry. Englishproficiencymay, however, be cine inEnglishyourmedicalschool,orhavespentatleastafewmonthsyears speaker, havestudiedinaU.S.orotherEnglish-speakingschool, learnedmedi- You maynothaveaproblemwithEnglishproficiencyifyouarenative to you. English clearlyandcomprehensiblytounderstandwhentheSPspeaks proficiency inspokenEnglish.InStep2CSterms,thisreferstotheabilityspeak manyIMGstakingtheStep2CS,acriticalconcernliesindemonstrationof For Improving Your EnglishProficiency your applicationonline,youcanuseOASIS(https://oasis2.ecfmg.org). make everyefforttosenditbyexpressmailorcourierservice.To checkthestatusof Once youhavecompletedyourapplicationanddouble-checkeditforerrors, Commonly encounterederrorsspecifictoIMGsincludethefollowing: An applicationthatdoesnotincludefullpayment not byyourmedicalschoolofficial An applicationthatdoesnotexplainwhyitwassignedbyanotarypublicbut lish. reaching anacceptableand even asuperiorlevelofclear, comprehensibleEng- correct yourmistakes.Themore youpractice,thebetteryourchanceswillbeof and askingsomeone(preferably anativeEnglishspeaker)tolistenyouand practicing commonstatementsandquestions,repeating them toyourselfaloud, Comprehensible pronunciation. grammatical errors. as muchpossible.Thiswillminimizethechancethatyou willmakesignificant with commonlyusedstatements,transitions,andquestions andtopracticethem The correctuseofgrammar. your comprehensibility. easier forSPstounderstandyouandwillminimizetheeffect youraccenthason you torememberandfortheSPunderstand.Speaking slowlywillalsomakeit stand. The abilitytospeakinamannerthatiseasyfortheSPfollowandunder- tion or toensurethatthetranslator’s stampcoversboththeoriginalandtransla- Failure tostapletogetheryourmedicalschooldiplomaanditsEnglishtranslation ploma iftheoriginalisnotinEnglish Failure tosendtheECFMGanEnglishtranslationofyourmedicalschooldi- face photographs Failure tosendtheECFMGacopyofyourmedicalschooldiplomawithtwofull- Toward thisgoal,choosephrasesthataresimple,direct,andeasybothfor The keytomasteringthiselementisbefamiliar Again, the key to good pronunciationliesin Again,thekeytogood GUIDE TO THE USMLE STEP 2 CS 17 questions. transitions, and used statements, spoken English is spoken English The key to better The key to practicing commonly practicing You may find it dif- may find You P.O. Box 6151 P.O. Princeton, NJ 08541-6151 609-771-7100 www.toefl.org TOEFL/TSE Services have said, simply repeat the phrase or question, or restate it in simple lay terms. the phrase or question, or restate have said, simply repeat The ability to correct and clarify your English if necessary. your English to correct and clarify The ability you and asks not understand in which an SP does for a situation ficult to prepare you can avoid said. Here again, you have just meaning of something you for the transitions; questions, and common statements, by practicing this situation words instead clearly as possible; and using nontechnical speaking as slowly and something you terms. If an SP still cannot understand of complicated medical necessary for the Step 2 CS. You may also consider taking the Test of English as a of English consider taking the Test may also necessary for the Step 2 CS. You doing so is no take the Step 2 CS. However, Foreign Language (TOEFL) before you 2 CS or to ECFMG certification. For more longer a prerequisite to taking the Step contact: information about the TSE and the TOEFL, oughly studying common cases and medical conditions (see Sections 3 and 4), you oughly studying common cases and medical can minimize this obstacle. the ECFMG suggests that you proficiency, If you are still unsure about your English you score of Spoken English (TSE) to get a measure of your abilities. If take the Test likely attained the level of English proficiency higher than 35 on this exam, you have increase the likelihood calm, concentrate, and take of making mistakes. So remain your time. time to do and say all the things you think are Fifteen minutes may seem like a short an organized plan. Most of but it will be more than enough if you follow necessary, so by thor- are the same in each encounter, the things you have to say in the exam Make every effort to remain calm throughout your clinical encounters. Nervousness remain calm throughout your clinical Make every effort to understand you. Likewise, making it difficult for the SP to can cause you to mumble, rushing, you will further and start looking at the clock and if you become nervous it more challenging for you to pass the Step 2 CS. Participating in a formal clinical it more challenging for you to pass the Step 2 CS. Participating in a formal This rotation in the United States is one of the best ways to polish your English skills. residency programs. Moreover, performing well on your rotation can earn you strong residency programs. Moreover, letters of recommendation, which are the most important part of your application better. after your USMLE scores. The more time you spend in such a rotation, the regard- Even if your Step 1 and Step 2 CK scores are impressive and you come highly make ed from a top international medical school, lack of proficiency in English will Many IMGs may lack basic familiarity with the workings of U.S. medical schools. A the Step clinical rotation or observership in the United States can prepare IMGs for 2 CS by introducing to the U.S. system and, in the process, immersing them in them performing a physical exam, and writing PNs. the “American” way of taking a history, Clinical rotations are also good to have on your curriculum vitae when you apply for Getting Clinical Rotations and Observerships GUIDE TO THE USMLE STEP 2 CS 18 your messagewillland e-mail blastcomposed way toguaranteethat medicine are thebest rotations forStep2 English isthebest Sending ageneric Internal medicine of poorlywritten in aspamfolder. CS preparation. and emergency here issomeadvicethatmayhelpyou: hospitals donothaveanysuchformalrotationortrainingprogram.Nonetheless, can stillbehighlyuseful.Gettinganobservershipisnoteasytaskbecausemost The observershipisperhapstheleastactivefunctionyoucanfillinahospital,butit ply forobservershipsandexternships. are nolongereligibleforclinicalelectiverotations(clerkships),butyoucanstillap- youarea If you areinterested. ever, itishighlyrecommendedthatyoualsodoarotationinthespecialtywhich send youanapplicationbymail.Forthepurposesofyourresidencyapplication,how- the personnelresponsibleforrotations.Mostoftime,suchwill United States,calltherelevantdepartmentsandmakeappointmentstomeetwith e-mail orwritetheprogramdirectorandchairmanofeach.Ifyouarealreadyin rotation. ChecktheWeb sitesoftheuniversitiesinwhichyouareinterestedand If youarestilla communication skillsifyouareinvitedforresidencyinterviews. will maketheStep2CSexperiencemoretolerableandmayultimatelyboostyour observership: some patientsandwritenotes.Hereisadvice for makingthemostofyour some topics,andattendconferences.Onrareoccasions, you maybeabletoexamine to doareobserve,roundswithyourteam,answeranoccasional question,present not touchapatientorwriteoncharts.Theonlythingsyou willofficiallybeallowed During yourrotation,youwill“officially”beanobserver, whichmeansthatyoucan- about them. Follow uponpatientsyourteam istakingcareofandlearneverythingyoucan Come earlyandstaylate(not verylate,though). Show ahighlevelofenthusiasm. where theydidsoandhowtoapply. Talk tootherphysicianswhoaredoingorhavedoneobservershipsandaskthem to meethimorher. Call theofficeofchairmanorprogramdirectorandtrytosetanappointment is thebestwaytoensurethatyourmessagewillendupinaspamfolder. the programandwhy. Agenerice-mailblastcomposedofpoorlywrittenEnglish better toaddressaphysicianbynameandspecificallymentionyourinterestin munication intheformofgrammaticallycorrectlettersore-mails.Itisalways pital. IMGsforwhomEnglishisnotafirstlanguageshouldsendtargetedcom- Send e-mailsand/orletterstothechairmanandprogramdirectorofeachhos- rotations. you mayknow. Connectionsareanimportantwaytouncovertheseunofficial Contact people(attendings,seniorresidents,secretaries,administrators)whom centers. types ofteachinghospitals:university, community, andVeterans Affairsmedical Prepare alistofhospitalsinyourareaoranythatinterestsyou.Includeall medical graduate, medical student, yourmissionismoredifficultbutnotimpossible.You it shouldnotbedifficultforyoutofindaclinical GUIDE TO THE USMLE STEP 2 CS 19 Practice writing PNs (see Section 4). Check other Web sites and discussion forums. They can be a good source of in- Check other Web formation. Review the steps of history taking (see Section 2). Choose and prepare common questions and cases (see Sections 3 and 4). Review the steps of the physical exam (see Section 2). Practice the physical exam as if you were performing the real exam. tion about the Step 2 CS. This will help you get a clear idea about regulations, tion about the Step 2 CS. This will help you get a clear idea about regulations, requirements, registration, examination dates, and all other details concerning the Step 2 CS. this Carefully prepare for the exam using the preparation materials included in book. Check and recheck the ECFMG and USMLE Web sites for the latest informa- Check and recheck the ECFMG and USMLE Web Memorial Hospital, Pawtucket, RI FL Miami, Mount Sinai Medical Center, DC Providence Hospital, Washington, University of Miami, Miami, FL DC Washington, Administration Medical Center, Veterans Harbor Hospital, Baltimore, MD (application and fees apply) Harvard Medical School, Boston, MA CT Hospital of St. Raphael, New Haven, Phoenix, AZ Maricopa Medical Center, physicians program) MN (visiting Mayo Clinic, Rochester, Banner Good Samaritan Medical Center, Phoenix, AZ Banner Good Samaritan Medical Center, Atlanta, GA Emory University, Hahnemann Hospital, Philadelphia, PA Talk to the nurses, secretaries, and support staff. This will improve your commu- secretaries, and support staff. This will to the nurses, Talk nication skills. observe the residents chance to examine patients, carefully If you do not get a during the physical exam. and medical students as you can. Do as many presentations Read about the cases your team is managing. the cases your team Read about that you are let them know patients, but always spend time with the Chat and to improve your histories and way to practice taking This is the best an observer. skills. language correct them, and compare PNs and orders, ask your residents to your own Write notes. them to the official Here is a partial list of hospitals that have been known to offer formal observerships of hospitals that have been known to Here is a partial list or externships: Some Final Tips you can take to help ensure your success on There are a few final practical measures the Step 2 CS: GUIDE TO THE USMLE STEP 2 CS 20 NOTES SECTION1 SUPPLEMENT The USMLE Step 2 CS Travel Guide

Introduction Traveling to the United States Atlanta (“The Big Peach”) Chicago (“The Windy City”) Houston (“Space City”) Los Angeles (“The City of Angels”) Philadelphia (“The City of Brotherly Love”) Useful Web Sites USMLE STEP 2 CS TRAVEL GUIDE 22 to thesesitesareprovidedattheendofthissection. cluded manyofthesehotelshere,butbesuretochecktheupdatedlistonline.Links ated reducedhotelrateswithmanynearbyhotelsfortheStep2CS.We havein- Also notethattheAssociationofAmericanMedicalColleges(AAMC)hasnegoti- Another greatsourceofinformationisthetravelsectiononUSMLEWeb site. by thetimeyougettoyourdestination,someofdetailsmaywellhavechanged. before youstartyourtrip.We havecompiledabroadrangeofsuggestionsforyou,but With thatsaid,makesuretoconfirmthedetailsofdestinationswe’vepresented enjoy yourselfinanewandexcitingcity. ter, thinkaboutgivingyourselfanextradayortwoaftertheexamtorelaxandreally your test,youmightwanttoscheduleatleastafewhoursseethesights.Evenbet- on whattheyhavetooffer. Althoughyoushouldnotletsightseeinggetinthewayof All fiveCSECdestinationsareamazingcities,andwedonotwantyoutomissout each destination. nally, wehaverecommendedafewwell-knownrestaurantsandtouristattractionsfor centers. Asacheaperoption,wehaveincludedoneyouthhostelforeachcity. Fi- tels, mostofwhicharereasonablypricedandwithinwalkingdistancetheCSEC CSEC testsitesandroutesfromtheairports.Alsolistedareanumberofnearbyho- Since mostofyouwillbeflying,wehaveplacedspecialemphasisondistancestothe best waystogetyourdestinationandsomethingsdoonceyouhavearrived. Skills EvaluationCollaboration,orCSEC,centers),wehaveprovideddetailsonthe travels. ForeachofthefivecitieswithaStep2CStestingcenter(calledClinical followingquickguidecanbeusedasaplanningtoolbothbeforeandduringyour The really important:doingagreatjobontheexam! ting allthedetailsinplacewelladvanceofyourtripwillhelpyoufocusonwhat’s ensure thateverythinggoesassmoothlypossibleontestdayistoplanahead.Get- extra travelstress—or, worsestill,problemsonthedayofexam.Thebestwayto After you’veworkedhardtopreparefortheStep2CS,lastthingyouneedis the following: quired aswell,especiallyifyou’recomingfromanother country. Thesemayinclude for theStep2CSisyour Arrange yourdocuments. help minimizetravelhassles. to theUnitedStatesforfirsttime—sohereareafew thingstokeepinmind We knowthatmanyofyouwhoareplanningtotake theStep2CSmaybecoming INTRODUCTION TRAVELING TOTHEUNITEDSTATES A U.S. Your your country). passport. tourist visa (usuallyaB-1/B-2visa;apply atthenearestU.S.embassyin scheduling permit. Generally, themostimportantdocumentyouwillneed However, other documentsmaybere- USMLE STEP 2 CS TRAVEL GUIDE 23 Be sure to make your reservations well Be sure to make your (consider getting one if you’re planning to getting one if you’re (consider When traveling abroad, particularly in major U.S. popula- When traveling abroad, Packing before air travel requires a lot of preparation. Here are Packing before air travel requires a lot Here are some guidelines for planning your visit and booking your international driver’s license driver’s international Before you choose a hotel, be sure to factor in the distance to the testing center Before you choose a hotel, be sure to factor in the distance to the testing center air- as well as the services each hotel offers—for example, whether it has a free Bear in mind that staying in a port shuttle, free breakfast, and access to Wi-Fi. to get a goodyour tickets and accommodations. price on sites and at Before you book a hotel or a flight, compare prices at multiple Web Sites” at the end of this supple- site (see “Useful Web Web each organization’s that include a combination of flight, lodging, packages and car ment). Vacation rental are usually cheaper than purchasing individually. Tag your baggage with brightly colored tape or a distinguishing mark so that you your baggage with brightly colored Tag can easily identify it at baggage carousels. Doing so will make it easier for you to schedule your exam well in advance. Try coat, or stethoscope in your checked baggage. Also remember to put a copy of coat, or stethoscope in your checked can locate you in the event that your itinerary in your baggage so that authorities your baggage is lost. to Using a mobile phone or a camera, take a photograph of your baggage to give the authorities in the event that your baggage is delayed or lost. There are many restrictions for carry-on luggage these days, particularly with There are many restrictions for carry-on Web Security Administration’s regard to liquids. Check the U.S. Transportation for the most up-to-date information. site (www.tsa.gov) Do not keep your scheduling permit, lab Prepare for lost or delayed baggage. Never carry anything in your baggage that doesn’t belong to you. You are respon- belong to you. You that doesn’t Never carry anything in your baggage including anything illegal that might have sible for the contents of your baggage, been placed there by someone else. walking alone on deserted streets at night. Avoid As a whole, Americans are friendly and willing to be of assistance, but not every- are friendly and willing to be of assistance, As a whole, Americans alert to individuals who seem a bit one has the best of intentions. Be particularly way for you. too eager to help or who go out of their by taxi, train, or any form of public Keep an eye on your baggage while traveling thieves tend to target visitors. transportation. Pickpockets and petty An permit. an international driving and potentially testing center) drive to your site (www.usa.gov/Topics/Foreign-Visitors- Web U.S. government Check the testing location. driving rules in your for links to the Driving.shtml) Plan, plan, plan. hotel: a few useful tips: Pack appropriately. in advance, and think about how you’re going to get around in the test city. Once in the test city. about how you’re going to get around in advance, and think how to get to the CSEC destination, make sure you know you’ve arrived at your to stay a bit farther away. your test, especially if you’re planning center on the day of Consider travel safety. general guidelines to ensure your safety: important to follow a few tion centers, it’s Make sure your travel plans are in place. Make sure your travel USMLE STEP 2 CS TRAVEL GUIDE 24 of Coca-Cola.We knowyou’llenjoyyourtimeinthisdiverseandthriving city! home totheCentersforDiseaseControlandPreventionaswellheadquarters Atlanta-Pacifica railwaythatranthroughthetownin1840s.Today, Atlantais main north-southandeast-westrailwayhub;infact,itsnamewasderivedfromthe tal cityofthegreatstateGeorgia.Throughouthistory, Atlantahasservedasa Atlantametroareahasapopulationofmorethanfivemillionandisthecapi- The Getting There all theCSECdestinationsareasfollows: nies requireyoutobe25yearsoldrentavehicle.Themostpopularcompaniesin Don’t forgetyourinternationaldriver’s license!Alsobearinmindthatmostcompa- Choosing todrive. ATLANTA (“THEBIGPEACH”) Atlanta, GA30349-5585 1745 PhoenixBoulevard,Suite500(5thFloor) Two CrownCenter Clinical SkillsEvaluationCollaborationCenter site mayhaveservicesgearedspecificallytowardexaminees. travel timetotheCSECcenterontestday. Also,hotelsthat areclosertothetest but may increase youth hostelmaysaveyoumoneyonhotelaccommodations Ground: downtown andonlyafewblocksfromtheCSECcenter. port (ATL) (www.atlanta-airport.com), located about9miles/14.5kmsouthof Air: Thrifty (www.thrifty.com): 800-847-4389 Payless (www.paylesscarrental.com): 800-729-5377 National (www.nationalcar.com): 877-222-9058 Hertz (www.hertz.com): 800-654-3131 Enterprise (www.enterprise.com): 800-261-7331 Dollar (www.dollar.com): 800-800-4000 Budget (www.budget.com): 800-218-7992 Avis (www.avis.com): 800-633-3469 Alamo (www.alamo.com): 877-222-9075 leans andNewYork. ing center. Atlantaisonthe CrescentLine,whichrunsbetweenNewOr- train stationisalsolocateddowntown,about16miles/25.7 kmfromthetest- Amtrak, 1688PeachtreeStreetNorthwest(www.amtrak.com): Themain center, which isapproximately20minutesbytaxiforaflatrateof$25. bus terminalislocateddowntown,about12miles/19.3 km fromtheCSEC Greyhound, 232ForsythStreetSouthwest(www.greyhound.com): Themain Atlanta’s majorairportistheHartsfield-JacksonAtlanta InternationalAir- Rentalcarsarealsoanoptionfortravelingtothesitelocation. USMLE STEP 2 CS TRAVEL GUIDE 25 Atlanta has a regional metro system called MARTA called MARTA Atlanta has a regional metro system All rental car companies are located at the Hartsfield-Jackson All rental car companies are located Most hotels offer free shuttle service to and from the airport. The from the airport. The service to and offer free shuttle Most hotels Taxis are available at the airport and at the bus/train terminals and cost at the airport and at the bus/train terminals are available Taxis Yellow Cab: 404-521-0200 Yellow Atlanta Checker Cab: 404-351-1111 National: 404-752-6834 Day and Night Cab Co.: 404-767-7464 Atlanta Airport Shuttle (www.taass.net) Atlanta Airport Shuttle (www.airportmetro.com) Airport Metro Shuttle the CSEC site via MARTA. For more information, check out this user-friendly For more information, check out this user-friendly the CSEC site via MARTA. guide: www.itsmarta.com/uploadedFiles/Using_Marta/How_to_ride_MARTA/ RookiesGuide2013.pdf. tions, or from many retail outlets. You simply use cash to add value to the card simply tions, or from many retail outlets. You A trip downtown will station entry points for service. and tap it at the MARTA will cost $2.50. Hours of operation can take approximately 15−20 minutes and planning can be easily accomplished site. Trip Web be found on the MARTA site. There is no access to iPhone or Android app or the Web using the MARTA Public transportation: and it is by www.itsmarta.com), Authority, (Metropolitan Atlanta Rapid Transit bus, ride the train or from the airport. To far the cheapest form of transit to and Breeze Card, which can be purchased you will need to purchase a stored-value Station (located near the Ride Store inside the Airport for $1 at the MARTA sta- from machines in MARTA baggage claim area in the domestic terminal), Rental cars: pick up your luggage, follow the signs to Rental Car Center (RCC). Once you site for spe- the airport Web for transport to the RCC. Visit SkyTrain the ATL cific details. $30–$35 from the airport to downtown. The ride from the airport to the CSEC to downtown. The ride from the $30–$35 from the airport companies include the following: center is about 10 minutes. Local taxi vices by destination. Airport Metro Shuttle serves most of the metro area, and Airport Metro Shuttle serves most vices by destination. city limits of At- serves the area within the corporate Atlanta Airport Shuttle and Buckhead). lanta (downtown, midtown, Taxis: Shuttles: service. There complimentary the hotels that offer site has a list of airport Web to downtown or other services available from the airport are also airport shuttle site lists shuttle ser- surrounding cities. The airport Web major attractions and The Atlanta CSEC center is located a few minutes to the south of the Hartsfield- Fayetteville Jackson airport, about 0.25 mile/0.4 km east of the intersection of West brown brick building that houses the Road and Phoenix Boulevard. The V-shaped The center is on the fifth CSEC center should be visible from the I-285 highway. and plenty of free parking is available. floor, CSEC Center Location Getting Around When You Arrive When You Around Getting USMLE STEP 2 CS TRAVEL GUIDE 26 favorites: TakeAtlanta hasamazingSouthernfood. advantageofthisandenjoysomeour Where toEatandPlay USMLE deals,listedontheAAMCWeb site(markedwithanasteriskbelow). highways around,soplanyourwalkcarefully. Remember toaskhotelsabouttheir youth hostelinthearea.Mostarelocatedjustoutsideairport,buttheresome The followinglistincludesafewhotelslocatedclosetothetestsiteaswell Where toStay you’re intown: Atlanta hasmuchtoseeanddo.Herearejustafewplaces toconsiderseeingwhile What toSee neighborhood. Underground Atlanta: in historicPiedmontPark. World ofCoca-Cola: gallons ofwaterand100,000speciessealife. Georgia Aquarium: live bluesmusiceverynight. 607-1622. AnAtlantahotspotservingupSouthern-stylebarbecueandplaying Fat Matt’s RibShack($): hot meals.Includesmanysmallethnicrestaurantsaswell. and town; 404-659-1665.Ahistoricmarketwithstallsthatfeaturefreshproduce Sweet AuburnCurbMarket($): serving burgersandhotdogssince1928. world’s largestdrive-inrestaurant,theVarsity isanAtlantaiconthathasbeen The Varsity ($): MARTA NorthAvenue station. A bitfartherawayfromtheCSECcenter, butonlyafewblocksaway fromthe miles/22.5 kmaway);404-875-9449.Acheaperoptionwithplentyofnightlife. Atlanta InternationalHostel($): Road (3.3miles/5.3kmaway);404-766-0303. *Hilton GardenInnAtlantaAirport/MilleniumCenter($$): km away);770-997-1100. *Sheraton GatewayAtlantaAirport($$): 1.5 miles/2.4kmaway);404-684-9898. *Comfort Inn&SuitesAirportSouth($): (about 0.7mile/1.1kmaway);770-996-5800. *Best Western Hotel&SuitesAirportSouth($$): CSEC center, makingitveryconvenient. Road (0.7mile/1.1kmaway);770-991-1099.Justaroundthecornerfrom *Country Inn&SuitesAtlantaAirportSouth($$): 61NorthAvenue Northwest,Downtown;404-881-1706.The Theworld’s largestaquarium,withmorethan8.5million Comeandcelebratetheoriginalhomeofthissugarydrink A malllocatedunderthestreets intheFivePoints 1811PiedmontAvenue Northeast,Midtown;404- 223PoncedeLeonAvenue Northeast(14 209 Edgewood Avenue209 Edgewood Southeast,Down- 2450OldNationalParkway(about 1900SullivanRoad(2.0miles/3.2 1556PhoenixBoulevard 5100 West Fayetteville 2301 Sullivan USMLE STEP 2 CS TRAVEL GUIDE 27 Home to the Martin Luther King, Jr., National Historic National Historic Jr., Martin Luther King, Home to the Most nearby hotels offer free shuttle service to and from the airport. Chicago has two major airports. The larger is O’Hare International Air- Chicago has two major airports. The Continental Airport Express (www.airportexpress.com): 888-284-3826 Continental Airport Express (www.airportexpress.com): 773-734-6688 Omega Airport Shuttle (www.omegashuttle.com): Amtrak, Canal Street between Adams and Jackson Boulevards (www.am- trak.com): The main train hub in Chicago is at downtown Union Station. This is a goodbut you’ll have to take public transportation or a taxi choice, Authority Blue Line runs to to get to the test site. The Chicago Transport Cumberland Station from Union Station, taking you very close to the CSEC center. Greyhound, 5800 North Cumberland Avenue (www.greyhound.com): The (www.greyhound.com): Avenue Greyhound, 5800 North Cumberland is just a few blocks Greyhound bus terminal Chicago Cumberland Avenue away from the CSEC center and is the closest of the six bus terminals in Chicago. 5 miles/8 km from the CSEC center. The other is Chicago Midway Airport 5 miles/8 km from the CSEC center. km southwest of downtown and roughly (MDW), located about 12 miles/19.3 Both have easy public transportation 20 miles/32.2 km from the testing center. its closer proximity but O’Hare may be more convenient given within the city, to the testing site. port (ORD), of downtown but only which is about 20 miles/32.2 km northwest Shuttles: of the airport shuttle services. Schedules and up-to-date can also use either You fares and booking are available online: Ground: Air: Sweet Auburn District: Sweet Auburn Site. www.lonelyplanet.com/worldguide/usa/atlanta/ www.atlanta.net Chicago, IL 60631 Clinical Skills Evaluation Collaboration Center Clinical Skills Evaluation Higgins Road, Suite 600 8501 West CHICAGO (“THE WINDY CITY”) CHICAGO (“THE For more information, check out: information, check For more Getting Around When You Arrive When You Getting Around Getting There life, so be sure to enjoy your stay! Located financial and on the shores of Lake Michigan, Chicago is the principal currently the third-largest city in the United cultural center of the Midwest and is lore, blues clubs, and biting cold winters. States. Chicago is known for its gangster City is bursting with Windy shopping, and culture, today’s plenty of history, With USMLE STEP 2 CS TRAVEL GUIDE 28 mute onthemorningoftest. west alongI-90.You’ll probablygetacheaperrateifyou’rewillingtomakecom- isk below).Ifyouhaveacar, youcanalsocheckoutsomeofthehotelsabitfarther tels abouttheirUSMLEdeals,listedontheAAMCWeb site(marked withanaster- optionsnearthetestingcenter.The followinghotelsaregood Remembertoaskho- Where toStay for othertenants,suchasFirstMidwestBank,ChicagoTitle, orWestwood College. park inthevisitorlotfordurationofexam,butdonotspacesreserved does Note fordrivers: the FirstMidwestBankBuilding.Thereshouldbeplentyoffreeparkingatsite. walk orcabridetothenorthoverhighway. TheCSECcenterislocatedwithin pressway). FromCumberlandStation,theCSECcenterisaquick0.8-mile/1.3-km km fromdowntownandjust5miles/8eastofO’HarealongI-90(KennedyEx- The examcenterislocatedonthenorthwestsideofChicago,about15miles/24.1 CSEC CenterLocation from O’HaretotheCSECcenter: Chicago. Theycostroughly$30–$40fromO’Haretodowntownandabout$10 Taxis: away); 847-671-6350. *Crown PlazaChicagoO’Hare ($$): mile/1.3 kmaway);773-693-5800. Very closetotheCumberlandBlueLinestop. *Holiday InnChicagoO’Hare($$): (0.7 mile/1.2kmaway);773-380-9600 *Renaissance ChicagoO’HareSuites($$$): mile/0.6 kmaway);773-867-0000 *SpringHill SuitesChicagoO’Hare($$): away); 773-693-4444.Rightnextdoortothetestingcenter! *Marriott ChicagoO’Hare($$$): sitchicago.com. are availableatallstations.Checkoutfaresandschedulesonlinewww.tran- Avenue). Single-ridefaresare$2.25;multiple-daypasses alsoavailable.Fares the CSECcenter, you’llwantCumberlandStation(5800North ter andO’Hareareonthe“L”BlueLine,bothconnecttodowntown.For light-rail system,ortakeaChicagoTransit Authoritybus.BoththeCSECcen- Public transportation: arrival terminaltotherentalcarsite. Rental cars: not Yellow Cab:312-829-4222 Flash Cab:773-561-4444 Des PlainesCabService:847-826-8424 American United:773-248-7600 applytoStep2CSexaminees,soyouarestillfreeparkthere.You may As inanybigcity, taxisareusuallythemostdirectway togetaround Rental carcompaniesatO’Hareofferfreeshuttleservicefromthe The signsinthevisitorlotindicateone-hour-only parking.This RidethefamousChicago“L,”aneasy-to-useandcheap . 83 etHgisRa 00 ie00 km 8535West HigginsRoad(0.02mile/0.03 5440NorthRiverRoad(2.1 miles/4.5km . 5615NorthCumberlandAvenue (0.8 8101 West HigginsRoad(0.04 8500 West BrynMawrAvenue USMLE STEP 2 CS TRAVEL GUIDE 29 135 East Lake Street; 312-616-1200. 135 East Lake Street; 24 East Congress Parkway (15 miles/24.1 Parkway (15 24 East Congress 6501 North Manheim Road (2.8 miles/3.3 km (2.8 miles/3.3 km Manheim Road 6501 North 736 North Clark Street; 312-642-6261. Check out one of 736 North Clark Street; 312-642-6261. 980 North Michigan Avenue; 312-280-2750. Try lunch at 312-280-2750. Try 980 North Michigan Avenue; The heart of the city, with upscale shopping and fantastic with upscale shopping and fantastic The heart of the city, Visit the Skydeck for amazing views from the second-tallest build- the Skydeck for amazing Visit Features museums, shops, restaurants, and even a Ferris wheel on the Features museums, shops, restaurants, www.lonelyplanet.com/worldguide/usa/chicago/ www.choosechicago.com www.cityofchicago.org/tourism shore of Lake Michigan. Magnificent Mile: restaurants running along Michigan Avenue. Willis Tower: Tower: Willis ing in North America. Navy Pier: Blue Chicago ($$): blues clubs, perhaps after you’re done with the test. world-famous Chicago’s Chicago is the town for pizza, and Giordano’s delivers some of the best. There delivers some of the best. There for pizza, and Giordano’s Chicago is the town to one that works for you. This is likely so find the the city, are branches all over had in a while. be the best and most filling meal you’ve Café Spiaggia ($$): next door to the world-famous Italian the café. This relaxed restaurant is just be a favorite of the Obamas. restaurant Spiaggia, which is known to Giordano’s Famous Chicago Pizza ($$): Giordano’s *Sheraton Chicago O’Hare: *Sheraton away); 847-699-6300. ($): International Chicago Hostelling about 15 is located downtown, large hostel 312-360-0300. This km away); plan at least an hour to make the trip so the testing center, miles/24.1 km from on the Blue Line. Clinical Skills Evaluation Collaboration Center Suite 700 400 North Sam Houston Parkway, Houston, TX 77060 HOUSTON (“SPACE CITY”) HOUSTON (“SPACE For more information, check out: and the testing center. After you’re done taking the exam, think about booking a late After you’re done and the testing center. downtown. flight and jumping on the “L” for an afternoon What to See city just because you’re staying near the airport miss the sights of this amazing Don’t Where to Eat and Play to Where here are just a few. of amazing restaurants of all varieties; Chicago has hundreds Sam Houston, then the president of the Republic of Texas. Today, Houston is one of Today, Sam Houston, then the president of the Republic of Texas. the largest cities in the United States and is home to many major energy companies It in addition to a substantial portion of the biomedical and aeronautical industries. Houston was founded in 1836 on land near the Buffalo Bayou and was named after Houston was founded in 1836 on land near the Buffalo Bayou and was named USMLE STEP 2 CS TRAVEL GUIDE 30 Getting Around WhenYou Arrive Getting There held hereeveryspringandfall.Enjoytheshow! also boastsoneofthebestartfestivalsincountry, the BayouCityArtFestival, the attachedgarage. see McDonald’s andArby’s restaurantsacrossthestreet.Freeparkingisavailable in a parkinggaragewithlarge“400”onthesidethatisvisible fromthestreet.You’ll at theintersectionofImperialValley DriveandBeltwayEastAccessRoad.Thereis The CSECcenterislocatedonthenorthsideofHouston inalargeofficebuilding CSEC CenterLocation trip planner, whichwillhelpyoufigureoutthedetails. 10–45 minutes,dependingontherouteandtimeofday. Visit theWeb sitefora serve theareaaroundairport,CSECcenter, andhotels.Buses runevery includes busroutesandlightrail(www.ridemetro.org). Lines102,56,and86 Public transportation: Web sitefordrivingdirections. see whiteandmaroonbusesthatwilltakeyoutotheCRCF. ChecktheUSMLE list ontheairportWeb site.Followthesignsinarrivalterminal andyouwill Rental CarFacility(CRCF)attheBushIntercontinentalAirport.Check Rental cars: ($20–$40 totheCSECcenterfromairport): Taxis: 713-523-8888. hotel doesn’t provideservice,checkoutSuperShuttle(www.supershuttle.com): Shuttles: Ground: km). CSEC center(8miles/12.9km);HOUissmallerandfartheraway(27miles/43.5 Hobby Airport(HOU).IAHisthelargeroftwoandmuchcloserto Air: Yellow Cab:713-236-1111 United Cab:713-699-0000 Square DealCab:713-659-5105 Liberty Cab:713-695-6700 Sunset Limitedline,whichrunsallthewayfromLouisianatoCalifornia. Amtrak, 902Washington Avenue (www.amtrak.com): Houstonis onthe to yourhotelshouldtakeabout30minutes. nal downtown.Outsidethestation,thereareplentyoftaxisavailable.Aride Greyhound, 2121MainStreet(www.greyhound.com): Houston hasatermi- Houstonhastwomajorairports:GeorgeBushIntercontinental(IAH)and AnumberoftaxicompaniesoperateinHouston;belowarejustafew MosthotelsaroundtheCSECcenterofferfreeshuttleservice.Ifyour MultiplerentalcarcompaniesareavailableattheConsolidated If you’rereallyadventurous,tryHouston’s Metro,which USMLE STEP 2 CS TRAVEL GUIDE 31 15700 JFK 15222 JFK Boulevard 300 Ronan Park Place (0.7 mile/1.1 1230 North Sam Houston Parkway East 1230 North Sam Houston Parkway East 12701 North Freeway (1.9 miles/3.1 km 12701 North Freeway (1.9 miles/3.1 5302 Crawford Street (18 miles/28.9 km 5302 Crawford Street (18 miles/28.9 4720 Almeda Road; 713-523-2861. A res- 500 North Sam Houston Parkway East (0.2 500 North Sam Houston 502 North Sam Houston Parkway East (0.2 502 North Sam 6 North Sam Houston Parkway East (0.5 mile/0.8 6 North Sam Houston 544 Yale Street; 713-426-2313. Relaxing and fun. Go for Street; 713-426-2313. 544 Yale 2040 West Gray Street, 832-200-1492. Américas serves Cen- 2040 West Located downtown with five great venues. Check out Bayou Theater district: Place, with its many theaters, bars, and restaurants. Dry Creek Café ($): one of their “Bad Ass” burgers. Américas ($$$): tral and South American cuisine with flair. Restaurant ($$): Spanish Village their delicious food since 1953. Try taurant that has been serving “Tex-Mex” margaritas. *Sheraton North Houston at George Bush Intercontinental ($$): *Sheraton North Houston at George Boulevard (4.5 miles/7.2 km away); 281-442-5100. Houston International Hostel ($): cheap ($15/night in a dormitory) and is away); 713-523-1009. The hostel is very CSEC center. located about 30 minutes away from the (1.8 miles/2.9 km away); 281-987-7100. *Comfort Inn Greenspoint ($$): away); 281-875-2000. Airport ($$): *Holiday Inn Houston Intercontinental (4.3 miles/6.9 km away); 281-449-2311. km away); 281-447-6888. Cheap with goodkm away); 281-447-6888. service, but there is no shuttle from the airport. ($$): *Hyatt Place Houston/Greenspoint a goodkm away); 281-820-6060. The Hyatt is to the CSEC center, deal, is close and is well recommended. ($): *Super 8 IAH West/Greenspoint mile/0.3 km away); 281-820-2101. The Baymont gets high marks for cleanliness 281-820-2101. The Baymont gets high mile/0.3 km away); great value. and service and is a North ($): *Park Inn Houston 281-931-0101. mile/0.3 km away); Inn & Suites ($): Venetian *Baymont Inn & Suites ($): *Baymont Inn & sights and sounds of Houston. What to See the If your exam is over by early afternoon, you might have some extra time to enjoy Where to Eat and Play Where the world but is especially well known for its Houston has cuisine from all around necessarily close to the test Latin American fare. The restaurants we’ve listed aren’t but we thought it might be fun for you to get out! center, Where to Stay Where from most, al- can walk site. You around the test hotels are located The following their USMLE deals, to ask hotels about great. Remember sidewalks aren’t though the an asterisk below). site (marked with Web listed on the AAMC USMLE STEP 2 CS TRAVEL GUIDE 32 favorite actors.Take insomestargazingwhile you’reintown! Theatre,theWaltcultural sites,suchastheKodak DisneyConcertHall,andallyour thehubofU.S.motionpictureindustry.wood, L.A.isalsohometosomeamazing tural andethnicdiversity. Oneofitsmostnotableattractions,course,isHolly- Angelesisoneofthebest-knowncitiesinUnitedStatesandrichcul- Los Getting Around WhenYou Arrive Getting There For moreinformation,checkout: LOS ANGELES(“THECITYOFANGELS”) El Segundo,CA90245 100 NorthSepulvedaBoulevard,13thFloor Clinical SkillsEvaluationCollaborationCenter seum. and parks.ItwouldbeashametopassuptheJohnC.FreemanWeather Mu- Museum district: These includeAdvantage,Alamo, Avis, Dollar, Enterprise,Hertz,andNational. Rental cars: $15 fromtheairporttoCSECcenter): Taxis: LAX. Ifyourhoteldoesn’t providesuchservice,check outthefollowing: Shuttles: Ground: miles/4.8 kmfromtheCSECcenter. (LAX). Itisoneofthebusiestairportsinworldandlocatedonlyabout3 Air: www.lonelyplanet.com/destinations/north_america/houston www.visithoustontexas.com game whileyou’rethere. Sports: L.A. Taxi/United CheckerCab: 213-627-7000 Independent Taxi OwnersAssociation:800-521-9294 Beverly HillsCab:310-273-6611 Prime Time Shuttle(www.primetimeshuttle.com): 800-733-8267 SuperShuttle (www.supershuttle.com): 800-258-3826 rail routesthatconnectittocitieslikeNewOrleans,Chicago,andSeattle. Amtrak, 800NorthAlamedaStreet(www.amtrak.com): L.A.isonmultiple your hotelshouldtakeabout30minutes. near downtown.Plentyoftaxisareavailableoutsidethestation.Arideto Greyhound, 1716East7thStreet(www.greyhound.com): L.A.hasaterminal LosAngelesisservedbyonemajorairport,International AnumberoftaxicompaniesoperateinL.A.;beloware justafew($10– Check outanAstros(www.astros.com) orRockets(www.rockets.com) ManyhotelsaroundtheCSECcenterofferfreeshuttle servicefrom Multiple rentalcarcompanies (nearly40!)areavailableinthearea. LocateddowntownnearRiceUniversity, withmanymuseums USMLE STEP 2 CS TRAVEL GUIDE 33 1985 East Grand 6101 West Century Boulevard (2.3 6101 West 2100 East Mariposa Avenue (0.9 mile/1.4 2100 East Mariposa Avenue 2135 East El Segundo Boulevard (0.4 mile/0.6 2135 East El Segundo Boulevard (0.4 mile/0.6 1624 Schrader Boulevard (24 miles/38.6 km 1804 East Sycamore Avenue (0.9 mile/1.4 km 1804 East Sycamore Avenue Despite its reputation, L.A. does have public transporta- reputation, L.A. does Despite its 525 North Sepulveda Boulevard (0.4 mile/0.6 km away); 525 North Sepulveda Boulevard (0.4 mile/0.6 Residence Inn El Segundo ($$): minutes away from the center by car if traffic is normal. It’s fun but loud, so if you minutes away from the center by car if traffic is normal. It’s plan on staying here, be sure to bring earplugs. km away); 310-726-0100. *Sheraton Gateway Los Angeles ($$): miles/3.7 km away); 310-642-1111. USA Hostels Hollywood ($): 30 away); 323-462-3777. This hostel is very cheap ($30–$80/night) and is about Avenue (0.6 mile/1 km away); 310-322-0999. The Doubletree is routinely rec- (0.6 mile/1 Avenue ommended by guests for its comfortable beds, clean rooms, and complimentary warm chocolate chip cookies. LAX South ($$): Travelodge away); 310-615-1073. An acceptable budget option, but service can be spotty. *Hilton Garden Inn El Segundo ($$): fast. *Hacienda Hotel ($$): An older but walk to the CSEC center. 310-615-0015. A convenient 10-minute decent choice—just be prepared for small elevators. *Doubletree Hotel Los Angeles International Airport ($$): * rooms and a goodkm away); 310-333-0888. Great, quiet break- complimentary site, www.metro.net. every 7–15 minutes during rush hour. Get off at the El Segundo/Nash station, Get off at the El during rush hour. every 7–15 minutes the park on the Boulevard (0.5 mile/0.8 km) toward walk west on El Segundo Boulevard. The and make a right on North Sepulveda south side of El Segundo, find can the stop. You generally within walking distance of hotels listed are also transportation Web and a Metro trip planner on the public more information For a full list of rental companies, visit www.lawa.org/lax and see “Ground Trans- and see visit www.lawa.org/lax of rental companies, For a full list portation.” Public transportation: is also a free bus stops. There not far from rail and the CSEC center is tion, and is on the Green Line, a rail line that station the Aviation shuttle from LAX to The Green Line runs from the El Segundo/Nash station. just two stops away The following hotels are situated around the test site. You can walk from most, al- the test site. You The following hotels are situated around Remember to ask hotels about their USMLE though not all have great walking routes. (marked with an asterisk below). site deals, listed on the AAMC Web veda and El Segundo Boulevards. You’ll see a series of large office towers; turn in the see a series of large veda and El Segundo Boulevards. You’ll visitors’ Follow the signs to get to Towers.” first driveway marked “Pacific Corporate parking ($9/day). to Stay Where CSEC Center Location side of L.A., only a few miles from LAX and The CSEC center is located on the west It is situated at the corner of North Sepul- about 20 miles/32.2 km from downtown. USMLE STEP 2 CS TRAVEL GUIDE 34 dependence wasfirstsignedon July4,1776.AndbesuretoeataPhillycheesesteak! While you’reintown,checkoutIndependenceHall,where theDeclarationofIn- Today, Philadelphiaisthriving,boastingthefifth-largestmetroareaincountry. Continental Congresses,andthereisawealthofplacesyou canvisittosoakitallup. activity duringtheAmericanRevolution,servingas site oftheFirstandSecond Philadelphia isagreatcitythatsteepedinU.S.history. Itwasanexusofpolitical reach withoutacar, butit’s worthatry. of L.A.Asthelocalssay, L.A.isvery“spreadout,”sothesightsaren’t alwayseasyto If yourexamisoverbyearlyafternoon,youmighthavesomeextratimetoseeabit What toSee almost everytaste.Herearejustafewofourfavorites: for L.A. isoneofthemostethnicallydiversecitiesinworld,soyoucanfindfood Where toEatandPlay For moreinformation,checkout: OF BROTHERLY LOVE”) PHILADELPHIA (“THECITY Clinical SkillsEvaluationCollaborationCenter Philadelphia, PA 19104 3624 MarketStreet,3rdFloor www.laweekly.com www.latourist.com with attractionslikeMuscleBeachandthearea’s renownedstreetperformers! Venice Beach: station. transfer totheRedLine.ExitatHollywood/Highland take sometime.Fromthetestcenter, taketheGreenLinetoBlue Lineand If youdon’t haveacar, youcanridetheMetroRail,butremember thatthiswill Hollywood: bratwurst. You’ll havetoseeitbelieveit. place togetdinnerandenjoythenightlyDJs.Offersgreatbeers,sushi,duck, Medusa Lounge($$$): with rice,beans,plantains,andcollardgreens.Simpledelicious. L.A., thisunassumingrestaurantservesupBrazilianfare.Grilledplatescome WoodSpoon ($): and enjoyagreatmealonthebeach. variety offare,includinggreathamburgersandsteaks.Hangoutwiththelocals 457-2503. SituatedoffPacificCoastHighway, thisseasiderestaurantoffersa Paradise CoveBeachCafé($$): Enjoy a stroll down Hollywood BoulevardandtheWalk EnjoyastrolldownHollywood ofFame. Only15minutesaway. Take insomeoftheuniquenessL.A. 107West NinthStreet;213-629-1765.Locatedindowntown 3211BeverlyBoulevard;213-382-5723.Anexciting 28128PacificCoastHighway, Malibu;310- USMLE STEP 2 CS TRAVEL GUIDE 35 Philadelphia has an extensive public transportation net- Philadelphia has an extensive public Multiple rental car companies are available. Follow directions at Multiple rental car companies are available. There is limited shuttle service from the airport, but one does cover There is limited shuttle service from A number of taxi companies operate in the city; below are just a few A number of taxi companies operate Philadelphia is served by one major airport, Philadelphia International Air- International major airport, Philadelphia is served by one Philadelphia Olde City Taxi Coach Association: 215-338-0838 Olde City Taxi 800-936-5111 PHL Taxi: Cab: 215-333-3333 Yellow Quaker City Cab: 215-728-8000 Liberty Cab: 215-389-8000 multiple Amtrak lines, including the high-speed Acela line, which connects including the high-speed Acela line, multiple Amtrak lines, DC. Other lines connect Philadelphia, and Washington, Boston, New York, Philadelphia to the South and the Midwest. Greyhound, 1001 Filbert Street (www.greyhound.com): The Greyhound ter- Street (www.greyhound.com): Greyhound, 1001 Filbert available outside the downtown area. Plenty of taxis are minal is located near to your hotel should take about five minutes. the station. A ride This bus 1001 Filbert Street (www.peterpanbus.com): Peter Pan Bus Lines, same address as Greyhound. line is located at the Philadelphia is on Market Street (www.amtrak.com): Amtrak, 30th and 2955 or you can transfer to the Market-Frankford line and take it to 34th and Market or you can transfer to the Market-Frankford line and take it to 34th and Market Multiple-use passes are Street, which is just two blocks from the testing center. available. Fares within the city vary depending on the destination and payment site for more information. method (cash vs. tokens). See the Web the airport to Zone 2 outside the baggage claim area for car pickup. the airport to Zone 2 outside the baggage Public transportation: Buses and a high-speed rail line connect (www.septa.org). work, called SEPTA The to the airport. Although the rail line is more expensive, it is easier to use. the Airport rail line (R1) costs an $8 (cash-only) one-way fare and connects all terminals to the 30th Street station, which is six blocks from the testing center— Rental cars: Shuttles: 215-724-8888. the area: Lady Liberty Company (www.ladylibertyshuttle.com): Taxis: ($28.50 flat rate from the airport to downtown): Air: world. PHL is country and the from all around the It serves flights port (PHL). km from the CSEC center. located about 10 miles/16.1 Ground: CSEC Center Location campus. The CSEC center is located downtown, near the University of Pennsylvania parking It can be found near the intersection of Market and 36th Streets. There is a lot right across the street ($14/day). Getting Around When You Arrive When You Getting Around Getting There Getting USMLE STEP 2 CS TRAVEL GUIDE 36 best: Philadelphia isagreatcitywithvarietyofrestaurants.Herearefewthe Where toEatandPlay their USMLEdeals,listedontheAAMCWeb site(markedbelowbyanasterisk). center isdowntown,thesehotelsarefairlyexpensive.Remembertoaskabout The followinghotelsarewithinwalkingdistanceofthetestsite.SinceCSEC Where toStay in theheartofit. the sightsofPhiladelphia.And sincetheCSECcenterisdowntown,you’realready If yourexamisoverbyearlyafternoon,youarelikelyto have someextratimetosee What toSee Streets andMarketChestnut Streets,IndependenceHallandtheLiberty Independence Hall/Liberty Bell: one, buttrythelobsterrisottoorchickentagine. experiences you’lleverhave.Therearetoomanygreatdishes tosingleoutjust Tangerine ($$$): restaurants intown,locatedtheheartofRittenhouse Square. Audrey Claire($$): with orwithoutfriedonionrings. “Philly” whileyou’reintown.Keystoaproperorder:CheeseWhizorprovolone inthecountry,best-known foods wasbornhere.Justremembertodropthe 0659/215-468-1547. ThePhillycheesesteak(or“hoagie”),perhapsoneofthe Geno’s Steaks/Pat’s KingofSteaks($): CSEC center. line, soyoucantaketheraillineto34thStreetstationandwalk is verywellrecommended.ItlocatedjustablockfromtheMarket-Frankford away); 877-275-1971.Thishosteloffersbothdorm-styleandprivaterooms Apple HostelsofPhiladelphia($): ever, ridetheMarket-FrankfordlinefromCityHallto34thStreetstation. 215-546-7000. Anacceptablebudgetoptionbutnotwalkable.You can,how- Rodeway InnPhiladelphia($$): km away);215-568-8300. *Best Western CenterCityHotel($$): delicious. away); 215-387-6065.ThisB&Bisawonderfulplacetostay. Thebreakfastsare *Cornerstone BedandBreakfast($$): close butexpensive. 222-0200. LocatedontheUniversityofPennsylvania’s campus,thishotelis Hilton InnatPenn($$$): priced hotelscloseby. blocks away);215-387-8000.Well recommendedandoneoftheonlymoderately *Sheraton PhiladelphiaUniversityCity($$): 232MarketStreet;215-627-5116.Thisisoneofthetastiest 276South20thStreet;215-731-1222.Oneofthebest 3600 SansomStreet(0.2mile/0.3kmaway);215- 1208Walnut Street(2.1miles/3.8kmaway); Locatedintheblockbetween 5thand6th 32SouthBankStreet(2.8miles/4.5km 3300BaringStreet(0.6mile/1km 501 North22ndStreet(1.7miles/2.7 1219 South9thStreet;215-389- 3549ChestnutStreet(two USMLE STEP 2 CS TRAVEL GUIDE 37 Close by and home of the Philadelphia Museum of Art, the Museum and home of the Philadelphia Close by www.orbitz.com www.hotwire.com www.hotels.com www.priceline.com www.expedia.com www.travelocity.com AAMC accommodations site: www.aamc.org/meetings/153904/clinicalskills_mtgs_homepage_teaser.html and hotel sites: Travel USMLE travel site: http://www.usmle.org/step-2-cs/#testcenters www.philly.com www.lonelyplanet.com/destinations/north_america/philadelphia Bell are two of the most iconic images in all of U.S. history. You may want to You U.S. history. images in all of of the most iconic Bell are two (www.nps.gov/inde). beforehand make a reservation District: Museum and Eastern Zoo, Fairmont Park, the Philadelphia of Science, Franklin Institute State Penitentiary. USEFUL WEB SITES USEFUL WEB For more information, check out: For more information, Here are a few other Web sites that you might find useful while you are planning your Here are a few other Web trip: NOTES USMLE STEP 2 CS TRAVEL GUIDE USMLE STEP 2 CS TRAVEL

38 SECTION 2

The Patient Encounter

Introduction Doorway Information Taking the History The Physical Exam Closure How to Interact with Special Patients Challenging Questions and Situations Counseling The Patient Note THE PATIENT ENCOUNTER 40 your timewisely. Themostcommonreasonsforrunningoutoftimeareasfollows: minutesshouldbeadequateforeachpatientencounteraslongyoubudget Fifteen the processstepbystep. this mayseemoverwhelming,butitneednotbe.Thischapterwillguideyouthrough and physicalexamtoformulateyourdifferentialdiagnosisworkupplan.All might have.Afterthat,youwillhave10minutestosummarizethepatienthistory exam, discussingyourfindingsandplans,answeringanyquestionsthepatient interviews includesmeetingthepatient,takinghistory, performingthephysical allottedforeachofyour five minutesremaininthesession.The15-minuteperiod Each encounterintheStep2CSlasts15minutes.You willbegivenawarningwhen gency departments. signed toreplicatesituationscommonlyseeninclinics,doctors’offices,andemer- cal encounterswithtrained“standardizedpatients”(SPs).Thesearede- describedinSection1,theStep2ClinicalSkills(CS)examconsistsof12clini- As exam, whileotherswillnecessitatethatmoretimebespent onclosureandpatient some encounterswillrequiremoretimefortakingthehistoryordoingphysical Of course,thisisonlyanapproximation.Inreality, eachencounterisdifferent,so judiciously. Arecommendedtimetableisasfollows: To bestmanageyourencounter, itisrecommendedthatyoudistributeyourtime some additionaltimemanagementtips: counseling. You shouldthustailoryourtimetothedemandsofeachcase.Hereare INTRODUCTION that arepertinenttothechief complaint. An organizedandwell-plannedhistoryiskey. Stayfocusedonaskingquestions do so. tor. Ifyouhave notbeguntoperformthephysicalexambythatpoint,youshould nouncement thatfiveminutesremainintheencounteras youronlytimeindica- Do notwastevaluabletimelookingattheclockonwall. Usetheofficialan- Closure: Physical exam: History: each room):10–20seconds information Doorway tients Failing toadaptorredirectchallenging(eg,unresponsive,angry, crying)pa- Allowing thepatienttostrayawayfromrelevanttopics Carrying outtheencounterinaslowordisorganizedfashion Conducting anunnecessarilydetailedphysicalexam Taking anoverlydetailedhistory 7–8minutes 2–3minutes 3–5minutes (assessingpreliminaryinformationpostedonthedoorof THE PATIENT ENCOUNTER 41 the patient note. can be used to write can be used to write Any time saved from Any time saved from the patient encounter the patient encounter History Closure Physical Exam Details of the chief complaint, Evaluation of the appropriate A more well-defined differential A final differential and an Forming a hypothesis (broad Forming Chief complaint, age, sex, and DoorwayInformation rule out each item in the differential. Leads to: diagnosis, which will help narrow down the procedures that should be performed and the systems that should be examined in the physical exam. Must get: associated symptoms, and any other relevant information that will help rule in or appropriate workup plan. Must get: systems to help rule in or rule out each item in the differential; any additional information on the patient’s history if required. Leads to: Explaining the findings, differential, and workup plan to the patient. Answering the patient’s questions and addressing his concerns. Must get: abnormal vital signs. Leads to: differential, relevant points that should be to examine). systems elicited in the history, Overview of the Clinical Encounter plan, and never try to save time by ignoring the patient’s questions, requests, or to save time by ignoring the patient’s plan, and never try emotional status. of and sense efficiency, way to improve your performance, Practice is the best timing. A brief and focused physical exam is also critical. There is no need to conduct a There is no need exam is also critical. focused physical A brief and points may be Remember that during encounters. physical exam comprehensive will be given no bonus points exam findings, but for omitting critical deducted low-yield maneuvers. for performing your ability to com- objectives of the Step CS is to evaluate One of the principal your management Make sure you leave time to discuss municate with patients. Figure 2-1 illustrates the key components and desired outcomes of the clinical en- the key components and desired outcomes Figure 2-1 illustrates each. sections will guide you through The following counter. FIGURE 2-1. THE PATIENT ENCOUNTER 42 contact withthepatient. room. Alwaysmakeeye Address thepatient when youenterthe by hisorhername do aphysicalexam.”Again,makeanefforttoestablisheye contactwiththepatient you.”). You mayalsoaddsomethinglike“Iwouldtoaskyousome questions and yourself inaconfidentyetfriendlymanner(eg,“Hi,IamDr. Morton. Nicetomeet After yourinitialentrance,youshouldshakehandswiththepatientandintroduce that theSPmighthavebeeninstructednottorespondhisorhername. your query, considerthepossibilitythattheremaybeachangeinmentalstatusand patient’s namefortheremainderofencounter. Ifthepatientdoesnotrespondto receive creditforhavingdonesoandwillnothavetoworryaboutrememberingthe patient ifheorsheisthepersonidentifiedondoor(eg,“Mr. Smith?”). You will and thenknockonthedoor. Onceyouhaveenteredtheexaminationroom,ask you entertheroom,besureto Your entranceintotheexaminationroomisacriticalpartofencounter. Sobefore time limitation. the timeyouspendreadingdoorwayinformationisincludedin15-minute so youcanreviewthatinformationattheendofeachencounter. Note, however, that copy ofthedoorwayinformationsheetwillbeavailableonothersidedoor, predictable andalreadyknowwhatyouareexpectedtodo.Rememberthatasecond bear inmindthatSPsareeasiertodealwiththanrealpatientstheymore counter isacommonscenariofoundinroutinemedicalpractice.You shouldalso Remain calmandconfidentbyremindingyourselfthatwhatyouareabouttoen- vital signsareaccurate. malities invitalsignswithouttryingtomemorizeactualnumbers.Assumethatthese the systemsyouwillhavetoexamine.Toward thisgoal,youshouldlookforabnor- complaint, andtryingtoorganizeinyourmindthequestionsyouwillneedask You shouldbeginbyreadingthedoorwayinformationcarefully, checkingthechief you willbecalledontoperform. sure, temperatureinbothCelsiusandFahrenheit,respiratoryrate);thetask age, andgender;thereasonforvisit;patient’s pres- vitalsigns(pulse,blood which ispostedonthedoorofexaminationroom,includespatient’s name, known as“doorwayinformation,”attheoutsetofeachencounter. Thisinformation, described,youwillbegivenachancetoreviewpreliminarypatientinformation, As the examinationroom.Itincludes thesimplebutessentialcomponentsdescribed Building atrustingrelationship withthepatientstartsfrommomentyouenter centers onthepatient’s concernsandneeds,notonthe diseaseorthedoctor. to establishatrustingdoctor-patient relationshipandtoensurethattheencounter cessfully completingtheencounterinStep2CS.The maingoalsofthePCIare Conducting apatient-centeredinterview(PCI)isanessential componentofsuc- The Patient-Centered Interview during thisinitialperiod. DOORWAY INFORMATION read andcommittomemorythepatient’s lastname, THE PATIENT ENCOUNTER 43 The interview is doctor centered. disease centered or disease centered patient centered, not patient centered, pol- A Once you The patient mpathy, mpathy, patient-centered E Building trust with Building trust with artnership, P ( connect with the patient. reflective listening. joint agenda with the patient. “PEARLS” upport). Look for opportunities to use PEARLS in interview. S doctor-centered means that you and the patient are working together to identify his egitimization, and is shown by acknowledging and showing understanding of the patient’s is shown by acknowledging and showing understanding of the patient’s refers to taking personal responsibility when it is appropriate to do so L means valuing the patient’s choices, behaviors, and decisions (eg, “You choices, behaviors, and decisions (eg, “You means valuing the patient’s espect, R Respect have obviously worked hard on this.”). Empathy feelings. For example, you might respond to a patient who expresses fear or anger look upset.” with “That sounds hard” or “You Apology (eg, “I’m sorry I was late” or “I’m sorry this happened to you”). Partnership or her main concerns and to come up with solutions. Phrases that help facilitate can do this.” deal with this together” and “We partnership include “Let’s on the nature of the case. A brief description of each PEARLS component is given on the nature of the case. A brief description of each PEARLS component below: every patient encounter. Of course, you will not need to use all six PEARLS elements every patient encounter. in each of your encounters; instead, you will likely use only one or two, depending previously: calling the patient by his or her name, introducing by his or her name, calling the patient previously: and shaking yourself, a respectful and courtesies; they set steps are not just that these hands. Remember tone to the entire encounter. attentive The step in conducting a PCI involves next an active part of the interview and that you are indeed conducting a an active part of the interview and that interview and not a The next step in the PCI involves setting a you need to prioritize them and establish concerns, have summarized the patient’s them. For example, you might say to the a joint agenda with the patient to address smoking. I am concerned are concerned about chest pain, cough, and patient, “You by addressing whichever of these things con- start about all these things as well. Let’s will make the patient feel that he or she is cerns you the most.” By doing this, you stated all of his or her concerns, summarize them using the patient’s own words as own words the patient’s her concerns, summarize them using stated all of his or that you are actively so builds trust by showing the patient much as possible. Doing particularly if appropriate to express empathy, listening. In some instances it is also like “This must be a difficult time for the patient is distraught, by saying something are going through.” you,” or “I can only imagine what you telling the patient what your role is and then asking about his or her concerns (eg, “I your role is and then asking about telling the patient what Once you have for your chest pain; what are your concerns?”). was asked to see you interrupting or interject- state his or her concerns, listen without asked the patient to these issues by using phrases Encourage the patient to express ing your own thoughts. patient has me more about that.” When the else?” or “Tell such as “Is there anything your patient requires that you be a good listener. Therefore, start the encounter by Therefore, start that you be a goodyour patient requires listener. Throughout the encounter, you should aim to Throughout the encounter, Be alert to these sadness, and anxiety. fear, is likely to express emotions such as anger, emotions, and be ready to respond with ogy, Once this is established, you can begin gathering information and developing a diag- Once this is established, you can begin section below). nosis (discussed in the history-taking THE PATIENT ENCOUNTER 44 Use simple,nontechnical speaking tothepatient. terminology when meanor. You shouldspeakclearlyandslowly, andyourquestions shouldbeshort,well Once theinterviewhasbegun,besuretomaintainaprofessional yetfriendlyde- Additional Tips 2-2 foranoverviewoftheprocess. SeeFigure or“HowcanIhelpyoutoday?” brought youtothehospital/clinictoday?” start yourinterviewbyaskingthepatientanopen-endedquestionsuchas“Sowhat As noted,theinterviewasawholeshouldtakenomorethan7−8minutes.You can proximately twofeetbetweenyourselfandthepatient. and keeptheclipboardonyourlap.Ifyoudecidetostand,maintainadistanceofap- with theclipboardinyourhands.Instead,itisbesttositdownonstool,relax, Don’t crossyourarmsinfrontofchestwhentalkingtothepatient,especially before takingthehistorytomakesureyougetcreditfordoingsoearlyon. placed onthisstool.Beginbyremovingthesheetanddrapingpatient.Do the stoolthatisprovided,whichusuallylocatednearbed.You willfindasheet You maytakethehistorywhilestandinginfrontofpatientorsittingon Guidelines you throughthisprocessinamannerthatwillmaximizeyourchancesofsuccess. differential diagnosisandworkupplan.Thediscussionthatfollowswillhelpguide Your abilitytotakeadetailedyetfocusedhistoryisessential totheformulationofa should alwaysremembertoask questionsinaneutralandnonjudgmentalway. Instead, givehimorherample timetorespond.Ininteractingwiththepatient,you you pleaserepeatwhat just said?”).Atthesametime,donotrushpatient. explain orrepeatit(eg,“Can youpleaseexplainwhatmeanbythat?”or“Can If youdon’t understandsomethingthepatienthas said, youmayaskhimorherto ask youforthemeaningofaterm,ormaylosecredit. patient maynotunderstand,offeraquickexplanation.Don’t waitforthepatientto use simple wordsthatalaypersoncanunderstand(eg,don’t usetheterm phrased, andsimple.Toward thatend,avoidtheuseofmedicalterms;instead, and developingthedifferentialdiagnosis. relationship. Fromthere,youcanmoveontomakingappropriatemedicaldecisions ing theseprinciplesintheCSexamwillhelpyouestablishatrustingdoctor-patient Again, thePCIispatientcentered,notdiseasecenteredordoctorcentered.Follow- TAKING THEHISTORY kidney stone like “Manyofuswouldbeconfusedorupsetbythissituation.” ings andchoices.Anexampleofalegitimizingstatementwouldbesomething Legitimization saying somethingassimple“I’llbeherewhenyouneedme.” Support shouldbecontinuallyofferedtothepatient.You canoffer supportby instead).Ifyoufindyourselfobligedtouseamedicalterm thatthe validates thepatientandshowsunderstandingofhisorherfeel- renal calculus; THE PATIENT ENCOUNTER 45 or questions. concerns nset ssociated symptoms elieving factors uration recipitating factors revious episodes rogression requency How to Get It How Introductions P A P P F O R D and address any patient concerns. eye contact. What HistoryWhat to Get the history. on the door. the patient and cover the legs. the patient and cover Before the Physical Exam interrupt or rush the patient. technical medical terms. Then focus on key organ systems and: Thenkey organ systems on focus Start with an open-ended question. open-ended Start with an Knock the patient’s name. Verify shake hands. Introduce yourself and Make eye contact. Drape empathy Summarize Ask if there is anything that was not covered. Ask if patient has any Avoid Show Maintain good Do not History-Taking Overview History-Taking but you may use it more often if you consider it necessary. It is recommended, how- but you may use it more often if you consider it necessary. that you give a summary (1) after you have finished taking the history and be- ever, the fore you start examining the patient, or (2) just after you have finished examining to the next step (eg, “So as I understand it, your abdominal pains are infrequent, last to the next step (eg, “So as I understand it, your abdominal pains are infrequent, Now tell me about . . .”). a short time, and are always in the middle of your belly. that you It is critical to summarize what the patient has told you, not only to verify need to use this have understood also to ensure that you receive credit. You him but credit, summary technique no more than once during the encounter in order to get sary. If the patient starts telling lengthy stories that are irrelevant to the chief com- If the patient sary. me, Mr. plaint, you can interrupt politely but firmly by saying something like “Excuse ask you Johnson. I understand how important those issues are for you, but I’d like to can also redirect the some additional questions about your current problem.” You move conversation by summarizing what the patient has told you thus far and then You should also remember not to interrupt the patient unless it is absolutely neces- should also remember You FIGURE 2-2. THE PATIENT ENCOUNTER 46 facts forthepatient will earnyoucredit. Look fornonverbal Summarizing key clues. should alsobesuretoaskonlyonequestionatatime.If youaskcomplexquestions choosing thequestionsyouaskinyoureffortstoobtainaconcise, relevanthistory. You list, nordoyouhavetouseallthequestionsoutlinedbelow. Instead,beselectivein in thecourseofeachyourpatientinterviews.Thisisnotintendedtobeacomplete In thissection,wewillcoverawidespectrumofquestionsthatyoumayneedtopose Common QuestionstoAskthePatient anything elseyouforgottotellmeabout?” something like“Isthereanythingelseyouwouldtotellmeabout?”or you finishyourinterviewandmovetothephysicalexam,mayaskpatient taking insidetheencounterwilldependonhowmuchyoutrustyourmemory. Before clipboard with12blankbluesheets,oneforeachencounter. Theextent ofyournote number ofyearsthepatientsmoked.To facilitatethisnotetaking,youwillbegivena to-forget piecesofinformationsuchasthedurationchiefcomplaintor Finally, takebriefnotesthroughouttheinterview, mainlytorecordrelevantyeteasy- onthetissuemaytakeyoubysurprise! check thecolorofsputum.Aspotblood as thereasonforvisit.Ifpatientisusingatissue,asktoseeitsothatyoucan ing youworry?”).Ifyourpatientiscoughing,askaboutthecoughevenifitisn’t cited sad; doyouknowthereason?”or“You lookconcerned;isthereanythingthatmak- By thesamelogic,youshouldaddresseverysignseeinpatient(eg,“You look she isintendingtodepictbronchitis,notbecauseofaninvoluntaryreflex. to hiccupinadvertently),anSPwillmostlikelycoughinencounterbecauseheor occur fornoreason.Althoughaccidentsdohappen(forexample,anSPoncestarted Remember thatclinicalencountersarestaged,soitisuncommonforsomethingto watch thepatientcarefully, payingattentiontohisorher every word,move,orsign. To ensurethatyoustayontrackingatheringinformation,willalsoneedto lifestyle andpersonalhabits.” tions aboutyourhealthinthepast,”or“I’dliketoaskyouafewquestions or socialandsexualhistory, youcansaysomethinglike“I needtoaskyousomeques- to movefromthehistoryofpresentillness(HPI)patient’s pastmedicalhistory transitionsmayalsobeusedduringthehistory. Forexample,whenyouwant Minor should includeonlythepointsthatarerelevanttopatient’s chiefcomplaint. patient andbeforeyougivehimyourmedicalopinion.Ineithercase,summary Opening oftheencounter: tion youposed.Instead,shouldslowdownandaskabout onesymptomatatime. (eg, “Isthereanyrednessorswelling?”),theSPwilllikely answeronlythelastques- “What areyourconcerns?” “What madeyoucomeintoday?” “What broughtyoutothehospital/clinictoday?” “How canIhelpyoutoday?” questions andexamineyoutoday.” “Mr. Jones,hello;IamDr. Singh.It’s nicetomeet you.I’dliketoaskyousome THE PATIENT ENCOUNTER 47 “What color is it?” “Is there any bloodit?” in A “Can you estimate the amount of the phlegm? A teaspoon? A tablespoon? cupful?” “Does anything make it better?” “Does anything make it worse?” “When did it start?” “How often do you cough?” “During what time of day does your cough occur?” “Do you bring up any phlegm with your cough, or is it dry?” “Does anything come up when you cough?” “Did you throw up?” “What color was the vomit?” “Did you see any blood in it?” “Do you have a cough?” “Do you feel nauseated?” “Do you feel sick to your stomach?” “Did you vomit?” “What brings the pain on?” start?” “Do you know what causes the pain to “Does anything make the pain better?” “Does anything make it worse?” “Have you had similar pain before?” “What is the character of the pain? For example, is it sharp, burning, cramping, of the pain? For example, is it sharp, “What is the character or pressure-like?” “Is it constant, or does it come and go?” the worst pain you have ever felt, how “On a scale of 1 to 10, with 10 being would you rate your pain?” “Where do you feel the pain?” “Where do you feel exactly where it is?” “Can you show me anywhere?” “Does the pain travel “What is the pain like?” it for me?” “Can you describe “Do you have pain?” “Do you have it start?” “When did pain?” have you had this “How long “How long does it last?” come on?” “How often does it Cough: Nausea: Vomiting: Pain: THE PATIENT ENCOUNTER 48 Headache: Shortness ofbreath: Fever: Urinary symptoms: “Tell meaboutyourheadaches.” “Do yougetheadaches?” “Do youhavetowakeupat night tourinate?” “How oftendoyouhavetourinate?” “Have younoticedanychange inthecolorofyoururine?” “Do youhaveanypainorburningduringurination?” “Has therebeenanychangeinyoururinaryhabits?” “Have younoticedanyswellingofyourlegsorankles?” “How fardoyouwalkonlevelgroundbeforehaveshortness ofbreath?” “Have youbeenwheezing?” do youuse?” “Do youhavetopropyourselfuponpillowssleepatnight? Howmanypillows “Do youwakeupatnightshortofbreath?” “What makesitbetter?” “What makesitworse?” “When doyoufeelshortofbreath?” “When diditfirststart?” “How manystepscanyouclimbbeforegetshortofbreath?” “Do yougetshortofbreathwhenyou’reclimbingstairs?” “Do yougetshortofbreath?” “How highisyourfever?” “Do youhavenightsweats?” “Do youhavechills?” “Do youhaveafever?” “Do younoticeanyfeverorstiffneckwithyourheadaches?” “Do youfeelnauseated?Dovomit?” “Do younoticeanynumbnessorweaknessbefore/during/aftertheheadaches?” “Do younoticeanychangeinyourvisionbefore/during/aftertheheadaches?” pulsating, pounding,orpressure-like?” “Can youdescribetheheadacheforme,please?Forexample,isitsharp,dull, “What makesitbetter?” “What makestheheadacheworse?” “Do yourheadacheswakeyouupatnight?” “Do youhaveheadachesatcertaintimesoftheday?” “What causestheheadachetostart?” “Can youshowmeexactlywherefeeltheheadache?” “When yourheadachestarts,howlongdoesitlast?” “How oftendoyougetthem?” “When doyourheadachesstart?” “Tell mewhathappensbefore/during/afteryourheadaches.” THE PATIENT ENCOUNTER 49 “Do you have any problems falling asleep?” “Do you have any problems staying asleep?” “What do you usually eat?” “Did you eat anything unusual lately?” “Are there any specific foods that cause these symptoms?” “Is there any kind of special diet that you are following?” “How is your appetite?” “Has there been any change in your appetite?” “Are you getting full too quickly during a meal?” “Has there been any change in your eating habits?” “Have you noticed any change in your weight?” “Have you noticed any change in your “How many pounds did you gain/lose?” “Over what period of time did it happen?” the weight gain/loss intentional?” “Was of feces comes out?” “Have you lost control of your bowels?” time to make it to the bathroom once “Do you feel as though you have very little you have the urge to have a bowel movement?” “What color is your stool?” “Is there any mucus or blood in it?” a bowel movement?” “Do you feel any pain when you have “Did you travel recently?” to the bathroom or a very small amount “Do you feel as though you strain to go “Has there been any change in your bowel movements?” “Has there been any “Do you have diarrhea?” “Are you constipated?” “How long have you had diarrhea/constipation?” have per day/week?” “How many bowel movements do you “What does your stool look like?” “Do you feel as though you have to urinate all the time?” “Do you feel as though to the bathroom once you have very little time to make it “Do you feel as though urinate?” you feel the urge to “Do you have any difficulty urinating?” “Do you have after urination?” emptied your bladder completely that you haven’t “Do you feel during urination?” to strain/push “Do you need in your stream?” noticed any weakness “Have you any blood“Have you noticed in your urine?” little urine comes out?” you need to urinate but then very “Do you feel as though Sleep: Diet: Appetite: Weight: Bowel symptoms: THE PATIENT ENCOUNTER 50 Past medicalhistory: Travel history: Joint pain: Dizziness: Family history: “Do yousnore?” “Do youfeelrefreshedwhenwakeup?” “Do youhaveanyproblemswakingup?” “Are yourbrothersorsisters alive?” “What didyourmother/father dieof?” health?” “Are theyingood “Are yourparentsalive?” “Does anyoneinyourfamilyhaveasimilarproblem?” “Do youhaveanyallergies?” “Are youtakinganyover-the-counter drugs, vitamins,orherbs?” “Are youtakinganymedications?” “Have youeverhadanyaccidentsorinjuries?” “Have youhadanysurgeriesbefore?” transfusion?” “Have youeverhadablood “Have youeverbeenhospitalized?” “Do youhaveanyothermedicalproblems?” “Have youhadanyothermajorillnessesbefore?” “Have youhadthisproblemoranythingsimilarbefore?” “Did anyoneelseonyourtripbecomesick?” “Have youtraveledrecently?” “Are youhavingdifficultymovingthejoint?” “Is thereanyrednessorswellingofthejoint?” “Have younoticedanyrashwithyourjointpain?” “Do youhavepaininanyofyourjoints?” “What makesyoufeelbetter?” “What causesthisdizzinesstohappen?” “Do youfeelnauseated?Dovomit?” “Do yourearsring?” “Did younoticeanychangeinyourhearing?” “Did youblackoutorloseconsciousness?” were goingtopassout?” “Did youfeeltheroomspinningaroundyou,ordidlightheadedasif “Tell meexactlywhatyoumeanbydizziness.” “Do youeverfeeldizzy?” “Do youtakeanypillstohelpgosleep?” “How manyhoursdoyousleep?” “Do youfeelsleepyduringtheday?” THE PATIENT ENCOUNTER 51 to steady (‘eye opener’) on drinking?” cut down by criticism of your drinking?” feelings about drinking?” guilty annoyed “Have you ever had a drink first thing in the morning “Have you ever had a drink first thing in “Have you ever felt a need to “Have you ever felt “Have you ever had your nerves or get rid of a hangover?” “Have you ever had a sexually transmitted disease?” “Do you have any problems with sexual function?” “Do you have any problems with erections?” “Do you use any contraception?” “Have you ever been tested for HIV?” “Are you sexually active?” “Do you use condoms? Always? Other contraceptives?” men, women, or both?” “Are you sexually active? With me about your sexual partner or partners.” “Tell “How many sexual partners have you had in the past year?” “Do you currently have one partner or more than one?” “I would like to ask you some questions about your sexual health and practice.” “Have you ever had a drinking problem?” “When was your last drink?” Administer the CAGE questionnaire: “Do you have a lot of stressful situations on your job?” “Do you have a lot of stressful situations on your job?” “Are you exposed to environmental hazards “How much alcohol do you drink?” your use of alcohol.” me about “Tell “What type of work do you do?” “What type of work whom?” “Where do you live? With your life at home.” me about “Tell “Are you married?” “Do you have children?” “How much do you drink per week?” “How much do you cocaine?” drugs such as marijuana or “Do you use any recreational use?” “Which ones do you use them?” “How often do you them?” “Do you smoke or inject “Do you smoke?” packs a day?” “How many have you smoked?” “How long “Do you drink alcohol?” “What do you drink?” Alcohol history: Social history: Sexual history: THE PATIENT ENCOUNTER 52 Gynecologic/obstetric history: Growth anddevelopment: Pediatric history: Routine pediatriccare: Feeding history: “How oftendoyougetyourmenstrualperiod?” “At whatagedidyouhaveyourfirstmenstrualperiod?” “When wasthedateofyour child’s last routinecheckup?” “Are yourchild’s immunizationsupto date?” “Are yougivingyourchildpediatric multivitamins?” “Is yourchild’s formulafortifiedwithiron?” “Does yourchildhaveanyallergies?” “How isyourchild’s appetite?” “When didyourchildstarteatingsolidfood?” “Did youbreast-feedyourchild?” “When didyourchildstartusingshortsentences?” “When didyourchildlearntodresshimself?” “When didyourchildstartwalking?” “When didyourchildstarttalking?” “When didyourchildstartcrawling?” “When didyourchildfirstsitup?” “When didyourchildfirstsmile?” “When didyourchildhavehisfirstbowelmovement?” “Did yourchildhaveanymedicalproblemsafterbirth?” “Was itavaginaldeliveryorC-section?” “Did yousmoke,drink,orusedrugsduringyourpregnancy?” “Was anultrasoundperformedduringyourpregnancy?” after delivery?” “Did youhaveanycomplicationsduringyourpregnancy/duringdelivery/ “Did youhaveroutinecheckupsduringyourpregnancy?Howoften?” “Was yourpregnancyfullterm(40weeksor9months)?” “Have youhadaPapsmearbefore?” “Do youhaveanyproblemscontrollingyourbladder?” “Do youhaveanyvaginaldischarge?” “Do youhavepainduringintercourse?” “Have youeverhadamiscarriageoranabortion?” “How manychildrendoyouhave?” “How manytimes?” “Have youeverbeenpregnant?” “Have younoticedanyspottingbetweenperiods?” “How manypadsortamponsdoyouuseperday?” “Do youhavecramps?” “Have younoticedanychangeinyourperiods?” “When wasthefirstdayofyourlastmenstrualperiod?” “How longdoesitlast?” THE PATIENT ENCOUNTER 53 “Tell me about your day yesterday.” “Tell “Do you need any help bathing/getting dressed/feeding yourself?” “Whom do you live with?” “How do they react to your behavior?” “Do you have any problems in your job?” “How is your performance on your job?” “Have you had any recent emotional or financial problems?” “Have you had any recent traumatic event in your family?” “Have you lost any interest in your social activities or relationships?” “Do you feel hopeless?” “Do you feel guilty about anything?” “How is your energy level?” “Can you still perform your daily functions or activities?” “Has anyone in your family ever experienced depression?” “Has anyone in your family ever experienced diagnosed with a mental illness?” “Has anyone in your family ever been with a counselor to help you with your problem?” you like to meet “Would you like to join a support group?” “Would “What do you think makes you feel this way?” “Do you have a plan to end your life?” me about it?” you mind telling “Would see or hear?” can’t “Do you ever see or hear things that others the world that other people would find “Do you hold beliefs about yourself or odd?” to harm or control you?” “Do you feel as if other people are trying “Do you have any memory problems?” “Do you have difficulty concentrating?” “Do you have hope for the future?” yourself or others?” “Have you ever thought about hurting your own life?” “Do you think of killing yourself or ending “Has your weight changed recently?” “Has your weight changed me how you spend your time/day.” “Tell asleep/waking up?” “Do you have any problems falling asleep/staying habits lately?” “Has there been any change in your sleeping “Do you enjoy any hobbies?” daily activities?” “Do you take interest or pleasure in your “How long have you been feeling unhappy/sad/anxious/confused?” “How long have you what might be causing this?” “Do you have any idea you feel this way?” you like to share with me what made “Would to for support?” or family members you can talk “Do you have any friends changed lately?” “Has your appetite “Has your child had any serious illnesses?” child had any serious “Has your taking any medications?” “Is your child child ever been hospitalized?” “Has your future goals.” me about yourself and your “Tell Daily activities (for dementia patients): Psychiatric history: THE PATIENT ENCOUNTER 54 touching oruncovering Ask permissionbefore the patient.Drape patient appropriately. The keyisafocused physical exam. use thetimetothinkaboutwhatyoushouldexamineand whetherthereisanything (You canwearglovesinsteadifyousochoose.)Whilearewashingyourhands, Then, don’t forgettowashyourhandswithsoapandwaterdrythemcarefully. Before youbegin,shouldinformthepatientofneedforphysicalexam. taking thehistory. SeeFigure2-3foranoverviewoftheprocess. look forphysicalfindingsthatcansupportthedifferentialdiagnosisyoumadeafter a completephysicalexam.Instead,youshouldaimatconductingfocusedexamto have fiveminutesremainingintheencounter. Bearinmindthatthereisnotimefor have startedthephysicalexambytimeyouhearannouncementthat history portionoftheencounterisestimatedtotake7−8minutes,youshouldalready described earlier, thephysicalexamcantakeuptofiveminutes.Giventhat As can performthephysicalexamwithoutmistakesorhesitation. youchoose,however,gardless ofthemethod itisessentialthatyoupracticeuntil You oranyothersystemwithwhich youfeelcomfortable.Re- canusethismethod thissection,wewillrecommendasystematicwaytoperformthephysicalexam. In Guidelines themselves. You shouldexpose onlytheareayouneedtoexamine.Donotexpose move thesheetdowntoexamine yourbelly?”).You mayalsoaskpatientstouncover (eg,“Isit okay ifIuntieyourgowntoexaminechest?”or“Can I his orherbody youproceed,besuretoaskthe patient’s permissionbeforeyouuncoveranypartof As warm itupbeforeyouuseit.Donotauscultateorpalpate throughthepatient’s gown. gether iftheyarecold).Inasimilarmanner, rubthediaphragmofyourstethoscopeto Before youtouchthepatient,makesureyourhandsare warm(rubyourhandsto- the waistdown. already doneso.Thedrapewillbeonthestool;unfoldit andcoverthepatientfrom you neglectedtoaskthepatient.You shouldthendrapethepatientif you havenot Abuse: THE PHYSICALEXAM “Do youeverhaveaccidentswithyoururineorbowelmovements?” “Do youneedanyhelptransferringfromyourbedtothechair?” “Do youneedanyhelpgoingtothetoilet?” “Can youtellmeaboutthebruisesonyourarm?” hurts youorthreatenstohurtyou?” “Does anyone(yourhusband/wife/parents/boyfriend)treatyouinawaythat “Is thereanythreattoyourpersonalsafetyathomeoranywhereelse?” “Are yousafeathome?” managing money?” yourhouse/doinglaundry/gettingfromplacetoplace/ preparing food/cleaning “Do youneedanyhelptakingyourmedications/usingthetelephone/shopping/ “What doyouneedhelpwithwheneat?” “Do youevernotmakeittothetoiletontime?” THE PATIENT ENCOUNTER 55 to untie the gown. After the Exam During the Exam Before the Physical Exam Before the your hands. repeat painful maneuvers. Tie the gown when you are done. Tie permission examine through the gown. examine through the far from any area of pain. far from any area of Wash patient what you are going to do. the Tell Ask Keep the exam focused and organized. focused the exam Keep Expose as little of the patient’s body as you can. Expose as little of the Do not Start Do not Physical Exam Overview Physical Exam not need to take a history. Should this occur, simply introduce yourself, proceed to Should this occur, not need to take a history. such examine the systems listed, and then leave the room. No PN is required under gave you circumstances; instead, you are required only to fill out the form the patient patient has answered your questions, you may resume the exam. remain alert to special situations that may not unfold as they you should Finally, for would in an ordinary physical exam. When you enter the examination room, certain example, the patient may hand you an insurance form requesting that only you do systems be examined. In such cases, the patient will usually tell you that for permission to proceed. The patient should always be made to feel that he or she for permission to proceed. The patient should always be made to feel that he is in control of his or her body. In the course of the physical exam, you may ask the patient any additional questions you that It is recommended, however, that you feel may be pertinent to the history. the pause the physical exam while communicating to reestablish eye contact. After not allowed to perform a corneal reflex, breast, rectal, pelvic, or genital exam. If you not allowed to perform a corneal reflex, are indicated, you should tell the patient think any of the above-mentioned exams later and then remember to add the exam that you will need to do the specific exam proce- to your orders on your patient note (PN). When you have concluded a given patient dure, remember to say “thank you.” Then explain the next step, and ask the it immediately. During physical exam, you will be scored both for performing a given procedure the an extra maneuver will not get credit for conducting You and for doing so correctly. but failure to perform a required procedure or for examining a nonrequired system, bear in mind that you are should also You will cost you a check mark on your list. large areas of the patient’s body you have examined a given area, cover at once. After large areas of the patient’s FIGURE 2-3. THE PATIENT ENCOUNTER 56 Not everypatientwill and aphysicalexam. require aninterview HEENTexam: 1. as toaskforconsentbeforeeachstep. Remember thatitiscrucialtokeepthepatientinformedofwhatgoingonaswell Included belowaresamplesofstatementsthatcanbeusedduringthephysicalexam. should remainwithyoudeepintoyourcareerasaphysician. be rewardedwiththediscoveryofuniquephysicalfindings.Thisimportantlesson lowing the“Look,Touch, Listen”approach, youwillappearthoughtfulandoften neck, theappearanceoflaboredbreathing,andmovementprecordium.Byfol- system shouldbeginwithinspectionforskincolor, cigarettestains, pulsations inthe ably itsmostimportantcomponent.Forexample,examinationofthecardiovascular targeted physicalexam,butthisdoesnotmeanthatyoushouldomitwhatisargu- can bestfocusonthetask.Thetimeconstraintsofpatientencountersnecessitatea ing ahistory, butitisimportanttodevoteafewsecondsformallyinspectwhenyou Part ofthegeneralinspectioncanbedonewhenyouaregreetingpatientortak- be overlookedbecausetheexaminerisfocusingontreeratherthanforest. general inspection.”Oftentimesabruise,surgicalscar, abandage,orasymmetrymay ing thepatientwhatyouaredoing—eg,“Ifdon’t mind,Iwouldliketoperforma simple butcrucialtaskbecausetheyfeelrushed.You should begintheprocessbytell- during thepatientencounter. Manystudents,examinees,andresidentsneglectthis Much canbelearnedfromtakingthetimetostepbackandperformabriefinspection conducting ageneralinspectionofthepatientaspartphysicalexam. body’s mainsystems.First,however, aspecialnoteisinorder abouttheimportanceof followingisareview ofthestepsinvolvedinexaminationeach The Physical ExamReview priate interactionwiththepatient. correct performanceofthephysicalexammaneuversandonprofessionalappro- with theappropriatefindings.Insuchencounters,emphasiswillbeplacedon toperformduringtheHEENTexam: What exam: What tosaythepatientbeforeandduring .Checkthepupilsforsymmetryandreactivitytolight. 2. Inspectthescleraeandconjunctivaeforcolorirritation. 1. Palpatetheheadfortendernessorabnormalities. 2. Inspecttheheadforsigns oftraumaandscars. 1. Eyes: Head: and yourthroat.” “Can youpleaseopenyourmouth?Ineedtochecktheinside ofyourmouth “I needtoexamineyourearsnow.” on thewall?” “I amgoingtoshinethislightinyoureyes.Canyouplease lookattheclock “I wouldliketoexamineyoureyesnow.” cheeks. Pleasetellmeifyoufeelpainanywhere.” “I needtoexamineyoursinuses,soIamgoingpressonforeheadand THE PATIENT ENCOUNTER 57 masses. toid. a new speculum for each patient.) (Don’t forget to use thalmoscope in examiner’s right hand—patient’s right eye—examiner’s right eye—examiner’s right hand—patient’s in examiner’s thalmoscope examiner’s left rule “left-left-left” (ophthalmoscope in right eye) and the eye—examiner’s left eye). hand—patient’s left Advanced techniques such as pulsus paradoxus or the Valsalva maneuver are Advanced techniques such as pulsus paradoxus or the Valsalva time-consuming and unlikely to provide essential information. Palpate the chest for the PMI, retrosternal heave, and thrills. Listen to at least two of the four cardiac areas. (Listen to the mitral area with the patient on his left side.) Listen to the base of the heart with the patient leaning forward. Check for pedal edema. Check the peripheral pulses. gown down the shoulder, exposing only the area to be examined. gown down the shoulder, Listen to the carotids for bruits. (Classically the bell of the stethoscope is used diaphragm is also acceptable but the turbulent blood flow, to listen for slow, in this scenario.) Look for JVD. Remember to raise the head of the bed to 45 degrees. “I am going to examine your legs to check for fluid retention. Is that okay “I am going to examine your legs to with you?” and legs now.” “I need to check the pulse in your arms pull the gown. Rather, When examining the heart, do not lift up the patient’s “I need to listen to your heart.” “Can you hold your breath, please?” “Can you sit, please?” “Can you turn to your left side, please?” Mouth and throat: Nose: Ears: 3. the nasal turbinates and the nasal septum with a light source. Inspect 1. with a light. Inspect 2. for mucosal ulcers, and inspect the uvula and under the tongue for Look 4. the Rinne and Weber tests. Conduct 1. the nose. Inspect 2. the nose and sinuses. Palpate 1. or masses. ear inspection for discharge, skin changes, Conduct an external 2. for the mas- for pain (otitis externa); do the same Palpate the external ear 3. otoscope. and the tympanic membrane using an Examine the ear canal 3. the eyes. movements of Check the extraocular 4. eye chart. with the Snellen Check visual acuity 5. (oph- the rule “right-right-right” exam. Remember Perform a funduscopic What to perform during the cardiovascular exam: What to say to the patient before and during theWhat to say to the patient before and exam: 2. Cardiovascular exam: THE PATIENT ENCOUNTER 58 .Neurologicexam: 5. Abdominalexam: 4. Pulmonaryexam: 3. What tosaythepatientbeforeandduringexam: exam questions: What tosaythepatientbeforeandduringexam—mini-mental status What toperformduringtheabdominalexam: What tosaythepatientbeforeandduringexam: What toperformduringthepulmonaryexam: ately, okay?Chair, bed,andpen.”(Tests immediatememory.) willnamethreeobjectsforyou, andIwantyoutorepeatthemimmedi- “I “Now Iwouldliketoaskyou somequestionstocheckyourmemory.” Show thepatientyourpenandask,“Doyouknowwhat this is?” “Do youknowthedatetoday?” “Do youknowwherearenow?” “Can youtellmeyournameandage?” questions.” “I wouldliketocheckyourmemoryandconcentration by askingyousome “I wouldliketoaskyousomequestionstestyourorientation.” Check theliverspan. sign. tenderness, CVA tenderness,theobturatorsign,psoasandMurphy’s the painfularea,anddon’t trytoreelicitthepain.Checkforrebound Palpate: Startfromthepointthatisfarthestpain;begentleon Percuss. Auscultate (alwaysauscultatebeforeyoupalpatetheabdomen). Inspect. “Do youfeelanypainwhenIpressinorletgo?Whichhurtsmore?” “Now Ineedtotaponyourbelly.” “I amgoingtopressonyourbelly. Tell meifyoufeelanypainordiscomfort.” “I amgoingtolistenyourbellynow.” “I needtoexamineyourbelly/stomachnow.” Allow afullinspirationandexpirationineachareaofthechest. Don’t percussorauscultateoverthescapula. Don’t percussorauscultatethroughthepatient’s gown. Examine boththefrontandbackofchest. Auscultate foregophony, wheezes,andcrackles. Percuss. Palpate: Lookfortendernessandtactilefremitus. Inspect: Examinetheshapeofchest,respiratorypattern,anddeformities. “I amgoingtotaponyourbackchecklungs.Isthatokaywithyou?” “Can yousay‘99’forme,please?” “Can youtakeadeepbreathforme,please?” “I needtolistenyourlungsnow.” THE PATIENT ENCOUNTER 59 forward and backward.” (Tests concentration.) forward and backward.” (Tests world wrists—flexion (“push down”), extension (“pull up”). Romberg’s sign, gait. Romberg’s sign. sign, Brudzinski’s Meningeal signs: Neck stiffness, Kernig’s Reflexes: Biceps, triceps, brachioradialis, patellar, Achilles, Babinski. Reflexes: Biceps, triceps, brachioradialis, patellar, Sensory system: Sharp (pin)/dull (cotton swab), vibration, position sense. Cerebellum: heel-to-shin, rapid alternating movements, Finger-to-nose, Motor system: “Can you walk across the room for me, please?” “Can you walk across the room for me, concentration. memory, Mental status examination: Orientation, Cranial nerves: “I am going to check your reflexes now.” muscles now.” “I am going to test the strength of your me which direction I am moving your big “This is up and this is down. Tell toe.” away from 100 and tell me what number you get; then keep taking 7 away tell me what number you get; then away from 100 and concentration.) until I tell you to stop.” (Tests “Spell coming out of a paper basket?” (Tests “What would you do if you saw a fire judgment.) “Are you left-handed or right-handed?” “Are you left-handed paper in your right I want you to take the of paper. “I will give you a piece command.) in half, and put it on the table.” (Three-step hand, fold the paper write your name on the paper.” “Now I want you to 7 100,” or “Take backward starting with the number “I want you to count “I will ask you to repeat the names of these three objects after a few minutes.” objects after a few of these three you to repeat the names “I will ask short-term memory.) (Tests memory.) recent (Tests had for lunch yesterday?” what you “Do you remember distant memory.) (Tests you get married?” “When did that I mentioned for me the names of the three objects “Now can you repeat short-term memory.) to you?” (Tests 4. Legs: Knee extension (“kick out”), knee flexion (“pull in”). 5. Ankles: “Push on the gas pedal.” 7. XII: “Stick out your tongue.” 1. Passive motion. 2. Active motion: Arms—flexion (“pull in”), extension (“push out”); 3. Hands: “Spread your fingers apart; close your fist.” 2.IV, VI: Extraocular movements. III, 3. sensation, muscles of mastication. V: Facial 4. “Smile, lift your brows, close your eyes and don’t let me open them.” VII: 5. IX, X: Symmetrical palate movement, gag reflex. 6. XI: “Shrug your shoulders.” 1. II: Vision. exam: What to perform during the neurologic What to say to the patient before and during the exam—neurologic exam What to say to the patient before questions: THE PATIENT ENCOUNTER 60 Jointexam: 6. What tosaythepatientbeforeandduringexam: Useful scales: What toperformduringthejointexam: 4: Poundingpulse 3: Increasedpulse 2: Regularpulse 1: Weakpulse 5: Hasfullstrength 4: Canliftlimbwithtwo-fingerresistancefromtheexaminer 3: Canliftlimbwithone-fingerresistancefromtheexaminer 2: Canliftlimbagainstgravity 1: Canmovelimb(wiggletoes) 4: Hyperreflexiaplusclonus(testtheankleandknee) 3: Hyperreflexia 2: Normalreflexes 1: Hyporeflexia Pulses (0–4),with0representingpulselessness: Strength (0–5),with0representinganinabilitytomovethelimb: Reflexes (0–4),with0beingcompletelyareflexic: For thelowerback:Conductalegraisetest. For thehip:Checkabduction,adduction,flexion,andextension. For theelbow:Checkforlateralandmedialepicondylitis. puytren’s contracture,andHeberden’s nodes. For thewrist:CheckforTinel’s sign,Phalen’s sign,signsorsymptomsofDu- tears, andO’Brien’s test. ternal rotation,Neer’s test,Hawkins’thedroparmtestforsupraspinatus For theshoulder:Checkadductionandinternalrotation,abductionex- lateral collateralligaments. drawer test,andMcMurray’s test,andcheckthestabilityofmedial For theknee:ConductaLachmantest,ananteriordrawerposterior Check gait. Check forinstability. Check forwarmth,swelling,andredness. tive) andbyhavingtheexaminermoveit(passive). Check jointrangeofmotionbothbyhavingthepatientmove(ac- Check forcrepitus. Check forjointeffusion. Palpate andcheckforjointtenderness. Inspect andcomparethejointwithoppositeside. “I amgoingtoexamineyourknee/anklenow.” “Tell meifyoufeelpainanywhere.” THE PATIENT ENCOUNTER 61 A patient in severe pain may ini- A patient in severe You can place the stethoscope any- can place You If mini-mental status exam, have time for a full you don’t Inquire about any bruises you see on the patient’s body, and body, patient’s Inquire about any bruises you see on the If you see a scar, a mole (nevus), a psoriatic lesion, or any a mole (nevus), a If you see a scar, Decreased respiratory sounds: The patient will move his chest without really Decreased respiratory sounds: The patient will move his chest without really inhaling any air so that you do not hear any respiratory sounds. CVA tenderness. CVA Shortness of breath. from Wheezing: This may often sound strange, as if the patient were whistling his mouth. exert. So don’t try to palpate the same area again; instead, move on, and consider try to palpate the same area again; instead, move on, and consider exert. So don’t the pain on palpation a positive sign. you Abdominal rigidity: The patient will contract his abdominal muscles when try to palpate the abdomen. Rebound tenderness of the abdomen. Abdominal tenderness: The patient feels pain when you press on his abdomen. Abdominal tenderness: The patient feels When you palpate the area, he will feel Remember that the patient is an actor. you pain where he is supposed to feel pain regardless of the amount of pressure other skin lesion during the exam, you should mention it and ask the patient other skin lesion during the exam, you complaint. patient’s about it even if it is not related to the Examining bruising: think about abuse as a possible cause. Running out of time: name, where they are, and what day it is. at least ask patients if they know their you are in severe pain, and I want to help you. The physical exam that I want to and I want to help you. The physical you are in severe pain, your pain. I will be as in helping determine what is causing do is very important I find the reason for your pain, I should be quick and gentle as possible, and once more comfortable.” able to give you something to make you Examining lesions: the PMI, if necessary ask the patient, “Can you please lift up your breast?” ask the patient, “Can you please lift the PMI, if necessary who is in severe pain: Examining a patient insist that you give him refuse the physical exam, or tially seem unapproachable, per- ask the patient’s his pain. In such cases, you should first something to stop that “I understand say, the physical exam. If he refuses, gently mission to perform Listening to the heart in a female patient: Listening to the heart auscultate or palpate To bra and between the breasts. where around the patient’s 2. Chest: it seriously. The following are some physical signs that may be simulated by the SP: The following are some physical signs that may it seriously. 1. Abdomen: SP Simulation of Physical Exam Findings SP Simulation of Physical Exam It that during the physical exam it is necessary to remain cautious bears repeating exhibit during the encounter are seldom and attentive, as the symptoms patients accidental and are usually reproducible. sign, take So when you notice any positive Special Challenges During the Physical Exam the Physical Challenges During Special problems. The any number of special may encounter physical exam, you During the to each: with potential responses challenges along are examples of such following THE PATIENT ENCOUNTER 62 .Musclesandjoints: 5. Eyes: 4. Nervoussystem: 3. .Bruitsandmurmurs: 6. fremitus thanusual. Increased fremitus:Thepatientwillsay“99”inacoarsevoice,creatingmore bruit. tiple antihypertensivemedications. Donotbesurprisedifyouhearanabdominal Renal arterystenosis:Apatient withhypertensionwhoisnotrespondingtomul- Restricted rangeofmotionjoints. wheel rigidity. steps, noswingingofthearms),restingtremor, masked facies,rareblinking,cog- Parkinsonism: Shufflinggait(difficultyinitiatingandstopping ambulation,small Spasticity. Rigidity. Muscle weakness. Nystagmus. Lid lag. in brightlight.”Dimthelighttomakepatientfeelmorecomfortable. Thepatientwillsay, “Ihatethelight”ordon’t feelcomfortable Photophobia: sis. Visual loss(central,peripheral):Inayoungpatient,thismaybemultiple sclero- Brudzinski’s sign. Kernig’s sign. Nuchal rigidity. Phalen’s sign. Tinel’s sign. Hearing loss. Chorea. Ataxia. Gait abnormalities. Hemiparesis. Facial paralysis. Tremor (resting,intentional). awkward hammerhit. make suretorespondwithanexaggeratedjerkeventhelightestandmost see it,itisnotthere.IfthepatientwantstoshowyouhyperactiveDTRs,hewill is strongenough.Inaclinicalencounter, trythereflexonlyonce;ifyoudon’t than oncetoensurethatyouhavenotmissedthetendonandyourstrike reflex intheSPisnotlikedoingsoarealpatient,whereyoumusttrymore Absent orhyperactivetendonreflexes(stroke,diabetesmellitus):Elicitingthe Extensor plantarresponse(Babinski’s sign). Dementia. Confusion. THE PATIENT ENCOUNTER 63 ) as well as to look for complications 1c seborrheic dermatitis of the scalp, or acne vulgaris. seborrheic dermatitis of the scalp, or acne be surprised to hear a real heart mur- heart, don’t When you listen to a patient’s mur. with enlarged tonsils. A patient with a sore throat may present You may see real C-section, appendectomy, cholecystectomy, or other scars. or other scars. cholecystectomy, may see real C-section, appendectomy, You see. Always inquire about any scar you overlook them. Don’t it and advise him to may see a real nevus (mole). Ask the patient about You in it. check it routinely and report any change Christmas-tree pattern, may see real skin lesions, such as pityriasis rosea in a You Skin lesions: You may see artificial skin discoloration (eg, painful red spots on discoloration (eg, painful red spots may see artificial skin Skin lesions: You nodosumthe shin for erythema with sarcoidosis or redness over an in a patient patient with arthritis). inflamed joint in a Thyroid bruit. stethoscope when you place the “Hush, hush” The patient says Carotid bruit: over his neck. heart, you will on the patient’s the stethoscope Once you place Heart murmur: hush.” hear him saying “Hush, CLOSURE 7. Skin: 8. Real physical exam findings: Here, both questions should be addressed, and the workup should aim to determine Here, both questions should be addressed, and the workup should aim to determine whether the diabetes is well controlled (HbA include the formulation of a differential diagnosis consisting of the most likely causes include the formulation of a differential diagnosis consisting of the most likely chest pain along with their associated workups. By contrast, if the of the patient’s case is patient has a history of diabetes mellitus and is presenting for follow-up, the diabetes well controlled? And second, is posing two questions: First, is the patient’s the patient experiencing complications such as diabetic retinopathy or nephropathy? as embodying and one or more key questions. Most of these questions are simple more complex. These questions straightforward, but others may be considerably should be addressed during closure. chief complaint is chest pain, the question that the As an example, if a patient’s case embodies is causing the chest pain? In this instance, closure should is, What Finishing that the patient encoun- the history and the physical exam does not mean closure is a critical part of the encounter. the contrary, To ter is over. is that each patient encounter can be viewed The first thing you should bear in mind Any signs of STDs? Any signs of pregnancy? Closure should include answers to all of Any signs of STDs? Any signs of pregnancy? Closure should include answers these questions along with a suitable workup for each. such as nephropathy (urine microalbuminuria). is presenting following a rape, the case is pos- cite another example, if the patient To injuries? ing the following questions: Are there any physical injuries? Psychological THE PATIENT ENCOUNTER 64 summarize keypoints Leave afewminutes to thepatient. for closure to FIGURE 2-4. Figure 2-4): From abroaderperspective,youareexpectedtodoseveralthingsduringclosure(see which youmustrespond(eg,“DothinkIhavecancer, doctor?” or “AmIgoingto During closure,almosteverypatientwillhaveatleastonechallengingquestionto any numberofdiseases. such awayastobemixtureofsignsandsymptomsthatcanconstruedindicate cases therewillactuallybenofinaldiagnosis;instead,thecaseconstructedin tell himthatyoustillneedtorunsometestsestablishthefinaldiagnosis.In You shouldnot,however, givethepatientadefinitivediagnosisatthistime.Instead, that youthinkisinvolvedandexplainasimplemechanismunderlyingthedisease. plicated medicaltermsyoumightuse.You mightalsopointouttheorganorsystem ential diagnoses(keeptoamaximumofthree)andexplainthemeaninganycom- give youmyimpression.”You shouldthentellthepatientaboutpossiblediffer- lettingmeexamineyou,Mrs.Jones.NowIwouldliketositdownwithyouand for To transitionintotheclosure,youshouldbeginbysayingsomethinglike“Thank Make atransitiontomarktheendofyourencounter. Leave theroom. Check toseeifthepatienthasanymorequestions. Address thepatient’s concerns. Answer anyquestionsthepatientmighthave. Suggest adiagnosticworkup. Give yourimpressionofthepatient’s clinicalconditionandmostlikelydiagnosis. Summarize yourfindingsfromthephysicalexam. before thephysicalexam. thechiefcomplaintandHPIifyouhavenotalreadydoneso Summarize Closure Overview Ask ifthepatienthasanyquestionsorconcerns. Do Briefly discussthediagnosticpossibilities. Briefly summarizethehistoryandphysicalfindings. Avoid Briefly explaintheplanneddiagnosticworkup. Shake thepatient’shandand say goodbye. Tell thepatientthatyouwillmeetagainwithtest results. not Handling ChallengingQuestionsorConcerns complicatedmedicalterms. give adefinitivediagnosis. give Avoid Be honestbutdiplomatic. giving falsereassurances. giving eoeLeaving Before Counseling THE PATIENT ENCOUNTER 65 once you leave. You cannot reenter cannot reenter You the examination room the examination room Encourage the patient to talk about his feelings. Ask about Stay calm and don’t be frightened. Remember that the pa- Stay calm and don’t also address the patient’s anger in a reasonable way. For example, if the patient is For example, if the patient anger in a reasonable way. also address the patient’s feel- complaining that he has been waiting for a long time, you can validate his ings by saying, “I can understand why anyone in your situation might become might react in this way, but I want you to know that I will do my best to address might react in this way, your concerns.” The angry patient: pa- tient is not really angry; he is just acting angry to test your response. Let the should You tient express his feelings, and inquire about the reason for his anger. The anxious patient: can Offer reasonable reassurance. You the things that are causing the anxiety. response by saying, “Any patient in your situation also validate the patient’s HOW TO INTERACT WITH SPECIAL PATIENTS The help you deal with atypical patients and uncommon following guidelines can encounters. completed the encounter. Once you have left the encounter room, you will not be completed the encounter. allowed to go back inside. ate closure, give priority to the execution of a proper closure. ate closure, give priority to the execution your encounter by looking the patient in Before you leave the room, you can finish Jones, I’ll contact you when I have Mr. the eye and saying something like “Okay, hand may then shake the patient’s you.” You your test results. It was nice meeting soon as you think you have are allowed to leave the room as and leave the room. You results of those tests, we will meet again to discuss them in detail, along with the final results of those tests, we will meet again by asking the patient you should conclude diagnosis and the treatment plan.” Finally, if he or she still has any questions. do not compromise the closure. If time con- If you find you are running out of time, a thorough physical exam and an appropri- straints dictate that you choose between planning to order. In doing so, you should again use nontechnical terms—for ex- In doing so, you should again use planning to order. need to run some blood of your liver and tests to check the function ample, “We might and a CT scan of the head.” You need to have a chest x-ray kidneys,” or “You CT scan is a form of x-ray imaging that further explain the latter by saying, “The should then add, “After we get the You body.” gives us clear images of sections of the If you do not know the answer to a patient’s question, you should state as much. See question, you the answer to a patient’s If you do not know patients might pose for examples of challenging questions the end of this section responses to each. along with potential During the diagnostic tests you are you should also explain to the patient closure, get better?”). In answering these questions, be honest yet diplomatic. Essentially, be- Essentially, yet diplomatic. questions, be honest In answering these get better?”). as “I am sure you reassurances such not giving false with the patient means ing honest it is not cancer.” I am sure worry, antibiotics,” or “Don’t after a week of will be cured of cancer at exclude the possibility I cannot is, “Well, might say instead What you how- Regardless of the final diagnosis, need to do additional testing. this point. We support you need.” you that I will be available for any I want to assure ever, THE PATIENT ENCOUNTER 66 now thatitishisturn,youwillfocusoncaseandtakecareofhim. and manypatientshadappointmentsbeforeyours.”Reassurethepatientthat angry underthesamecircumstances.Iamsorrylate.Thecliniciscrowded, ted tocalltheSPonlyonce. Treat thislikeanormalencounterandgather all the patient.Donotdialanynumbers ortouchanyotherbuttons.You arepermit- the telephone,andpush speaker buttonbytheyellowdottobeconnected enter theexaminationroom.Onceyouareinside,sitin frontofthedeskwith with otherencounters,patientinformationwillbeposted onthedoorbeforeyou The phoneencounter: may Icontacthimtoobtainsomeinformation?” cases, askyourpatient,“Isthereanyonewhodoesknowabout yourproblem,and swer yourquestionsbystating,“Idon’t know”or“I can’t remember.” Insuch The confusedpatient: bring thislistwithhimassoonpossible. tion orawrittenlistofthemedicationsheiscurrentlytaking. Ifnot,askhimto cations whosenames The patientwhodoesn’t knowthenamesofhismedicationsoristakingmedi- necessary, youcanalsowriteyourquestiondownandshowittohim. words. Ifthepatienthasunilateralhearingloss,sitclosetoside. lips. Speakslowly, anddonotcoveryourmouth.Usegesturestoreinforce The hard-of-hearingpatient: maneuver, anddocumenthisrefusalyourcounselinginthePN. at adiagnosis.Ifthepatientstillrefusestocooperate,skipquestionor him thattheyarenecessarytoallowyouunderstandtheproblemandarrive Explain tothepatientwhyquestionorphysicalexamisimportant.Tell The patientwhorefusestoansweryourquestionorletyouexaminehim: resources.” You mightalsoadd,“We willreferyoutoasocialworkerwhocanhelpfind by saying,“Nothavingenoughmoneydoesn’t meanyoucan’t gettreatment.” The patientwhocan’t payforthetestsortreatment: willbeasquickandgentlepossible.” “I your painandgiveyoutherighttreatment.”Reassurepatientbysaying, to helpyou.Ineedexamineyou,though,beablelocatethesourceof severe painheisexperiencing,tellhim,“Iknowthatyouareinpain,andIwant neuvers. Ifthepatientdoesnotallowyoutotouchhisabdomenbecauseof I candoforyoutohelpfeelmorecomfortable?”Donotrepeatpainfulma- thing like“Iknowthatyouareinpain.”Offerhelpbyasking,“Isthereanything The patientwhoisinpain: encounter inpeaceifyourespondcorrectly. patient isanactorandthathiscryingtimed.Hewillallowyoutocontinuethe about it?”Don’t worryabouttimeconstraintsinsuchcases.Rememberthatthe arm andsaysomethinglike“Iknowthatyoufeelsad.Would youliketotellme expressions. You mayalsoplaceyourhandlightlyonthepatient’s shoulderor silence forhimtofinish.Offeratissue,andshowempathyinyourfacial The cryingpatient: Allow thecryingpatienttoexpresshisfeelings,andwaitin you fthepatientisforgetful orconfused,hewilllikelyan- If h Step 2CSmayincludeatelephoneencounter. As The don’t recognize: Showcompassionforthepatient’s pain.Saysome- Facethepatientdirectlytoallowhimreadyour Askthepatientifhehasaprescrip- Reassurethepatient THE PATIENT ENCOUNTER 67 a definitive diagnosis. Do not give the patient you are in a safe environment and that you are not a victim of abuse.” “I know that you are concerned about “I know that you are concerned about medical costs, but your life will be in Let have surgery. danger if you don’t our social workers help you with the cost issues.” that you have There is a chance “Yes. already transmitted the disease to your or he or she may be the source partner, of your infection. The most important step is to have both of you evaluated and appropriately treated.” “I am primarily concerned about your and my goal is to make sure that safety, Don’t give the patient a final diagnosis. Instead, tell the patient about your initial give the patient a final diagnosis. Don’t have in mind to reach a conclusive diag- impressions and about the workup you nosis. Do not give false reassurances. question, tell him so, but reassure patient’s If you do not know the answer to the him that you will attempt to find out. Be honest and diplomatic. you might restate the issue back to the pa- issue, Before addressing the patient’s tient to let him know that you understand. the necessary information. To end the call, press the speaker button above the press the speaker end the call, information. To the necessary leave this physical exam, so there is no encounter, As in the pediatric yellow dot. the PN blank. portion of An anxious patient who you suspect has been abused asks, “Why are you asking me these questions?” “Should I tell my sexual partner about my venereal disease?” A patient who needs emergent surgery A patient who needs emergent surgery afford the cost of staying says, “I can’t in the hospital. I have no insurance. Just give me something to relieve the pain and I will leave.” Challenging Question Possible Response CHALLENGING QUESTIONS AND SITUATIONS CHALLENGING The following are examples of challenging questions: The following are examples of challenging Confidentiality/Ethical Issues an underlying concern. When answering the challenging questions, try to remember When answering the challenging an underlying concern. the following guidelines: During challenging ques- every patient will ask you one or more your encounters, questions will be scored. Such questions reactions and answers to these tions. Your take the or they may that you are expected to answer directly, may be explicit ones addressed to reveal or statements that must be properly form of indirect comments THE PATIENT ENCOUNTER 68 Patient Belief/BehavioralIssues hlegn usinPossibleResponse HIV asks,“DoIhavetotellmywife?” A patientrecentlydiagnosedwith Challenging Question for thisdiseaseisherbalcompounds.” “I readinajournalthatthetreatment PossibleResponse getting old.” problem” (impotence)or“Iamjust that itisnormalatmyagetohavethis An elderlymalepatientsays,“Ithink Challenging Question me atmyplace?” do youhavetimetodinnerwith her malephysicianbysaying,“Doctor, A femalepatientattemptstoseduce a femalepatient. but whenyouentertheroom,find and thathepresentswithdizziness, cates thatthepatientisMr. Smith A doorwayinformationsheetindi- during thephysicalexam. you willneedtotakethepatient’s vitals sheet arethoseofadifferentpatient,so signs listedonthedoorwayinformation complaint, butrememberthatthevital go ontodiscussthepatient’s presenting What isyourproblem?”You canthen me thewrongchart.Hello,Mrs.Black. “Oh, Ithinkthenursemusthavegiven “No, IamMrs.Black,”youcansay, Smith?” Whenthepatientresponds, Begin bysaying,“Excuseme,Mrs. ease.” prevent thetransmissionofdis- appropriate precautionstotreatand will allowyouandyourwifetotakethe “I knowthatit’s difficult,butdoingso ease.” its potentialtreatmentroleforthisdis- herbal medicineandIwillcheckinto clear-cut. Letmeknowthenameof safety andefficacymaynotalwaysbe for manydiseases.However, their “Herbal medicineshavebeensuggested prove yoursexualfunction.” We alsohave medicationsthatmayim- pertension, diabetes)ormedications. rule out,suchascertaindiseases(hy- may haveothercausesthatweshould your sexualfunction,butproblem in thechangeyouareexperiencing “Not necessarily. Agemayplayarole of adoctor-patient relationship.” would notbepermissibleinthecontext priate, sinceyouaremypatient,andit “I amsorry, butthatwouldbeinappro- THE PATIENT ENCOUNTER 69 to do to help us make the final diagno- sis.” “It is never too late to seek help, and I am glad you made the decision to pur- will sue treatment options with me. We do our best to help you, but next time I want you to feel comfortable coming to me as soon as you feel you might have a problem.” Explain the meaning of the term using Explain the meaning of the term using simple words. For example, “Bronchos- copy is using a thin tube connected to a camera to look into your respiratory airways and parts of your lungs,” or “An MRI is a machine that uses a large mag- net to obtain detailed pictures of your brain or body.” and very common to have these feel- common to have and very Is there anything ings before surgery. concerned about?” specific that you are put off want to “I know that you don’t may have a serious your trip, but you benefit from early problem that may Also, it is diagnosis and management. suffer complica- possible that you could while you are tions from this problem do not effectively on vacation if we deal with it before you leave.” If the pa- Repeat the question slowly. comprehend the ques- tient still doesn’t tion, ask if there is any specific word he failed to understand, and try to explain it or use a simpler one. ?” going “It means all the tests that we are workup late for recovery?” A patient who is late in seeking medi- cal advice asks, “Do you think it is too “What do you mean by “What is a bronchoscopy?” (MRI, CT, “What is a bronchoscopy?” (MRI, CT, colonoscopy) x-ray, “I did not understand your question, “I did not understand your question, Could you repeat it, please?” doctor. A patient who has a serious prob- A patient who has colon cancer) lem (unstable angina, on a trip with my asks, “I want to go tests after I come wife. Can we do the back?” Challenging QuestionChallenging “I am afraid of surgery.” Response Possible It is normal “I understand your feelings. THE PATIENT ENCOUNTER 70 hlegn usinPossibleResponse asks, “Isthisaheartattack?AmIgo- A patientwithpleuriticchestpain Challenging Question could becomemalignant?” “Do youthinkthatthistumorIhave health asks,“CanIgobacktowork?” at ajobthatcannegativelyaffecthis A personwhowantstoreturnwork penses ofthistest?” “Will myinsurancecovertheex- to refillmyprescription.” fine doctor. That’s why Icametoyou “My friendtoldmethatyouareavery “Do Ihaveendometrialcancer?” “Do youthinkIhaveabraintumor?” “Do youthinkIhavecoloncancer?” ing todie?” if thechiefcomplaintisinconsistent fore makingadiagnosis.”However, symptoms thatweshouldruleoutbe- there areotherexplanationsforyour “That isoneofthepossibilities,but sistent withhisquestion,tellhim, If thepatient’s chiefcomplaintiscon- problems.” firm thediagnosisandruleoutheart we stillneedtodosometestscon- life-threatening condition.However, ing yourpain,andthisisusuallynota branes surroundingyourlungsiscaus- likely thatinflammationofthemem- cion foraheartattackislow. Itismore clinical examandfindings,mysuspi- “On thebasisofyourhistoryandmy we getanyinformation.” it. We willkeepyouinformedassoon the tumorandgetapathology reporton “We really won’t knowuntilweremove ployer explainingyoursituation.” now. Icanwritealettertoyourem- would preferthatyoustayathomefor worsen yourcondition.Therefore,I “Unfortunately, workmayactually the importanceofthistest.” to yourinsurancecompanyindicating mation. Ifnecessary, Icanwriteanote social workerwhodoeshavethatinfor- “I’m notsure,butIcanreferyoutoa order sometests.” need todoaphysicalexamandperhaps need forthismedication.Iwillalso your historytobetterunderstand give youarefillwithoutfirstreviewing but sincethisisyourfirstvisit,Ican’t “I amhappythatyoucametoseeme, by conductingsometests.” worried aboutit,Iwilltrytoruleitout this typeofcancer, butifyouarereally a patientwithyourcomplainttohave with hisconcern,say, “Itisunlikelyfor THE PATIENT ENCOUNTER 71 “It is hard to tell right now, but these “It is hard to tell right now, fractures usually heal well, and with physical therapy you should regain the normal range of motion of your arm.” Is “Life can certainly be challenging. is there something in particular that bothering you? Have you thought of can then con- ending your life?” You tinue screening for depression. “Having multiple sexual partners does put you at risk for STDs, including HIV infection, but this rash may be due to many other causes. I agree that we should do an HIV test on you in addi- tion to a few other tests.” “I understand your concern about your child, but right now staying in the hos- your pital is in your best interests. With permission, one of our social workers can make some phone calls to arrange for child care.” “There are healthier ways to lose healthier ways “There are as exercise smoking, such weight than will increase your and diet. Smoking heart problems, and lung risk of cancer, disease.” will do our best to help you re- “We With injury. cover from your shoulder will communicate your permission, I employer.” the situation to your can vary from “Yes, of the disease and most people with this disease are com- pletely cured” to “A complete cure may be difficult to achieve at this advanced stage, but we have a lot to offer in terms of controlling the symptoms and im- proving your quality of life.” A patient who needs hospitalization says, “My child is at home alone. I have to leave now.” A young man with multiple sexual A young man with multiple sexual partners and a recent-onset skin rash says, “I am afraid that I might have AIDS.” “I think that life is full of misery. Why “I think that life is full of misery. do we have to live?” A person who has a broken arm asks, A person who has a broken arm asks, do you think I will be able to “Doctor, move my arm again like before?” “Will I ever feel better, doctor?” better, I ever feel “Will The answer depends on the prognosis A patient with a shoulder injury says, A patient with a shoulder my job if my “I am afraid of losing better.” get shoulder doesn’t Challenging QuestionChallenging I have stopped smoking, “Since I back to I want to go gained weight. lose weight.” smoking in order to Response Possible THE PATIENT ENCOUNTER 72 hlegn usinPossibleResponse me feelbetter?Please,doctor, Iamin “Do youhaveanythingthatwillmake Challenging Question waiting hereforthewholeday.” have youbeen,doctor?Ibeen A patientisshoutingangrily, “Where bly becheatingonme?” STD. Sheasksyou,“Couldhepossi- partner, andsheisdiagnosedwithan A femalepatienthasonlyonesexual “Iwilltrytomanageyourproblemmed- surgery?” need will I think you “Do “Afterwegettheresultsofyourtests, “So what’s the plan, doctor?” this pain.” tions andjustgivemesomethingfor “Stop askingmeallthesestupidques- in mybackisterrible.” need aweekofffromwork.Thepain (malingering) says,“Please,doctor, I A patientyoubelieveispretending pain.” After Iamdonewithmyevaluation, I amtogiveyoutherighttreatment. to determinethecauseofpainif “I knowthatyou’reinpain,butIneed tinuing yournormaldailyactivities.” part ofyourrecoverywillinvolvecon- medication andexercises,butalarge you outofwork.Iplantoprescribepain disability significantenoughtokeep but afterexaminingyou,Idon’t find “I knowthatyouareuncomfortable, help manageyourpain.” tion, wecandecideonthebestwayto ment. AfterIamdonewithmyevalua- order togiveyoutheappropriatetreat- to knowwhatiscausingyourpainin “I knowthatyouareinpain,butIneed much timewithyouasneed.” on youandyourconcernsspendas ing. ButI’mherenow, andIwillfocus had someunexpecteddelaysthismorn- “I amsorryyouhadtowaitsolong.We reinfected.” and treated,orelseyouriskbecoming clear thingsup.Heneedstobetested to talkyourpartneraboutthis tion fromyourpartner. Itwouldbebest “You mostlikelycontractedthisinfec- gether inthefuture.” and thentrytomakethatdecisionto- need surgery. We canseehowthingsgo ically, butifthatdoesn’t work,youmay have.” try toansweranyquestionsyoumight we willmeetagain.Atthattime,I ment.” we willgiveyoutheappropriatetreat- THE PATIENT ENCOUNTER 73 giving you any blood.” “I have had a blood transfusion be- fore, and I had a serious reaction.” “Thank you for telling me say, You this. I will determine the reason you had this reaction and will treat it before giving you any blood.” “I have a religious objection to re- “I have a religious “I respect say, ceiving blood.” You will make sure your opinion and a bloodyou do not receive trans- explained its fusion until we have obtained your benefits and have permission.” a blood“My brother died following transfusion, and I’m afraid the same thing will happen to me.” respond, “I am sorry for your You loss, but I want you to know that it is rare for patients to die as a result of a blood transfusion. I will take all necessary precautions before First determine the reason for the pa- the reason for First determine respond ac- and then reaction, tient’s For example: cordingly. him. “I can only imagine how any patient “I can only imagine how any patient in your situation might feel, but if you speak with me, I will not be able don’t to help you. So please have a seat and help me determine what is going on.” The patient may have a problem un- derstanding or hearing you. Ask the patient why he is repeating your ques- tions. If the problem relates to com- prehension and you are not a native ask him to stop you English speaker, whenever he has difficulty understand- ing what you are saying. If the problem relates to his hearing, draw closer to Challenging QuestionChallenging angrily patient reacts A bleeding may need mention that she when you a blood transfusion and states that she any blood.refuses to be given Response Possible fore answering them. A patient is wandering around the A patient is wandering around the room ignoring you and is not answer- ing your questions or listening to you. A patient repeats your questions be- THE PATIENT ENCOUNTER 74 hlegn usinPossibleResponse you arewashingyourhands. A patientasksyouaquestionwhile Challenging Question with asonwhoisgay. A patientwantstoknowhowdeal tor madeamistakeinhistreatment. A patientasksyouifhispreviousdoc- tions asksifyouthinkheiscrazy. A patientwithauditoryhallucina- another doctor. A patientwantstobeexaminedby he cantakeabathroombreak. During theencounter, apatientasksif “There isnosuchdiagnosisinmedi- fit foryou.” another doctorwhomightbeabetter ion, Iwouldbehappytohelpyouselect concerns. Ifwestillneedanotheropin- you ifwillallowmetoaddressyour am awell-qualifieddoctorandcanhelp vider, butIwanttoreassureyouthat tainly yourrighttochooseanotherpro- Find outwhy. You cansay, “Itiscer- tance. nation room,andofferhimyourassis- Do notforcehimtostayintheexami- to him. establish eyecontactwhenyourespond him yourfullattention.Makesureyou Tell himthatyouwouldliketogive tal healthprofessional. or hersontoseekguidancefromamen- sexual orientation.Ifso,encourageher any guiltorconfusedfeelingsabouthis Ask thepatientifsheorhersonhas safe foryou.” that wethinkwillbemosteffectiveand but wewanttogiveyouthetreatment sorry ifthismaybefrustratingforyou, of whatwehavediscoveredtoday. I’m our besttomakeadecisiononthebasis different treatmentplan,wehavetodo your previousdoctormayhavehada fers fromyours,youcansay, “Although ous doctor’s findingsortreatmentdif- If thepatientindicatesthatprevi- we canaddressit.” chancethat andthereisagood mood, ological problemoradisorderinyour cine. Ithinkyoumayhaveaphysi- THE PATIENT ENCOUNTER 75 one of several possible causes that we will investigate.” “I know that you are in pain, but I need “I know that you are in pain, but I need to ask you a few questions first to better understand your pain. Then we will get you some medications to help ease your discomfort.” “It appears that you have a common cold, which is caused by a virus. Antibi- otics do not treat viruses, and they have adverse effects that could make you feel should focus on treating worse. We your symptoms.” risk, but it doesn’t are at increased “You mean that you will get it. There are other risk factors that need to be con- sidered, and regular screening tests will be very important.” need to it should be repeated. We “Yes, screen for more polyps, and in this way we hope to prevent the development of colon cancer.” disease is Alzheimer’s know. “I don’t “Studies do in fact show a slight increase in fact show a slight “Studies do breast cancer in the risk of developing years of combina- after more than four progesterone use for tion estrogen and The therapy. hormonal replacement are to use current recommendations therapy solely hormonal replacement flashes, and only for for the relief of hot a limited period of time.” could be explained by a small stroke, but we need to wait for the results of your MRI.” but we still need to do ad- possibility, ditional tests.” moved a polyp. Do you think that I have to repeat the colonoscopy?” A patient with headache or confusion asks, “Do you think I have Alzheim- disease?” er’s cancer too?” man says, “I had a colo- A 55-year-old noscopy six years ago, and they re- “My mother had breast cancer. What “My mother had breast cancer. is the possibility that I will have breast tor, I need some Demerol now or I will I need some Demerol tor, die from pain.” A patient with symptoms of a com- mon cold says, “I think I need antibi- otics, doctor.” “Do I have lung cancer?”lung have I “Do An African American man with sickle cell anemia presents with back do not know at this point. It is a “We and chest pain and says, “Please, doc- “Did I have a stroke?”a have I “Did symptoms know yet. Your don’t “We breast cancer. What do you think of What breast cancer. that, doctor?” Challenging QuestionChallenging woman asks, 58-year-old An educated journal that hor- “I read in a scientific therapy causes monal replacement Response Possible Disease-Related Issues Disease-Related THE PATIENT ENCOUNTER 76 hlegn usinPossibleResponse tubes aretied?” “Can Igetpregnanteventhoughmy Challenging Question get betterifIstopsmoking?” A patientwithCOPDasks,“Will I What doyouthinkthereasonis?” “I amdrinkingalotofwater, doctor. you thinkIhaveanSTD?” A youngmanwithdysuriaasks,“Do think thatiswhyIamoverweight?” “Obesity runsinmyfamily. Doyou think itismythyroid?” mother hadathyroidproblem;doyou A patientwithpalpitationssays,“My lon canceraswell?” are thechancesthatIwilldevelopco- “My brotherhascoloncancer. What pregnancy?” asks, “DoyouthinkIamlosingmy of pregnancywithvaginalbleeding A womanwhoisinherfirsttrimester more informationaboutyourpersonal get coloncancerforsure.Ineedto risk, butitdoesn’t meanthatyouwill reditary, andyoumaybeatincreased “Some typesofcoloncancerarehe- without anyproblems.” bleeding carrythepregnancytoterm the sametime,mostwomenwhohave your riskoflosingthepregnancy, butat “Bleeding earlyinpregnancyincreases ectopic pregnancy.” if suchapregnancyoccurs,itwillbean occur. Thereisahighprobabilitythat than 1%,butonrareoccasionsitdoes of pregnancyaftertuballigationisless thatis100%effective.Therisk method “There isnosinglecontraceptive of lungcancerinthefuture.” dition toasignificantlydecreased risk improvement intheirsymptoms,ad- stop smokingwillexperienceagradual “Most patientswithyourconditionwho tests todeterminethecause.” such asdiabetes.We needtodosome tion, oritmaybeasignofdisease “This maysimplybeduetodehydra- urinary tractinfection.” since yoursymptomsmaybeduetoa and wewillalsocheckaurinesample, do someculturestofindoutforsure, “That isoneofthepossibilities.We will way thatcanhelpyouloseweight.” weight. Thesefactorscanbeusedina its arealsomajorfactorsinfluencing sity, butlifestyle,diet,anddailyhab- “Genes playanimportantroleinobe- palpitations.” consider manyotherpossiblecausesof test,butwewillalso a thyroidblood “That isapossibility. We alwayscheck level ofrisk.” and familyhistorytodetermineyour THE PATIENT ENCOUNTER 77 are are 5 A’s smoking. patients quit ssist in the quit rrange for sk the patient dvise him or her to ssess the patient’s The recommended recommended attempt. follow-up. about tobacco use. quit. willingness to make an attempt to quit. A A A A A guidelines to help 4. 5. 1. 2. 3. tell us if you are at a healthy weight.” “I am sorry, but I can’t give you any- give but I can’t “I am sorry, You now. eat or drink right thing to and anes- surgery, may need emergent if your stomach is thesia is much safer completely empty.” recovered from the “Now that you have disease, you can go acute stage of the I want you to stay back to school, but exercise or away from any strenuous may rupture your contact sports, as you spleen.” Even if the patient appears to be thin, do not state as much. Instead, respond First by saying, “I cannot tell right now. I need to determine your height and weight and calculate your body mass index, and then we can let the numbers Well, I strongly recommend that you quit smoking. Smoking is a major Well, Do you smoke cigarettes? Have you ever tried to quit? I would be happy to help you quit smoking. We have many tools to help I would be happy to help you quit smoking. We ready to quit smoking yet, but I want to as- I understand that you aren’t Yes. (If the answer is “no,” see below.) (If the answer Yes. want to quit. No, I don’t Yes, I have smoked one pack a day for 20 years. I have smoked Yes, but it never works. Yes, COUNSELING A very thin patient with weight loss A very thin patient with weight loss do you think I am too asks, “Doctor, fat?” A patient with infectious mononu- A patient with infectious go back to school, cleosis asks, “Can I doctor?” Challenging QuestionChallenging with possible appendicitis A patient cup of water to drink. asks for a Response Possible sure you that whenever you are ready, I will be here to help you. sure you that whenever you are ready, SP1: Examinee: set up an appointment Let’s you do that, and I will be with you every step of the way. and we can get started on it then. Is that okay with you? for two weeks from today, SP2: Examinee: SP: Examinee: SP: Examinee: cause of cancer and heart disease. Are you interested in trying to quit now? conversations you might have with your patient. Try to practice saying some of these to practice saying some patient. Try conversations you might have with your fit your personality and style. aloud, making sure to change them to The Smoker Examinee: During at least one of your encounters, you are likely to find a patient who smokes, During at least one of your encounters, adversely affect his or her health. Although drinks, or has another habit that may to your primary diagnosis, it is impor- these behaviors may or may not be relevant Here are some examples of yet caring manner. tant that they be addressed in a rapid THE PATIENT ENCOUNTER 78 SP2: be abletoarrangeforanursecomeyourhome.Areyouinterestedinthat? who couldhelpyoutakeyourmedications?Ifnot,wehaveasocialworkermight Examinee: to takethem. SP1: ance.) controlled. Howoftendoyouforgettotakeyourmedication?(Checkfornoncompli- Examinee: The PatientwithUncontrolled Diabetes felt Examinee: SP: Examinee: SP: Examinee: The Alcoholic SP: Examinee: The SexuallyPromiscuous Patient who maybeabletohelpyou.Areyouinterestedinmeeting withher? ing infections,heartattacks,andstrokes.Fortunately, wehaveadiabeteseducator damage toyoureyes,kidneys,andnerves.You willalsobeathigherriskfordevelop- sugarcan cause sugarwithinnormallimits.Persistentlyhighblood keep yourblood Examinee: SP2: Examinee: that ifyoudodecidetotry, Iwillbehereforyou.Okay? Examinee: SP2: of resources,andmyofficeassistantwillbringittoyou. later thisweektotalkaboutyouroptions.Inthemeantime,Ihaveprintedupalist quit drinking,andIwouldliketodiscussthemwithyou.Let’s make anappointment Examinee: SP1: in cuttingdownorquitting? problems withbleeding,orevenpredisposeyoutoearlydementia.Areinterested Examinee: SP: and promptfurtherquestioning.) answer toanyoneofthequestionsinCAGEquestionnaireshouldraisesuspicion admits tomanydrinksperweekshouldreceivetheCAGEquestionnaire.A“yes” ing? Haveyoueverhadtotakeamorning Itishardtosay. Too many. Oh,maybefiveorso. Yes. annoyed All ofthesethingsapply. Ihavebeentakingmymedicationsexactlyastheywere prescribedtome. Taking allthesemedicationsjustgetssoconfusing. Icanneverrememberwhen I eat regular meals, but I really like to drink soda. Diet soda tastesawful! Dietsoda Ieatregularmeals,butreallyliketodrinksoda. Yes. (Iftheansweris“no,”seebelow.) No, Iamnotreadytoquit. Howmanydrinksdoyouhaveperday? You mustbeverycarefulaboutyoursugarconsumption.Itisprudentto Tell meaboutyourdiet.(Checkfordietarymanagement.) Diabetescancertainlybeachallengetomanage.Doyouhavesomeone glucosereadings,yourdiabetesisnotadequately Accordingtoyourblood Iamgladyouwanttoquit.Avarietyofresourcesareavailablehelp Iamconcernedaboutyourdrinking.Itcanleadtoliverdisease,cause Have youeverfelttheneedto How manydrinksdoyouhaveinaweek? Are youcurrentlyinasexual relationship? I realizethatyouarenotreadytoquitdrinking,butwantassure bycriticismofyourdrinking?Haveyoueverfelt cut down eye opener? onyourdrinking?Haveyouever (Ingeneral,anypatientwho guilty aboutdrink- THE PATIENT ENCOUNTER 79 Has it been in things that energy? interest changed? Have you felt agitated Have you lost ) (If you suspect depression, ask the ques- appetite Do you lack your usual sleeping? suicide? guilty? Has your SIG E CAPS. disturbances.) Do you feel as though you want to hurt disturbances.) Do you feel as though concentrate? Psychomotor Your symptoms are due to an infection called trichomoniasis, a sexually symptoms are Your Do you have problems Condoms reduce the risk of sexually transmitted infections. Do you think risk of sexually transmitted infections. Condoms reduce the I am concerned you may not like to use condoms, but I understand that Can you tell me about your partner or partners? tell me about your Can you partners? with these any type of protection Are you using You answered “yes” to many of my questions. I believe that you may be answered “yes” to many of my questions. I believe You I have a girlfriend, but I also see a couple of other women on the side. see a couple of other but I also I have a girlfriend, I tried them, but I just don’t like them. like just don’t I tried them, but I My girlfriend is on the pill, but I don’t use anything with the other women I see. with the other use anything I don’t is on the pill, but My girlfriend (Answers affirmatively to many of these questions.) (Answers affirmatively to many of these THE PATIENT NOTE THE PATIENT The Patient with an STD (Trichomoniasis) Figure 2-5 for a detailed overview of the clinical encounter and PN). Toward this Figure 2-5 for a detailed overview of the clinical encounter and PN). Toward goal, you will find a desk with a computer on it immediately outside the encounter ics and your partner gets treated. your final task will be to compose a PN (see Once you have completed an encounter, transmitted infection that has been given to you by one of your sexual partners. This transmitted infection that has been given to you by one of your sexual partners. will also infection responds well to treatment with antibiotics and is curable. You partner needs to be informed and treated as need to be tested for all other STDs. Your you use well; otherwise you will be at risk of contracting the infection again. Unless antibiot- condoms, you should avoid sexual intercourse until you finish the course of Examinee: Examinee: used to interest you? Do you feel whom you have sexual contact has an STD, you could share it among all of them, contact has an STD, you could whom you have sexual consider using a condom in the future. Do including your girlfriend. I hope you will you have any questions for me? Patient The Depressed SP: Examinee: con- could for STDs. You yourself and your partners at risk that you may be putting complications or any of a number of other STDs. The herpes, chlamydia, tract HIV, with infections, or even death. If anyone painful infertility, of these diseases include Examinee: SP: Examinee: SP: Examinee: condoms? you could try to use combined with counseling. I can write you a prescription and also give you a referral combined with counseling. I can write are interested in? to see a therapist. Is this something you tions posed in the mnemonic SP: Examinee: it is due to a chemical imbalance in the depressed. Depression is a common disease; we you have described to me. Fortunately, brain that causes many of the symptoms these medications work best when they are have medications that can help; however, difficult for you to or lethargic? ( yourself or someone else or commit THE PATIENT ENCOUNTER 80 FIGURE 2-5. 3–5 10–20 SECONDS 7 8MINUTES –8 MINUTES of the 15-minute period allottedforyourpatientencounter,of the15-minuteperiod youcandevotetheextra be notifiedwhentwominutesremain.Ifyouleavetheencounter roombeforetheend type, nothandwrite,thePN. room. the spacewisely. portions ofyournoteifyou run outofspace,sousethistoyouradvantage,and note isthatitallowsyoutodelete extraneousinformationinfavorofmorepertinent fields inDifferentialDiagnosisandDiagnosticWorkup. Onebenefitofthecomputer character limitsare950forHistory, 950forPhysicalExam,and100eachofthe tic Workup. onlyacertainnumberofcharacters: The Eachfieldcanaccommodate formation andfieldsforHistory, PhysicalExam,DifferentialDiagnosis,andDiagnos- The PNscreenlocatedoutsidetheencounterroomwill have youridentificationin- while youaretypingthePN. time youhavetotypingthePN.You areallowedtoreviewthedoorwayinformation Summary OverviewofthePatientEncounter Remember thatallexamineestakingtheStep2CSwillnowberequiredto “Examinees, youmayentertheroom.” First announcement: First PHYSICAL EXAM PHYSICAL DOORWAY HISTORY Use arespectfuldrapingtechnique. Never Ask Tell chiefcomplaint. Develop alistoflikelydifferentialdiagnoses. history. Note theencounterobjectives:historyandphysicalexamvs.just Read thedoorwayinformationandnote Obtain past medical, surgical, medication,family,Obtain pastmedical,surgical, social,sexual, Do not withan Start Put the drape onthepatientandcoverlegs. Show Avoid Make Greet and allergy history. andallergy permission thepatientwhatyouaregoingtodo. examine throughthegown. thepatient/shakehands/introduceyourself. eye technicalterms. empathy interrupt/rush thepatient. contact;maintaina KNOC You willbegiven10minutestotypethePNand open to start thephysicalexam. tostart andaddressthepatient’sconcernsexpressions. K -endedquestion. ntedoo the on WASH yourhands. proper posture. r n ente and r the vital signs/age/ r oom. (continued) THE PATIENT ENCOUNTER 81 exams if needed. for this encounter.” (continued) negatives. breast through your exam. or and challenging questions. halfway finished.” five minutes left diagnostic tests. possible diagnoses with supporting history CC, HPI, PMH, PSH, meds, SH, ROS, FH. physical findings. eight three key key your diagnostic possibilities/workups. pertinent positives counseling. consult, hospitalize, or treat the patient. rectal, pelvic, genital, complicated medical terms. if the patient has any concerns. not the Document Include and physical findings. Order up to Do Order Document Give up to Explain Avoid Ask Be prepared to handle Avoid giving false reassurances. Do patient, and leave the encounter. Say goodbye, thank the TYPED PATIENT NOTE CLOSURE Fourth announcement: Fifth announcement: “You have two minutes left.” “You is now finished time “Your .” Second announcement: “Examinees, you have be By this time you should Third announcement: “This encounter is now Summary Overview of the Patient Encounter Overview of the Patient Summary 10 MINUTES 2–3 MINUTES You will be required to fill out four main sections in your PN: the history, physical to fill out four main sections in your PN: the history, will be required You exam, differential diagnosis, and initial diagnostic workup. become garbled, and you will risk losing the point of your story. figure Note that you will not be able to render diagrams such as the neurology stick site. simulate typing the PN online at the USMLE Web can for reflexes. You Patient Note the Writing Before you start typing the PN, take a few seconds to review the history, including the Before you start typing the PN, take a few seconds to review the history, take chief complaint, how it started, its progression, and the main symptoms. Then may a deep breath and try to relax. If you get nervous and try to rush, your thoughts FIGURE 2-5. THE PATIENT ENCOUNTER 82 The componentsthatshouldbeincludedareasfollows: Also bearinmindthatitisnotnecessarytowriteadetailed,all-inclusivehistory. avoid longandcomplexphrases.Makesurethehistoryflowsinalogicalsequence. Summarizing thehistory. why yourdiagnosisislikely. You donotneedtolistthreefindingsforeach,andin Below eachdiagnosis,youneed tolisthistoricalandphysicalfindingsthatsupport your diagnosesbelistedinorder ofprobability, fromthemosttoleastprobable. mon chiefcomplaintwillhaveatleastthreepossibleetiologies. Itispreferablethat You arenotrequiredtolistthatmanyiftwodiagnosessuffice,butingeneralanycom- exam datathatsupportthem. following tablestolistyourthreepossiblediagnosesandthehistoricalphysical Developing adifferential. of howtodocumentphysicalexamfindings. litus ifyoudidnotevengettoseetheeyefundus.SeeFigure2-6forsomeexamples ample, donotclaimthatyousawdiabeticretinopathyinapatientwithdiabetesmel- and pretendthatyoudid.Behonestlistonlytheitemsexamined.Forex- If youdidnotperformamaneuverthatthinkwasnecessary, itisbetternottolie systems thatyouexamined,outliningalltherelevantpositiveandnegativefindings. Outlining thephysicalexam. this book’s samplecases. candidate orientationmanual,andmultipleexamplesofbothstylesareincludedin your historyisbothcomprehensiveandcoherent.Two examplescanbefoundinthe In general,twostylesofwriting—narrativeand“bullet”—areacceptableaslong better tospelloutthewordorphrase. commonly usedinU.S.hospitals.Ifyouareunsureofthecorrectabbreviation,itis find acopyofthislistoneachdesk.You areallowedtouseanyabbreviationsthat breviations thatarelistedintheUSMLEStep2CSorientationmaterials.You will way tosavetimeismakeampleuseofabbreviations.Train yourselftousetheab- When youaresummarizingthehistory, youneedtobeefficient withyourtime.One itr idn() Physical Exam Finding(s): History Finding(s): Diagnosis Family history(FH) Social history(SH) Past surgicalhistory(PSH) Past medicalhistory(PMH) Review ofsystems(ROS) History ofpresentillness(HPI) Chief complaint(CC) Inwritingthehistory, beclear, direct,andconcise, Inwritingthedifferential,youshouldusethreeof To summarizethephysicalexam,writealist ofthe THE PATIENT ENCOUNTER 83 . , no hepatosplenomegaly. RRLA, normal eye fundus. Romberg sign, intact finger to nose. Supple, no JVD, normal thyroid, no cervical LAD. Supple, no JVD, normal Distant memories are impaired. Oriented to person, date, and place. Completed three-step command. Right-handed. 1/5 on serial 7s. judgment. Poor No murmurs, rubs, or gallops. nondistended, BS Soft, nontender, No clubbing, cyanosis, or edema. speaks slowly. Patient No hostile behavior toward the interviewer. Blunt affect with poor eye contact. Inattentive to interviewer. 3/3 registration, 3/3 recall at 3 minutes. Sensation: Intact to sharp and dull. Cerebellum: Clear to auscultation bilaterally. No rales, rhonchi, wheezing, or rubs. No tenderness to palpation. fremitus WNL. Tactile PMI not displaced. Regular rate and rhythm. Normal S1, S2. Head: Atraumatic, normocephalic. Eyes: EOMI, PE Nose: No nasal congestion. No tonsillar erythema,Throat: exudates, or enlargement. membranes, good dentition, no lesions. Mouth: Moist mucous Mental status: Alertconcentration. and oriented x 3, good Cranial nerves II–XII grossly intact. Motor: Strength5/5 in all muscle groups. symmetric, Babinski 2+ intact and DTRs: Examples of How to Document Physical Exam Findings of How to Document Examples ࠗ ࠗ ࠗ ࠗ ࠗ ࠗ ࠗ Abdomen: ࠗ Extremities: ࠗ Mental Status Exam: ࠗ ࠗ ࠗ ࠗ ࠗ ࠗ ࠗ Chest/Lung: ࠗ ࠗ ࠗ ࠗ Heart: ࠗ ࠗ ࠗ HEENT: ࠗ ࠗ ࠗ ࠗ ࠗ Neck: Nervous System: ࠗ ࠗ ࠗ ࠗ ● ● ● ● ● ● ● ● FIGURE 2-6. THE PATIENT ENCOUNTER 84 Tests inthediagnostic workup should be specific. should belistedtogether. may, however, tests,x-rays) orderelectrolytes.Eachgroupofrelatedtests(blood your notewithours.Askyourselfthefollowingquestions: When usingthecasespresentedinthisbook,trytowriteyourPNandthencompare Imagine thatyouareintheactualexam,andtrytotypePNwithin10minutes. The cardinalruleforpreparingtowriteaPNispractice,andpractice. How toPrepare score ofall10scoredencounters. formation, andinterpretationofdata.ThefinalscorewillrepresenttheaveragePN PNwillbescoredbyaphysicianonthebasisofitsorganization,qualityin- The Scoring thePatientNote tests,” youshouldspecify“Na,K,”“TSHandtotalT Be specificinyourorders.Insteadof“chem7,”“thyroidpanel,”or“liverfunction treatments, hospitalizations,orconsults,asthesewillnotbescored. straightforward testsandendingwiththemostcomplex.Donotincludereferrals, state therequiredlaboratoryandradiologictests,startingwithmostsimple (eg, rectalexam,pelvicexam)ifyoufeelthatsuchproceduresareindicated.Then your differential.Itisbesttostartwiththe“forbidden”physicalexammaneuvers mum ofeightteststhatwouldhelpconfirmorruleoutthediagnosesyoulistedon Specifying theinitialdiagnosticworkup. at all. some cases,suchastelephoneinteractions,youwillnothaveanyphysicalexamdata familiarize yourselfwithboth. give yousamplesofbullet-styleandtraditionalnarrative-style formatssothatyoucan memorizeit,andstickwithit.Inthisbook,wewill the PN.Sochooseamethod, There aretwostylesyoucanusebothtodocumentthephysicalexamandcompose the reviewofsystems(listingonlypositivesfirst). ferential diagnosis,thetestsconducted,physicalexam, andthenthehistory If youarerunningoutoftime,startfromthebottom PN.Write downthedif- Are thetestscorrectandinrightorder? Is thedifferentialdiagnosiscorrect? Are thephysicalexamresultscomplete? Does itmakesense? Is thehistorycomplete? Insummarizingyourworkup,listamaxi- 4 ,” and“ASTALT.” You SECTION 3

Minicases

Headache / 87 Abdominal Pain / 112 Confusion/Memory Loss / 89 Constipation/Diarrhea / 116 Loss of Vision / 91 Upper GI Bleeding / 118 Depressed Mood / 92 Blood in Stool / 119 Psychosis / 93 Hematuria / 119 Dizziness / 94 Other Urinary Symptoms / 120 Loss of Consciousness / 95 Erectile Dysfunction / 122 Numbness/Weakness / 96 Amenorrhea / 123 Fatigue and Sleepiness / 98 Vaginal Bleeding / 124 Night Sweats / 100 Vaginal Discharge / 126 Insomnia / 100 Dyspareunia / 126 Sore Throat / 101 Abuse / 127 Cough/Shortness of Breath / 102 Joint/Limb Pain / 128 Chest Pain / 105 Low Back Pain / 132 Palpitations / 107 Child with Fever / 133 Weight Loss / 108 Child with GI Symptoms / 134 Weight Gain / 109 Child with Red Eye / 136 Dysphagia / 110 Child with Short Stature / 136 Neck Mass / 111 Behavioral Problems in Childhood / 137 Nausea/Vomiting / 111 MINICASES 86 exam arereviewed.Eachclinicalcaseconsistsofthreecomponents: patient. Aftereachchiefcomplaint,keypointspertinenttothehistoryandphysical examination roomoracomplaintthatyoumayhavetoelicitfromthestandardized complaint thatyoumayseeonthedoorwayinformationsheetbeforeenter likely toencounterontheStep2CS.Themaintitleofeachcaserepresentsachief In thissection,wewillattempttocovermostoftheclinicalcasesthatyouare figure outthedifferentialdiagnosisandworkup. we suggestthatyoutaketurnsreadingthevignettealoudandalloweachotherto physical findingsforeachdiagnosis.Ifyouarestudyingwithapartneroringroup, to figureoutthediagnosisandworkup.Thinkthroughsupportinghistory If youarestudyingbyyourself,wesuggestthatreadthevignetteandthentry from whichtochooseintheexam. chief complaint,whereasthesumof The sumofthe time ornotatall. order ofpriority. Inclinicalpractice,manytestsmaybeperformed atthesame Note thatthediagnostictestsinthirdcolumnaregenerallylistedrough Workup: are notprovided. pears inboldface.Thesupportinghistoryandphysicalfindingsforeachdiagnosis Differential: tives. Presentation: Themaindiagnosticteststhatshouldbeconsideredforeachdisease. Differential Anappropriatedifferentialdiagnosis;themostlikelydiagnosisap- Abriefclinicalvignettewithsomepertinentpositivesandnega- columnwillgiveyouawidedifferentialdiagnosisforthe Workup columnwillgiveyouapooloftests MINICASES 87 CT—head MRI—brain LP—CSF analysis CBC CT—head MRI—brain LP—CSF analysis ESR ESR CBC CRP artery biopsy Temporal Doppler U/S—carotid MRI—brain LP—CSF analysis CBC ESR (TMJ) disorder cell arteritis) Cluster headache Pseudotumor cerebri CNS vasculitis Partial seizure Intracranial neoplasm Cluster headache Migraine headache Tension Intracranial neoplasm Pseudotumor cerebri arteritis (giant Temporal Migraine Cluster headache headache Tension Meningitis Carotid artery dissection Pseudotumor cerebri neuralgia Trigeminal Intracranial neoplasm joint Temporomandibular Migraine (complicated) headache Tension same time every night for the past week and last for same time every night for the past week 45 minutes. right tem- 65 yo F presents with severe, intermittent blurred vision in her right eye, poral headache, fever, and pain in her jaw when chewing. associated with nausea and vomiting. She has a fam- associated with nausea and vomiting. ily history of migraine. head- 26 yo M presents with severe right temporal eye aches associated with ipsilateral rhinorrhea, tearing, and redness. Episodes have occurred at the 21 yo F presents with several episodes of throbbing Before on- left temporal pain that last for 2–3 hours. visual field set, she sees flashes of light in her right right side of and feels weakness and numbness on the her body for a few minutes. Her headaches are often HEADACHE Key Physical Exam neurologic exam, including fundu- head; ENT inspection; complete signs; inspection and palpation of entire Vital scopic exam. Presentation Differential Workup esthesias, visual stigmata, weakness, numbness, ataxia, photophobia, dizziness, auras, neck stiffness); nausea/vomit- dizziness, auras, neck stiffness); weakness, numbness, ataxia, photophobia, esthesias, visual stigmata, factors (stress, fatigue, menses, sinusitis symptoms; exacerbating recent trauma, dental surgery, ing, jaw claudication, exercise, certain foods) family history of headache; history of (rest, medications); patient and and alleviating factors trauma. Key History (is it the “worst quality (dull vs. stabbing), intensity location (unilateral vs. bilateral), Onset (acute vs. chronic), symptoms (par- sleep?), presence of associated neurologic duration, timing (does it disturb headache of their life”?), MINICASES 88 Workup Differential Presentation HEADACHE She istakingOCPs. vomiting, andblurredvisionforthepast2–3weeks. 18 yoobeseFpresentswithapulsatileheadache, confusion, photophobia,andnuchalrigidity. 25 yoMpresentswithhighfever, severeheadache, gidity. vomiting, confusion,lefthemiplegia,andnuchalri- 35 yoMpresentswithsuddensevereheadache, decreased herintakeofcaffeine. periencing significantstressinherlifeandrecently typically towardtheendofherworkday. Sheisex- squeezing headachesthatoccur3–4timesaweek, ofbilateral 50 yoFpresentswithrecurrentepisodes allergies. frontal andmaxillarysinuses.Shehasahistoryof nasal discharge.Thereispainonpalpationofthe 30 yoFpresentswithfrontalheadache,fever, and (cont’d) Intracranial neoplasm Pseudotumor cerebri Cluster headache Hypertension Caffeine oranalgesic Depression Migraine Tension headache Intracranial neoplasm Meningitis Tension headache Migraine Sinusitis Intracranial neoplasm Intracranial venous Meningitis Cluster headache Migraine Tension headache Pseudotumor cerebri Intracranial orepidural Sinusitis/encephalitis Subarachnoid hemorrhage Migraine Meningitis Intracranial neoplasm Acute hypertension Intracranial venous Vertebral arterydissection Intracranial hemorrhage Meningitis/encephalitis Migraine Subarachnoid withdrawal thrombosis abscess thrombosis hemorrhage LP—CSF analysis CT—head ESR Electrolytes CBC LP—CSF analysis CT—sinus XR—sinus CBC LP—opening pressureand CT—head CBC Urine hCG LP—CSF analysis(cell MRI—brain CT—head CBC Urine toxicology PT/PTT/INR CBC LP—CSF analysis Noncontrast CT—head CSF analysis culture) specific pathogens, Gram stain,PCRfor count, protein,glucose, MINICASES 89 12 CBC VDRL/RPR Serum B TSH MRI—brain CT—head LP—CSF analysis CBC ESR MRI—brain deficiency infarct”) dementia hydrocephalus hematoma 12 Neurosyphilis Hypothyroidism Vascular (“multi- Vascular disease Alzheimer’s Normal pressure Chronic subdural Intracranial neoplasm Depression B Trigeminal neuralgia Trigeminal headache Tension Migraine Cluster headache TMJ disorder Intracranial neoplasm (cont’d) deficiency; family history of Alzheimer’s disease or other neurologic disorders. deficiency; family history of Alzheimer’s stroke (stepwise decline in cognitive function). 81 yo M presents with progressive confusion for the 81 yo M presents with progressive confusion and past several years accompanied by forgetfulness diabe- clumsiness. He has a history of hypertension, tes mellitus, and 2 strokes with residual left hemi- paresis. His mental status has worsened after each ter and occurs while he is shaving. Each episodeter and occurs while lasts 2–4 minutes. 57 yo M c/o daily pain in the right cheek for the past 57 yo M c/o daily pain electric and stabbing in charac- month. The pain is HEADACHE 12 CONFUSION/MEMORY LOSS CONFUSION/MEMORY Vital signs; complete neurologic exam, including mini-mental status exam and gait; general physical exam, including exam, including mini-mental status exam and gait; general physical signs; complete neurologic Vital and extremities. heart, lungs, abdomen, ENT, Presentation Differential Workup (acute vs. chronic/gradual onset); associated symptoms (constitutional, incontinence, ataxia, hypothyroid symp- (acute vs. chronic/gradual onset); associated (waxing/waning level of alertness); falls, medications (and recent medication toms, depression); screen for delirium vascular disease, syphilis, HIV risk factors, alcohol use, or vitamin changes); history of stroke or other atherosclerotic B Key Physical Exam Key History Must when available. Detailed time course of cognitive deficits include history from family members/caregivers Presentation Differential Workup MINICASES 90 Workup Differential Presentation CONFUSION/MEMORY LOSS response. are associatedwithmyoclonus,ataxia,andastartle ory impairmentforthepast2months.Hissymptoms in mentalstatus,inabilitytoconcentrate,andmem- 55 yoMpresentswitharapidlyprogressivechange and urinaryincontinenceforthepast6months. 72 yoMpresentswithmemoryloss,gaitdisturbance, the pastfewyears. phone). Theproblemhasprogressedgraduallyover (eg, bathing,dressing,managingmoney, usingthe and difficultyperformingsomeofherdailyactivities forgets phonenumbers,losesherwaybackhome) 84 yoFbroughtbyhersonc/oforgetfulness(eg, (cont’d) Vascular dementia Alzheimer’s disease Normal pressure B Intracranial neoplasm Normal pressure Chronic subdural Hypothyroidism Depression Vascular dementia Alzheimer’s disease B Delirium Depression Intracranial neoplasm Chronic subdural Normal pressure Wernicke’s dementia Lewy body Vascular dementia Creutzfeldt-Jakob disease B Depression Intracranial neoplasm Chronic subdural Neurosyphilis Neurosyphilis Hypothyroidism Neurosyphilis 12 12 12 hydrocephalus hydrocephalus hematoma hematoma hydrocephalus encephalopathy hematoma deficiency deficiency deficiency LP—CSF analysis CT—head MRI—brain (preferred) TSH Brain biopsy LP—CSF analysis EEG CT—head MRI—brain (preferred) VDRL/RPR TSH VDRL/RPR LP—opening pressureand MRI—brain CT—head Serum B VDRL/RPR CBC Serum B Electrolytes, calcium CBC Serum B CSF analysis 12 12 12 MINICASES 91 Fluorescein angiogram Echocardiography Doppler U/S—carotid Intraocular tonometry ESR artery biopsy Temporal CBC Glucose CBC Electrolytes CPK-MB, troponin Echocardiography ECG MRI—brain Doppler U/S—carotid CT—head CBC Electrolytes MRI—brain LP—CSF analysis glaucoma cell arteritis) hyponatremia) Retinal artery occlusion Retinal vein occlusion Acute angle-closure Retinal detachment arteritis (giant Temporal attack ischemic Transient Arrhythmia Delirium Angina Subdural hematoma SIADH (causing Creutzfeldt-Jakob disease Intracranial neoplasm (cont’d) eye, palpitations, and shortness of breath. He has a history of atrial fibrillation and cataracts in his right eye. He has no eye pain, discharge, redness, or pho- tophobia. He has not experienced headache, weak- ness, or numbness. 73 yo M presents with acute loss of vision in his left 55 yo F presents with gradual altered mental status 55 yo F presents with gradual altered hit her weeks ago she slipped, and headache. Two for 2 head on the ground, and lost consciousness minutes. phoresis, and weakness. 70 yo insulin-dependent diabetic M presents with 70 yo insulin-dependent episodes dizziness, palpitations, dia- of confusion, LOSS OF VISION CONFUSION/MEMORY LOSS LOSS CONFUSION/MEMORY Vital signs; cardiovascular, HEENT, funduscopic, and neurologic exams. HEENT, signs; cardiovascular, Vital Presentation Differential Workup Acute vs. chronic, progression, ability to see light; associated symptoms (eye pain, discharge, itching, tearing, photo- Acute vs. chronic, progression, ability numbness, floaters, sparks); history of cardiac, rheumatic, thrombotic, autoim- phobia, redness, headache, weakness, medications, trauma. mune, or neurologic disorders; jaw claudication, Key Physical Exam Key History Presentation Differential Workup MINICASES 92 Workup Differential Presentation status exam),insight,andjudgment. behavior, affect,thoughtprocess,content,cognition(measuredbythe30-pointmini-mental speech,mood, Vital signs;headandneckexam;neurologicmentalstatusexam,includingdocumentationofappearance, Key PhysicalExam medications. disorders;priorepisodes; agitation orretardation;familyhistoryofmood cidality, social function,decreasedinterest(anhedonia),energy, decreasedconcentration,psychomotor Onset, duration;sleeppatterns;appetiteandweightchange;drugalcoholuse;lifestresses,excessiveguilt,sui- Key History DEPRESSED MOOD her presentation. and racingthoughtsapproximately6monthsbefore energy, sexualpromiscuity, irresponsiblespending, butreportsseveralweeksofincreased sive episode dation. Shedoesnotidentifyatriggerforthedepres- awakening, excessiveguilt,andpsychomotorretar- the past2months,accompaniedbyearly-morning 26 yoFpresentswitha6.5-lb(2.9-kg)weightlossin admits toincreasedalcoholuseinthepastmonth. has made2previoussuicideattempts.Shefurther inthepast,firsther20s,and 5 similarepisodes she thinksconstantlyaboutdeath.Shehassuffered fatigue, insomnia,andanhedonia.Shestatesthat 42 yoFpresentswitha4-weekhistoryofexcessive ing dinner. at thesupermarketorstandinginkitchenmak- admits tothinkinghehasseenhisdeadwifeinline He cannotenjoytimewithhisgrandchildrenand (6.8-kg) weightloss,allfollowinghiswife’s death. spells, excessivesleep,poorhygiene,anda15-lb 68 yoMpresentswitha2-monthhistoryofcrying Major depressivedisorder Adjustment disorderwith Normal bereavement Schizoaffective disorder Major depressivedisorder Cyclothymic disorder Bipolar IIdisorder Bipolar Idisorder Dysthymic disorder Substance-induced mood Major depressivedisorder Depressive disordernot Schizoaffective disorder depressed mood disorder otherwise specified with psychoticfeatures Urine toxicology CBC TSH Physical exam Urine toxicology Mental statusexam Physical exam Urine toxicology CBC TSH alcohollevel Blood Beck Depression Mental statusexam Physical exam Beck Depression Inventory Inventory MINICASES 93 Inventory Electrolytes Urine toxicology Mental status exam TSH CBC Electrolytes, BUN/Cr AST/ALT Mental status exam Beck Depression TSH CBC Electrolytes Mental status exam Urine toxicology TSH CBC to a general medical condition to a general medical condition psychosis features psychosis to a general medical condition psychotic features personality disorder Psychotic disorder due Substance-induced Depression with psychotic Substance-induced Brief psychotic disorder Schizophreniform disorder Schizophrenia Psychotic disorder due Schizoaffective disorder Mood disorder with Schizophrenia Schizophreniform disorder Psychotic disorder due Schizophrenia Schizoid or schizotypal Schizophreniform disorder of weight loss, early-morning awakening, decreased motivation, and overwhelming feelings of guilt. 48 yo F presents with a 1-week history of auditory hallucinations that state, “I am worthless” and “I should kill myself.” She also reports a 2-week history past 2 days and hearing loud voices when she is past 2 days and hearing loud voices any- alone in her room. She has never experienced ingested an thing similar in the past. She recently unknown substance. recently been hearing a voice from his television set recently been hearing a voice from his telling him to “guard against the evil empire.” bed for the 28 yo F c/o seeing bugs crawling on her in high school. In college, he started to suspect his in high school. In college, he started stopped going roommate of bugging the phone. He were to classes because he felt that his professors no one else saying horrible things about him that room and has noticed. He rarely showered or left his 19 yo M c/o receiving messages from his television 19 yo M c/o receiving messages from many friends set. He reports that he did not have PSYCHOSIS grooming, oddpoorly fitting clothing). or Presentation Differential Workup (disorganized speech or thought patterns, paranoia); age at first symptoms and/or hospitalization; previous psychiatric first symptoms and/or hospitalization; or thought patterns, paranoia); age at (disorganized speech and substance use. medications; alcohol Key Physical Exam to general appearance (eg, poor physical exam, pay particular attention signs; mental status exam; during Vital Key History behavior), negative thoughts, disorganized or catatonic (delusions, hallucinations, disorganized Positive symptoms cognitive symptoms decreased speech/thought), affect, social withdrawal, decreased motivation, symptoms (blunted MINICASES 94 Workup Differential Presentation gait, hearing,andWeber andRinnetests;ENTexam;cardiovascularexam. Vital signs;completeneurologicexam,includingRombergtest,nystagmus,tilttest(eg,Dix-Hallpikemaneuver), Key PhysicalExam jury; medications;historyofatheroscleroticvasculardisease. positioning, followingheadtrauma);otherassociatedsymptoms(visualdisturbance,URI,nausea);neckpainorin- context(occurswith Lightheadedness vs.vertigo,±auditorysymptoms(hearingloss,tinnitus),durationofepisodes, Key History DIZZINESS A tilttestresultsinnystagmusandnausea. She feelsthattheroomisspinningaroundherhead. 44 yoFc/odizzinessonmovingherheadtotheleft. hydrochlorothiazide 2daysago. steadiness. Hehashypertensionandwasstartedon 65 yoMpresentswithposturaldizzinessandun- takes furosemideforhypertension. and hasseverediarrheathatstarted2daysago.She 55 yoFc/odizzinessforthepastday. Shefeelsfaint past week. tigo, tinnitus,nausea,andhearinglosswithinthe ofver-35 yoFpresentswithintermittentepisodes Acoustic neuroma Benign positionalvertigo Labyrinthitis Vestibular neuronitis Ménière’s disease Ménière’s disease Labyrinthitis Vestibular neuronitis Benign positionalvertigo Acute renalfailure Brain stemorcerebellar Benign positionalvertigo Labyrinthitis Vestibular neuronitis Drug-induced orthostatic Vertebrobasilar Benign positionalvertigo Labyrinthitis Vestibular neuronitis Orthostatic hypotension tumor hypotension insufficiency (diarrhea, diureticuse) due todehydration MRI—brain VDRL/RPR (syphilisis CBC Audiogram MRI—brain Dix-Hallpike maneuver MRI—brain Echocardiography Electrolytes CBC Orthostatic vitalsigns Stool leukocytes Rectal exam,stoolfor Electrolytes CBC Orthostatic vitalsigns Dix-Hallpike maneuver disease) a causeofMénière’s occult blood MINICASES 95 CT—head LP—CSF analysis ECG CBC Electrolytes, glucose Urine toxicology EEG MRI—brain CBC Electrolytes Electronystagmography MRI/MRA—brain Audiogram Electronystagmography MRI/MRA—brain seizure cervical spine disease or cervical spine disease injury insufficiency insufficiency Hypoglycemia Generalized tonic-clonic Convulsive syncope Substance abuse/overdose Malingering Vestibular neuronitis Vestibular Labyrinthitis disease Ménière’s Benign positional vertigo associated with Vertigo Vertebrobasilar Labyrinthitis neuronitis Vestibular disease Ménière’s Acoustic neuroma Vertebrobasilar (cont’d) limbs, bit his tongue, and lost control of his bladder. limbs, bit his tongue, and lost control of his bladder. He was subsequently confused after regaining con- sciousness (as witnessed by his colleagues). 26 yo M presents after falling and losing conscious- ness at work. He had rhythmic movements of the ago. 55 yo F c/o dizziness that started this morning and 55 yo F c/o dizziness that started this and has of “not hearing well.” She feels nauseated She had a URI 2 days vomited once in the past day. She had a URI 2 days ago and has experienced no She had a URI 2 days hearing loss. 55 yo F c/o dizziness that started this morning. She 55 yo F c/o dizziness vomited once in the past day. is nauseated and has LOSS OF CONSCIOUSNESS LOSS OF CONSCIOUSNESS DIZZINESS Presentation Differential Workup of heart disease, arrhythmia, hypertension, or diabetes; alcohol and drug use. of heart disease, arrhythmia, hypertension, Key Physical Exam signs, including orthostatics; complete neurologic exam; carotid and cardiac exam; lung exam; exam of the Vital lower extremities. Key History context (ex- lightheadedness), (nausea, diaphoresis, palpitations, pallor, Presence or absence of preceding symptoms painful, or claustrophobic experience; dehydration); associated tongue biting ertional, postural, traumatic; stressful, prolonged confusion; dyspnea or pulmonary embolism risk factors; history or incontinence, tonic-clonic movements, Presentation Differential Workup MINICASES 96 rsnainDfeeta Workup Differential Presentation Vital signs;neurologicandmusculoskeletalexams;relevantvascular exam. Key PhysicalExam disease. pain); constitutionalsymptoms,otherneurologicsymptoms;historyofdiabetes,alcoholism,atheroscleroticvascular Distribution (unilateral,bilateral,proximal,distal),duration,±progression,pain(especiallyheadache,neckorback Key History Workup Differential Presentation LOSS OFCONSCIOUSNESS NUMBNESS/WEAKNESS mellitus, andheavysmoking. partment. Hehasahistoryofhypertension,diabetes resolved bythetimehegottoemergencyde- and righthandweakness.Hissymptomshadtotally slurred speech,rightfacialdroopingandnumbness, of 68 yoMpresentsfollowinga20-minuteepisode includes acoronaryarterybypassgraft. passing outbutrecentlyhadpalpitations.Hishistory ness forafewseconds.Hehadnowarningbefore 65 yoMpresentsafterfallingandlosingconscious- and diabetesmellitus. past medicalhistoryissignificantforhypertension He experiencedtransientlossofconsciousness.His 55 yoMc/ofallingafterfeelingdizzyandunsteady. (cont’d) Seizure Syncope (othercauses) Severe aorticstenosis Cardiac arrhythmia Pulmonary embolism MI Stroke Syncope (vasovagal,other Cardiac arrhythmia Hypoglycemia Drug-induced orthostatic Facial nervepalsy Stroke Seizure Hypoglycemia Transient ischemicattack Pulmonary embolism (causing syncope) causes) syncope) hypotension (causing (TIA) Echocardiography Electrolytes, glucose CBC Holter monitoring ECG CTA—chest withIV V/Q scan ECG CT—head Echocardiography Electrolytes, glucose CBC Orthostatic vitalsigns EEG Echocardiography Doppler U/S—carotid MRI—brain ECG lipidpanel Electrolytes, glucose CBC CT—head CT—head D -dimer contrast MINICASES 97 12 12 1c electrophoresis test HbA ESR Calcium Serum B UA Serum and urine protein Nerve conduction studies (edrophonium) Tensilon Serum B CBC ESR VDRL/RPR MRI—brain, spine LP—CSF analysis Retinal evoked potentials CT—head CBC Electrolytes PT/PTT/INR Fasting lipid panel MRI—brain Doppler U/S—carotid Echocardiography ECG CBC Electrolytes CPK LP—CSF analysis MRI—spine EMG deficiency canal neuropathy neuropathy hematoma 12 Hypocalcemia Hyperventilation Paraproteinemia/myeloma Multiple sclerosis Stroke Conversion disorder Malingering CNS tumor Neurosyphilis Syringomyelia CNS vasculitis Diabetic peripheral Alcoholic peripheral B Stroke TIA Seizure Intracranial neoplasm Subdural or epidural Guillain-Barré syndrome Multiple sclerosis Polymyositis Myasthenia gravis Peripheral neuropathy in the vertebral Tumor (cont’d) the feet. 55 yo M presents with tingling and numbness in his hands and feet (glove-and-stocking distribution) for the past 2 months. He has a history of diabetes mellitus, hypertension, and alcoholism. There is de- and position sense in creased soft touch, vibratory, sion, and double vision. She reports feeling “electric sion, and double vision. She reports feeling head. shocks” down her spine upon flexing her 30 yo F presents with weakness, loss of sensation, 30 yo F presents with weakness, loss this morning. and tingling in her left leg that started decreased vi- She is also experiencing right eye pain, 33 yo F presents with ascending loss of strength in 33 yo F presents with ascending loss She had a re- her lower legs over the past 2 weeks. cent URI. Babinski’s sign is present on the right. He has a his- sign is present Babinski’s diabetes mellitus, and heavy tory of hypertension, smoking. 68 yo M presents with slurred speech, right facial 68 yo M presents and right hand weakness. drooping and numbness, NUMBNESS/WEAKNESS Presentation Differential Workup MINICASES 98 rsnainDfeeta Workup Differential Presentation testing. blood dominal, neurologic,andextremity(pallor, coolnessatdistalextremities)exams;considerrectalexamandoccult Vital signs; ENTexam(conjunctivalpallor, oropharynx/palate,lymphadenopathy, thyroidexam);heart,lung,ab- Key PhysicalExam toms; symptomsofthyroiddisease;historybleedingoranemia;medications;alcohol,caffeine,anddruguse. other emotionalproblems;lifestylechanges,shiftchangesatwork;diet,weightchanges;constitutionalsymp- Duration; sleephygiene,snoring,wakingupchoking/gasping,witnessedapnea;overexertion;stress,depression, or Key History Workup Differential Presentation NUMBNESS/WEAKNESS FATIGUE ANDSLEEPINESS been havingfightswithherhusbandaboutmoney. of havingsuicidalthoughts.Shelostherjobandhas 40 yoFc/ofeelingtired,hopeless,andworthless a 20-lb(9-kg)weightgainoverthesameperiod. works asasecretary. Shehasahistoryoffatigueand gressively worsened,andarerelievedwithrest.She 5 months.Hersymptomsareconstant,havepro- thumb, indexfinger, andmiddlefingerforthepast 56 yoobeseFc/otinglingandnumbnessofher clonic seizurethatresolvedwithinafewhours. 25 yoMpresentswithhemiparesisafteratonic- ing. droopy eyelidsatnightwithnormalizationbymorn- 40 yoFpresentswithoccasionaldoublevisionand (cont’d) Carpal tunnelsyndrome Malingering Complicated migraine Stroke TIA Todd’s paralysis Amyotrophic lateral Intracranial neoplasm Multiple sclerosis Horner’s syndrome Myasthenia gravis Anemia Hypothyroidism Adjustment disorder Depression Medial epicondylitis Overuse injuryofmedian sclerosis or VI compressing CNIII,IV, nerve hypothyroidism secondary to Phalen’s maneuverand Doppler U/S—carotid MRI—brain EEG Electrolytes CBC EMG MRI—brain CT—chest CXR Serum AChreceptor Tensilon (edrophonium) Beck Depression HIV/STD testing TSH CBC CBC TSH Nerve conductionstudies antibodies test Inventory Tinel’s sign MINICASES 99 4 , FT 3 1c 1c glucose Inventory occult blood calcium CBC Fasting glucose HbA UA CBC Electrolytes, BUN/Cr, Nocturnal pulse oximetry Polysomnography CBC TSH Nocturnal pulse oximetry Polysomnography ECG Glucose tolerance test HbA CBC Colonoscopy Barium enema CT—abdomen/pelvis TSH, FT CBC TSH Urine toxicology Beck Depression Rectal exam, stool for CBC Electrolytes, BUN/Cr, AST/ALT TSH disorder disorder disorder Sleep apnea Depression Anemia Obstructive sleep apnea Hypothyroidism Chronic fatigue syndrome Narcolepsy Diabetes mellitus Atypical depression Primary polydipsia Diabetes insipidus Shift work sleep disorder Hypothyroidism Depression Diabetes Anemia Posttraumatic stress Posttraumatic stress Depression Generalized anxiety Psychotic or delusional Hypothyroidism Colon cancer Hypothyroidism Renal failure Hypercalcemia Depression (cont’d) 35 yo M policeman c/o feeling tired and sleepy dur- to the night shift last week. He changed ing the day. 20 yo M presents with fatigue, thirst, increased ap- petite, and polyuria. 50 yo obese F presents with fatigue and daytime 50 yo obese F presents with fatigue 3–4 times sleepiness. She snores heavily and naps has hyper- per day but never feels refreshed. She also tension. 40 yo F presents with fatigue, weight gain, sleepi- 40 yo F presents with fatigue, weight dry skin. ness, cold intolerance, constipation, and 55 yo M presents with fatigue, weight loss, and con- 55 yo M presents with fatigue, weight cancer. stipation. He has a family history of colon year ago. Since then, he has avoided the mall and year ago. Since then, night. has not gone out at 44 yo M presents with fatigue, insomnia, and night- 44 yo M presents with that he witnessed in a mall 1 mares about a murder FATIGUE AND SLEEPINESS AND SLEEPINESS FATIGUE Presentation Differential Workup MINICASES 100 irritability). or lifestyle(jetlagshiftwork),stressors,sleephygiene; presenceofpsychiatricsymptoms(eg,grandiosedelusions, loudsnoring,nightmares,anddepression;caffeine,alcohol,medication, andrecreationaldruguse;work including betes (polyuria);evidenceofacommonsleepdisorder(eg, sleepapnea,restlesslegsyndrome);associatedsymptoms, ening); daytimesleepiness;othermedicalproblemskeeping patientawakeatnight,suchasarthritis(pain)ordia- Primary vs.secondary, duration,description(troublefallingasleepvs.multipleawakeningsearly-morningawak- Key History Workup Differential Presentation exam; abdominalexamforhepatosplenomegaly;skinmusculoskeletaljointpain. Vital signs;HEENTexam,includinginspectionofthethroatandotherareasforlymphadenopathy;heartlung Key PhysicalExam status, travelhistory. sick contacts,exposuretohigh-riskpopulationssuchasprisonersorhomelesspeople;menstrualhistory, menopausal significant riskfactors(eg,travelingtoareaswithendemicinfections,IVdruguse);alcoholhistory, sexualexposure, lymphadenopathy, rash,malaise,weightloss,itching,diarrhea, nausea/vomiting,earlysatiety, anorexia;presenceof tions); associateddiseasesandsymptoms(fever, recentURIs,associatedcough,hemoptysis,pleuriticchestpain); Onset, duration,severity, frequency, medica- timing,patterns(escalating,waxing,waning),precipitants(eg,food, Key History NIGHT SWEATS INSOMNIA ness ofbreath. tional weightloss,palpitations,diarrhea,andshort- 45 yoFpresentswithexcessivesweating,uninten- emigrated fromtheAfricansubcontinent. swollen glandsof1month’s duration.Herecently 30 yoMpresentswithnightsweats,cough,and Acute HIVinfection Tuberculosis Tuberculosis Carcinoid syndrome Pheochromocytoma Hyperthyroidism Hyperthyroidism Leukemia Lymphoma CBC PPD/QuantiFERON Gold TSH, FT TSH, FT HIV antibody Sputum Gramstain,acid- CXR PPD CBC 5-HIAA 24-hour urinary fast stain,andculture catecholamines 4 4 MINICASES 101 Mental status exam TSH CBC Polysomnography Polysomnography Mental status exam Urine toxicology CBC TSH CBC TSH Polysomnography ECG depressive disorder rhythm sleep disorder rhythm sleep disorder depressive disorder insomnia rhythm sleep disorder depressive disorder hypersomnia Insomnia related to major Primary hypersomnia Insomnia with circadian Insomnia related to major Stress-induced insomnia Caffeine-induced Insomnia with circadian Insomnia related to major Obstructive sleep apnea Daytime fatigue in primary Insomnia with circadian (cont’d) . and cannot return to sleep. She also reports . and cannot return to sleep. She also M . A an unintentional weight loss of 8 lbs (3.6 kg) and an an unintentional weight loss of 8 lbs (3.6 liked to do. inability to enjoy the things she once 33 yo F c/o 3 weeks of fatigue and trouble sleeping. 33 yo F c/o 3 weeks of fatigue and trouble wakes up at She states that she falls asleep easily but 3 wife reports that he snores loudly. 3–4 cups of coffee a day. 3–4 cups of coffee a of poor 55 yo obese M presents with several months His sleep, daytime fatigue, and morning headaches. 25 yo F presents with a 3-week history of difficulty 25 yo F presents with sleeps 7 hours per night without falling asleep. She She recently began college nightmares or snoring. with her boyfriend. She drinks and is having trouble SORE THROAT INSOMNIA on splenomegaly), and skin exams. Duration, fever, other ENT symptoms (ear pain, nasal or sinus congestion), odynophagia, swollen glands, ± cough, Duration, fever, rash; sick contacts, HIV risk factors. Key Physical Exam signs; ENT exam, including oral thrush, tonsillar exudate, and lymphadenopathy; lung, abdominal (focusing Vital Key History Presentation Differential Workup Key Physical Exam Key Physical signs; mental status exam; thyroid exam. Vital MINICASES 102 heart failure;allergies;medications (especiallyACEinhibitors). burn); exacerbatingandalleviating factors,exposures;smokinghistory;historyoflung disease,posttussiveemesis,or ence ofhemoptysis;associatedsymptoms(constitutional, URI,postnasaldrip,dyspnea,wheezing,chestpain,heart- Acute/subacute vs.chronic,increasedfrequencyofcough ifchronic,timing;presence/descriptionofsputum,pres- Key History Workup Differential Presentation SORE THROAT COUGH/SHORTNESS OFBREATH 46 yoFpresentswithfeverandsorethroat. sharing needles. weight loss.HehasahistoryofIVdrugabuseand 26 yoMpresentswithsorethroat,fever, rash,and recently experiencedsimilarsymptoms. cervical lymphadenopathyandarash.Herboyfriend also reportsepigastricandLUQdiscomfort.Shehas tigue, andlossofappetiteforthepastweek.She 26 yoFpresentswithsorethroat,fever, severefa- (cont’d) Viral pharyngitis Hepatitis Infectious mononucleosis HIV, acuteretroviral Secondary syphilis Acute HIVinfection Viral orbacterial Hepatitis Infectious mononucleosis Infectious mononucleosis Acute HIVinfection Mycoplasma Pharyngitis (bacterialor Secondary syphilis Streptococcal tonsillitis/ syndrome pharyngitis viral) scarlet fever pneumonia CD4 count andviral HIV antibody CBC withperipheral VDRL/RPR Anti-EBV antibodies andviral HIV antibody AST/ALT// Throat culture Monospot test CBC withperipheral HIV antibody andviral HIV antibody Serologic test(cold CBC Monospot test Throat swabforculture AST/ALT/bilirubin/ VDRL/RPR Throat culture Monospot test load smear load smear load Mycoplasma agglutinin titer)for antigen and rapidstreptococcal alkaline phosphatase MINICASES 103 antigen Legionella culture stain and culture Mycoplasma pneumoniae culture, and cytology test culture CT—chest PPD ANCA Bronchoscopy Echocardiography PFTs CT—chest PPD CBC Sputum Gram stain and CXR CT—chest ECG PPD CBC Induced sputum Gram CXR IgM detection for Urine CBC Sputum Gram stain, CXR CBC CXR Peak flow measurement PFTs Methacholine challenge CBC Sputum Gram stain and CXR granulomatosis) (bronchitis) (“postinfectious”) bronchitis Vasculitis (eg, Wegener’s (eg, Wegener’s Vasculitis Interstitial lung disease CHF Tuberculosis Pneumonia COPD exacerbation Lung abscess Lung cancer Tuberculosis Pericarditis Atypical pneumonia Reactive airway disease URI-associated cough Postnasal drip GERD Lung cancer Tuberculosis Lung abscess COPD Asthma GERD Bronchitis Pneumonitis Foreign body COPD—chronic Bronchiectasis Lung cancer (cont’d) 6 months accompanied by hemoptysis, dyspnea, weakness, and weight loss. He is a heavy smoker. 65 yo M presents with worsening cough for the past 25 yo F presents with 2 weeks of nonproductive cough. Three weeks ago she had a sore throat and a runny nose. 58 yo M presents with 1 week of pleuritic chest pain, 58 yo M presents with 1 week of pleuritic yellow spu- chills, and cough with purulent fever, tum. He is a heavy smoker with COPD. each year over the past 2 years. She is a heavy each year over the past 2 years. She smoker. 56 yo F presents with shortness of breath and a pro- 56 yo F presents with shortness of breath 3 months ductive cough that has lasted for at least 30 yo M presents with shortness of breath, cough, 30 yo M presents He has had worsen in cold air. and wheezing that several such episodes in the past 4 months. COUGH/SHORTNESS OF BREATH OF BREATH COUGH/SHORTNESS Presentation Differential Workup Key Physical Exam Key Physical nodes, mucosa, oropharynx, heart, lungs, lymph signs ± pulse oximetry; exam of nasal (clubbing, and extremities Vital cyanosis, edema). MINICASES 104 Workup Differential Presentation COUGH/SHORTNESS OFBREATH frothy sputum. duration andacoughthatisaccompaniedbypink, 60 yoMpresentswithworseningdyspneaof6hours’ 3 pillows.Healsoreportsexertionaldyspnea. lying downatnightandimprovedbyproppingupon 50 yoMpresentswithacoughthatisexacerbatedby active tuberculosis. partners andwasrecentlyexposedtoapatientwith He hashadunprotectedsexwithmultiplesexual 35 yoMpresentswithshortnessofbreathandcough. with tuberculosispatientsatwork. night sweats,andfever. Shehasahistoryofcontact weeks’ durationaccompaniedbyweightloss,fatigue, 34 yoFnursepresentswithworseningcoughof6 smoking 2daysago. and stoppedusinghisinhalerslastweek.He forthepast3days.HehasCOPD tum production 55 yoMpresentswithincreaseddyspneaandspu- (cont’d) HIV/AIDS Metastatic cancer Lymphoma Vasculitis Lung abscess Pneumonia Tuberculosis CHF URI Pneumonia Lung cancer COPD exacerbation Pneumonia Asthma Arrhythmia Mitral valvestenosis CHF Pulmonary edema Postnasal drip COPD Pulmonary fibrosis GERD Cardiac valvulardisease CHF CHF (cardiomyopathy) Acute HIVinfection Asthma Bronchitis Pneumonia (including Tuberculosis Sarcoidosis (bronchitis) Pneumocystis jiroveci ) Bronchoscopy CT—chest CXR Sputum Gramstain,acid- PPD/QuantiFERON Gold CBC Echocardiography CT—chest Sputum Gramstainand PFTs ABG CXR CBC BNP PFTs ABG CBC CXR ECG CT—chest BNP PFTs Echocardiography ECG CXR CBC Echocardiography HIV antibody CXR Sputum Gramstain,acid- PPD/QuantiFERON Gold CBC Lymph biopsy node HIV antibody fast stain,andculture culture and culture fast stain,silver MINICASES 105 contrast count and peripheral smear -dimer -dimer Helical CT CXR CPK-MB, troponin ECG IV with CTA—chest D D CBC Electrolytes Echocardiography Cardiac catheterization CBC with reticulocyte LDH ABG ECG Barium swallow Upper endoscopy Esophageal pH monitoring ECG CPK-MB, troponin × 3 CXR chest syndrome (MI) MI Angina Angina Costochondritis Aortic dissection Pericarditis Pulmonary embolism Pneumothorax Sickle cell disease—acute Pulmonary embolism Pneumonia MI Pneumothorax Aortic dissection GERD Esophagitis Peptic ulcer disease Esophageal spasm Myocardial infarction GERD tion that occurs after heavy meals and when lying down. Her symptoms are relieved by antacids. 45 yo F presents with a retrosternal burning sensa- of sickle cell disease and multiple hospitalizations for of sickle cell disease and multiple hospitalizations pain and anemia management. 20 yo African American F presents with acute onset 20 yo African American F presents with has a history of severe chest pain for a few hours. She heavy chest pain that has lasted for 30 minutes and heavy chest pain that has lasted for 30 by radiates to the left arm. The pain is accompanied a history of dyspnea, diaphoresis, and nausea. He has hypertension, hyperlipidemia, and smoking. 60 yo M presents with sudden onset of substernal 60 yo M presents with sudden onset CHEST PAIN CHEST Vital signs ± BP in both arms; complete cardiovascular exam (JVD, PMI, chest wall tenderness, heart sounds, pulses, wall tenderness, heart sounds, pulses, exam (JVD, PMI, chest signs ± BP in both arms; complete cardiovascular Vital (inspection for signs of DVT). exams; lower extremity exam edema); lung and abdominal Presentation Differential Workup factors (especially medications); history of similar symptoms; known heart or lung disease or history of diagnostic known heart or lung disease or medications); history of similar symptoms; factors (especially pulmonary embolism smoking, family history of early MI); factors (hypertension, hyperlipidemia, testing; cardiac risk recent immobilization). malignancy, coagulopathy, of DVT, risk factors (history Key Physical Exam Key History cocaine use, trauma); postprandial, positional, radiation, duration, context (exertional, severity, Location, quality, and alleviating sense of doom, fever); exacerbating (sweating, nausea, dyspnea, palpitations, associated symptoms MINICASES 106 Workup Differential Presentation CHEST PAIN surgery. mild fever. Sheisrecoveringfromhipreplacement ents withtachycardia,hypotension,tachypnea,and breath atrestandpleuriticchestpain.Shealsopres- 70 yoFpresentswithacuteonsetofshortness noted. She hadaURI1weekago.Chestwalltendernessis ens withdeepinspirationandisrelievedbyaspirin. 33 yoFpresentswithstabbingchestpainthatwors- with deepinspiration.ShehadaURI1weekago. that improveswhensheleansforwardandworsens 34 yoFpresentswithretrosternalstabbingchestpain intake. is relievedbyrestandnotrelatedtofood that lastsfor2minutesandoccurswithexercise.It 55 yoMpresentswithretrosternalsqueezingpain (cont’d) Esophageal rupture GERD Costochondritis MI Aortic dissection Pericarditis Esophagitis Esophageal spasm Stable angina Aortic dissection CHF MI Costochondritis Pneumonia Pulmonary embolism Muscle strain Pleurisy Pericarditis Pulmonary embolism MI Pneumonia Costochondritis Upper endoscopy CBC Echocardiography CXR CPK-MB, troponin ECG Cardiac catheterization Upper endoscopy/pH Exercise stresstest Electrolytes CBC CXR CPK-MB, troponin ECG CBC CXR CPK-MB, troponin ECG ESR D Doppler U/S—legs CTA—chest withIV Electrolytes, BUN/Cr, CBC CPK-MB, troponin ABG CXR ECG -dimer monitor contrast glucose MINICASES 107 catecholamines contrast echocardiography (TEE) 5-HIAA Glucose CBC Electrolytes TSH ECG Holter monitor 24-hour urinary ECG CPK-MB, troponin CXR CBC , with IV CTA—chest Transesophageal MRI/MRA—aorta Aortic angiography Upper endoscopy Hypoglycemia Cardiac arrhythmia Angina Hyperthyroidism Hyperventilation episodes Panic attack Pheochromocytoma Carcinoid syndrome Aortic dissection MI Pericarditis Esophageal rupture Esophageal spasm GERD Pancreatitis Fat embolism (cont’d) 70 yo diabetic M presents with episodes of palpita- tions and diaphoresis. He is on insulin. uncontrolled hypertension. 55 yo M presents with sudden onset of severe chest 55 yo M presents with to his back. He has a history of pain that radiates PALPITATIONS CHEST PAIN PAIN CHEST Vital signs; endocrine/thyroid exam, including exophthalmos, lid retraction, lid lag, gland size, bruit, and tremor; exam, including exophthalmos, lid retraction, lid lag, gland size, bruit, signs; endocrine/thyroid Vital complete cardiovascular exam. Presentation Differential Workup Gradual vs. acute onset/offset, context (exertion, caffeine, anxiety); associated symptoms (lightheadedness, loss of (exertion, caffeine, anxiety); associated symptoms (lightheadedness, loss Gradual vs. acute onset/offset, context of sweating, pale skin, flushing, diarrhea); hyperthyroid symptoms; history consciousness, chest pain, dyspnea, fever, hypertension, or diabetes. bleeding or anemia; history of heart disease, Key Physical Exam Key History Presentation Differential Workup MINICASES 108 tobacco, alcohol, and drug use; medications; history of cancer; blood inurineorstool. tobacco, alcohol,anddruguse;medications;historyofcancer; blood symptoms; hyperthyroidsymptoms(palpitations,tremor, diarrhea);familyhistoryofthyroiddisease;HIVriskfactors; image,anxietyordepression;otherconstitutional Amount, duration,±intention;dietandexercisehistory; body Key History Workup Differential Presentation PALPITATIONS WEIGHT LOSS pain. panied bylightheadednessandsharp,atypicalchest palpitationsaccom- 34 yoFpresentswithepisodic because sheisafraidofhumiliatingherself. ing socialsituation.Nowsheavoidsallevents trouble sleepingfordaysorweeksbeforeanupcom- rapid breathing.Shecomplainsofintenseworryand tremor. Shealsoexperiencesexcessivesweatingand include heartpounding,facialblushing,andhand during presentationsinfrontofherclass.Episodes palpitations,especially 19 yoFpresentswithepisodic tion anxietyasachild. any historyofpsychiatricillnessexceptforsepara- lasts2–3minutes.Hedoesnothave Each episode permonthforseveralmonths. has had4–5episodes unexpectedly, andhedoesnotrecallanytriggers.He occur tions, sweating,andrapidbreathing.Episodes ofpalpita- 35 yoMpresentswithseveralepisodes (cont’d) Agoraphobia/specific Avoidant personality Social phobia Pheochromocytoma Mitral valveprolapse Substance abuse/ Agoraphobia Hyperthyroidism Specific phobia Acute stressdisorder Generalized anxiety Panic attack Pheochromocytoma Panic attack Cardiac arrhythmia Mitral valveprolapse Hyperthyroidism Substance abuse/ Generalized anxiety Panic attack disorder dependence disorder dependence disorder phobia Echocardiography ECG Electrolytes CBC TSH, FT Electrolytes CBC 24-hour urinary Holter monitor Echocardiography ECG TSH, FT 24-hour urinary Urine toxicology Echocardiography ECG Mental statusexam catecholamines catecholamines 4 4 MINICASES 109 4 suppression test insulin CBC Electrolytes HIV antibody Urine toxicology TSH, FT CBC Electrolytes, glucose TSH 24-hour urine free cortisol Dexamethasone Blood glucose and plasma Glucose tolerance test 24-hour urine free cortisol hypoglycemia Smoking cessation Drug side effect Hypothyroidism syndrome Cushing’s Polycystic ovary syndrome Diabetes mellitus Atypical depression Insulinoma Reactive postprandial syndrome Cushing’s Hyperthyroidism Cancer HIV infection Dieting/diet drugs Anorexia nervosa Malabsorption (cont’d) ety, and hunger that is relieved by eating. She exhib- and hunger that ety, its proximal muscle weakness and easy bruising. 30 yo F presents with weight gain over the past 3 palpitations, anxi- months. She also reports tremor, 44 yo F presents with a weight gain of > 25 lbs (11.3 44 yo F presents with a weight gain of smoking 3 kg) within the past 2 months. She quit months ago and is on amitriptyline for depression. She also reports cold intolerance and constipation. 42 yo F presents with a 15.5-lb (7-kg) weight loss 42 yo F presents with and She has a fine tremor, within the past 2 months. her pulse is 112. WEIGHT GAIN WEIGHT LOSS WEIGHT Presentation Differential Workup Key Physical Exam moon face, buffalo syndrome (hypertension, central obesity, including signs of Cushing’s signs; complete exam, Vital abdominal striae); edema resulting from water retention in renal disease. hump, supraclavicular fat pads, purple Key History medication changes, smoking cessation, depression); diet history; hypothyroid Amount, duration, timing (relation to hirsutism; medical history; alcohol changes); menstrual irregularity, symptoms (fatigue, constipation, skin/hair/nail and drug use. Presentation Differential Workup Key Physical Exam Key Physical signs; complete physical. Vital MINICASES 110 Workup Differential Presentation Vital signs;headandneckexam;heart,lung,abdominalexams;skinexam(forsignsofscleroderma/CREST). Key PhysicalExam phagia, GERDsymptoms;medications;HIVriskfactors;historyofanxiety, smoking,Raynaud’s phenomenon. odyno- tutional symptoms(especiallyweightloss);hoarseness,drooling,regurgitationofliquidsvs.undigestedfood, Solids orliquidsvs.bothsolidsandliquids,±progression,occurringatthebeginningmiddleofswallow;consti- Key History DYSPHAGIA thrush. lowing solidsmorethanliquids.Examrevealsoral 38 yoMpresentswithdysphagiaandpainonswal- months. at night.Shehaslost5.5lbs(2.5kg)inthepast2 especially ing andregurgitationofundigestedfood, the pastyear. Itisassociatedwithdifficultybelch- liquids thathasslowlyprogressedinseveritywithin 48 yoFpresentswithdysphagiaforbothsolidsand craving foriceandclay. panied bymouthandthroatpain,fatigue,a 45 yoFpresentswithdysphagiafor2weeksaccom- months. weight lossof15lbs(6.8kg)withinthepast4 and aheavysmoker. Hehashadanunintentional solids andprogressedtoliquids.Heisanalcoholic 75 yoMpresentswithdysphagiathatstarted Amyotrophic lateral Esophageal stricture Systemic sclerosis Esophagitis Achalasia Esophageal cancer Esophagitis Zenker’s diverticulum Esophageal stricture Mitral valvestenosis Systemic sclerosis Esophagitis Esophageal cancer Plummer-Vinson Achalasia Mitral valvestenosis Systemic sclerosis Achalasia Esophagitis Esophageal cancer Plummer-Vinson Zenker’s diverticulum Esophageal stricture GERD Systemic sclerosis syndrome syndrome sclerosis HIV, pill-induced) (CMV, HSV, CT—chest Barium swallow Upper endoscopywith CXR CBC XR—neck Esophageal manometry Barium swallow Upper endoscopy CXR Video fluoroscopy Upper endoscopy Barium swallow ,,TIBC CBC CD4 count andviral HIV antibody Barium swallow Upper endoscopy CBC biopsy load MINICASES 111 glucose UA, urine culture HIV antibody U/S—thyroid Upper endoscopy Urine hCG Pelvic exam U/S—pelvis CBC Electrolytes, calcium, CBC with differential Electrolytes ESR, CRP node biopsy Lymph PPD CXR TSH Hodgkin’s lymphoma Hodgkin’s Depression Pregnancy Gastritis Hypercalcemia Diabetes mellitus UTI Hodgkin’s/non- Hodgkin’s/non- Tuberculosis Thyroid nodule Gastric carcinoma strual period ago, and her breasts are was 6 weeks She is sexually active with her boy- full and tender. friend, and they occasionally use condoms for con- traception. 20 yo F presents with nausea, vomiting (especially in the morning), fatigue, and polyuria. Her last men- palpitations, hoarseness, cough, difficulty breathing, palpitations, hoarseness, cough, difficulty Her hus- difficulty swallowing, or abdominal pain. and her band was recently discharged from prison, mother has a history of gastric cancer. 39 yo F presents with a single 2-cm mass on the 39 yo F presents with sweats, fever, right side of her neck along with night The satiety. weight loss, loss of appetite, and early not changed mass is painless and movable and has tremor, in size. She does not report heat intolerance, NAUSEA/VOMITING NECK MASS NECK Presentation Differential Workup neurologic symptoms (headache, stiff neck, vertigo, focal numbness or weakness); urinary symptoms; other associ- neurologic symptoms (headache, stiff and alleviating factors; medications; history of prior abdominal surgery. ated symptoms (GI, chest pain); exacerbating Key Physical Exam consider funduscopic exam (increased intracranial pressure); complete abdominal exam; consider signs; ENT; Vital heart, lung, and rectal exams. Key History to meals, sick contacts, possible foodAcuity of onset, ± abdominal pain, relation poisoning, possible pregnancy; Key Physical Exam Key Physical Exam nodes, signs; HEENT exam; exam of lymph heart, lung, and abdominal exams. spleen, and tonsils; Vital Presentation Differential Workup Key History of symptoms (dysphagia, shortness movement with swallowing; obstructive pain, mobility, Onset, size, location, ill contacts. hematologic, GI, endocrine, pulmonary); associated symptoms (constitutional, breath); other masses; MINICASES 112 Workup Differential Presentation and obturatorsigns,CVA percussion;bowelsounds, aorticbruits;rectalexam;pelvicexam(women). Vital signs;heartandlungexams;abdominalexam,includingtenderness,guarding,rebound,Murphy’s sign,psoas Key PhysicalExam corticosteroids); alcoholanddruguse;domesticviolence,stress/anxiety, sexualhistory, pregnancyhistory. abdominal surgeries,trauma,gallstones,renalstones,atheroscleroticvasculardisease;medications(eg,NSAIDs, GI, cardiac,pulmonary, renal,pelvic);exacerbatingandalleviating factors;historyofsimilarsymptoms; Location, quality, intensity, duration,radiation,timing(relationtomeals);associatedsymptoms(constitutional, Key History ABDOMINAL PAIN past 3daysbingedrinking. and vomiting.Heisanalcoholichasspentthe he leansforward.Healsoreportsanorexia,nausea, nal painthatradiatestothebackandimproveswhen 56 yoMpresentswithseveremidepigastricabdomi- and smoker. Heappearsjaundicedonexam. urine, andclay-coloredstool.Heisaheavydrinker ates totheback,accompaniedbyweightloss,dark 60 yoMpresentswithdullepigastricpainthatradi- tenderness. panied bynausea,vomiting,hematuria,andCVA sided flankpainthatradiatestothetesticles,accom- 45 yoMpresentswithsuddenonsetofcolickyright- GI etiology(eg, Pyelonephritis Renal cellcarcinoma Nephrolithiasis Boerhaave syndrome Alcoholic hepatitis Mesenteric ischemia Abdominal aortic Gastritis Cholecystitis/ Peptic ulcerdisease Acute pancreatitis Peptic ulcerdisease Abdominal aortic Cholecystitis/ Chronic pancreatitis Acute alcoholichepatitis Acute viralhepatitis Cholangiocarcinoma Pancreatic cancer aneurysm choledocholithiasis aneurysm choledocholithiasis appendicitis) BUN/Cr UA, urinecultureand ECG Upper endoscopy CT—abdomen U/S—abdomen AST/ALT/bilirubin/ Amylase, lipase Electrolytes, BUN/Cr CBC U/S—abdomen CT—abdomen AST/ALT/bilirubin/ Amylase, lipase Electrolytes CBC culture Blood IVP KUB U/S—renal CT—abdomen cytology sensitivity, urine alkaline phosphatase alkaline phosphatase MINICASES 113 H pylori alkaline phosphatase occult blood alkaline phosphatase (including testing) alkaline phosphatase alkaline phosphatase MRCP ERCP CBC Amylase, lipase AST/ALT/bilirubin/ hepatitis serologies Viral UA U/S—abdomen Rectal exam, stool for Amylase, lipase, lactate AST/ALT/bilirubin/ Upper endoscopy Upper GI series CBC AST/ALT/bilirubin/ U/S—abdomen CT—abdomen Blood culture CBC AST/ALT/bilirubin/ Blood culture hepatitis serologies Viral U/S—abdomen syndrome syndrome appendicitis cholangitis Fitz-Hugh–Curtis Acute hepatitis Acute cholecystitis Ascending cholangitis Choledocholithiasis Pancreatitis Acute glomerulonephritis Peptic ulcer disease Gastritis GERD Cholecystitis Chronic pancreatitis Mesenteric ischemia Acute cholecystitis Choledocholithiasis Hepatitis Ascending cholangitis Peptic ulcer disease Fitz-Hugh–Curtis Acute subhepatic Ascending cholangitis Acute gallstone Acute cholecystitis Hepatitis Sclerosing cholangitis (cont’d) 35 yo M presents with burning epigastric pain that 35 yo M presents with burning epigastric starts 2–3 hours after meals. The pain is relieved by food and antacids. 25 yo M presents with RUQ pain, fever, anorexia, 25 yo M presents with RUQ pain, fever, and clay- nausea, and vomiting. He has dark urine colored stool. potensive on exam. 43 yo obese F presents with RUQ abdominal pain, 43 yo obese F presents with RUQ abdominal with asymp- and jaundice. She was diagnosed fever, to be hy- tomatic gallstones 1 year ago. She is found with nausea, vomiting, and a fever of 101.5°F. The and a fever of 101.5°F. with nausea, vomiting, ate fatty food.pain started after she She has had sim- episodesilar but less intense few hours. that lasted a sign. Murphy’s Exam reveals a positive 41 yo obese F presents with RUQ abdominal pain 41 yo obese F presents right scapula and is associated that radiates to the ABDOMINAL PAIN PAIN ABDOMINAL Presentation Differential Workup MINICASES 114 Workup Differential Presentation ABDOMINAL PAIN exam. ceived digitalis.Herpainisoutofproportiontothe of CHFandatrialfibrillation,forwhichshehasre- massive darkbowelmovement.Shehasahistory abdominal pain.Sherecentlyvomitedandhada 70 yoFpresentswithacuteonsetofsevere,crampy surgeries. tus orstool.Hehasahistoryofmultipleabdominal iting, abdominaldistention,andinabilitytopassfla- 40 yoMpresentswithcrampyabdominalpain,vom- tious mononucleosis3weeksago. pain thatradiatestotheleftscapula.Hehadinfec- 18 yoMboxerpresentswithsevereLUQabdominal and takesaspirinonadailybasis. andantacids.Healsosmokesheavily lieved byfood a historyofintermittentepigastricpainthatisre- vomiting, andmildfever. Heappearstoxic.has 37 yoMpresentswithsevereepigastricpain,nausea, (cont’d) Kidney stone Splenic rupture Mesenteric ischemia Gallstone cholangitis Cholecystitis Hepatitis Acute pancreatitis Perforated pepticulcer Cholecystitis Acute pancreatitis Gastroenteritis Peptic ulcerdisease Diverticulitis Mesenteric ischemia/ Hernia Ileus poisoning Food Gastroenteritis Volvulus Small bowelorcolon Intestinal obstruction Splenic infarct Perforated pepticulcer Pneumonia Rib fracture infarction cancer Electrolytes CBC culture Blood Upper endoscopy CT—abdomen KUB CXR AST/ALT/bilirubin/ Amylase, lipase,lactate Electrolytes CBC Rectal exam Barium enema Mesenteric angiography CT—abdomen AXR ECG Amylase, lipase,lactate CBC Rectal exam Colonoscopy CT—abdomen/pelvis with AXR Electrolytes CBC Rectal exam U/S—abdomen (if CT—abdomen CXR testing) (including alkaline phosphatase contrast unstable) hemodynamically H pylori MINICASES 115 Entamoeba antigen parasitology, parasitology, histolytica occult blood testing, VDRL/RPR CT—abdomen Blood culture CBC Electrolytes CT—abdomen AXR U/S—abdomen Blood culture Rectal exam, stool for Pelvic exam Urine hCG CBC Electrolytes Colonoscopy CT—abdomen/pelvis Stool for ova and Pelvic exam Urine hCG Doppler U/S—pelvis Rectal exam UA CBC CT—abdomen Laparoscopy Chlamydia and gonorrhea Rectal exam CBC Electrolytes CXR AXR intestinal obstruction (amebiasis, giardiasis) disease perforation Acute appendicitis Gastroenteritis Diverticulitis disease Crohn’s Nephrolithiasis or other Volvulus Perforation Acute cholecystitis Irritable bowel syndrome disease Crohn’s Celiac disease Chronic pancreatitis GI parasitic infection Endometriosis Ovarian torsion Appendicitis Nephrolithiasis Ectopic pregnancy Ruptured ovarian cyst Pelvic inflammatory Bowel infarction or Diverticulitis disease Crohn’s Ulcerative colitis Gastroenteritis Abscess (cont’d) months. The pain never awakens her from sleep. It is relieved by defecation and worsens when she is upset. She has alternating constipation and diarrhea but no nausea, vomiting, weight loss, or anorexia. 30 yo F presents with periumbilical pain for 6 20 yo M presents with severe RLQ abdominal pain, 20 yo M presents with severe RLQ abdominal started yester- nausea, and vomiting. His discomfort As the day as a vague pain around the umbilicus. to the pain worsened, it became sharp and migrated psoas signs are positive. and RLQ. McBurney’s onset of alternating diarrhea and constipation. He onset of alternating diarrhea and constipation. diet. high-fat consumes a low-fiber, 68 yo M presents with LLQ abdominal pain, fever, pain, fever, 68 yo M presents with LLQ abdominal reports recent and chills for the past 3 days. He also nary symptoms, or vaginal bleeding and has never nary symptoms, or taken OCPs. Her last menstrual period was regular, of STDs. She has been told and she has no history on her right ovary. that she had a cyst 21 yo F presents with acute onset of severe RLQ 21 yo F presents with uri- vomiting. She has no fever, pain, nausea, and ABDOMINAL PAIN PAIN ABDOMINAL Presentation Differential Workup MINICASES 116 rsnainDfeeta Workup Differential Presentation Vital signs;relevantthyroid/endocrineexam;abdominalandrectal exams;±femalepelvicexam. Key PhysicalExam risk factors;historyofabdominalsurgeries,diabetes,pancreatitis;alcoholanddruguse;familycoloncancer. Workup (especially fiberandfluidintake);medications(includingrecentantibiotics);sickcontacts,travel,camping,HIV plete evacuation,melenaorhematochezia);thyroiddiseasesymptoms(eg,feelinghot,palpitations,weightloss);diet of changeinbowelhabits;associatedsymptoms(constitutional,abdominalpain,bloating,tenesmus,senseincom- Frequency, color, odor, andvolumeofstools;presencemucusorflatulence;whetherstoolsfloatinbowl;duration Key History Differential Presentation ABDOMINAL PAIN CONSTIPATION/DIARRHEA ago, heandhisfatherhaveeatenonlyjunkfood. the past3weeks.Sincehismotherdied2months 28 yoMpresentswithconstipation(hardstool)for was 12yearsago. family historyofcoloncancer. Hislastcolonoscopy tional weightloss.Heisonalow-fiberdietandhas stool forthepast8months.Healsoreportsuninten- stipation, decreasedstoolcaliber, inthe andblood 67 yoMpresentswithalternatingdiarrheaandcon- protected sexwithmultiplesexualpartners. greenish-yellow vaginaldischarge.Shehashadun- Thepainisassociatedwithfeverandathick, riod. that startedwiththefirstdayofhermenstrualpe- 24 yoFpresentswithbilaterallowerabdominalpain (cont’d) Pyelonephritis Spontaneous abortion Cystitis Vaginitis Dysmenorrhea Endometriosis Pelvic inflammatory Hypothyroidism Irritable bowelsyndrome Substance abuse(eg, Depression Low-fiber diet Inflammatory bowel GI parasiticinfection Diverticulosis Irritable bowelsyndrome Colorectal cancer disease heroin) disease (ascariasis, giardiasis) U/S—pelvis UA, urineculture ESR CBC Cervical cultures Urine hCG Pelvic exam Urine toxicology Electrolytes TSH Rectal exam CT—abdomen/pelvis Barium enema Colonoscopy AST/ALT/bilirubin/ Electrolytes CBC Rectal exam,stoolfor alkaline phosphatase occult blood MINICASES 117 toxin antigen, C difficile occult blood culture culture Giardia Entamoeba histolytica antigen alkaline phosphatase occult blood parasitology occult blood culture Rectal exam, stool for Stool leukocytes and CBC Electrolytes Stool for Hydrogen breath test TSH Rectal exam Stool leukocytes, culture, CBC Electrolytes AST/ALT/bilirubin/ hepatitis serologies Viral Rectal exam Stool leukocytes and Rectal exam, stool for Rectal exam, stool CBC Electrolytes Colonoscopy Stool for ova and CT—abdomen/pelvis Rectal exam, stool for Stool leukocytes and CBC Electrolytes CT—abdomen/pelvis ) — Clostridium difficile colitis disease disease ( disease (ascariasis, giardiasis) (gastroenteritis) bacterial, viral, parasitic, protozoal Gastroenteritis Cryptosporidiosis Food poisoning Inflammatory bowel Inflammatory bowel Irritable bowel syndrome Hyperthyroidism Pseudomembranous diarrhea Traveler’s Giardiasis Amebiasis Food poisoning Hepatitis A Lactose intolerance Gastroenteritis Irritable bowel syndrome Inflammatory bowel Celiac disease Chronic pancreatitis GI parasitic infection Lactose intolerance Infectious diarrhea Food poisoning (cont’d) 30 yo F presents with watery diarrhea, abdominal cramping, and bloating. Her symptoms are aggra- vated by milk ingestion and are relieved by fasting. 25 yo M presents with watery diarrhea and abdomi- 25 yo M presents with watery diarrhea nal cramps. He was recently in Mexico. 40 yo F presents with watery diarrhea and abdominal 40 yo F presents with watery diarrhea and for a UTI. cramps. Last week she was on antibiotics 33 yo M presents with watery diarrhea, vomiting, 33 yo M presents with watery diarrhea, He yesterday. and diffuse abdominal pain that began coworkers are also reports feeling hot. Several of his also ill. lieved by defecation. She has no nausea, vomiting, lieved by defecation. weight loss, or blood in her stool. 30 yo F presents with alternating constipation and 30 yo F presents with by abdominal pain that is re- diarrhea accompanied CONSTIPATION/DIARRHEA CONSTIPATION/DIARRHEA Presentation Differential Workup MINICASES 118 rsnainDfeeta Workup Differential Presentation Vital signs,includingorthostatics;ENT, heart,lung,abdominal,andrectalexams. Key PhysicalExam history ofpepticulcerdisease,liverabdominalaorticaneurysmrepair, easybleeding. thinners,NSAIDs,andcorticosteroids); tional, nausea,abdominalpain,dyspepsia);medications(especiallyblood Workup Amount, duration,context(afterseverevomiting,alcoholingestion,nosebleed);associatedsymptoms(constitu- Key History Differential Presentation CONSTIPATION/DIARRHEA UPPER GIBLEEDING alcoholic. thritis thathasbeentreatedwithNSAIDs.Sheisan ground emesis.Shehasahistoryofrheumatoidar- 40 yoFpresentswithepigastricpainandcoffee- andantacids. by food tory ofintermittentepigastricpainthatisrelieved past 3days.Herstoolisdarkandtarry. Shehasahis- 45 yoFpresentswithcoffee-groundemesisforthe not respondedtoantibiotics. weeks. Hehasahistoryofaphthousulcers. dominal pain,andweightlosswithinthepast3 33 yoMpresentswithwaterydiarrhea,diffuseab- (cont’d) Carcinoid syndrome Small bowellymphoma Hyperthyroidism Pseudomembranous colitis Celiac disease Ulcerative colitis Gastroenteritis Crohn’s disease Mallory-Weiss tear Esophageal varices Gastric cancer Bleeding pepticulcer Gastritis Esophageal varices Gastric cancer Gastritis Bleeding pepticulcer 5-HIAA Small bowelseries TSH CT—abdomen Colonoscopy Electrolytes CBC Stool leukocytesand Rectal exam,stoolfor Upper endoscopy INR AST/ALT/bilirubin/ Electrolytes CBC, typeandcross Rectal exam Upper endoscopy INR AST/ALT/bilirubin/ Electrolytes CBC, typeandcross Rectal exam culture occult blood alkaline phosphatase confirmed) testing ifulceris (including alkaline phosphatase H pylori MINICASES 119 alkaline phosphatase Colonoscopy CEA CT—abdomen/pelvis Rectal exam CBC PT/PTT Colonoscopy CT—abdomen/pelvis Rectal exam CBC, type and cross PT/PTT Electrolytes Colonoscopy RBC scan Tagged CT—abdomen/pelvis Rectal exam CBC AST/ALT/bilirubin/ INR disease Angiodysplasia Upper GI bleeding Inflammatory bowel Ulcerative colitis disease Crohn’s Proctitis Anal fissure Hemorrhoids Diverticulosis Dysentery Diverticulosis Anal fissure Hemorrhoids Angiodysplasia Colorectal cancer Colorectal cancer Anal fissure Hemorrhoids Diverticulosis Ischemic bowel disease low-fiber diet. 58 yo M presents with painless bright red blood per rectum and chronic constipation. He consumes a 33 yo F presents with rectal bleeding and diarrhea 33 yo F presents with rectal bleeding pain for the past week. She has had lower abdominal and tenesmus for several months. 67 yo M presents with blood in his stool, weight loss, of colon and constipation. He has a family history cancer. HEMATURIA BLOOD IN STOOL BLOOD point along the stream (initial vs. terminal vs. throughout); medications; history of vigorous exercise, trauma, smok- point along the stream (initial vs. terminal vs. throughout); medications; history or easy bleeding; skin bruising (purpura). ing, stones, cancer, Key History dysuria, irritative voiding symptoms); Amount, duration, ± clots; associated symptoms (constitutional, renal colic, Presentation Differential Workup (especially blood disease, renal disease, aortic valve bleeding or atherosclerotic vascular thinners); history of easy cancer. repair; family history of colon alcoholism, or abdominal aortic aneurysm disease, liver disease, Key Physical Exam rectal exams. signs ± orthostatics; abdominal and Vital Key History blood;Melena vs. bright red or rectal pain, tenes- symptoms (constitutional, abdominal amount, duration; associated medications of similar symptoms; prior colonoscopy; menstrual cycle; trauma; history mus, constipation/diarrhea); MINICASES 120 tion; stones,diabetes,alcoholism. constitutional symptoms;bone pain;medications;historyofUTIs,urethralstricture, orurinarytractinstrumenta- postvoid dribbling,leakagewith coughorsneeze,incontinence),irritativesymptoms (urgency, frequency, nocturia), Duration, obstructivesymptoms(hesitancy, diminishedstream,senseofincompletebladderemptying,straining, Key History Workup Differential Presentation Vital abdominalexam;genitourinaryandrectalexams;extremities. signs;lymphnodes; Key PhysicalExam HEMATURIA OTHER URINARY SYMPTOMS a flankmass. loss andfeveroverthepast2months.Examreveals urine withoutdysuria.Hehasexperiencedweight inhis 55 yoMpresentswithflankpainandblood family historyofkidneydisease. 35 yoMpresentswithpainlesshematuria.Hehasa heavy smokerandworksasapainter. 65 yoMpresentswithpainlesshematuria.Heisa (cont’d) Polycystic kidneydisease Coagulation disorder(ie, Prostate cancer Acute glomerulonephritis Nephrolithiasis Renal cellcarcinoma Bladder cancer Prostate cancer Pyelonephritis Acute glomerulonephritis Nephrolithiasis Bladder cancer Renal cellcarcinoma Bladder cancer Coagulation disorder UTI Acute glomerulonephritis Nephrolithiasis factor VIIIantibodies) (eg, IgAnephropathy) Genitourinary exam Prostate biopsy CT—abdomen/pelvis U/S—renal/bladder Cystoscopy PT/PTT CBC PSA BUN/Cr UA, urinecytology Genitourinary exam Cystoscopy CT—abdomen/pelvis U/S—renal PT/PTT CBC PSA BUN/Cr UA, urinecytology Genitourinary, rectal CT—abdomen/pelvis U/S—renal PT/PTT CBC PSA BUN/Cr UA, urinecytology exams MINICASES 121 (transrectal) urethral discharge PCR sensitivity (transrectal) CT—abdomen CBC BUN/Cr PSA U/S—prostate Prostate biopsy Alkaline phosphatase CT—pelvis MRI—spine Genital, rectal exams UA, urine culture Gram stain and culture of Chlamydia and gonorrhea UA, urine culture and Blood culture CBC BUN/Cr U/S—renal Rectal exam UA CBC BUN/Cr Alkaline phosphatase U/S—prostate PSA Rectal exam UA urethritis) hypertrophy (BPH) Renal cell carcinoma UTI Bladder stones Urethritis Cystitis Prostatitis Acute pyelonephritis Nephrolithiasis Lower UTI (cystitis, Renal cell carcinoma Benign prostatic Prostate cancer UTI Bladder stones Prostate cancer BPH (cont’d) 45 yo diabetic F presents with dysuria, urinary fre- chills, and nausea for the past 3 days. fever, quency, tenderness on exam. There is left CVA 18 yo M presents with a burning sensation during 18 yo M presents with a burning sensation recently had urination and urethral discharge. He unprotected sex with a new partner. 71 yo M presents with nocturia, urgency, a weak 71 yo M presents with nocturia, urgency, and lower stream, terminal dribbling, hematuria, also experi- back pain for the past 4 months. He has enced weight loss and fatigue. urinary retention that required catheterization. urinary retention that 60 yo M presents with nocturia, urgency, weak with nocturia, urgency, 60 yo M presents dribbling. He denies any weight stream, and terminal pain. He has had 2 episodesloss, fatigue, or bone of OTHER URINARY SYMPTOMS SYMPTOMS URINARY OTHER Presentation Differential Workup Key Physical Exam Key Physical bladder); genital and rectal percussion to assess for a distended signs; abdominal exam (including suprapubic Vital exam. exams; focused neurologic MINICASES 122 rsnainDfeeta Workup Differential Presentation Vital signs;cardiovascularexam;genitalandrectalexams. Key PhysicalExam smoking, alcohol,anddruguse. known atheroscleroticvasculardisease,priorprostatesurgery, liverdisease,thyroidneurologicdisease); hair;medications(andrecentchanges);medicalhistory(hypertension,diabetes,highcholesterol, or lossofbody Duration, severity, ±nocturnalerections,libido,stressordepression,trauma,associatedincontinence;gynecomastia Key History Workup Differential Presentation OTHER URINARY SYMPTOMS ERECTILE DYSFUNCTION(ED) on insulin. atenolol 4monthsago.Healsohasdiabetesandis months ago.Hehashypertensionandwasstartedon 47 yoMpresentswithimpotencethatstarted3 ago. der stresssincehersisterpassedawayafewmonths She drinks8cupsofcoffeeperday. Shehasbeenun- of UTIsandafamilyhistorydiabetesmellitus. amounts ofurinewithoutwarning.Shehasahistory able tosuppresstheurgeurinateandloseslarge 33 yoFpresentswithurinaryleakage.Sheisun- the onsetofmenopause. liveries, andhermotherhadthesameproblemafter back pain.Shehasahistoryofmultiplevaginalde- laughs, orsneezes.Shealsocomplainsofvaguelow She losesasmallamountofurinewhenshecoughs, 55 yoFpresentswithurinaryleakageafterexercise. (cont’d) Diabetes mellitus UTI Functional incontinence Overflow incontinence Stress incontinence Mixed incontinence Urge incontinence Diabetes mellitus UTI Functional incontinence Overflow incontinence Urge incontinence Mixed incontinence Stress incontinence Peyronie’s disease Psychogenic ED ED causedbydiabetes ED causedby Drug-related ED mellitus hypertension Cystourethroscopy IVP testing Urodynamic UA, urineculture Electrolytes, BUN/Cr, CBC Cystourethroscopy IVP testing Urodynamic BUN/Cr UA, urineculture Testosterone level CBC Glucose Rectal exam Genital exam glucose MINICASES 123 cervical cultures, HIV rubella antibody, B hepatitis antibody, surface antigen, VDRL/ RPR Baseline Pap smear, Baseline Pap smear, Urine hCG LH/FSH, TSH, prolactin Pelvic exam DHEAS Testosterone, Urine hCG LH/FSH, TSH, prolactin MRI—brain Pelvic and breast exams Urine hCG U/S—abdomen/pelvis Pelvic exam CBC UA, urine culture Prolactin, TSH syndrome malignancy to prolactinoma Polycystic ovary Thyroid disease Hyperprolactinemia Pregnancy Ovarian or adrenal Premature ovarian failure Amenorrhea secondary Pregnancy Thyroid disease Premature ovarian failure Pituitary tumor Pregnancy Anovulatory cycle Hyperprolactinemia UTI Hypothyroidism 35 yo F presents with amenorrhea, galactorrhea, visual field defects, and headaches for the past 6 months. years. 23 yo obese F presents with amenorrhea for 6 23 yo obese F presents with amenorrhea and infertility for the past 3 months, facial hair, ception. 40 yo F presents with amenorrhea, morning nausea 40 yo F presents with amenorrhea, morning Her last men- and vomiting, fatigue, and polyuria. strual period was 6 weeks ago, and her breasts are full She uses the rhythm method for contra- and tender. AMENORRHEA Presentation Differential Workup nervosa, excessive dieting, vigorous exercise, pregnancies, D&Cs, uterine infections; drug use; medications. D&Cs, uterine infections; drug use; dieting, vigorous exercise, pregnancies, nervosa, excessive Key Physical Exam exam. signs; breast exam; complete pelvic Vital Key History (headache, decreased peripheral vision, pregnancy; associated symptoms duration, possible Primary vs. secondary, history of anorexia dryness, symptoms of thyroid disease); virilization, hot flashes, vaginal galactorrhea, hirsutism, MINICASES 124 Vital signs; abdominalexam;completepelvicexam. Key PhysicalExam tives); historyofeasybleedingorbruising;abnormal Papsmears. thinners,contracep- discharge, pelvicorabdominalpain,urinarysymptoms; trauma;medications(especiallyblood associated Pre- vs.postmenopausalstatus,duration,amount;menstrual historyandrelationtolastmenstrualperiod; Key History Workup Differential Presentation AMENORRHEA VAGINAL BLEEDING which shewasapassenger. and dizziness.Shelostherfiancéinacaraccident months. Shehasahistoryofoccasionalpalpitations 29 yoFpresentswithamenorrheaforthepast6 and heatintolerance. weighs 90lbs(40.9kg).Shehasahistoryofexercise 4 months.Sheis5feet,6inches(167.6cm)and 18 yoFpresentswithamenorrheaforthepast shock andfailureoflactation2yearsago. tory ofabruptioplacentaefollowedbyhypovolemic coarse hair, weightloss,andfatigue.Shehasahis- 35 yoFpresentswithamenorrhea,coldintolerance, emotional lability, anddyspareunia. months accompaniedbyhotflashes,nightsweats, 48 yoFpresentswithamenorrheaforthepast6 (cont’d) Asherman’s syndrome Thyroid disease Pituitary tumor Premature ovarianfailure Sheehan’s syndrome Thyroid disease Pituitary tumor Pregnancy Menopause Hyperthyroidism Depression Posttraumatic stress Anxiety-induced Hyperthyroidism Pregnancy Anorexia nervosa disorder amenorrhea TSH, FT Pelvic exam CBC LH/FSH, prolactin Urine hCG MRI—brain CBC Pelvic exam Testosterone, DHEAS LH/FSH, TSH,prolactin Urine hCG TSH, FT CBC TSH, FT CBC Urine hCG Hysteroscopy MRI—brain ACTH LH/FSH, estradiollevels Progesterone challenge Urine cortisollevel LH/FSH test 4 4 4 MINICASES 125 Colposcopy Hysteroscopy Pelvic exam Pap smear Colposcopy and biopsy HPV testing Endometrial biopsy Urine hCG Quantitative serum hCG U/S—abdomen/pelvis Pelvic exam CBC PT/PTT Urine hCG Quantitative serum hCG U/S—abdomen/pelvis Pelvic exam Cervical cultures Urine hCG Pelvic exam Cervical culture Pap smear CBC ESR Glucose PT/PTT LH/FSH, TSH, prolactin U/S—pelvis Pelvic exam Pap smear Endometrial biopsy Endometrial curettage U/S—pelvis laceration) disease bleeding (eg, von Willebrand’s disease, hemophilia) Atrophic vaginitis Cervical cancer Endometrial cancer Cervical polyp Cervicitis (eg, cervical Trauma Spontaneous abortion Ectopic pregnancy Molar pregnancy Ectopic pregnancy Ruptured ovarian cyst Ovarian torsion Pelvic inflammatory Dysfunctional uterine Dysfunctional uterine Coagulation disorder Cervical cancer Molar pregnancy Hypothyroidism Diabetes mellitus Endometrial cancer Cervical cancer Atrophic endometrium Endometrial hyperplasia Endometrial polyps (cont’d) menstrual period weeks ago. She has a history was 5 of pelvic inflammatory disease and unprotected in- tercourse. 32 yo F presents with sudden onset of left lower ab- dominal pain that radiates to the scapula and back and is associated with vaginal bleeding. Her last 28 yo F who is 8 weeks pregnant presents with lower 28 yo F who is 8 weeks pregnant presents abdominal pain and vaginal bleeding. 45 yo G5P5 F presents with postcoital bleeding. She 45 yo G5P5 F presents with postcoital is a cigarette smoker and takes OCPs. 10 years ago. She has a history of hypertension and 10 years ago. She has a history of hypertension diabetes mellitus. She is nulliparous. 61 yo obese F presents with profuse vaginal bleeding 61 yo obese F presents with profuse vaginal periodfor the past month. Her last menstrual was 6 months. 17 yo F presents with prolonged, excessive men- 17 yo F presents irregularly within the past strual bleeding occurring VAGINAL BLEEDING VAGINAL Presentation Differential Workup MINICASES 126 Vital signs; abdominalexam;completepelvicexam. Key PhysicalExam endometriosis, pelvicinflammatorydisease,orpriorabdominal/pelvic surgeries. of lubrication,menopausalstatus,libido;sexualhistory, historyofsexualtraumaordomesticviolence; of Duration, timing;associatedsymptoms(vaginaldischarge,rash,painfulmenses,GIsymptoms,hotflashes);adequacy Key History Workup Differential Presentation Vital signs;abdominalexam;completepelvicexam. Key PhysicalExam useofcontraceptives,tampons,anddouches;historysimilarsymptoms; STDs. of lastmenstrualperiod; Amount, color, consistency, odor, duration;associatedvaginalburning,pain,orpruritus;recentsexualactivity;onset Key History VAGINAL DISCHARGE DYSPAREUNIA ing anddiscomfort. greenish vaginaldischargewithintenseitch- profuse,frothy,35 yoFpresentswithamalodorous, vaginaldischargeanditching. like, odorless 30 yoFpresentswithathick,white,cottagecheese– smelling vaginaldischarge. 28 yoFpresentswithathin,grayish-white,foul- Bacterial vaginosis Vaginitis—candidal Cervicitis (chlamydia, Vaginitis—trichomonal Vaginitis—candidal vaginosis Bacterial Cervicitis (chlamydia, Bacterial vaginosis Vaginitis—candidal Vaginitis—trichomonal Vaginitis—trichomonal gonorrhea) gonorrhea) Wet mount,KOHprep, Pelvic exam Cervical cultures pH ofvaginalfluid Wet mount,KOHprep, Pelvic exam Cervical cultures pH ofvaginalfluid Wet mount,KOHprep, Pelvic exam Cervical cultures pH ofvaginalfluid “whiff test” “whiff test” “whiff test” MINICASES 127 XR—skeletal survey CT—maxillofacial Urine toxicology CBC Pelvic exam LH/FSH mount, KOH prep Wet Cervical cultures Pelvic exam mount, KOH prep Wet Cervical cultures U/S—pelvis Laparoscopy Endometrial biopsy behavior disease Domestic violence Osteogenesis imperfecta Substance abuse Consensual violent sexual Atrophic vaginitis Endometriosis Cervicitis Depression Domestic violence Endometriosis Cervicitis Vaginismus Vulvodynia Pelvic inflammatory Depression Domestic violence (cont’d) ized for physical injuries 7 months ago. She presents with her husband. 28 yo F c/o multiple facial and bodily injuries. She claims that she fell on the stairs. She was hospital- 37 yo F presents with dyspareunia, inability to con- 37 yo F presents with ceive, and dysmenorrhea. 54 yo F c/o painful intercourse. Her last menstrual 54 yo F c/o painful period ago. She has hot flashes. was 9 months ABUSE DYSPAREUNIA DYSPAREUNIA Vital signs; complete exam ± pelvic exam. signs; complete exam Vital Presentation Differential Workup Establish confidentiality; directly question about physical, sexual, or emotional abuse and about fear, safety, backup safety, about physical, sexual, or emotional abuse and about fear, Establish confidentiality; directly question mental illness, drug use; firearms in the home. plan; history of frequent accidents/injuries, Key Physical Exam Key History Presentation Differential Workup MINICASES 128 rsnainDfeeta Workup Differential Presentation Vital signs;HEENTandmusculoskeletalexams;relevantneurovascular exam. Key PhysicalExam DVT riskfactors;alcoholanddruguse;familyhistoryofrheumaticdisease. ness, morningstiffness);exacerbatingandalleviatingfactors;trauma(includingvigorousexercise);medications; associated symptoms(constitutional,redeye,oralorgenitalulceration,diarrhea,dysuria,rash,focalnumbness/weak- Location, quality, intensity, duration,pattern(smallvs.largejoints;numberinvolved;swelling,redness,warmth); Key History Workup Differential Presentation ABUSE JOINT/LIMB PAIN story. a table.Shelooksanxiousandgivesaninconsistent tripping, falling,andhittingherheadontheedgeof 30 yoFpresentswithwristpainandablackeyeafter by 2men. injuries. Shestatesthatshewasattackedandraped 30 yoFpresentswithmultiplefacialandphysical (cont’d) Domestic violence Rape Substance abuse Factitious disorder Domestic violence Urine toxicology CT—head XR—wrist Viral hepatitisserologies HIV antibody CBC XR—skeletal survey Chlamydia andgonorrhea Cervical cultures Wet mount,KOHprep Urine hCG Pelvic exam Forensic exam(sexual testing collection kit) evidence [SAFE] assault forensic MINICASES 129 (DEXA) parvovirus B19 CCP RF, synovial fluid analysis joints vitamin D Tinel’s sign Tinel’s CCP C3, C4, RF, XR—hands, left knee ANA, anti-dsDNA, ESR, CBC Cervical culture Arthrocentesis and CCP ANA, ESR, RF, CBC XR—hands XR—pelvis/sacroiliac Uric acid XR—hip/pelvis CT or MRI—hip CBC, type and cross Serum calcium and Bone density scan Phalen’s maneuver and maneuver Phalen’s test Finkelstein’s studies Nerve conduction EMG ANA, anti-dsDNA, ESR, CBC XR—hands UA Antibody titers for inflammatory bowel disease compression in the compression in the forearm or arm the roots C6 and C7 in cervical spine tenosynovitis erythematosus (SLE) Rheumatoid arthritis SLE Disseminated gonorrhea Arthritis associated with Psoriatic arthritis Rheumatoid arthritis SLE Gout Hip fracture Hip dislocation Pelvic fracture Carpal tunnel syndrome Median nerve Radiculopathy of nerve De Quervain’s Systemic lupus Rheumatoid arthritis Psoriatic arthritis Parvovirus B19 infection (cont’d) HRT or calcium supplements. Her left leg is exter- HRT nally rotated, shortened, and adducted, and there is tenderness in her left groin. 65 yo F presents with inability to use her left leg or bear weight on it after tripping on a carpet. Onset of menopause was 20 years ago, and she did not receive 18 yo M presents with pain in the interphalangeal 18 yo M presents with pain in the interphalangeal salmon-pink joints of both hands. He also has scaly, lesions on the extensor surface of his elbows and knees. geal joints of both hands. Her left knee is also pain- geal joints of both hands. Her left knee that lasts ful and red. She has morning joint stiffness arthritis. Her mother had rheumatoid for an hour. 28 yo F presents with pain in the metacarpophalan- 28 yo F presents with pain in the interphalangeal 28 yo F presents with pain in the interphalangeal loss and a joints of her hands accompanied by hair rash on her face. first, second, and third fingers of her right hand. The first, second, and third and is relieved by loose shak- pain worsens at night is sensory loss in the same ing of the hand. There sign. a positive Tinel’s fingers. Exam reveals 30 yo F secretary presents with wrist pain and a sen- 30 yo F secretary presents and burning in her palm and the sation of numbness JOINT/LIMB PAIN PAIN JOINT/LIMB Presentation Differential Workup MINICASES 130 Workup Differential Presentation JOINT/LIMB PAIN test) withtheelbowinextension. as wellpainonresistedwristdorsiflexion(Cozen’s pain. Thereistendernessoverthelateralepicondyle plays tennis.Hisgripisimpairedasaresultofthe 55 yoMpresentswithpainintheelbowwhenhe known trauma. then improvesashecontinueswalking.Hehasno foot thatismostnotablewithhisfirstfewstepsand 65 yoMpresentswithpainintheheelofright training foramarathon. 65 yoMpresentswithrightfootpain.Hehasbeen with swellingandredness. 45 yoMpresentswithfeversandrightkneepain is havingdifficultywalking. has noticedswellinganddeformityofthejoint have graduallyworsenedoverthepast10years.She pain thatincreaseswithmovement.Hersymptoms 56 yoobeseFpresentswithrightkneestiffnessand hip, adducted,andinternallyrotated. motor vehicleaccident.Hisrightlegisflexedatthe 40 yoMpresentswithpainintherightgroinaftera (cont’d) Meniscal orligament Gout Pseudogout Osteoarthritis Hip fracture Hip dislocation— Stress fracture Tennis elbow(lateral Splinter/foreign body Heel fracture Plantar fasciitis Foot sprainorstrain Plantar fasciitis Stress fracture Reiter’s syndrome Trauma Lyme arthritis Pseudogout Gout Septic arthritis damage traumatic epicondylitis) (reactive arthritis) MRI—knee Uric acid Knee arthrocentesisand ESR CBC XR—knee Urine toxicologyand PT/PTT CBC, typeandcross CT orMRI—hip XR—hip MRI—elbow Bone scan XR—arm Bone scan—foot XR—heel MRI—foot Bone scan—foot XR—foot Lyme titers—IgGandIgM Uric acid XR—knee urethralcultures Blood, Knee arthrocentesisand CBC stain, culture,crystals) (cell count,Gram synovial fluidanalysis alcohollevel blood stain, culture,crystals) (cell count,Gram synovial fluidanalysis MINICASES 131 synovial fluid analysis synovial fluid analysis (cell count, Gram stain, culture) CCP RF, extremity -dimer D Doppler U/S—right leg CBC ECG CBC XR—shoulder CXR Echocardiography Stress test XR—shoulder XR—arm MRI—shoulder Knee arthrocentesis and Knee arthrocentesis ESR, ANA, anti-dsDNA, CBC Blood, cervical cultures XR—knee Ankle-brachial index Doppler U/S—lower Angiography MRI—L-spine (pseudoclaudication) thrombosis (reactive arthritis) disease (intermittent claudication) (aortoiliac occlusive disease) Lumbar spinal stenosis Osteoarthritis DVT cyst rupture Baker’s Myositis Cellulitis Superficial venous Angina/MI Tendinitis Osteoarthritis Shoulder dislocation Fracture of the humerus Rotator cuff injury Disseminated gonorrhea Rheumatoid arthritis SLE syndrome Reiter’s Peripheral vascular Leriche syndrome (cont’d) ticed deformity of his shoulder and had to hold his right arm. 50 yo M presents with right shoulder pain after fall- ing onto his outstretched hand while skiing. He no- 60 yo F c/o left arm pain that started while she was 60 yo F c/o left arm pain that started swimming and was relieved by rest. 45 yo F presents with right calf pain. Her calf is ten- 45 yo F presents with right calf pain. Her to the left warm, red, and swollen compared der, side. She was started on OCPs 2 months ago for dys- functional uterine bleeding. to smoke heavily. 60 yo F presents with pain in both legs that is in- 60 yo F presents with pain in both legs rest. She had duced by walking and is relieved by and continues cardiac bypass surgery 6 months ago a rash on her limbs, and vaginal discharge. She is a rash on her limbs, multiple partners and occasion- sexually active with ally uses condoms. 27 yo F presents with painful wrists and elbows, a 27 yo F presents with joint that is painful on flexion, swollen and hot knee JOINT/LIMB PAIN PAIN JOINT/LIMB Presentation Differential Workup MINICASES 132 rsnainDfeeta Workup Differential Presentation diagnostic utility);hipexam(canreferpaintotheback);considerrectalexam. Vital signs;neurologicexam(especiallyL4–S1nerveroots);backpalpation andrangeofmotion(althoughrarely Key PhysicalExam Workup cer, recurrentUTIs,diabetes,renalstones,IVdruguse,smoking. associated symptoms(especiallyconstitutional,incontinence);exacerbatingandalleviatingfactors;historyofcan- Location, quality, intensity, radiation,context(movingfurniture,bending/twisting,trauma),timing(disturbssleep); Key History Differential Presentation JOINT/LIMB PAIN LOW BACKPAIN pushing agrocerycart). pain isrelievedbysittingandleaningforward(eg, legs duringprolongedstandingandwalking.The 45 yoMpresentswithpaininthelowerbackand also noted. legs. Paraspinalmuscletendernessandspasmare and thereisnosensorydeficitorweaknessinher she cleanedherhouse.Thepaindoesnotradiate, 45 yoFpresentswithlowbackpainthatstartedafter raise ispositive.Thepatientunabletotiptoe. the lateralaspectofherleftfoot.Thestraightleg 45 yoFpresentswithlowbackpainthatradiatesto statin andclofibrateforhyperlipidemia. calf pain,fatigue,anddarkurine.Heisonsimva- 55 yoMpresentswithcrampybilateralthighand (cont’d) Inclusion body myositis Inclusion body Polymyositis Rhabdomyolysis dueto Peripheral vasculardisease Tumor inthevertebral Lumbar musclestrain Lumbar spinalstenosis Vertebral compression Disk herniation Lumbar musclestrain Tumor inthevertebral Lumbar musclestrain Disk herniation statins canal fracture canal Urine UA Aldolase CPK Phosphate, potassium, CBC Ankle-brachial index CT—L-spine XR—L-spine MRI—L-spine (preferred) MRI—L-spine XR—L-spine MRI—L-spine XR—L-spine calcium, uricacid BUN/Cr, glucose, MINICASES 133 pneumatic otoscopy) UA, urine culture Physical exam CBC Electrolytes Blood culture LP—CSF analysis CXR UA, urine culture Physical exam (including CBC Blood culture culture Tympanocentesis LP—CSF analysis XR—L-spine/knee MRI—L-spine Neonatal sepsis Meningitis Pneumonia Pyelonephritis Acute otitis media URI Meningitis Pyelonephritis Malingering Lumbar muscle strain Disk herniation Knee or leg fracture Ankylosing spondylitis (cont’d) recently had a cold. The child attends a day care center. 3 yo M presents with a 2-day history of fever and and He is otherwise healthy, pulling on his right ear. his immunizations are up to date. His older sister 20-day-old M presents with fever, decreased breast- decreased 20-day-old M presents with fever, He was born at 36 weeks as a feeding, and lethargy. result of premature rupture of membranes. during gym class. He also describes areas of loss of during gym class. He foot. The pain and sensory loss sensation in his left distribution. He insists on do not match any known from school because of his in- requesting a week off jury. 17 yo M presents with low back pain that radiates 17 yo M presents with began after he fell on his knee to the left leg and CHILD WITH FEVER LOW BACK PAIN PAIN LOW BACK Key Physical Exam neck, heart, lung, abdominal, and skin exams. signs; HEENT, Vital Presentation Differential Workup Key History as rash, wheezing, cough, and ear discharge; poor appetite, duration; associated localizing symptoms such Severity, immunizations. sleepiness; sick contacts, day care, convulsions, lethargy, No child will be present; the mother will relate the story. When you enter the examination room, you may see a will relate the story. No child will be present; the mother the handset. Upon doing so, you will be speaking to the parent of the child. telephone with instructions to pick up Presentation Differential Workup MINICASES 134 exam; pelvicexam(women). Vital signs;examforsignsofdehydration(BP, heartrate,skinturgor);and lungexams;abdominalexam;rectal Key PhysicalExam sick contacts,daycare,immunizations. color; exacerbatingandalleviatingfactors;historyofsimilarsymptoms;abdominalsurgeries;medications; tional, GI,cardiac,pulmonary, stools,changeinstool renal,pelvic);changesinweight,skinrash,bloody/mucoid Onset, location,quality, intensity, duration,radiation,timing(relationtomeals);associatedsymptoms(constitu- Key History No childwillbepresent;onlytheparentrelatestory, eitherinpersonorbytelephone. Workup Differential Presentation CHILD WITHFEVER CHILD WITHGISYMPTOMS symptoms. center whereseveralchildrenhavehadsimilar weakness, andfever. Thechildattendsadaycare 4 yoMpresentswithdiarrhea,vomiting,lethargy, vaccine. face andbody. HehasnotyetreceivedtheMMR days accompaniedbyamaculopapularrashonhis 12-month-old Mpresentswithfeverforthepast2 (cont’d) Gastroenteritis Meningitis Scarlet fever Varicella Fifth disease Roseola Rubella Measles (orotherviral Intussusception Volvulus URI UTI poisoning Food exanthem) bacterial, parasitic) (viral, LP—CSF analysis Throat swabforculture Viral antibodies/titers CBC Physical exam AXR UA, urineculture Electrolytes CBC Stool examandculture Physical exam MINICASES 135 4 occult blood alkaline phosphatase Stool exam Suction rectal biopsy Anorectal manometry TSH, FT Physical exam Rectal exam, stool for CBC Electrolytes Contrast enema U/S—abdomen CT—abdomen Physical exam CBC Electrolytes U/S—abdomen CT—abdomen Amylase, lipase Physical exam CBC Electrolytes U/S—abdomen Barium swallow pH probe Endoscopy AST/ALT/bilirubin/ UA, urine culture culture Tympanocentesis Physical exam Rectal exam Stool exam and culture Barium enema CBC Electrolytes Serum lead level Lead poisoning Intussusception Appendicitis diverticulum Meckel’s Volvulus Gastroenteritis Enterocolitis Blunt abdominal trauma Somatoform disorder Malingering Irritable bowel syndrome Lactose intolerance Child abuse Pyloric stenosis Partial duodenal atresia GERD Gastroenteritis Hepatitis UTI Otitis media disease Hirschsprung’s Low-fiber diet Anal stenosis Hypothyroidism (cont’d) has missed many school days because of the pain. Growth and development are normal. His parents recently divorced. 7 yo M presents with abdominal pain that is general- in the morning, and seemingly worse ized, crampy, less prominent during weekends and holidays. He abdominal pain with vomiting. The episodes are 20 well be- minutes apart, and the child is completely tween episodes. She had loose stools several hours before the pain, but her stools are now bloody. 8-month-old F presents with sudden-onset colicky 8-month-old F presents with sudden-onset had 1 bowel movement per week since birth despite had 1 bowel movement per week since did not pass the use of stool softeners. At birth, he weight gain. meconium for 48 hours. He has poor There is a family history of this problem. 3 yo M presents with constipation. The child has 3 yo M presents with constipation. The iting episodes and have increased in frequency is nonbloodyforcefulness. Emesis and nonbilious. The episodes occur immediately after breast- usually gaining weight. feeding. She has stopped 1-month-old F is brought in because she has been 1-month-old F is brought for the last 10 days. The vom- spitting up her milk CHILD WITH GI SYMPTOMS WITH GI SYMPTOMS CHILD Presentation Differential Workup MINICASES 136 rsnainDfeeta Workup birth history, growthhistory; past medicalhistory;familycognitiveabilities,school performance. Associated symptoms(constitutional,GI,cardiac,pulmonary, renal,pelvic,endocrine);medications;prenataland Key History Differential No childwillbepresent;onlytheparentrelatestory, eitherinpersonorbytelephone. Presentation Vital signs;HEENTexam. Key PhysicalExam Workup leviating factors;medications;sickcontacts,daycare,immunizations;historyofsimilarsymptoms. symptoms (constitutional,dermatologic,GI,cardiac,pulmonary, renal,pelvic,rheumatologic);exacerbatingandal- Onset, location,duration,affectingoneorbotheyes;eyedischarge,itching,pain,photophobia,tearing;associated Key History Differential No childwillbepresent;onlytheparentrelatestory, eitherinpersonorbytelephone. Presentation CHILD WITHGISYMPTOMS CHILD WITHSHORT STATURE CHILD WITHREDEYE atopic dermatitis. the flulastweek.Shehasahistoryofasthmaand opening hereyesinthemorning.Hermotherhad She hasmucoiddischarge,itching,anddifficulty began intherighteyebutnowinvolvesbotheyes. 3 yoFpresentswitha3-dayhistoryof“pinkeye.”It ver. or growth.Thereisnovomiting,constipation,fe- eructation. Thereisnochangeinappetite,weight, subsideafterpassingflatusor 2 weeks.Theepisodes 2-month-old Mpresentswithpersistentcryingfor (cont’d) Gastroenteritis Testicular torsion Strangulated hernia Lactose intolerance GERD Formula allergy Colic Uveitis Seasonal allergies Keratitis Viral conjunctivitis Bacterial conjunctivitis U/S—testicular U/S—abdomen Rectal exam,stoolfor Physical exam Slit lampexam Discharge cultures Electrolytes CBC Ophthalmoscopic eye Physical exam occult blood exam MINICASES 137 4 4 XR—hand U/S—renal and cardiac Sweat chloride testing BUN/Cr Karyotype Physical exam CBC Electrolytes GH stimulation test IGF-1, IGFBP-3 levels TSH, FT EEG Physical exam Mental status exam TSH, FT hyperactivity disorder (ADHD) disorder stature deficiency insufficiency Hyperthyroidism Attention-deficit Oppositional defiant Manic episode Conduct disorder Constitutional short Constitutional short Growth hormone (GH) Hypothyroidism Chronic renal Genetic causes Cystic fibrosis (cont’d) regularly interrupts his classmates, and has trouble making friends. 9 yo M presents with a 2-year history of angry out- bursts both in school and at home. His mother com- plains that he runs around “as if driven by a motor.” His teacher reports that he cannot sit still in class, were young, but they are now of normal stature. were young, but they 14 yo M presents with short stature and lack of sex- 14 yo M presents with birth weight and length were ual development. His shortest child in his class. His normal, but he is the the same problem when they father and uncles had CHILD WITH SHORT STATURE STATURE WITH SHORT CHILD BEHAVIORAL PROBLEMS IN CHILDHOOD PROBLEMS IN BEHAVIORAL Key Physical Exam exam. signs; neurologic Vital Presentation Differential Workup No child will be present; only the parent will relate the story, either in person or by telephone. will relate the story, No child will be present; only the parent Key History changes of weapons; substance use, developmental history, duration, triggers; physical violence or use Onset, severity, anhedonia. change in personality, in environment or school performance; Presentation Differential Workup Key Physical Exam Key Physical neurologic exams. heart, lung, abdominal, and signs; height, weight; HEENT, Vital MINICASES 138 rsnainDfeeta Workup Differential Presentation BEHAVIORAL PROBLEMSINCHILDHOOD care. does notdisplaythesebehaviorswhenheisatday trouble sleeping,hashadnochangeinappetite,and follow adisappointmentordiscipline.Hehasno tantrums thatlast5–10minutesandimmediately 5 yoMpresentswitha6-monthhistoryoftemper he doesgoout,itiswithanewsetoffriends. most ofhistimealoneinroom,addingthatwhen cluding shoplifting.Hisparentsreportthathespends grades, schoolabsenteeism,andlegalproblems,in- 15 yoMpresentswitha1-yearhistoryoffailing average studentwithnobehavioralproblems. new district.Beforeherparentsdivorced,shewasan recently divorced,andshejuststartedschoolina in school,truancy, andbreakingcurfew. Herparents 12 yoFpresentswitha2-monthhistoryoffighting ADHD Age-appropriate behavior Adjustment disorder Oppositional defiant Conduct disorder Substance abuse Conduct disorder Oppositional defiant Manic episode Substance intoxication, Adjustment disorder Oppositional defiant disorder disorder abuse, ordependence disorder (cont’d) Mental statusexam Physical exam Physical exam Mental statusexam Urine toxicology Urine toxicology Mental statusexam Physical exam SECTION 4

Practice Cases

Case 1 46-year-old man with chest pain / 142 Case 2 57-year-old man with bloody urine / 152 Case 3 51-year-old man with back pain / 161 Case 4 25-year-old man presents following motor vehicle accident / 170 Case 5 28-year-old woman presents with positive pregnancy test / 180 Case 6 10-year-old girl with new-onset diabetes / 189 Case 7 74-year-old man with right arm pain / 197 Case 8 56-year-old man presents for diabetes follow-up / 206 Case 9 25-year-old woman presents following sexual assault / 216 Case 10 35-year-old woman with calf pain / 225 Case 11 62-year-old man with hoarseness / 235 Case 12 67-year-old woman with neck pain / 243 Case 13 48-year-old woman with abdominal pain / 251 Case 14 35-year-old woman with headaches / 260 Case 15 36-year-old woman with menstrual problems / 269 Case 16 28-year-old woman with pain during sex / 278 Case 17 75-year-old man with hearing loss / 287 Case 18 5-day-old boy with jaundice / 296 Case 19 7-month-old boy with fever / 305 Case 20 26-year-old man with cough / 314 Case 21 52-year-old woman with jaundice / 323 Case 22 53-year-old man with dizziness / 332 PRACTICE CASES 140 Case 42 Case 41 Case 40 Case 39 Case 38 Case 37 Case 36 Case 35 Case 34 Case 33 Case 32 Case 31 Case 30 Case 29 Case 28 Case 27 Case 26 Case 25 Case 24 Case 23 Case 44 Case 43 49-year-old manwithlossofconsciousness/532 21-year-old manwithsore throat /523 11-month-old girlwithseizures /515 8-year-old boywithbed-wetting/507 6-month-old girlwithdiarrhea/499 30-year-old womanwithweightgain/490 66-year-old manwithtremor /481 55-year-old manwithbloodystool/472 32-year-old manpresents forpreemployment physical/463 27-year-old manwithvisualhallucinations/454 32-year-old womanwithfatigue/445 46-year-old manwithfatigue/436 65-year-old womanwithforgetfulness andconfusion/426 21-year-old womanwithabdominalpain/416 2-year-old girlwithnoisybreathing /407 20-year-old womanwithsleepingproblems /397 54-year-old manpresents forhypertensionfollow-up/388 61-year-old manwithfatigue/379 54-year-old womanwithcough/369 18-month-old girlwithfever/360 31-year-old manwithheelpain/350 33-year-old womanwithkneepain/340 PRACTICE CASES 141 This sheet includes a sample patient note This sheet includes a sample patient Designed to simulate the actual information that information that to simulate the actual Designed This sheet outlines information that standardized patients information that standardized patients This sheet outlines A blank form is supplied on which you can write your own A blank form is supplied on which you for you to review after you have written your own, as well as a discussion of for you to review after you have written tests to consider in each case. reasonable differential diagnoses and diagnostic changes, this form includes blank matrices that you can use to outline the three changes, this form includes blank matrices history and exam findings that support most likely differential diagnoses; the each; and the initial testing modalities to establish a that you have proposed definitive diagnosis. sample checklist that SPs will use to evaluate your performance in the areas of SPs will use to evaluate your performance sample checklist that recommendations, as diagnosis, closure, and follow-up entrance, history taking, interview. well as your ability to conduct a patient-centered In accordance with recent exam note after you complete the patient encounter. ing the exam. You should read this sheet just before starting the 15-minute sheet just before starting the 15-minute should read this ing the exam. You encounter. questions SPs might them during the interview and lists (SPs) will use to guide It also includes a potential responses to these questions. ask you, along with you will find on the doorway of each examination room, this sheet contains doorway of each examination room, you will find on the to perform dur- vital signs, and the tasks you are required the opening scenario, in accordance with the guidelines provided in Sections 1 and 2 and compare each of in accordance with the guidelines provided in the text. your patient notes with those provided note after reviewing the you can try to formulate a patient For a quicker self-review, sample doorway sheet and the SP checklist, and then compare your note with the note provided. Because the cases in this section are designed to simulate the actual exam, you will Because the cases in this section are designed To them with a friend who can act as an SP. derive the most benefit by practicing cases, you should also time each encounter maximize the effectiveness of these practice 4. Sample patient note and discussion: 3. Blank patient note: 2. Checklist/SP sheet: This section consists of 44 commonly encountered cases that approximate those you cases that approximate encountered consists of 44 commonly This section of four parts: Each case consists Step 2 CS exam. on the actual USMLE might find 1. sheet: Doorway information PRACTICE CASES 142 Entrance Building theDoctor-Patient Relationship Examinee Checklist Notes fortheSP Patient isa46yoM. Patient Description Checklist/SP Sheet Examinee Tasks HR: RR: Temp: BP: Signs Vital Joseph Short,a46-year-old male,comestotheEDcomplainingofchestpain. Opening Scenario DOORWAY INFORMATION CASE 1 if yourpainislifethreatening.” “As yoususpect,yoursymptomsareofsignificantconcern. We needtolearnmoreaboutwhat’s goingontoknow Sample ExamineeResponse “Is thisaheartattack?AmIgoingtodie?” Challenging QuestionstoAsk Examinee introduced selfbyname. Examinee introduced Examinee knockedonthedoor beforeentering. 4. Write thepatientnoteafterleavingroom. 3. Explainyourclinicalimpressionandworkupplantothepatient. 2. Performafocusedphysicalexam(donotperformrectal,genitourinary, orfemalebreastexam). 1. Take afocusedhistory. 165/85mmHg 22/minute 90/minute,regular If ECGismentionedbytheexaminee,ask,“WhatanECG?” Exhibit difficultybreathing. Place yourhandsinthemiddleofchest. Lie onthebedandexhibitpain. 98.6°F(37°C) PRACTICE CASES 143 None. Patient Response this pain. Precipitating eventsAlleviating factorsAssociated symptomsCurrent medications up the stairs, strenuous work, and heavy meals. Walking Antacids. None. Maalox, diuretic. Previous episodes of similar painOnsetSeverity but not exactly the same. Yes, Frequency The past 3 months. Less severe. I have had 2−3 episodes a week, each lasting 5–10 minutes. Nausea/vomitingSweatingAssociated symptoms (cough, wheez- ing, abdominal pain, diarrhea/constipa- tion) vomit. I feel nauseated, but I didn’t Yes. RadiationQualityAlleviating/exacerbating factorsShortness of breath Nothing. my neck, upper back, and left arm. To Yes. Pressure. Like something sitting on my chest. Precipitating eventsProgressionSeverity on a scaleLocation morning having Nothing; I was asleep and woke up at 5:00 in the 7/10. Constant severity. right underneath the bone. Middle of the chest. It feels as if it’s Chief complaintOnset Chest pain. ago. Forty minutes Question

Examinee asked the SP to list his/her concerns and listened to the response without interrupting. SP to list his/her concerns and listened Examinee asked the own words. SP’s concerns, often using the the SP’s Examinee summarized efficiently and accurately. Examinee elicited data Examinee identified his/her role or position. identified his/her Examinee name. correctly used patient’s Examinee with the SP. made eye contact Examinee to the response. open-ended question and actively listened Examinee asked an ✓ Information Gathering Reflective Listening PRACTICE CASES 144 Physical Examination Connecting withthePatient ✓ ✓ Examinee didnotrepeatpainfulmaneuvers. Examinee usedrespectfuldraping. Examinee askedpermissiontostarttheexam. Examinee washedhis/herhands. Examinee recognizedtheSP’s emotionsandrespondedwithPEARLS.

ExamComponent Question Extremities Checked peripheral pulses, checked blood pressureinbotharms, Checkedperipheral pulses,checkedblood Auscultation, palpation,percussion Inspection,auscultation,palpation Auscultation, palpation,percussion LookedforJVD,carotid auscultation Extremities Abdominal exam Pulmonary exam CV exam Neck exam Mydoctorgavemeastrictdietlastyeartolowermycholesterol,but No. No. Onepackaday. Well, doctor, tobehonest,Ihaven’t hadsexwithmywifeforthe Twenty-five years. Drug allergies Stopped3monthsago. Diet Exercise Cocaine,onceaweek. Sexual activity Yesterday afternoon. Onceinawhile. TenAmount years. Accountant. Duration Tobacco Myfatherdiedoflungcanceratage72.motherisaliveandhas Last timeofcocaineuse Duration ofcocaineuse None. Illicit druguse Alcohol use Occupation Hypertensionfor5years,treatedwithadiuretic.Highcholesterol, Family history Past surgicalhistory Past medicalhistory a pepticulcer. Noearlyheartattacks. GERD 10yearsago,treatedwithantacids. managed withdiet;Ihavenotbeenverycompliantthediet. Patient Response looked foredemaandcyanosis Maneuver I alwayscheat. past 3monthsbecauseIgetthispaininmychestduringsex. PRACTICE CASES 145 ify the Follow-up tests. Lifestyle modification (diet, exercise). Examinee asked if the SP had any other questions or concerns. Examinee asked if the Examinee discussed initial diagnostic impressions. discussed initial diagnostic Examinee plans: discussed initial management Examinee source of your problem. We will start with an ECG and some blood work, but more complex tests may be needed as well. In the ECG and some blood work, but more complex will start with an We source of your problem. stop using cocaine, since use of this drug can lead to a variety of medical problems, meantime, I strongly recommend that you questions for me? including heart attacks. Do you have any Sample Closure be due to acid reflux, such as a heart attack or angina, or it may of your pain can be a cardiac problem Short, the source Mr. some tests to ident your chest. It is crucial that we perform related to the large blood vessels in lung problems, or disorders Closure PRACTICE CASES 146 Physical Examination History USMLE STEP2CS Patient Note PRACTICE CASES 147 Patient Note Patient Diagnosis #3 History Finding(s): Physical Exam Finding(s): Diagnosis #2 History Finding(s): Physical Exam Finding(s): Diagnosis #1 History Finding(s): Physical Exam Finding(s): USMLE STEP 2 CS STEP USMLE Diagnostic Workup Differential Diagnosis Differential USMLE STEP 2 CS Patient Note

History HPI: 46 yo M c/o substernal chest pain. The pain started 40 minutes before the patient presented to the ED. The pain woke the patient from sleep at 5:00 A.M. with a steady 7/10 pressure sensation in the middle of his chest that radiated to the left arm, upper back, and neck. Nothing makes it worse or better. Nausea, sweating, and dyspnea are also present. Similar episodes have occurred during the past 3 months, 2–3 times/week. These episodes were precipitated by walking up the stairs, strenuous work, sexual intercourse, and heavy meals. Pain during these episodes was less severe, lasted for 5–10 minutes, and disappeared spontaneously or after taking antacids. ROS: Negative except as above. Allergies: NKDA. Medications: Maalox, diuretic. PMH: Hypertension for 5 years, treated with a diuretic. High , managed with diet. GERD 10 years ago, treated with antacids. SH: 1 PPD for 25 years; stopped 3 months ago. Occasional EtOH, occasional cocaine for 10 years (last used yesterday afternoon). No regular exercise; poorly adherent to diet. FH: Father died of lung cancer at age 72. Mother has peptic ulcers. No early coronary disease.

Physical Examination Patient is in severe pain. VS: BP 165/85 mm Hg (both arms), RR 22/minute. Neck: No JVD, no bruits. Chest: No tenderness, clear symmetric breath sounds bilaterally. Heart: Apical impulse not displaced; RRR; normal S1/S2; no murmurs, rubs, or gallops. Abdomen: Soft, nondistended, nontender, BS, no hepatosplenomegaly. Extremities: No edema, peripheral pulses 2+ and symmetric.

Differential Diagnosis Diagnosis #1: Myocardial ischemia or infarction History Finding(s): Physical Exam Finding(s): Pressure-like substernal chest pain Pain radiates to left arm, upper back, and neck Pain awakens patient at night

Diagnosis #2: Cocaine-induced myocardial ischemia History Finding(s): Physical Exam Finding(s): History of cocaine use Last used yesterday afternoon Pressure-like substernal chest pain PRACTICE CASES

148 USMLE STEP 2 CS Patient Note

Diagnosis #3: GERD History Finding(s): Physical Exam Finding(s): Pain in midchest Previous pain was relieved by antacids Previous pain occurred after heavy meals

Diagnostic Workup

ECG Cardiac enzymes (CPK, CPK-MB, troponin) Transthoracic echocardiography Upper endoscopy Urine toxicology PRACTICE CASES

149 PRACTICE CASES 150 Diagnostic Workup Additional Differential Diagnoses Patient NoteDifferential Diagnoses CASE DISCUSSION Cardiac catheterization: Urine toxicology: together withendoscopicvisualizationconstitutesaneffective diagnostictechnique. normal inuptoone-halfofsymptomaticpatients;esophageal probe(pHandmanometrymeasurements) Upper endoscopy: acute MI(infarctionisunlikelyintheabsenceofwallmotion abnormalities). Transthoracic echocardiography (TTE): positive asearly4–6hoursafteronsetofpain. Cardiac enzymes(CPK,CPK-MB,troponin): ECG: palpation orpleuriticpain. Costochondritis (orothermusculoskeletalchestpain): this patienthasnoapparentriskfactorsforpulmonaryembolism. Pulmonary embolism: but itislesslikelyinthiscasegiventhatbreathsoundsaresymmetric. Pneumothorax: friction rubmakepericarditislesslikely. Pericarditis: aortic dissection). aortic regurgitantmurmur(althoughphysicalexamfindingshavepoorsensitivityandspecificitytodiagnose pressuresarenotdiminishedorunequal,andthereisno the back.Inaddition,hisperipheralpulsesandblood thrombolytic therapy).However, thepatient’s painisnottheclassicsudden,tearingchestthatradiatesto the highpotentialfordeathifmissed(andharmmistakenacuteMIandtreatedwith Aortic dissection: heartburn, whichmaybeexacerbatedbymeals. Other atypicalsymptomsincludechroniccough,wheezing,anddysphagia.TheclassicsymptomofGERDis GERD: myocardial energyrequirements. induce myocardialischemiaandinfarctioneitherbycausingcoronaryarteryvasoconstrictionorincreasing Cocaine-induced myocardial ischemia: hypertension, andhyperlipidemia,hissymptomsareclassicforcardiacischemia. Myocardial ischemiaorinfarction: done rapidlyatthebedside. Transesophageal echocardiography (TEE): Acutemyocardialischemia,infarction,andpericarditishavecharacteristicchangesonECG. SeverechestpainisatypicalbutnotuncommonforGERDandmayworsenwithrecumbencyovernight. Theabsenceofpainthatchangeswithpositionorrespirationandtheapericardial Thisdiagnosisshouldbeconsideredinapatientwithacutechestpainanddifficultybreathing, To helpconfirmthepatient’s historyofrecentcocaine use. With thesuddenonsetofseverechestpain,aorticdissectionshouldbesuspectedgiven Canbeusedtodocumenttissuedamagecharacteristicof GERD.However, itcanbe Asabove,thisisonthedifferentialforacutechestpainanddifficultybreathing,but Candiagnoseandtreatcoronary arterydisease. Thepatienthasmultiplecardiacriskfactors,includingsmoking, Cocainecanpredisposetoprematureatherosclerosisand Candemonstratesegmentalwallmotionabnormalitiesin suspected Highlyspecificandsensitive foraorticdissection,andcanbe Specifictestsformyocardialtissuenecrosisthatcanturn Thisismoretypicallyassociatedwithpainon PRACTICE CASES 151 Another rapidly available diagnostic study that can rule out aortic dissection or can rule out aortic study that rapidly available diagnostic Another Can identify a critical risk factor for cardiovascular disease. Can identify a critical A widened mediastinum suggests aortic dissection and may reveal other causes of chest pain, including pain, including other causes of chest and may reveal aortic dissection mediastinum suggests A widened CT—chest with IV contrast: CT—chest pulmonary embolism. panel: Cholesterol CXR: and pneumonia. pneumothorax PRACTICE CASES 152 Entrance Building theDoctor-Patient Relationship Examinee Checklist Notes fortheSP Patient isa57yoM. Patient Description Checklist/SP Sheet Examinee Tasks HR: RR: Temp: BP: Signs Vital Carl Fisher, a57-year-old urine. male,comestotheEDcomplaining ofbloody Opening Scenario DOORWAY INFORMATION CASE 2 “No. Bloody urineisrarelynormal.We“No. Bloody willneedtorunafewmoretestsdetermine thecauseofthisfinding.” Sample ExamineeResponse inmyurineisbecauseofoldage.Isthattrue?” “They toldmethathavingblood Challenging QuestionstoAsk Examinee madeeyecontact with theSP. Examinee correctlyusedpatient’s name. Examinee identifiedhis/her role orposition. selfbyname. Examinee introduced Examinee knockedonthedoorbeforeentering. 4. Write thepatientnoteafterleavingroom. 3. Explainyourclinicalimpressionandworkupplantothepatient. 2. Performafocusedphysicalexam(donotperformrectal,genitourinary, orfemalebreastexam). 1. Take afocusedhistory. 130/80mmHg 13/minute 72/minute,regular If theexamineementionsprostatedisease,ask,“What’s prostatedisease?” Show painwhentheexamineechecksforCVA tendernessontheright. 98.5°F(36.9°C) PRACTICE CASES 153 None. None. alive and healthy. Patient Response normal. Illicit drug useTobacco No. I have smoked a pack a day for 30 years. Yes, Family historyOccupationAlcohol use My mother is My father died at age 80 because of a kidney problem. Painter. A couple of beers after work, 2–3 times a week. Previous similar episodesCurrent medicationsPast medical historyPast surgical history No. Allopurinol. Gout. at age 23. Appendectomy History of renal stonesAssociated symptoms (nausea/vomit- ing, diarrhea/constipation) Constitutional symptoms (weight loss, appetite changes, night sweats) No. Nocturia streamWeak DribblingOnset of the previous symptoms They told me I am getting old; am I? years ago. Two Yes. Yes. Yes. FeverAbdominal/flank painPolyuria, frequencyStraining during urination None. Yes. every 2−3 hours now. I have to go to the bathroom Yes. None. How did he know it was blood?How did he know it OnsetProgression It was bright red and later had some clots. Pain/burning on urination None. time it has ever happened; my urine is back to That was the only morning. Yesterday Chief complaint I have blood in my urine, doctor. Question

Examinee elicited data efficiently and accurately. Examinee elicited data Examinee asked an open-ended question and actively listened to the response. listened to the question and actively asked an open-ended Examinee interrupting. response without and listened to the his/her concerns asked the SP to list Examinee own words. SP’s concerns, often using the the SP’s Examinee summarized ✓ Reflective Listening Reflective Information Gathering PRACTICE CASES 154 determine ifthereisastoneinyourkidneys,ananatomicabnormality, oratumor. Doyouhaveanyquestionsforme? order aurinetesttolookforsignsofinfection.Dependingon theresultsweobtain,Imayalsoordersomeimagingstudiesto First Iwilldrawsomeblood,andthenperformagenital examaswellarectaltoassessyourprostate.Iwillt Mr. Fisher, thebloodinyoururinecouldbecausedbyavarietyoffactors,soIwouldliketodofewtestselicitanans Sample Closure Closure Physical Examination Connecting withthePatient ✓ ✓ Examinee askediftheSPhadanyotherquestionsorconcerns. Examinee discussedinitialmanagementplans: Examinee discussedinitialdiagnosticimpressions. Examinee didnotrepeatpainfulmaneuvers. Examinee usedrespectfuldraping. Examinee askedpermissiontostarttheexam. Examinee washedhis/herhands. Examinee recognizedtheSP’s emotionsandrespondedwithPEARLS.

ExamComponent Question xrmte Inspection Auscultation,palpation,percussion,checkedforCVA tenderness Auscultation Auscultation Extremities Abdominal exam Pulmonary exam CV exam No. None. Regularly. Women only. Ihaveagirlfriend;mether2yearsagothroughmutualfriend. Drug allergies History ofSTDs Use ofcondoms Sexual orientation Sexual activity Follow-up tests:Examineementionedtheneedforagenitalexamandrectalprostate. Patient Response Maneuver wer. hen PRACTICE CASES 155 Patient Note Patient USMLE STEP 2 CS STEP USMLE Physical Examination History USMLE STEP 2 CS Patient Note

Differential Diagnosis Diagnosis #1 History Finding(s): Physical Exam Finding(s):

Diagnosis #2 History Finding(s): Physical Exam Finding(s):

Diagnosis #3 History Finding(s): Physical Exam Finding(s):

Diagnostic Workup PRACTICE CASES

156 USMLE STEP 2 CS Patient Note

History HPI: 57 yo male c/o 1 episode of painless hematuria yesterday morning. He has no fever, no abdominal or flank pain, and no dysuria. No history of renal stones. He has a 2-year history of straining on urination, polyuria, nocturia, weak urinary stream, and dribbling. No nausea, vomiting, diarrhea, or constipation. No change in appetite or weight loss. No previous similar episodes. ROS: Negative except as above. Allergies: NKDA. Medications: Allopurinol. PMH: Gout. PSH: Appendectomy, age 23. SH: 1 PPD for 30 years, 2 beers 2–3 times/week, no illicit drugs. Works as a painter. Heterosexual, has a partner, and uses condoms regularly. FH: Father died from kidney disease at age 80.

Physical Examination Patient is in no acute distress. VS: WNL. Chest: Clear breath sounds bilaterally. Heart: RRR; normal S1/S2; no murmurs, rubs, or gallops. Abdomen: Soft, nondistended, nontender, BS, no hepatosplenomegaly. Mild right CVA tenderness. Extremities: No edema.

Differential Diagnosis Diagnosis #1: Bladder cancer History Finding(s): Physical Exam Finding(s): Hematuria Straining on urination Weak urinary stream and dribbling Works as painter (exposure to industrial solvents) History of smoking 1 PPD × 30 years

Diagnosis #2: Urolithiasis History Finding(s): Physical Exam Finding(s): Hematuria CVA tenderness PRACTICE CASES Straining on urination

157 USMLE STEP 2 CS Patient Note

Diagnosis #3: Benign prostatic hypertrophy History Finding(s): Physical Exam Finding(s): Polyuria, nocturia Weak urinary stream and dribbling Straining on urination

Diagnostic Workup

Genital exam Rectal exam Cystoscopy U/S—renal UA CT—abdomen/pelvis PSA PRACTICE CASES

158 PRACTICE CASES 159 ematologic or H tones. Gynecologic sources may tones. Gynecologic S —etiologies include —etiologies include HITTERS enal disorders, and enal disorders, and R xercise, E The patient’s urinary symptoms are classic for this diagnosis except urinary symptoms are classic The patient’s umor, umor, T rauma, T To evaluate the urinary tract. Can identify neoplasms and a variety of benign evaluate the urinary tract. Can identify neoplasms and a variety To The classic triad is hematuria, flank pain, and a palpable mass. Constitutional symptoms The classic triad is hematuria, flank pain, The absence of hypertension or signs of volume overload (eg, edema) argues against The absence of hypertension or signs nfection, I As above, this diagnosis is plausible but is hard to reconcile with the presence of CVA is hard to reconcile with the presence of CVA As above, this diagnosis is plausible but Hematuria and irritative voiding symptoms are consistent with this diagnosis, and the voiding symptoms are consistent Hematuria and irritative To exclude a urologic source of bleeding in men. To To detect masses as well as prostatic enlargement or nodules. detect masses as well as prostatic enlargement or To The gold standard for the diagnosis of bladder cancer. Despite the presence of hematuria and CVA tenderness, this very common diagnosis is unlikely in tenderness, this very common of hematuria and CVA Despite the presence Can detect bladder and renal masses and stones, but is operator dependent and less sensitive in Can detect bladder and renal masses and stones, but is operator dependent The serum level correlates with the volume of both benign and malignant prostatic tissue. It can be The serum level correlates with the volume of both benign and malignant This can cause hematuria but is uncommon in males. The patient has no other symptoms to suggest acute This can cause hematuria but is uncommon To assess hematuria, pyuria, bacteriuria, and the like. Dysmorphic RBCs or casts are signs of glomerular assess hematuria, pyuria, bacteriuria, and the like. To conditions, such as stones. PSA: normal in about 20% of patients with nonmetastatic prostate cancer. detecting ureteral disease. UA: disease. CT—abdomen/pelvis: Genital exam: Rectal exam: Cystoscopy: U/S—renal: symptoms, or a flulike illness. UTI: infection. Renal cell carcinoma: other urinary symptoms may be due to coexisting BPH. may be prominent. The patient’s Glomerulonephritis: remember that IgA nephropathy is the most common acute glomerulonephritis intrinsic renal disease. However, and most often presents with an episodePresentation is usually concurrent with URI, GI of gross hematuria. pathology. cancer: Prostate to a right posterior rib). tenderness (could postulate metastasis Urolithiasis: flank pain. Pain may migrate to the groin and is not alleviated by changes the absence of sudden, severe colicky in position. (BPH): hypertrophy Benign prostatic may signal upper urinary tract tenderness Again, CVA that hematuria (if present) is usually microscopic. Bladder cancer: However, to industrial solvents are risk factors. smoking and possible occupational exposure cigarette patient’s urinary tract disease. is unusual and could be a sign of upper tenderness CVA the finding of right Diagnostic Workup Additional Differential Diagnoses Additional Differential need to be excluded in women. The passage of clots often localizes the source of bleeding to the lower urinary tract. localizes the source of bleeding to the in women. The passage of clots often need to be excluded otherwise. adults represents malignancy until proven Gross hematuria in CASE DISCUSSION CASE Diagnoses Note Differential Patient is for the differential diagnosis of hematuria A useful mnemonic coagulation disorders, PRACTICE CASES 160 urography tocircumventtheneedforcontrastadministration. IVP: BUN/Cr: tumor. Threevoidedsamplesshouldbeexaminedtomaximizesensitivity. Urine cytology: Urine culture: Providesanassessmentofthekidneys,ureters,andbladder. IVPhasgenerallybeenreplacedbyCT To evaluatekidneyfunction. To excludeUTI. Hasvariablesensitivityindetectingbladdercancers,dependingonthegradeandstageof PRACTICE CASES 161 98.2°F (36.8°C) Pretend that you have paraspinal lower back tenderness when examined. Pretend that you have paraspinal lower strength in both lower extremities. Show normal reflexes, sensation, and Lean forward slightly when walking. 80/minute, regular 20/minute 120/85 mm Hg Examinee introducedself by name. Examinee identified his/her role or position. Examinee knocked on the door before entering. 2. Perform a focused physical exam (do not perform rectal, genitourinary, or female breast exam). not perform rectal, genitourinary, 2. Perform a focused physical exam (do workup plan to the patient. 3. Explain your clinical impression and the patient note after leaving the room. 4. Write 1. Take a focused history. a focused 1. Take “I don’t think I can go to work, doctor. Can you write a letter to my boss so that I can have some time off?” think I can go to work, doctor. “I don’t Sample Examinee Response ask your I will construction work can worsen your back pain or cause it to heal more slowly. right; heavy “You’re boss to reassign you to light duty for a while.” Challenging Questions to Ask DOORWAY INFORMATION DOORWAY CASE 3 CASE Examinee Checklist Relationship Building the Doctor-Patient Entrance Checklist/SP Sheet Patient Description girlfriend. Patient is a 51 yo M who lives with his Notes for the SP RR: HR: Examinee Tasks Rick Meyer, a 51-year-old male construction worker, comes to the office complaining of back pain. comes worker, male construction a 51-year-old Rick Meyer, Vital Signs BP: Temp: Opening Scenario PRACTICE CASES 162 Information Gathering Reflective Listening ✓ Examinee eliciteddataefficientlyandaccurately. Examinee summarizedtheSP’s concerns,oftenusingtheSP’s ownwords. Examinee askedtheSPtolisthis/herconcernsandlistenedresponsewithoutinterrupting. Examinee askedanopen-endedquestionandactivelylistenedtotheresponse. Examinee madeeyecontactwiththeSP. Examinee correctlyusedpatient’s name.

Question lii rgueNever. Yes, acoupleofbeers ontheweekends. No(toall4). Constructionworker. Myfatherdied ofaheartattackatage65,andmymotherishealthy. Illicit druguse None. CAGE questions None. Alcohol use Itakeibuprofen. Ithelps,butthepainisstillthere. Occupation Family history Past surgicalhistory Past medicalhistory Well, forthe pastyearIhavebeenhavingbackpainonandoff, Current medications No. Inoticedthatoverthepast6monthshavehadtostraininorder No. None. History ofbackpaininthepast Sharp. Walking, sittingforalongtime,coughing. Fever, nightsweats,weightloss Lying still in bed. Itradiatestomyleftthighandsometimesreachesfoot. Urinary orfecalincontinence Themiddleofmylowerback. Difficulty urinating Weakness/numbness Ithasbeenthesame. 8/10. Exacerbating factors Oneweekago. Alleviating factors Quality Radiation Location Paininmyback. Severity onascale Iwasliftingsomeheavyboxes;thenmybackstartedhurtingright Progression Associated/precipitating events Onset Chief complaint away. That paingoesawaywhenIstopwalkingandsitdown. mainly whenIwalk.Itisusuallyaccompaniedbypaininmy legs. to urinate.SometimesIfeelasifhaven’t emptied mybladderfully. Patient Response PRACTICE CASES

g 163 Patient Response Patient Maneuver passive straight leg raising, sensory exam Follow-up tests: Examinee mentioned the need for a rectal exam. Follow-up tests: Examinee mentioned Back examExtremitiesNeurologic exam Inspection, palpation, range of motion sign, gait (including toe and heel walking), DTRs, Babinski’s Motor, of peripheral pulses, hip exam Inspection, palpation TobaccoDrug allergies Penicillin gives me a rash. a day for the past 18 years. a pack Yes, Exam Component Question

Examinee asked if the SP had any other questions or concerns. Examinee asked if the SP had any other Examinee discussed initial diagnostic impressions. Examinee discussed initial diagnostic plans: Examinee discussed initial management Examinee asked permission to start the exam. Examinee asked permission Examinee used respectful draping. Examinee did not repeat painful maneuvers. Examinee recognized the SP’s emotions and responded with PEARLS. emotions and the SP’s Examinee recognized hands. Examinee washed his/her ✓ ✓ I can better determine the cause of your pain. In the meantime, as we discussed, I will write a note to your employer requestin I can better determine the cause of your pain. In the meantime, as we discussed, I for me? that you be given only light duties while you are at work. Do you have any questions Sample Closure I would like to do a rectal exam and assess your prostate for I am concerned about your difficulty urinating, so Meyer, Mr. I would also like to run some blood tests and order an x-ray and possibly an MRI of your back so that benign growths or cancer. Closure Connecting with the Patient Connecting with Physical Examination PRACTICE CASES 164 Physical Examination History USMLE STEP2CS Patient Note PRACTICE CASES 165 Patient Note Patient Diagnosis #3 History Finding(s): Physical Exam Finding(s): Diagnosis #2 History Finding(s): Physical Exam Finding(s): Diagnosis #1 History Finding(s): Physical Exam Finding(s): USMLE STEP 2 CS STEP USMLE Diagnostic Workup Differential Diagnosis Differential USMLE STEP 2 CS Patient Note

History HPI: 51 yo M construction worker c/o low back pain that started after he lifted heavy boxes 1 week ago. The pain is 8/10 and sharp, and it radiates to the left thigh and sometimes to the left foot. Pain worsens with movement, cough, and sitting for a long time. It is relieved by lying still and partially by ibuprofen. He denies urinary/stool incontinence or weakness/loss of sensation in the lower extremities. No fever, night sweats, or weight loss. He does report difficulty urinating and incomplete emptying of the bladder for 6 months as well as a 1-year history of intermittent low back pain. The pain is exacerbated by sitting for long periods but is relieved by sitting after ambulation. ROS: Negative except as above. Allergies: Penicillin, causes rash. Medications: Ibuprofen. PMH: None. PSH: None. SH: 1 PPD for 18 years, 1–2 beers on weekends, CAGE 0/4. FH: Noncontributory.

Physical Examination Patient is in mild distress due to back pain. VS: WNL. Back: Mild paraspinal muscle tenderness bilaterally, normal range of motion, no warmth or erythema. Extremities: 2+ popliteal, dorsalis pedis, and posterior tibial pulses bilaterally. Hips normal, nontender range of motion bilaterally. Neuro: Motor: Strength 5/5 throughout, including left great toe dorsiflexion. DTRs: 2+ symmetric, Babinski bilaterally. Gait: Normal (including toe and heel walking), although he walks with back slightly bent forward. Straight leg raising bilaterally. Sensation: Intact.

Differential Diagnosis Diagnosis #1: Disk herniation History Finding(s): Physical Exam Finding(s): Low back pain Pain started after lifting heavy boxes Pain radiates to left thigh and foot Pain worsens with movement and is relieved by lying still PRACTICE CASES

166 USMLE STEP 2 CS Patient Note

Diagnosis #2: Lumbar spinal stenosis History Finding(s): Physical Exam Finding(s): History of intermittent low back pain and leg Walks with back slightly bent forward pain with ambulation Pain resolves with sitting

Diagnosis #3: Metastatic prostate cancer History Finding(s): Physical Exam Finding(s): Difficulty urinating Incomplete emptying of the bladder Low back pain

Diagnostic Workup

XR—L-spine MRI—L-spine Rectal exam PSA PRACTICE CASES

167 PRACTICE CASES 168 equina syndrome). caused byinfection,cancer, abdominalaorticaneurysm,recurrentsymptoms,orneurologicemergency(eg,cauda within fourweeks.Patientswhorequiremoreextensive orurgentevaluationarethosesuspectedofhavingpain The historyandphysicalexamareoftenallthatisrequired,asmostpatientswithacutelowbackpainwillimprove Diagnostic Workup Additional Differential Diagnoses Patient NoteDifferential Diagnoses CASE DISCUSSION integrity ofsacralnerveroots. Rectal exam(including“saddle area” sensoryexam): correlate withclinicaldisease. or malignancy. Rememberthatasymptomaticdiskherniationiscommon,soitspresencedoesnotnecessarily MRI—L-spine: expected inolderpatientsandcorrelatepoorlywithclinical symptoms. XR—L-spine: Malingering: presenting complaint.Anemia,neuropathy, hypercalcemia,andrenalfailurearealsocommon. Multiple myeloma: activity andalleviatedbyrest.Radicularsymptomsmaybepresent. Degenerative arthritis: the extremities.Paraspinalmuscletendernessisoftenpresent. Lumbar musclestrain: prostatic disease. may beworseatnightandunrelievedbyrest.Thispatient’s urinarysymptomsandlowbackpainmaybesigns of pain (oroccasionallyacuteinthecaseofpathologicfracture)withorwithoutneurologicsymptoms.Pain breast, lung,multiplemyeloma,andlymphoma.Inmetastaticdisease,patientscomplainofgradual-onsetback Metastatic prostate cancer: case). Pain usuallyoccurswithwalkingorprolongedstandingandsubsidessittingleaningforward(asinthis gradual onsetofbackpainthatradiatestothebuttocksandlegswithorwithoutlegnumbnessweakness. Lumbar spinalstenosis: may havediskherniationbuthasnoobjectiveevidenceofneurologiccompromiseatthispoint. (with thelegraised<60degrees)ishighlysensitivebutnonspecificinherniationsattheselevels.Thispatient radicularsymptoms dorsiflexion (L5),andankle-jerkreflex(S1).Ipsilateralstraightlegraisingthatproduces L5–S1 vertebrallevels.Thesenerverootsarequicklyassessedbycheckingtheknee-jerkreflex(L4),greattoe degenerative arthritis,spinalstenosis,orothercausesofsciatica.MostdiskherniationsoccurattheL4–L5 due todiskherniation.However, thispatternisnonspecificandcanalsobecausedbysacroiliitis,facetjoint Disk herniation: Thisisdefinedasintentionalfakingofsymptomsforsecondarygain(eg,gettingoutwork). Canshowevidenceofvertebralosteomyelitis,cancer, orfractures.Degenerativechangesare Providesthebestanatomicdetailandistestofchoice forsuspectedherniation,infection, Lowbackpainradiatingdownthebuttockandbelowkneesuggestsnerverootirritation Typically, patientsareolderthan50yearsofage.Backandbonepainmaybetheonly Thisoftenfollowsstrenuousorunusualexertion,butpainusuallydoesnotradiateto Degenerativebackdiseasesarecommon,andclassicallypainisexacerbatedby Thisismostoftenseeninpatientsolderthan60yearsofage.Theypresentwith The most common cancers leading to vertebral body metastasesare prostate, Themostcommoncancersleadingtovertebralbody To evaluatetheprostate,rectalsphinctertone,and PRACTICE CASES 169 To detect a monoclonal paraprotein in myeloma. Both tests must paraprotein in myeloma. Both tests detect a monoclonal To To detect anemia, hypercalcemia, and renal failure, all of which may be clues to all of which may and renal failure, anemia, hypercalcemia, detect To Screening test for prostate cancer. Screening PSA: BUN/Cr: CBC, calcium, multiple myeloma. underlying electrophoresis: protein Serum and urine could be negative. be done because one PRACTICE CASES 170 Notes fortheSP Patient isa25yoM. Patient Description Checklist/SP Sheet Examinee Tasks HR: RR: Temp: BP: Signs Vital John Matthews,a25-year-old male,comestotheEDfollowingamotorvehicleaccident. Opening Scenario DOORWAY INFORMATION CASE 4 you verycloselyfromthispoint on,andifyouhaveanysignificantproblems,wewill beavailabletohelp.” aroundyourlungs,thereisaprocedurewecanperformtoreleasethepressure.Weis airorblood willbemonitoring once wehaveabetterideaofwhatiswrong,cangive you somemedicationtohelpwithyourpain.Ifthere “Your conditionraisesconcernandisobviouslyurgent.We willstart by takingsomeimagesofyourchest.Then, Sample ExamineeResponse “Do youthinkIamgoingtodie?” Challenging QuestionstoAsk 4. Write thepatientnoteafterleavingroom. 3. Explainyourclinicalimpressionandworkupplantothepatient. 2. Performafocusedphysicalexam(donotperformrectal,genitourinary, orfemalebreastexam). 1. Take afocusedhistory. 123/88mmHg 22/minute 85/minute,regular Moan occasionallyandanswerquestionsinshortsentences. Occasionally coughhardintoatissue. Take fast,shallowbreaths. Exhibit painwhenyourleftupperabdomenisbeingpalpated. Exhibit painwhenyourleftchestisbeingpalpated. your sideandstopbreathingwithashortgasp). Exhibit painintheleftchestthatworsensduringinspirationandmovement(ie,whenyoubreathein,hold 100°F(38°C) PRACTICE CASES 171 Patient Response No. chest, left side). the road. I immediately I looked up, I found that I had veered off going very fast, and my tried to slow down but hit a tree. I wasn’t call the police. so I didn’t I was embarrassed, car was basically okay. think I at first, so I didn’t I was wearing my seat belt and felt okay needed to come to the hospital. think I’ve had any shaking or chills. ache, but I don’t Convulsions No. Other injuriesHead traumaDischarge from the ears, mouth, or nose (clear or bloody) Loss of consciousness I have a few scratches on my arms from the car accident. No. No. CoughSputum productionFever/chills I keep bringing up all this yellow junk. I have to use a tissue because for a couple of days, I guess. I have been coughing I have been feeling a little warm and have noticed that my muscles Alleviating factorsExacerbating factors Nothing I do makes it better. try to move. It gets even worse when I take a deep breath or OnsetSeverity on a scaleContext the worst pain I’ve ever had. At least 8/10. some of It’s It started last night. I was driving my car and was trying to answer my cell phone. When Chief complaint have this excruciating pain (holds I’m having trouble breathing and Question

Examinee elicited data efficiently and accurately. Examinee elicited data efficiently and Examinee asked an open-ended question and actively listened to the response. open-ended question and actively listened Examinee asked an concerns and listened to the response without interrupting. Examinee asked the SP to list his/her own words. often using the SP’s concerns, Examinee summarized the SP’s Examinee introduced self by name. his/her role or position. Examinee identified name. used patient’s Examinee correctly contact with the SP. Examinee made eye Examinee knocked on the door before entering. Examinee knocked ✓ Examinee Checklist Examinee Relationship the Doctor-Patient Building Entrance Information Gathering Reflective Listening PRACTICE CASES 172 Physical Examination Connecting withthePatient ✓ Examinee didnotrepeatpainfulmaneuvers. Examinee usedrespectfuldraping. Examinee askedpermissiontostarttheexam. Examinee washedhis/herhands. Examinee recognizedtheSP’s emotionsandrespondedwithPEARLS.

Question oac No. No. No. Occasionally, on theweekends. I’mabanker. Mymotherand fatherarebothhealthy. Drug allergies Tobacco None. Illicit druguse Ihadasorethroat,mildfever, andfatigue2weeksago.Mydoctor Alcohol use None. Occupation No. Family history Past surgicalhistory Ihadbreakfast thismorning,about5hoursago.Ididn’t haveany Past medicalhistory Current medications Yes, Ihavesharppainrighthere(pointstotheLUQ). Pain onurination hol orrecreationaldrugs? No. Were youundertheinfluenceofalco- Last meal/drink No. Nausea/vomiting orstiffneck No. No. Abdominal pain Heart symptoms(palpitations) ties Weakness ornumbnessintheextremi- personality Confusion, memoryloss,orchangein Change invision Headache No. trouble keepingitdown. No. No. told meIhadinfectiousmononucleosis,butitisgonenow. Patient Response PRACTICE CASES 173 Maneuver organomegaly such as splenomegaly) organomegaly such Follow-up tests. Abdominal examNeurologic examSkin exam (examined specifically for Inspection, auscultation, palpation Mental status, cranial nerves, gross motor Inspection for abrasion, lacerations, bruising Head and neck examHead and CV examPulmonary exam Inspection percussion palpation, Inspection, auscultation, Auscultation Exam Component

Examinee asked if the SP had any other questions or concerns. Examinee asked if the SP had any other Examinee discussed initial diagnostic impressions. Examinee discussed initial diagnostic plans: Examinee discussed initial management ✓ will also give you something for your pain and will observe your breathing to make sure you are getting enough oxygen. Do you and will observe your breathing to make sure you are getting enough oxygen. Do will also give you something for your pain have any questions for me? Sample Closure you closely until must now observe medical treatment after an accident like this. We Matthews, you should always seek Mr. We are going to run a few tests and take some imaging studies of your chest. We we can determine what is causing your pain. Closure PRACTICE CASES 174 Physical Examination History USMLE STEP2CS Patient Note PRACTICE CASES 175 Patient Note Patient Diagnosis #3 History Finding(s): Physical Exam Finding(s): Diagnosis #2 History Finding(s): Physical Exam Finding(s): Diagnosis #1 History Finding(s): Physical Exam Finding(s): USMLE STEP 2 CS STEP USMLE Diagnostic Workup Differential Diagnosis Differential USMLE STEP 2 CS Patient Note

History HPI: 25 yo M c/o left chest pain and LUQ pain following an MVA. The patient struck a tree with his car at a slow speed. The chest pain is 8/10. It is exacerbated with movement or when he takes a deep breath, and nothing relieves it. He reports dyspnea and a productive cough with a low-grade fever but denies LOC, headache, change in mental status, or change in vision. No cardiovascular or neurologic symptoms. No nausea, vomiting, neck stiffness, or unusual fluid from the mouth or nose. No dysuria. His last meal was 5 hours ago. He denies being under the influence of alcohol or drugs. ROS: As per HPI. Allergies: NKDA. Medications: None. PMH: Infectious mononucleosis. PSH: None. SH: No smoking, occasional EtOH, no illicit drugs. FH: Noncontributory.

Physical Examination Patient is in acute distress, dyspneic. VS: Temp 100°F, RR 22/minute. HEENT: No JVD, no bruises, PERRLA, EOMI, no pharyngeal edema or exudates. Chest: Two large bruises on left chest, left rib tenderness, decreased breath sounds over left lung field, right lung fields clear. Heart: RRR; normal S1/S2; no murmurs, rubs, or gallops. Abdomen: Soft, nondistended, BS, LUQ tenderness, no rebound or guarding, no organomegaly. Skin: No bruises or lacerations. Neuro: Mental status: Alert and oriented × 3. Cranial nerves: 2–12 grossly intact. Motor: Strength 5/5 in all muscle groups. Sensation: Intact to pinprick and soft touch.

Differential Diagnosis Diagnosis #1: Pneumothorax History Finding(s): Physical Exam Finding(s): Left-sided chest pain following an MVA Decreased breath sounds over left lung field Pain is exacerbated by movement and deep RR 22/minute breaths Dyspnea PRACTICE CASES

176 USMLE STEP 2 CS Patient Note

Diagnosis #2: Hemothorax History Finding(s): Physical Exam Finding(s): Left-sided chest pain following an MVA Decreased breath sounds over left lung field Dyspnea RR 22/minute Cough

Diagnosis #3: Pneumonia History Finding(s): Physical Exam Finding(s): Unilateral chest pain Temperature 100°F Productive cough RR 22/minute Low-grade fever

Diagnostic Workup

CXR Sputum and blood Gram stain and culture PRACTICE CASES

177 PRACTICE CASES 178 dyspnea areserioussymptomsthatrequireswiftevaluationandintervention. able towalkandtalk,somewhatnegatingtheurgencyofatypicalEDevaluation.Atsametime,chestpain Diagnostic Workup Additional Differential Diagnoses (neurologic), and The mostimportantstepsinanytraumaaretoassessthe Patient NoteDifferential Diagnoses CASE DISCUSSION bacterial pathogenssuchas Sputum andbloodGramstainculture: fractures canalsobediagnosedfromtheCXRiftheyare present. tension pneumothoraxwillshowmediastinalshiftandtracheal deviationawayfromthepneumothorax.Rib CXR: a simpleviralpleuritis,butmoreemergentcausesneedto beruledoutfirst. movement. Thephysicalexamisgenerallynegativewith theexceptionofchestpain.Thispatientmayhave Pleuritis: physical exam,itisimportanttoevaluateforanysignsorsymptomsoforganomegaly. the spleenshouldbeevaluatedwithanultrasoundexamfollowedbyfurtherimagingabdominalCT. On injury orbleedingaregreater. Giventhatthispatient’s painisprimarilyleft-sidedinthechestareaandLUQ, lossveryquickly.significant blood Ifthispatientwasexposedtoinfectiousmononucleosis,hischancesofsplenic Splenic rupture: pneumothorax. TheycanbediagnosedwithaCXR. wall. Asimplefracturecouldcausethispatient’s painoninspiration andcough.Ribfracturescanalsoleadto Rib fracture: CXR willshowalobarinfiltrate,andsputumculturesmayhelpidentifythebacterialpathogen. should beruledoutfirst.Physicalsignsincludetachypnea,crackles,egophony, anddullnesstopercussion.The cough,low-gradefever,productive andunilateralchestpainsuggestiveofpneumonia.However, traumaticcauses can presentwithacuterespiratorydistress,fever, cough,pleuriticpain,andshakingchills.Thispatienthasa Pneumonia: chest tubeplacement. and occasionallysignssymptomsofhypovolemicshock.Thefinaldiagnosiscanbemadebypleurocentesisor trauma. Itpresentswithchestpain,shortnessofbreath,cough,decreasedbreathsoundsontheinvolvedside, Hemothorax: acute onsetanddistresssuggestpneumothorax.CXRisthefastestdiagnostictoolavailable. the injuredside(incaseoftensionpneumothorax),andJVD.AlthoughnoJVDispresent,thispatient’s Physical findingsincludeaunilaterallossofbreathsoundswithhyperresonance,shiftthetracheaawayfrom Pneumothorax: grade fever. culture andGramstainwouldalsobeusefulgiventhatthe patienthasalow- that thesearepossibilities.A blood identify tuberculosisand OnCXR,lobarconsolidationmayindicatepneumonia, hemothoraxmaycauselinearconsolidation,and Inflammationofthepleuralmembranecancauseseverepainthatincreaseswithinspirationor Mostoftencommunityacquiredandcausedby Ribfracturesarethemostcommonchestinjuryandcanresultfromalmostanyinsultto This is defined by the presence of blood inthepleuralspaceandis mostcommonlydueto Thisisdefinedbythepresenceofblood E xposure. Inthiscase,theexamisseparatedfromtraumabyseveralhoursandpatient Apneumothoraxformswhenaircollectsbetweenthepleuralandviscerallayersofthorax. Splenicinjuriesarealwaysofgreatconcernfollowingatraumabecausetheycancause Pneumocystis jiroveci S pneumoniae. Other stains, such as acid-fast stains and monoclonal antibodies, can Otherstains,suchasacid-fast stainsandmonoclonalantibodies, (formerly Usedtoscreensputumsamplesfortheidentificationof P carinii BDs A ABCDEs: Streptococcus pneumoniae, ) andshouldbeobtainedifthe historysuggests irway, B reathing, bacterialpneumonia C irculation, D isability PRACTICE CASES 179 These tests should be considered for any driver following a for any driver should be considered These tests Although a CT scan may be a more effective means of assessing patients for internal may be a more effective means of assessing Although a CT scan Although not as sensitive as ABG analysis, pulse oximetry is a fast, noninvasive measure of as ABG analysis, pulse oximetry Although not as sensitive abdominal bleeding, which may be advisable for this patient given his history of infectious mononucleosis. An given his history of infectious mononucleosis. which may be advisable for this patient abdominal bleeding, way to rule out free air in the abdomen. AXR remains a quick Pulse oximetry: chronically lung disease such as COPD may have that a patient with long-standing oxygenation. Remember drive. which is necessary to maintain respiratory suppressed oxygenation, Urine toxicology and blood alcohol level: and blood Urine toxicology to it is still necessary a while ago, car accident occurred though this patient’s accident. Even motor vehicle current situation. evaluate his XR/CT—abdomen: assess for intra- sonography for trauma) can quickly scan (focused assessment with a FAST abdominal injury, PRACTICE CASES 180 Entrance Building theDoctor-Patient Relationship Examinee Checklist wish, Iwouldbehappytodiscussyouroptionswithboth ofyou.” your physician,Iwanttoassureyouthatamheresupport andadviseyouinwhateverdecisionmake.If “I understandyouranxietyaboutthisunplannedpregnancy. Isuggestthatyoudiscussthiswithyourhusband.As Sample ExamineeResponse “We hadnotplannedtohaveababysosoonaftermarriage.What should Ido,doctor?” Challenging QuestionstoAsk If asked,tellthedoctorthatyoufeeltiredalltime. Notes fortheSP Patient isa28yoF, marriedwithnochildren. Patient Description Checklist/SP Sheet Examinee Tasks HR: RR: Temp: BP: Signs Vital Tanya Parker, a28-year-old female,comestotheclinicwithapositivepregnancytest. Opening Scenario DOORWAY INFORMATION CASE 5 Examinee madeeyecontact with theSP. Examinee correctlyusedpatient’s name. Examinee identifiedhis/her role orposition. selfbyname. Examinee introduced Examinee knockedonthedoorbeforeentering. 4. Write thepatientnoteafterleavingroom. 3. Explainyourclinicalimpressionandworkupplantothepatient. 2. Performafocusedphysicalexam(donotperformrectal,genitourinary, orfemalebreastexam). 1. Take afocusedhistory. 120/70mmHg 14/minute 76/minute 98.6°F(37°C) PRACTICE CASES 181 or itching. with some smells of food. bloated all the time. Patient Response I checked a had one yet. So I haven’t light, and this month pregnancy test, and it was positive. 1 or 2 days. Usually I have Last month I had some spotting for only moderate 4–5 pads per day. flow and use Shortness of breath No. Last Pap smearFeverBowel habitsUrinary habits Eight months ago, and it was normal. Once a day. No. I have no burning now. I feel I have to use the bathroom frequently Weight changesWeight FatigueBreast discharge/tenderness but I have been feeling checked my weight recently, I haven’t My breasts are a little fuller than before. I’m really easily tired out by doing my daily activities. Yes. Nausea/vomitingPostcoital bleedingAbdominal painAppetite changes but I have not been vomiting. lately, I do feel nauseated No. No. feel like eating anything because of the nausea, especially I don’t ContraceptionPregnancy/miscarriagesSexual activity/partnersHistory of STDs None. My husband withdraws before ejaculation. with my husband. I am sexually active only None. Menstrual historyPain with periodsSpotting between periods My periods every month. last 3–4 days and occur at the same time No. No. Onset/durationLast menstrual periodMenarche but last month it was very My periods have always been regular, Six weeks ago, and it was only light spotting. At the age of 14. Chief complaint Positive pregnancy test. Question

Examinee elicited data efficiently and accurately. Examinee elicited data Examinee asked an open-ended question and actively listened to the response. listened to the question and actively asked an open-ended Examinee interrupting. response without and listened to the his/her concerns asked the SP to list Examinee own words. SP’s concerns, often using the the SP’s Examinee summarized ✓ Reflective Listening Reflective Information Gathering PRACTICE CASES 182 Physical Examination Connecting withthePatient ✓ ✓ Examinee didnotrepeatpainfulmaneuvers. Examinee usedrespectfuldraping. Examinee askedpermissiontostarttheexam. Examinee washedhis/herhands. Examinee recognizedtheSP’s emotionsandrespondedwithPEARLS.

ExamComponent Question knea Inspectedforpigmentationorpallor Inspection, auscultation, palpation Auscultation Auscultation Inspection/palpation ofthyroid Skin exam Abdominal exam Pulmonary exam CV exam HEENT exam No. Unsure. None. None. No. None. Occasionally1or2beersaweek. Domestic abuse Graduatestudent. Desired pregnancy Myfatherisadiabetic.momhasthyroidandobesityproblems. Planned pregnancy Drug allergies Tobacco MyappendixwasremovedwhenI20. Illicit druguse None. Inormallyrun5milesaday, butlatelyI’vehadto cutbackbecauseI Alcohol use Multivitamins. Occupation Ihavenotnoticedanything. Family history Past surgicalhistory Past medicalhistory Current medications Exercise Skin changes feel sotiredallthetime. Patient Response Maneuver PRACTICE CASES

183 of will Follow-up tests: Examinee mentioned the need for a pelvic/breast exam. mentioned the need for a pelvic/breast Follow-up tests: Examinee Examinee discussed initial diagnostic impressions. discussed initial diagnostic Examinee plans. discussed initial management Examinee SP had any other questions or concerns. Examinee asked if the are not very effective. We will also need to perform a pelvic ultrasound to estimate the age of the fetus and the expected date ultrasound to estimate the age of the fetus will also need to perform a pelvic We are not very effective. and some vaginal cultures that we routinely If you are pregnant, we will check some more blood tests, a Pap smear, delivery. intense exercises and excess I recommend stopping alcohol consumption and avoiding For now, perform in every pregnancy. we will schedule your future prenatal visits. I and multivitamins to take orally, caffeine. I will be giving you some prenatal receive these tests. Do you have any questions or concerns? be able to advise you further as soon as we Sample Closure I will have to repeat have told me, it appears that you are pregnant. basis of my observations and what you on the Mrs. Parker, can a real menstrual period, as spotting last period may not have been to confirm the diagnosis. Your a urine pregnancy test pulling out before ejaculation methods of contraception such as natural Unfortunately, first trimester. frequently occur in the Closure PRACTICE CASES 184 Physical Examination History USMLE STEP2CS Patient Note PRACTICE CASES 185 Patient Note Patient Diagnosis #3 History Finding(s): Physical Exam Finding(s): Diagnosis #2 History Finding(s): Physical Exam Finding(s): Diagnosis #1 History Finding(s): Physical Exam Finding(s): USMLE STEP 2 CS STEP USMLE Diagnostic Workup Differential Diagnosis Differential USMLE STEP 2 CS Patient Note

History HPI: 28 yo G0 presents with a positive pregnancy test. Her LMP was 6 weeks ago and was unusually scant. She reports bilateral breast engorgement, poor appetite, nausea with no vomiting, increased urinary frequency, and feeling bloated and fatigued. She is sexually active with her husband only, with coitus interruptus as the only method of contraception. This is an unplanned pregnancy, and she is unsure whether she will continue. OB/GYN: G0, menarche at age 14, has regular periods 4–5/30. No history of STDs; last Pap smear was taken 8 months ago and was normal. ROS: Denies abnormal bleeding, abdominal pain, fever, shortness of breath, or change in bowel habits. Allergies: NKDA. Medications: Multivitamins. PMH: None. PSH: Appendectomy at age 20. SH: No smoking, 1–2 beers/week, no illicit drugs. Married graduate student; denies domestic violence. FH: Father is a diabetic. Mother has thyroid problems and obesity.

Physical Examination Patient appears comfortable. VS: WNL. HEENT: NC/AT, PERRLA, no icterus, no pallor, mouth and oropharynx normal. Neck: No thyroid enlargement. Chest: Clear breath sounds bilaterally. Heart: RRR; normal S1/S2; no murmurs, rubs, or gallops. Abdomen: Soft, nontender, nondistended, BS, no evidence of guarding or hepatosplenomegaly.

Differential Diagnosis Diagnosis #1: Normal pregnancy History Finding(s): Physical Exam Finding(s): Amenorrhea for 6 weeks Positive pregnancy test Bilateral breast engorgement Nausea and weight gain

Diagnosis #2: Ectopic pregnancy History Finding(s): Physical Exam Finding(s): Amenorrhea for 6 weeks Positive pregnancy test PRACTICE CASES

186 USMLE STEP 2 CS Patient Note

Diagnosis #3: Molar pregnancy History Finding(s): Physical Exam Finding(s): Positive pregnancy test Nausea

Diagnostic Workup

Urine hCG U/S—pelvis Breast/pelvic exams Blood type, Rh, antibody screen PRACTICE CASES

187 PRACTICE CASES 188 Diagnostic Workup Patient NoteDifferential Diagnoses CASE DISCUSSION a UTI. pyelonephritis. Therefore,allpregnantwomenneedtobe screenedeveniftheydonotcomplainofsymptoms UA, urineculture: serious neonatalinfections. Cervical gonorrheaandchlamydiaDNAtesting: another fourmonths. smear eightmonthsago,arepeatPapisnotnecessarily indicatedatthisvisitandcouldbepostponedfor Pap smear: every womandiagnosedwithpregnancyshouldreceive. should bediscussedseparately, andthepatient’s consentisrequiredinsomestates.Thesearestandardteststhat detection allowsformeasuresthatcoulddecreasethepossibilityoftransmissiontofetus.HIVscreening RPR, rubellaIgG,HBsAg,HIVantibody: while maternalhyperthyroidismcanleadtofetalandcomplications. TSH: CBC: newborn. Rh(D)-negativewomenshouldreceiveanti-Dimmuneglobulinasindicated. Blood type,Rh,antibodyscreen: PCR studies. the uterus(size,fibroids),andadnexa(masses)tocollectnecessaryspecimensforcytology, cultures,and pregnancy. Apelvicexamneedstobeperformedevaluatethecervix(lesions,length,dilation,consistency), Breast/pelvic exams: pregnancies, uterinefibroids,andadnexalmasses. Thiscanalsoaidinthediagnosisofmolar gestational ageinpatientswithanuncertainLMPorirregularperiods. U/S—pelvis: ordered ifanabnormalpregnancy(eg,abortion,ectopicpregnancy, molarpregnancy)issuspected. Urine hCG: suspicion formolarpregnancy. Thediagnosisisusuallyconfirmed bypelvicultrasound. vomiting, new-onsethyperthyroidism,andauterusthatislargerthanexpectedforgestationalageshouldraise Molar pregnancy: differential diagnosisofwomenwithapositivepregnancytestuntilintrauterineisidentified. Ectopic pregnancy: of nausea,weightgain,andbreastengorgement,allsignsearlypregnancy. sexually activeshouldpromptthediagnosisofpregnancyunlessotherwiseruledout.Thispatienthassymptoms pregnancy:Normal Neurologicdevelopmentmaybeadverselyaffectedinchildrenborntomotherswithhypothyroidism, To ruleoutanemiaandtoobtainabaseline forhemoglobinandplatelets. AurinehCGtestcanconfirmpregnancy. Alternatively, aquantitativeserum To screenforcervicaldysplasiaandcancer. However, sincethispatienthadanormal Pap Itisimportanttoconfirmthelocationofpregnancy(intrauterinevs.extrauterine)and Molarpregnanciesareuncommon.Very highserum Pregnantwomenwithuntreatedasymptomaticbacteriuria areathighriskofdeveloping Extrauterineimplantationresultinginectopicpregnancyshouldalwaysbethe Any history of delayed periods or amenorrhea in a reproductive-age womanwhois oramenorrheainareproductive-age Anyhistoryofdelayedperiods Breastengorgementandgalactorrheaaresomeofthephysiologicchangesthatoccurin To thatcouldpotentiallycausehemolyticdiseaseofthe detectantibodies Theseinfectionscanbetransmittedperinatally, andearly EarlydiagnosisandtreatmentoftheseSTDscanprevent β -hCG levels,severenauseaand β -hCG canbe PRACTICE CASES 189 “Doctor, I have no history of diabetes in my family. Why is this happening to my daughter?” Why in my family. I have no history of diabetes “Doctor, my child ever be able to eat sweets again?” “Will Examinee identified his/her role or position. name. Examinee correctly used patient’s Examinee made eye contact with the SP. Examinee knocked on the door before entering. Examinee introducedself by name. 2. Explain your clinical impression and workup plan to the mother. impression and workup plan to the 2. Explain your clinical the patient note after leaving the room. 3. Write 1. Take a focused history. 1. Take necessary to have a family history of diabetes. With regard to sweets being the cause of your daughter’s diabetes, daughter’s regard to sweets being the cause of your necessary to have a family history of diabetes. With this is a myth. In fact, your daughter can still eat sweets, but in moderation. She will need to see a dietitian to develop healthy meal plans as well as to learn to recognize which foods contain carbohydrates and how much.” led her to get full-blown diabetes. Your daughter may have either type 1 or type 2 diabetes. In type 1 diabetes, the daughter may have led her to get full-blown diabetes. Your destroys the cells that are responsible for making insulin. Since insulin immune system attacks the and On the other levels of blood an insulin deficiency will lead to high sugar. regulates and maintains blood sugar, 2 diabetes, which is a combination hand, if your child is overweight and is not physically active, she may have type overweight. In either case, it is not of insulin deficiency and resistance to the action of insulin resulting from being Sample Examinee Response diabetes. Then certain unknown environmental factors daughter probably had a genetic tendency to develop “Your DOORWAY INFORMATION DOORWAY CASE 6 CASE Building the Doctor-Patient Relationship Building the Doctor-Patient Entrance Examinee Checklist Patient Description the history; her daughter is at school. mother offers The patient’s Notes for the SP None. Challenging Questions to Ask Checklist/SP Sheet The mother of Louise Johnson, a 10-year-old female child, comes to the office because she is concerned that her child, comes to the office because she female Johnson, a 10-year-old The mother of Louise diagnosed with diabetes. daughter was recently Examinee Tasks Opening Scenario PRACTICE CASES 190 Information Gathering Reflective Listening ✓ Examinee eliciteddataefficientlyandaccurately. Examinee summarizedtheSP’s concerns,oftenusingtheSP’s ownwords. Examinee askedtheSPtolisthis/herconcernsandlistenedresponsewithoutinterrupting. Examinee askedanopen-endedquestionandactivelylistenedtotheresponse.

Question egtcagsShehaslostabout 9poundswithinthepast3months,butnowher Sheweighs180 pounds andhasbeenonthe heavy sideforalong No. Normalatpresent,butshehadtogothebathroom alot,whichis Patient’s weightandheight glucosereadingwas80inthemorning. Notreally;thelowestblood No. Weight changes Vision problems(blurringof vision) No,butshewasexcessivelythirsty earlier. Weakness orfatigue Today hermorningglucosewas96inthefastingstate. Abnormal thirstorextremehunger Urination Two types:onewithmealsandatbedtime. Hypoglycemia Yes, beforeeachmealandatbedtime. sugarlevelsarenormallyinthe low100sinthemorning Herblood Recent levelofglucose Idoitwhensheisathome,butawayfromme,does Yes. glucosereadings Ranges ofblood Insulininjections. Inthetummy. Measuring glucoseathome No. Schedule ofinsulin We wereshocked. I’mnotsure,butsheseemsmoreconcernedthandepressed. Compliance withinsulin Insulin injector Sheisconcernedabouttheeffectthiswillhaveonhernormal Site ofinjection Amonthago. Medication Effect onparents Irritability Iamnotsure. Depression Mychildwasrecentlydiagnosedwithdiabetes. Effect onchild diagnosis Presenting symptomsatthetimeof Onset Type ofdiabetes Chief complaint activities, suchasplayingtennisandattendingschool. Excessive thirstandurination. time. Sheis5feet,1inchtall. weight isstable. how shewasfirstdiagnosed. and inthehigh100sbeforemeals. the injectionsherself. Patient Response PRACTICE CASES

191 ase ase Patient Response Patient dietitian. She walked early, She was always up to date with her development. talked on time, and is doing well in school. Further examination. Follow-up tests. development Past medical historyPast surgical historyDrug allergies None. None. No. Last menstrual periodSleeping problemsBirth historyChild weight, height, and language She has not yet started menstruating. No. Normal. Any specific diet activitiesExercise and playful When does she play? while playingLoss of consciousness she is active and plays tennis. Yes, No. are trying to give her a balanced diet with the help of the We Evenings. Tingling or numbness in limbs or numbness Tingling of skin or gumsInfections Itchy skin No. No. No. Question

Examinee asked if the SP had any other questions or concerns. Examinee discussed initial diagnostic impressions. Examinee discussed initial management plans: Examinee recognized SP’s emotions and responded with PEARLS. emotions and Examinee recognized SP’s ✓ and should know how to provide appropriate care. Your daughter should always carry a snack or juices as an “emergency kit.” and should know how to provide appropriate care. Your have any Do you what we discussed today. Her teachers and friends should also be aware of her disease. I hope you understood additional questions or concerns? Sample Closure with diabetes. Diabetes may alter the Mrs. Johnson, I can understand how you have felt since your daughter was diagnosed can manage this dise We different now. dynamics of the entire family and affects everyone, so your life is going to be a little of all, you should understand the dise very well through a combination of insulin, a balanced diet, and regular exercise. First Second, everyone in your family, will need to attend diabetes classes with your daughter. and know how to manage it. You should learn to recognize signs of low glucose levels, such as confusion, disorientation, or fainting, including your daughter, None. Closure Connecting with the Patient Physical Examination PRACTICE CASES 192 Physical Examination History USMLE STEP2CS Patient Note PRACTICE CASES 193 Patient Note Patient Diagnosis #3 History Finding(s): Physical Exam Finding(s): Diagnosis #2 History Finding(s): Physical Exam Finding(s): Diagnosis #1 History Finding(s): Physical Exam Finding(s): USMLE STEP 2 CS STEP USMLE Diagnostic Workup Differential Diagnosis Differential USMLE STEP 2 CS Patient Note

History HPI: The source of the information is the patient’s mother. The mother of a 10 yo F states that her child was diagnosed with DM 1 month ago, when she presented with excessive thirst and frequent urination. The parents were shocked after the diagnosis was made. The child seems concerned but not irritable or depressed. She is active, plays tennis, and is currently on a diet prescribed by a dietitian. She is on insulin injections and regularly monitors her blood glucose levels at home. Her compliance is good; she checks her blood glucose before each meal and at bedtime. Fasting glucose levels are usually 80 to the low 100s and in the high 100s before meals. She has not had any episodes of hypoglycemia. She has lost 9 lbs in the past 3 months, but her weight is stable now at about 180 lbs. She denies any weakness, fatigue, tingling over the limbs, visual symptoms, or rash/itch at the injection sites. She has not yet started menstruating. ROS: Negative. Allergies: NKDA. Medications: Insulin. PMH: None. PSH: None. Birth history: Normal. Developmental history: Normal. FH: No family history of diabetes.

Physical Examination None.

Differential Diagnosis Diagnosis #1: Type 1 diabetes mellitus History Finding(s): Physical Exam Finding(s): Polyuria, polydipsia Recent weight loss

Diagnosis #2: Type 2 diabetes mellitus History Finding(s): Physical Exam Finding(s): Polyuria, polydipsia Obesity Hyperglycemia PRACTICE CASES

194 USMLE STEP 2 CS Patient Note

Diagnosis #3: Secondary causes of diabetes (eg, Cushing’s syndrome) History Finding(s): Physical Exam Finding(s): Obesity

Diagnostic Workup

Insulin and C-peptide levels Islet cell antibodies

HbA1c Electrolytes, glucose UA and urine microalbumin 24-hour urine free cortisol PRACTICE CASES

195 PRACTICE CASES 196 Diagnostic Workup Patient NoteDifferential Diagnoses CASE DISCUSSION 24-hour urinefree cortisol: UA, urinemicroalbumin: in conjunctionwithHbA Electrolytes, glucose: HbA Islet cellantibodies: Insulin andC-peptidelevels: hemochromatosis, andacromegaly. conditions, suchasdrugs(eg,thiazidediuretics,glucocorticoids),Cushing’s syndrome,pancreatitis,cysticfibrosis, Secondary causesofdiabetes(hyperglycemia): DM amongchildren.IneverysuspectedcaseofDM,itismandatorytoruleoutothercauses. prevalence ofobesityandphysicalinactivityintheurbanpopulationhasledtoagrowingincidencetype2 Diabetes mellitus(DM): within thepast2–3months. glucoselevels preceding 2–3months.ElevatedlevelssuggestexistingDMaswelllackofcontrolblood 1c : UsedtodiagnoseDMandmonitortreatment.HbA Thisfindingwillsupportthediagnosisoftype1DM. To assessforhypernatremia,whichmaybe seeninDM,aswellforglycemiccontrol 1c AlthoughmostcasesofDMinthepediatricpopulationaretype1,increasing . Arandomglucosetest To screenfordiabeticnephropathy. To ruleoutcoexistingCushing’s syndrome. Whencombined,canbeausefultoolinidentifyingtype1DM. ≥ 200mg/dLcanhelpmakethediagnosisofDM. DMcanbesecondarytootherfactorsormedical 1c estimates blood glucosecontrolduringthe estimatesblood PRACTICE CASES 197 98.0°F (36.7°C) Sit up on the bed. Hold your right arm close to your body rotated and slightly ab- with your left hand and keep it externally ducted. Show pain when the examinee tries to move your right shoulder in any direction. in flexion, extension, abduction, Do not allow the examinee to bring your shoulder to its full range of motion or external rotation. 76/minute, regular 12/minute 135/85 mm Hg Examinee knocked on the door before entering. Examinee introducedself by name. 2. Perform a focused physical exam (do not perform rectal, genitourinary, or female breast exam). not perform rectal, genitourinary, 2. Perform a focused physical exam (do workup plan to the patient. 3. Explain your clinical impression and the patient note after leaving the room. 4. Write 1. Take a focused history. a focused 1. Take “I hope so, but first we need to find out exactly what is causing your problem.” Challenging Questions to Ask do you think I will be able to move my arm again like before?” “Doctor, Sample Examinee Response DOORWAY INFORMATION DOORWAY CASE 7 CASE Entrance Examinee Checklist Relationship Building the Doctor-Patient Checklist/SP Sheet Patient Description Patient is a 74 yo M. Notes for the SP RR: HR: Examinee Tasks Richard Green, a 74-year-old male, comes to the ED complaining of pain in his right arm. to the ED complaining of pain in male, comes Richard Green, a 74-year-old Vital Signs BP: Temp: Opening Scenario PRACTICE CASES 198 Information Gathering Reflective Listening ✓ Examinee eliciteddataefficientlyandaccurately. Examinee summarizedtheSP’s concerns,oftenusingtheSP’s ownwords. Examinee askedtheSPtolisthis/herconcernsandlistenedresponsewithoutinterrupting. Examinee askedanopen-endedquestionandactivelylistenedtotheresponse. Examinee madeeyecontactwiththeSP. Examinee correctlyusedpatient’s name. Examinee identifiedhis/herroleorposition.

Question oyufe aea oe Yes (looksanxious). Iamawidower. Mywifedied3yearsago,andsincethenIhave No(looksanxious). Theyareallnice. Ilivewithmy son.Heismarriedandhas3children.Lifebeen Do youfeelsafeathome? Bad treatmentinhisson’s house Social history Movingmy arm. NotmovingmyarmandTylenol. Living conditions Iusedasling and tooksomeTylenol, butthepaindidn’t getthat tion Reason fornotseekingmedicalatten- No. Exacerbating factors No. Alleviating factors Ididn’t feelanypainatthetime,andthenstarted Theupperandmiddlepartsofthearm. Any treatments None. Seen byadoctorsincethen None. Pain anywhereelse No. Progression ofpain Threedaysago. Numbness/loss ofsensation Itripped overatoyonthegroundandfellmyhand.Myarmwas Weakness/paralysis Iwasplayingwithmygrandchildreninthegardenwhentripped Location Painintherightarm. Loss ofconsciousness Description ofthefall Precipitating events Onset Chief complaint and fell. lived withmyson. hard onhimlately. Helosthisjob andislookingforanewone. hospital; hewasbusy. (looks anxious).Also,mysondidn’t havetimetobringmethe Well, itwasn’t thatbad,andIthoughtitwouldgetbetteronitsown much better. gradually. Itisstablenow, butitisstillthere. outstretched. Patient Response PRACTICE CASES 199 Patient Response Patient Maneuver sensation, DTRs, pulses wrist), joint stability, elbow, (shoulder, for me to urinate, but that has gotten much better. They said there They that has gotten much better. for me to urinate, but cancer. was no evidence of Follow-up tests. Alternative living options such as assisted living. Social work assistance. Head and neck examCV examPulmonary examExam of the arms Checked for bruises, neck movements Auscultation Auscultation motion Compared both arms in terms of strength, range of Alcohol useTobaccoExercise No. No. and back. walk for 20 minutes to the grocery store Every day I Past medical historyPast surgical historyOccupation Asthma. was very difficult They removed part of my prostate 2 years ago. It Retired schoolteacher. Current medicationsAllergies aspirin Nature of reaction to albuterol inhaler. Tylenol, my body. I get an itchy rash all over I am allergic to aspirin. Yes, Exam Component Question

when you need it.” Examinee asked if the SP had any other questions or concerns. safety is my primary concern, and I am here for help and support Examinee offered a statement of support: “Your Examinee discussed initial diagnostic impressions. Examinee discussed initial management plans: Examinee asked permission to start the exam. Examinee asked permission to start the Examinee used respectful draping. Examinee did not repeat painful maneuvers. Examinee recognized the SP’s emotions and responded with PEARLS. Examinee recognized the SP’s Examinee washed his/her hands. ✓ ✓ Closure Connecting with the Patient Physical Examination PRACTICE CASES 200 social situationandhelpyoufindtheresourcesneed.Dohaveanyquestionsforme? an apartmentcomplexforseniors?Ifyouareinterested,Icanarrangeameetingwithoursocialworker, whocanassessyour with afamilycanbestressfulforthewholehousehold.Haveyoueverconsideredmovingtoanassisted-livingcommunityor well. Your safetyismyprimaryconcern,andIamheretooffer youhelpandsupportwheneverneedit.Sometimesliving x-ray ofyourshoulderandarmtomakeadiagnosis,morepreciseimagingstudiessuchasanMRImaybenecessary Mr. Green,youmayhaveafracturedbone,simplesprain,ordislocationoftheshoulderjoint.We willneedtoobtainan Sample Closure PRACTICE CASES 201 Patient Note Patient USMLE STEP 2 CS STEP USMLE Physical Examination History USMLE STEP 2 CS Patient Note

Differential Diagnosis Diagnosis #1 History Finding(s): Physical Exam Finding(s):

Diagnosis #2 History Finding(s): Physical Exam Finding(s):

Diagnosis #3 History Finding(s): Physical Exam Finding(s):

Diagnostic Workup PRACTICE CASES

202 USMLE STEP 2 CS Patient Note

History HPI: 74 yo M c/o right arm pain for the past 3 days. The pain started after he fell on his outstretched right arm and persisted despite his use of Tylenol and a sling at home. No loss of consciousness before or after the fall. No paralysis or loss of sensation. The pain is in the upper and middle part of the arm, increases with any movement of the arm, and is alleviated by rest. When asked why he delayed seeking medical assistance, the patient looked anxious and stated that his son didn’t have time to take him to the hospital. ROS: Negative except as above. Allergies: Aspirin (rash). Medications: Tylenol, albuterol inhaler. PMH: Asthma, probable BPH s/p prostate surgery. PSH: As above. SH: No smoking, no EtOH. Widower for the past 3 years; lives with his son, who recently lost his job. Walks 20 minutes every morning.

Physical Examination Patient is in no acute distress. VS: WNL. HEENT: Normocephalic, atraumatic, no bruises. Neck: Supple, full range of motion in all directions, no bruises. Chest: Clear breath sounds bilaterally. Heart: RRR; normal S1/S2; no murmurs, rubs, or gallops. Extremities: Right arm held closely against chest wall. Nonlocalized tenderness over middle and upper right arm and right shoulder; pain and restricted range of motion on flexion, extension, abduction, and external rotation of right shoulder. Right elbow and wrist are normal. Pulses normal and symmetric in brachial and radial arteries. Unable to assess muscle strength due to pain. DTRs intact and symmetric. Sensation intact to pinprick and soft touch.

Differential Diagnosis Diagnosis #1: Humeral fracture History Finding(s): Physical Exam Finding(s): Pain following recent fall on outstretched arm Tenderness over upper and middle right arm Pain increases with arm movement Restricted range of motion PRACTICE CASES

203 USMLE STEP 2 CS Patient Note

Diagnosis #2: Shoulder dislocation History Finding(s): Physical Exam Finding(s): Pain following recent fall on outstretched arm Right arm externally rotated and slightly abducted Pain increases with arm movement Pain and restricted range of motion on shoulder exam

Diagnosis #3: Osteoporosis History Finding(s): Physical Exam Finding(s): Advanced age

Diagnostic Workup

XR—right shoulder and arm MRI—shoulder Bone density scan (DEXA) PRACTICE CASES

204 PRACTICE CASES 205 AP and lateral views that include the joints above and below the injury can AP and lateral views that include the To diagnose and quantify osteoporosis. To The glenohumeral joint is the most commonly dislocated joint in the human body. It joint in the human body. joint is the most commonly dislocated The glenohumeral Most commonly occurs in elderly persons, usually after a fall. The axillary nerve can be in elderly persons, usually after a Most commonly occurs Patients often but pain is with nonspecific pain localized to the shoulder, usually present Required to diagnose rotator cuff tears, labral disease, and other disorders. Required to diagnose rotator cuff tears, Suspect presenting with fractures in elderly patients (especially women) underlying osteoporosis The history contains red flags (bruises, anxious behavior) that may point to elder abuse. The history contains red flags (bruises, XR—right shoulder and arm: view is useful to help diagnose proximal humeral fracture or dislocation. show fracture or dislocation. An axillary MRI—shoulder: Bone density scan (DEXA): Rotator cuff tear: Rotator cuff owing to shared innervation. There may be an inability to abduct or flex referred down the proximal lateral arm significant weakness in internal or external rotation strength. Patients may also demonstrate the shoulder. Elder abuse: defined elder abuse as “an act or omission which results in harm or The American Medical Association has of an elderly person.” The diagnosis of elder abuse is not readily made threatened harm to the health or welfare victim deny abuse. Thus, diagnosis is often inferential, and supporting because often both the abuser and the evidence must be sought. Osteoporosis: lumbar fractures are the thoracic and trauma. The most common sites of osteoporotic following minimal and the distal radius. vertebral bodies, the neck of the femur, radial nerve can be injured in a fracture of the midshaft/distal third of the humerus, causing wrist drop. third of the humerus, causing wrist injured in a fracture of the midshaft/distal radial nerve can be Shoulder dislocation: arm with forceful results from a fall on an outstretched anteriorly and inferiorly and usually most often dislocates arm is typically externally On exam, the patient’s and external rotation of the shoulder. abduction, extension, to pain. abducted. Movement is avoided owing rotated and slightly Humeral fracture: region. The along the lateral aspect of the deltoid humerus fracture, causing sensory loss injured in a proximal Diagnostic Workup Diagnoses Additional Differential CASE DISCUSSION CASE Diagnoses Note Differential Patient PRACTICE CASES 206 Entrance Building theDoctor-Patient Relationship Examinee Checklist Notes fortheSP Patient isa56yoM. Patient Description Checklist/SP Sheet Examinee Tasks HR: RR: Temp: BP: Signs Vital Raymond Stern,a56-year-old male,comestotheclinicfordiabetesfollow-up. Opening Scenario DOORWAY INFORMATION CASE 8 well controlled,weshouldbeabletoavoidamputation. We’ll discussmoreonhowtodothislaterinthevisit.” take thepropermeasurestoprotectyourfeetfrominjury. sugarandcholesterol Ifwecontinuetokeepyourblood not atthatpoint.Thenervedamagetoyourfeetisuncomfortable, butitwillnotleadtoamputationaslongyou “Amputation isalastresortinpatientswithdiabeteswho developaninfectionintheirfeet,andfortunatelyweare Sample ExamineeResponse “Will Ilosemyfeet,doctor?” Challenging QuestionstoAsk Examinee introduced selfbyname. Examinee introduced Examinee knockedonthedoor beforeentering. 4. Write thepatientnoteafterleavingroom. 3. Explainyourclinicalimpressionandworkupplantothepatient. 2. Performafocusedphysicalexam(donotperformrectal,genitourinary, orfemalebreastexam). 1. Take afocusedhistory. 139/85mmHg 15/minute 75/minute,regular Pretend tohaveanormalkneejerkandabsentanklereflex. (stocking distribution). Pretend thatyouhavealossofsharpanddullsensations,vibrationsense,positionsenseinbothfeet 98.0°F(36.7°C) PRACTICE CASES 207 Patient Response The last was 6 months ago, and it was 7. I start sweating. Sometimes I feel my heart racing, and juice. No. I have tingling and numbness in my feet all the time, especially at gotten worse over the past 2 months. night, and it’s No. twice a week. It usually ranges between 120 and 145. twice a week. It usually ranges between 6 months ago. 1c Foot infectionMarital or work problems No, my wife is great, and I am very happy in my job. No. Abdominal complaints (pain, dyspep- sia, nausea) Change in bowel habits problems (blurred vision)Visual No. No. Pulmonary complaints (shortness of breath, cough) Neurologic complaints (headaches, dizziness, weakness, numbness) Polyuria, dysuria, hematuria No. Medication side effectsHeart symptoms (chest pain, palpita- tions) Description of these symptoms No. I feel better after drinking orange It happens rarely if I miss a meal. Last time eyes were checkedHow he is feeling today there were no signs of diabetic eye disease. One year ago, and Good. Last HbA TreatmentCompliance with medicationsLast blood sugar readingBlood sugar monitoring I never miss any doses. days ago, and it was 135. Three in the evening. 20 units in the morning and 15 units NPH insulin, I have a blood sugar monitor at home, and I check my blood sugar Chief complaintOnset my doctor was I am here for a diabetes checkup. The last time I saw I have had diabetes mellitus for the past 25 years. Question

Examinee asked the SP to list his/her concerns and listened to the response without interrupting. SP to list his/her concerns and listened Examinee asked the own words. SP’s concerns, often using the the SP’s Examinee summarized efficiently and accurately. Examinee elicited data Examinee identified his/her role or position. identified his/her Examinee name. correctly used patient’s Examinee with the SP. made eye contact Examinee to the response. open-ended question and actively listened Examinee asked an ✓ Information Gathering Reflective Listening PRACTICE CASES 208 Physical Examination Connecting withthePatient ✓ Examinee didnotrepeatpainful maneuvers. Examinee usedrespectfuldraping. Examinee askedpermissionto starttheexam. Examinee washedhis/herhands. Examinee recognizedtheSP’s emotionsandrespondedwithPEARLS.

Question rgalrisNo. Oneortwoyearsago. Good. No. Iamnotdoing myjobthewayIusedto,butwifeunderstands Ican’t getitup,doc.Idon’t evenwakeupwitherectionsanymore. Drug allergies No. Iammarriedand livewithmywife. Feelings ofdepression Duration No. Libido Type ofsexualproblem Yes, whiskeyonthe weekends. No(toall4). No. Ieateverythingthatmywifecooks—meat,vegetables,etc.don’t Sexual activity Social history Tobacco Clerk. Illicit druguse Myfatherdied atage60ofastroke.Mymotherishealthy. CAGE questions Alcohol use None. Exercise Heartattacklastyear;highcholesterolfor2years. Diet Insulin,lovastatin,aspirin,atenolol. Occupation Family history No. No. Past surgicalhistory Ihadaheartattacklastyear. Past medicalhistory No. Current medications No. Yes, itwasdiagnosed2yearsago. History ofTIAorstroke Previous heartproblems History ofhypercholesterolemia No. Hypertension Appetite changes Weight changes Feelings ofanxietyorstress follow anyspecialdiet. and issupportive.Theytoldmeitthediabetes.Isit? Patient Response PRACTICE CASES 209 Maneuver Lifestyle modification (diet, exercise). Follow-up tests. Pulmonary examAbdominal examExtremitiesNeurologic exam Auscultation Auscultation, palpation, percussion and strength in lower extremities sign, sensation DTRs, Babinski’s pulses Inspected feet, peripheral Eye examNeck examCV exam exam Funduscopic Carotid auscultation Palpation, auscultation Exam Component

questions or concerns. Examinee asked if the SP had any other Examinee discussed initial diagnostic impressions. Examinee discussed initial diagnostic plans: Examinee discussed initial management ✓ Mr. Stern, the palpitations and sweating you have experienced are most likely due to episodes of low blood sugar, which may which you have experienced are most likely due to episodes of low blood sugar, Stern, the palpitations and sweating Mr. meals. The numbness you describe in your dose of insulin or from skipping or delaying have resulted from a higher-than-normal on your nervous system; better control of your blood sugar may help improve feet is probably related to the effect of diabetes an can cause the erection difficulties you describe. I will need to perform this problem. Many factors, including diabetes, some blood tests, and at some point we may also need to conduct some more complex examination of your genital area and run Do you have any questions for me? tests to identify the cause of your problems. Sample Closure Closure PRACTICE CASES 210 Physical Examination History USMLE STEP2CS Patient Note PRACTICE CASES 211 Patient Note Patient Diagnosis #3 History Finding(s): Physical Exam Finding(s): Diagnosis #2 History Finding(s): Physical Exam Finding(s): Diagnosis #1 History Finding(s): Physical Exam Finding(s): USMLE STEP 2 CS STEP USMLE Diagnostic Workup Differential Diagnosis Differential USMLE STEP 2 CS Patient Note

History HPI: 56 yo M presents for diabetes follow-up. 25-year history of DM, treated with insulin. Compliant with medications. Monitors blood glucose twice a week, readings between 120 and 145 mg/dL. Last HbA1c 6 months ago was 7%. Occasional episodes of palpitations and diaphoresis, occurring after missing meals and resolving with drinking orange juice. Tingling and numbness in feet all the time, especially at night, worse over past 2 months. Loss of erections × 2 years; absence of early-morning erections. No weight or appetite changes. No special diet. ROS: Negative except as above. Allergies: NKDA. Medications: Lovastatin, NPH insulin, aspirin, atenolol. PMH: Hypercholesterolemia diagnosed 2 years ago; MI 1 year ago. PSH: None. SH: No smoking, drinks whiskey on weekends (CAGE 0/4), no illicit drugs. Works as a clerk. He is married and lives with his wife. FH: Father died of a stroke at age 60.

Physical Examination Patient is in no distress. VS: WNL. HEENT: PERRLA, no funduscopic abnormalities. Neck: No carotid bruits, no JVD. Chest: Clear breath sounds bilaterally. Heart: Apical impulse not displaced; RRR; normal S1/S2; no murmurs, rubs, or gallops. Abdomen: Soft, nondistended, nontender, BS, no bruits, no organomegaly. Extremities: No edema, no skin breakdown, 2+ dorsalis pedis pulses. Neuro: Motor: Strength 5/5 in bilateral lower extremities. DTRs: Symmetric 2+ knee jerks, absent ankle jerks and Babinski bilaterally. Sensation: Decreased pinprick; soft touch, vibratory, and position sense in bilateral lower extremities. PRACTICE CASES

212 USMLE STEP 2 CS Patient Note

Differential Diagnosis Diagnosis #1: Insulin-induced hypoglycemia History Finding(s) Physical Exam Finding(s) Episodes of palpitations and diaphoresis that resolve with drinking orange juice Tight glycemic control

Diagnosis #2: Diabetic peripheral neuropathy History Finding(s) Physical Exam Finding(s) History of diabetes mellitus Absent ankle jerk Constant numbness and tingling in feet

Diagnosis #3: Organic erectile dysfunction History Finding(s) Physical Exam Finding(s) Loss of erection for 2 years with absence of early-morning erection History of diabetes mellitus History of alcohol use Taking lovastatin and atenolol

Diagnostic Workup

Genital exam

Serum glucose, HbA1c UA, urine microalbumin, BUN/Cr Doppler U/S—penis Nerve conduction studies PRACTICE CASES

213 PRACTICE CASES 214 Diagnostic Workup some ofthecommoncausesdiscussedbelow. and paraneoplasticdisorders.Nospecificcauseisdeterminedinupto50%ofcases.Thehistoryexamguideus to The differentialfornondiabeticperipheralneuropathyincludeshereditary, toxic,metabolic,infectious,inflammatory, Additional Differential Diagnoses Patient NoteDifferential Diagnoses CASE DISCUSSION axonal vs.demyelinatingmechanism. Nerve conductionstudies: Doppler U/S—penis: UA, urinemicroalbumin, BUN/Cr: Serum glucose,HbA Genital exam: and canbedetectedbymonitoringESR,ANCA,RF, andanti−cycliccitrullinatedpeptide(anti-CCP)antibody. Vasculitides: hypothyroidism. Hypothyroidism: Renal failure: neuropathy. Multiple myeloma: vitamin B paresthesias andisdirectlyattributabletoalcoholorassociatednutritionaldeficiencies(eg,thiamine Alcoholic peripheralneuropathy: statins and origin. AlcoholalsocausesanautonomicneuropathyandmaycontributetoED,ascanmedicationssuch loss oflibidowithabsenceearly-morningerectionssuggestsorganicEDeitheravascularorneurologic medications takenforassociatedconditions(eg,antihypertensives).Ingeneral,impotenceunaccompaniedby Erectile dysfunction(ED): that worsenatnightandlossofanklereflexes,asseeninthiscase,areclassic. symmetric sensoryormixedpolyneuropathy(amongotherpatternsofneuropathy).Burningfootparesthesias Diabetic peripheralneuropathy: educated abouthowtorecognizeandtreatthiscomplication. seizures. Maintainingtightglycemiccontrolmayoccasionallyresultinhypoglycemia,andpatientsshouldbe symptoms ofhypoglycemiaincludesweating,tachycardia,palpitations,tremor, anxiety, weakness,confusion,and Insulin-induced hypoglycemia: 12 β ). -blockers. Polyarteritis nodosa, rheumatoidarthritis,andothervasculitidesmaycauseperipheralneuropathy Polyarteritisnodosa, Uremiamaycauseasensoryperipheralneuropathythataffectdiabeticpatients. To ruleoutPeyronie’s disease(eg,penilescarringor plaqueformation). Peripheralneuropathyandotherneurologicsymptomsmaybeassociatedwith Myelomaorotherparaproteinemiasmustberuledoutinapatientwithperipheral 1c A helpful noninvasive test to measure penile blood flow. Ahelpfulnoninvasivetesttomeasurepenileblood : To assessglycemiccontrol. Indiabetics,EDisusuallyrelatedtovasculardisease,autonomicneuropathy, or To confirmthatsymptomsarisefromaperipheralnerveoriginandtoindicatean Thepatient’s ofhypoglycemia.Typical historysuggestsepisodes signsand Involvementoftheperipheralnervoussystemindiabetesmayleadto Thiscausesadistalsensorimotorpolyneuropathymarkedbypainfulleg To screenfordiabeticnephropathy. PRACTICE CASES 215 To detect paraproteinemias (eg, multiple paraproteinemias detect To In select cases, other studies used to evaluate peripheral neuropathy include ESR, BUN/Cr, neuropathy include ESR, BUN/Cr, studies used to evaluate peripheral In select cases, other urine heavy metal screen, CSF examination, CXR, and cutaneous nerve biopsy (eg, to diagnose amyloidosis). nerve biopsy (eg, to diagnose CSF examination, CXR, and cutaneous urine heavy metal screen, CBC, serum calcium, ESR, serum protein electrophoresis: serum protein calcium, ESR, CBC, serum elevated blood Other findings include associated finding. anemia is often an myeloma); and an calcium levels elevated ESR. Other studies: serologies, RPR, HIV antibody, hepatitis B and C ANA, ANCA, anti-CCP antibody, RF, TSH, liver enzymes, PRACTICE CASES 216 Entrance Building theDoctor-Patient Relationship Examinee Checklist Notes fortheSP Patient isa25yoF. Patient Description Checklist/SP Sheet Examinee Tasks HR: RR: Temp: BP: Signs Vital Julia Melton,a25-year-old female,comestotheEDafterbeingassaulted. Opening Scenario DOORWAY INFORMATION CASE 9 not yourfaultbyanymeans.Whoeverdidthistoyoushould beheldaccountable.” “I amsosorryforwhathappenedtoyou;itishorrificand mustbeverydifficultforyoutohandle.However, itis Sample ExamineeResponse “This isallmyfault,doctor. Doyouthinkmyfriendswilleveracceptmeagain?” Challenging QuestionstoAsk Examinee identifiedhis/her role orposition. selfbyname. Examinee introduced Examinee knockedonthedoorbeforeentering. 4. Write thepatientnoteafterleavingroom. 3. Explainyourclinicalimpressionandworkupplantothepatient. 2. Performafocusedphysicalexam(donotperformrectal,genitourinary, orfemalebreastexam). 1. Take afocusedhistory. 120/85mmHg 17/minute 90/minute,regular Pretend tohaverightchestpainwithdeepinspiration,cough,andpalpation. Start weepingwhenaskedaboutphysicaland/orsexualassaults. Look depressedandtearful. 98.0°F(36.7°C) PRACTICE CASES 217 Patient Response into a storage room. I started screaming, but the men started to slap into a storage room. I started screaming, me and beat me up with their fists. against them. move at all even though I tried to struggle Headache No. Alleviating factorsExacerbating factorsBleeding or bruisesLoss of consciousness It improves when I sit still. It gets worse whenever I move or take a deep breath. No. No. PainLocation of the worst painRadiationSeverity on a scale The right chest. my chest. especially on the right side of I feel sore all over, About 8/10. Yes, No. Type of intercourse (oral, vaginal, anal)Type Vaginal. Foreign objects usedLast menstrual periodContraceptives None. Three weeks ago. I’m not on the pill or anything. Assault objectsSexual assaultDid they use condoms?Did ejaculation occur? They used their fists and their bodies to hold me down. I couldn’t No. Yes. know. I don’t Did you recognize the assailants?Did you report the incident?Description of the assaultthe bar but never talked to them. I have seen them in No. and then all of a sudden I was pulled I was walking toward my car, Chief complaintOnsetIncident location I was attacked by 2 men. I usually go to. It happened outside the bar that About 3 hours ago. I came to the ED right away. Question

Examinee summarized the SP’s concerns, often using the SP’s own words. SP’s concerns, often using the the SP’s Examinee summarized efficiently and accurately. Examinee elicited data Examinee correctly used patient’s name. correctly used patient’s Examinee with the SP. made eye contact Examinee to the response. open-ended question and actively listened Examinee asked an to the response without interrupting. SP to list his/her concerns and listened Examinee asked the ✓ Information Gathering Reflective Listening Reflective PRACTICE CASES 218 Physical Examination Connecting withthePatient ✓ Examinee didnotrepeatpainfulmaneuvers. Examinee usedrespectfuldraping. Examinee askedpermissiontostarttheexam. Examinee washedhis/herhands. Examinee recognizedtheSP’s emotionsandrespondedwithPEARLS.

Question oac No. No. Never. Occasionally. Student. None. Drug allergies Tobacco None. Illicit druguse Mywristshurtwheretheywereholdingmedown. None. Alcohol use None. Occupation Yes, ithurtseverywhere. Family history No. Past surgicalhistory No. Past medicalhistory Yes. Current medications No,butIhaven’t gonetothebathroomsinceincident. Joint pain No. Abdominal pain No,Iamjusttired. Yes, Ifeelthatcan’t getenoughair. No. Nausea/vomiting Vaginal bleeding instool/urine Blood No. Palpitations Shortness ofbreath Numbness Weakness Dizziness Change invision Patient Response PRACTICE CASES

l n 219 Maneuver Follow-up tests: Examinee mentioned the need for a pelvic exam. Follow-up tests: Examinee mentioned Pulmonary examAbdominal examNeurologic examMusculoskeletal exam palpation, percussion Inspection, auscultation, Inspection, auscultation, palpation Inspection, palpation Mental status, cranial nerves, gross motor Head and neck examHead and Mouth examCV exam Inspection, palpation Inspection Auscultation Exam Component

Examinee asked if the SP had any other questions or concerns. Examinee asked if the SP had any other Examinee discussed initial diagnostic impressions. Examinee discussed initial diagnostic plans: Examinee discussed initial management ✓ other resources. Do you have any questions for me? other resources. Do you have any questions to make sure you have no injuries in the genital area. In addition, I will need to collect some specimens and swabs from your genital area. In addition, I will need to collect some specimens and swabs from your to make sure you have no injuries in the run will used as evidence if you choose to file charges, and also to look for STDs. We body and genital area so that they can be wil order a pregnancy test and some x-rays. If your pregnancy test is negative, we some blood tests for potential STDs and will It would also be prudent to give you some antibiotics to protect you from offer you some options for emergency contraception. you with phone numbers for support groups and I can have our social worker come talk to you and provide infections. Finally, Sample Closure feel to you. I want to emphasize that it is not your fault, and you should not Ms. Melton, I am really sorry for what happened the incident to the police. In the meantime, I will need to do a pelvic examinatio guilty about it. I recommend that you report Closure PRACTICE CASES 220 Physical Examination History USMLE STEP2CS Patient Note PRACTICE CASES 221 Patient Note Patient Diagnosis #3 History Finding(s): Physical Exam Finding(s): Diagnosis #2 History Finding(s): Physical Exam Finding(s): Diagnosis #1 History Finding(s): Physical Exam Finding(s): USMLE STEP 2 CS STEP USMLE Diagnostic Workup Differential Diagnosis Differential USMLE STEP 2 CS Patient Note

History HPI: 25 yo F comes to the ED after being sexually and physically assaulted. The event happened about 3 hours ago as she was leaving a bar. She was beaten and raped by 2 unknown men. They had vaginal intercourse with her without using condoms, and she is unsure if ejaculation occurred. Her LMP was 3 weeks ago. She does not use any form of contraception. She also c/o shortness of breath, palpitations, and right chest pain that is nonradiating. The chest pain is exacerbated by movement and deep breaths and is relieved by sitting still. No nausea or vomiting. No dizziness or headache. No weakness or numbness in her extremities; no vaginal, rectal, or urinary bleeding. ROS: Negative except as above. Allergies: NKDA. Medications: None. PMH: None. PSH: None. SH: No smoking, occasional EtOH, no illicit drugs. FH: Noncontributory.

Physical Examination Patient is anxious and in acute distress. VS: WNL. HEENT: No JVD, PERRLA, EOMI. Chest: Clear breath sounds bilaterally; tenderness on palpation of right chest wall. Heart: Normal S1/S2; no murmurs, rubs, or gallops. Abdomen: Soft, nontender, nondistended, BS, no rebound or organomegaly. Neuro: Mental status: Alert and oriented × 3. Cranial nerves: 2–12 grossly intact. Motor: Strength 5/5 in all muscle groups.

Differential Diagnosis Diagnosis #1: Rib fracture History Finding(s) Physical Exam Finding(s) Physical assault Tenderness on palpation of right chest wall Right chest pain Pain is exacerbated by movement and deep breaths

Diagnosis #2: STD History Finding(s) Physical Exam Finding(s) Sexual assault by 2 men No condom use PRACTICE CASES

222 USMLE STEP 2 CS Patient Note

Diagnosis #3: Pregnancy History Finding(s) Physical Exam Finding(s) Unprotected vaginal intercourse with possible ejaculation No OCP use Last menstrual period 3 weeks ago

Diagnostic Workup

Pelvic exam XR—skeletal survey CXR Urine hCG Wet mount, KOH prep, cervical culture, gonorrhea and chlamydia tests HIV antibody, VDRL, HBV antigen PRACTICE CASES

223 PRACTICE CASES 224 Diagnostic Workup Additional Differential Diagnoses Patient NoteDifferential Diagnoses CASE DISCUSSION Careful consideration should be given to maintaining a set chain of custody oftheevidencecollected. Careful considerationshouldbegiventomaintainingaset chainofcustody process. Tissue swabsshouldbecollectedfromthevictimassoonpossibleto assist inevidencecollection. Evidence collectionusingrapekit: HIV antibody, VDRL,HBVantigen: identification oftheassailants. is indicativeofbacterialvaginosis.Ifspermaredetectedinthevictim,testingDNAmayaid aftertheadditionofKOHtodischarge Motile organismsareseenintrichomonalinfection.A“fishy”odor (clue cells)suggestsbacterialvaginosis,andthepresenceofhyphaesporespointstocandidalinfection. is examinedmicroscopicallytoevaluateforinfection.Thepresenceofepithelialcellscoveredwithbacteria Wet mount,KOHprep, cervicalculture, gonorrheaandchlamydiatests: Urine hCG: CXR: XR—skeletal survey: medical andforensicpurposes. Pelvic exam: Muscle rupture: traumatic pneumothorax.ACXRisafastandeasytoolwithwhichtoevaluatepatientsfor the tracheaawayfrominjuredside,andJVD.Becausethispatientsufferedphysicaltrauma,shemayhavea and parietalpleurae.Physicalfindingsincludeunilaterallossofbreathsoundswithhyperresonance,shifting Pneumothorax/hemothorax: emergency contraception. Pregnancy: chlamydia, gonorrhea,HIV, andhepatitisB. STDs: and cough. Rib fracture: To detectribfractures,pneumothorax,and pleuraleffusions. Sexualassaultvictimsmayacquireavarietyofpathogensduringtheincident,includingtrichomoniasis, To ruleoutpregnancy. Allsexualassaultvictimsshouldbeevaluatedforpossibleexistingpregnancyandoffered Thiscanresultfromanyinsulttothechestwall.Asimplefracturecausepainoninspiration To evaluateforanypossiblephysicalinjury ofthegenitaloranalareaandtocollectspecimensfor Chestpainintraumavictimsmaybemusculoskeletalorigin. To detectpossibleboneorribfractures. Defined as the presence of air or blood inthepleuralspacebetween thevisceral Definedasthepresenceofairorblood To ruleoutHIV, syphilis,and hepatitis Binfection. Rapekitsareavailabletofacilitateandguidetheevidence collection Thevaginaldischarge PRACTICE CASES 225 99.9°F (37.7°C) Exhibit pain in your calf when the doctor dorsiflexes your right ankle. Exhibit pain in your calf when the doctor the cuts that you got after a fall. Place a bandage on your right leg to cover 88/minute 13/minute 130/70 mm Hg Examinee knocked on the door before entering. Examinee introducedself by name. 2. Perform a focused physical exam (do not perform rectal, genitourinary, or female breast exam). not perform rectal, genitourinary, 2. Perform a focused physical exam (do workup plan to the patient. 3. Explain your clinical impression and the patient note after leaving the room. 4. Write 1. Take a focused history. a focused 1. Take obesity increases your risk of having a clot, so I suggest that you exercise regularly and manage your diet.” obesity increases your risk of having a clot, so I suggest that you exercise regularly Sample Examinee Response a clot. Above all, you should avoid “There are several measures you can take that may prevent you from having immobilization for long periods example, while sitting at your computer desk or on long-distance of time—for perhaps take a short walk. If you are on oral contraceptive pills, I strongly to move in place and plane trips. Try Studies have also shown that recommend that you stop taking them, as they are known to precipitate clotting. Challenging Questions to Ask one too?” get I don’t “My father had a clot in his leg. What do you think I should do to make sure DOORWAY INFORMATION DOORWAY CASE 10 CASE Examinee Checklist Relationship Building the Doctor-Patient Entrance Checklist/SP Sheet Patient Description married with two children. Patient is a 35 yo F, Notes for the SP RR: HR: Examinee Tasks Riva George, a 35-year-old female, comes to the hospital complaining of pain in her right calf. complaining of pain in her right female, comes to the hospital Riva George, a 35-year-old Vital Signs BP: Temp: Opening Scenario PRACTICE CASES 226 Information Gathering Reflective Listening ✓ Examinee eliciteddataefficientlyandaccurately. Examinee summarizedtheSP’s concerns,oftenusingtheSP’s ownwords. Examinee askedtheSPtolisthis/herconcernsandlistenedresponsewithoutinterrupting. Examinee askedanopen-endedquestionandactivelylistenedtotheresponse. Examinee madeeyecontactwiththeSP. Examinee correctlyusedpatient’s name. Examinee identifiedhis/herroleorposition.

Question ee Ihavefeltwarmrecentlybuthaven’t measuredmytemperature. Ihavebeentaking oralcontraceptivesfor2years. Two Regular. weeks ago. last3days,andIuse 3–4pads.Theyarenot My periods No. Frequency ofmenstrualperiods Itravelfrequentlyaspartofmyconsultingbusiness,andaweek ago Contraceptives Myrightlegfeelswarmerthanmyleft. No. No. Last menstrualperiod Yes. Yes, Ifelldownandscratchedmyrightleg(pointsto bandage). Fever Attheendofday, mylegsfeelheavyandpit on pressure. Recent immobilization Chest pain Shortness ofbreath Varicose veins Pressure,spasms. Walking andextendingmyknee. Warmth Painmedication(ibuprofen).ItalsohelpsifIpropupmylegwitha No. Redness Injury Swelling 8/10. Thepainwasmildinthebeginning,butnowithurtsevenwhenI Exacerbating factors Itispresentallthetime. Thepainstartedafewdaysagoandhasgottenworse. Alleviating factors Quality Radiation Paininmyrightcalfmuscle. Severity onascale Progression Frequency Onset Chief complaint take justasinglestep. accompanied bypain. I tooka15-hourflighttomeetanimportantclient. pillow. Patient Response PRACTICE CASES 227 Maneuver Patient Response Patient popliteal, dorsalis pedis) on both sides both sides) Follow-up tests. ExtremitiesNeurologic exam Sensory and motor reflexes (knee, ankle) Inspection, palpation; checked for Homans’ sign CV/pulmonary examJoint exam Inspection, auscultation, palpation; compared pulses (femoral, joint on Inspection, palpation, range of motion (knee, ankle, hip Drug allergies/herbal medication No. Alcohol useIllicit drug useTobaccoSexual activity No. No. my husband. With No. Past medical historyPast surgical historyFamily historyOccupation None. None. My dad had a clot in his leg. Executive consultant. Obstetric historyObstetric Last Pap smear changesWeight delivery. kids, both with a normal I have had 2 it was normal. One year ago; my last child 3 years ago. after having I gained 50 pounds Exam Component Question

Examinee asked if the SP had any other questions or concerns. Examinee discussed initial diagnostic impressions. Examinee discussed initial management plans: Examinee did not repeat painful maneuvers. Examinee washed his/her hands. exam. Examinee asked permission to start the Examinee used respectful draping. Examinee recognized the SP’s emotions and responded with PEARLS. Examinee recognized the SP’s ✓ ✓ Closure Physical Examination Connecting with the Patient PRACTICE CASES 228 Do youhaveanyquestionsforme? clot, wewillstartyouonbloodthinnerstopreventfurthercomplications,suchasthepossibilityofaclottravelingyour running afewbloodtestsaswellsomeimagingstudiesthatshouldhelpusmakefinaldiagnosis.Ifyourtestresultsshow However, wewillalsolookforotherpossiblecausesofyoursymptoms,suchasaninfectionorarupturedcyst.We willbe Mrs. George,onthebasisofyourhistoryandmyphysicalexamination,Ibelieveitispossiblethatyouhadabloodclot. Sample Closure lungs. a PRACTICE CASES 229 Patient Note Patient USMLE STEP 2 CS STEP USMLE Physical Examination History USMLE STEP 2 CS Patient Note

Differential Diagnosis Diagnosis #1 History Finding(s): Physical Exam Finding(s):

Diagnosis #2 History Finding(s): Physical Exam Finding(s):

Diagnosis #3 History Finding(s): Physical Exam Finding(s):

Diagnostic Workup PRACTICE CASES

230 USMLE STEP 2 CS Patient Note

History HPI: 35 yo F c/o right calf pain of a few days’ duration. The pain is constant, 8/10 in intensity, not radiating, aggravated on walking and extending the knee, and associated with swelling, redness, and warmth. It is alleviated on elevation of the foot and with ibuprofen. The patient took a 15-hour flight 1 week ago. She has a history of weight gain postpartum and cuts to the right leg secondary to a fall. She has 2 children, both normal deliveries. LMP was 2 weeks ago. The patient says she has gained 50 lbs in the past 3 years. She has been on OCPs for 2 years. No history of chest pain or shortness of breath. ROS: Negative except as above. Allergies: NKDA. Medications: OCPs, ibuprofen. PMH: None. PSH: None. SH: No smoking, no EtOH, no illicit drugs. FH: Father had DVT. No history of sudden deaths in the family.

Physical Examination Patient is in severe pain. VS: WNL except for low-grade fever. Chest: Clear breath sounds bilaterally; no rales or rhonchi. Heart: RRR; normal S1/S2; no murmurs, rubs, or gallops. Abdomen: Soft, nontender, nondistended, BS. Extremities: Inspection: Right calf appears red and swollen compared to left; contours of the muscles appear normal; no ulcers or pigmentation. Palpation: Right leg is warmer compared to left; pitting pedal edema on right side; multiple healing cuts covered with bandage on right leg; dorsalis pedis pulse felt and equal on both sides; mobility normal at ankle joint, knee, and hip joint; Homans’ sign on right side. Neuro: Mental status: Alert and oriented. DTRs: Symmetric 2+. Motor/sensation: Normal. Cranial nerves: 2–12 intact. Gait: Normal.

Differential Diagnosis Diagnosis #1: Deep venous thrombosis History Finding(s) Physical Exam Finding(s) Recent 15-hour airplane flight Homans’ sign Weight gain of 50 lbs over past 3 years Pitting edema Taking OCPs for 2 years Swollen, tender, red, warm right calf Father with DVT (possible familial thrombophilia) PRACTICE CASES

231 USMLE STEP 2 CS Patient Note

Diagnosis #2: Cellulitis History Finding(s) Physical Exam Finding(s) Cuts to right leg secondary to fall Swollen, tender, red, warm right calf Low-grade fever Temperature 99.9°F

Diagnosis #3: Rupture of Baker’s cyst History Finding(s) Physical Exam Finding(s) Spasmodic pain in right calf Swollen, tender, warm right calf

Diagnostic Workup

Doppler U/S—legs D-dimer Hypercoagulability testing CBC with differential Wound and blood cultures PRACTICE CASES

232 PRACTICE CASES 233 This presents with sudden pain associated with rupture at the This presents with sudden pain associated DVT is common in the lower limbs and may arise under conditions of the lower limbs and may arise under DVT is common in Several autoantibodies are implicated in thrombophilic states. Proteins Baker’s cysts (also known as popliteal cysts) are seen in the popliteal fossa. Arthritis cysts (also known Baker’s To detect infections such as cellulitis. To An initial diagnostic test that is noninvasive and can visualize clots in the veins of the An initial diagnostic test that is noninvasive and can visualize clots in the Undue strain may cause physical tearing of muscles or tendons, inducing spasm and pain. Undue strain may cause physical tearing Injuries can cause bleeding intramuscularly (in which no bruising occurs) or intermuscularly (in Injuries can cause bleeding intramuscularly Trauma can lead to cellulitis of the skin and subcutaneous tissue or to myositis of the calf muscle. All tissue or to myositis of the calf muscle. cellulitis of the skin and subcutaneous can lead to Trauma A cross-linked fibrin degradation product that may be increased in DVT. It is usually indicated in A cross-linked fibrin degradation product that may be increased in DVT. -dimer: sufficiently high to rule out DVT. cases with a low to intermediate probability of thromboembolism. The negative predictive value of this test is cases with a low to intermediate probability of thromboembolism. The negative factors, recurrent DVT, or a family history of DVT. or a family history of DVT. factors, recurrent DVT, CBC with differential: Hypercoagulability testing: Hypercoagulability prothrombin gene mutations, factor V Leiden, partial antithrombin deficiency, C and S deficiency, hyperhomocysteinemia, antiphospholipid antibody syndrome, and paroxysmal nocturnal hemoglobinuria may Hypercoagulability testing should be done on patients with no predisposing all lead to increased coagulability. Doppler U/S—legs: leg. D bruising and pain on standing on the tips of the toes. Patients also present with pain on dorsiflexion of the ankle bruising and pain on standing on the against resistance. Spasm/sprain: Ligaments can be ruptured or torn as a result of overstretching or injuries. Hematoma: present with pain, swelling, and restricted movement. The condition which bruising is usually present). Patients may lead to posterior compartment syndrome. muscle: of the gastrocnemius Rupture muscle, halfway between the knee and the heel. There may be musculotendinous junction of the gastrocnemius or a cartilage tear of the knee joint may cause excess synovial fluid to be accumulated, forming a cyst. A ruptured cause excess synovial fluid to be accumulated, forming a cyst. A ruptured or a cartilage tear of the knee joint may the knee, pain on knee Ruptures can present with tightness and swelling behind cyst may mimic a DVT. Baker’s extension, and stiffness of the calf muscle. the classic signs of inflammation associated with fever (calor, dolor, rubor, tumor) may point to this diagnosis. tumor) may point to this rubor, dolor, inflammation associated with fever (calor, the classic signs of Regional lymph node ossificans may occur as a are commonly seen. Myositis enlargement and tenderness hardening of the muscle and pain on contraction. Radiographs may show causing complication of this disorder, ossification in the muscle. cyst: of Baker’s Rupture (eg, postsurgery, trauma, sedentary jobs, extended airplane or automobile travel) are predisposing factors. extended airplane or automobile travel) trauma, sedentary jobs, (eg, postsurgery, and malignancy, obesity, DVT, synthetic estrogens, prior advancing age, pregnancy, Other risk factors include may producethrombophilia. DVT be asymptomatic. A positive edema of the affected limb or may pain and of DVT but not diagnostic. on dorsiflexion of the ankle) is suggestive Homans’ sign (pain Cellulitis: Deep venous thrombosis (DVT): Deep venous thrombosis that result in prolonged immobilization Conditions endothelial injury. and venous stasis, hypercoagulability, Diagnostic Workup Additional Differential Diagnoses Additional Differential CASE DISCUSSION CASE Diagnoses Note Differential Patient PRACTICE CASES 234 or tendonrupture. pulmonary embolism.MRIisnoninvasiveandcandetectacute,symptomaticproximalDVTs aswellmuscle CT/MRI: CPK andmyoglobin: Wound andbloodcultures: CTvenographyisusedtodiagnoseDVTinconjunctionwithcontrast-enhancedspiralruleout Bothcanbeelevatedinmuscleinjury(myositis). To workupaninfectiousetiologyofcellulitis. PRACTICE CASES 235 99.9°F (37.7°C) 74/minute, regular 16/minute 115/75 mm Hg Examinee correctly used patient’s name. Examinee correctly used patient’s Examinee made eye contact with the SP. Examinee knocked on the door before entering. Examinee introducedself by name. Examinee identified his/her role or position. 2. Perform a focused physical exam (do not perform rectal, genitourinary, or female breast exam). not perform rectal, genitourinary, 2. Perform a focused physical exam (do workup plan to the patient. 3. Explain your clinical impression and the patient note after leaving the room. 4. Write 1. Take a focused history. a focused 1. Take DOORWAY INFORMATION DOORWAY CASE 11 CASE Entrance “I see that you are very concerned about your voice, and I am concerned too. I am not yet sure what has caused “I see that you are very concerned about your voice, and I am concerned too. will need to do some tests to find out what the problem is and decide on your treatment.” your hoarseness. We Examinee Checklist Relationship Building the Doctor-Patient Speak slowly and in a hoarse voice. Challenging Questions to Ask “Am I going to get my voice back?” Sample Examinee Response Checklist/SP Sheet Patient Description Patient is a 62 yo M, married with 4 children. Notes for the SP RR: HR: Examinee Tasks Oliver Jefferson, a 62-year-old male, comes to the office complaining of hoarseness. male, comes Oliver Jefferson, a 62-year-old Vital Signs BP: Temp: Opening Scenario PRACTICE CASES 236 Information Gathering Reflective Listening ✓ Examinee eliciteddataefficientlyandaccurately. Examinee summarizedtheSP’s concerns,oftenusingtheSP’s ownwords. Examinee askedtheSPtolisthis/herconcernsandlistenedresponsewithoutinterrupting. Examinee askedanopen-endedquestionandactivelylistenedtotheresponse.

Question itTheusual.Nochangeinmydiet.Justeatingless. None. Highcholesterol, butIdon’t takeanymedicationforit. Yes, Idon’t havethesameenergyasbefore. Diet Past surgicalhistory Past medicalhistory breath, hemoptysis,cough) No. Pulmonary symptoms(eg,shortnessof Ifeelhot, but Ididn’t measuremytemperature,andIdon’t have Cardiac symptoms(eg,palpitations) Ihaveapoorappetite. constipation) Ihavelost10poundsoverthepast3months. GI symptoms(eg,nausea/vomiting, Fatigue Yes, Ifeellikethere’s alumpinmythroat. Yes, Ihaveheartburnallthetime,butdon’t take anymedication Fever, nightsweats No. Swollen glandsorlymphnodes Nothing. Appetite changes Nothing. Weight changes History ofstrokeorTIA Heartburn Exacerbating factors Iwasateacherfor20years,butnowamretired. Itisgettingworse. Alleviating factors No. No. sore throat,runnynose) Recent upperrespiratoryinfection(eg, Threemonthsago. No. Exposure tocoldweatherordust Voice overuserecently Itis allthetime. Itstartedgradually. Pain duringspeaking Hoarseness. inthepast Similar episodes Progression Constant orintermittent Suddenly orgradually Onset Chief complaint No. No. chills ornightsweats. for it. I hadtheflu4weeksago. Patient Response PRACTICE CASES 237 to Maneuver Patient Response Patient thyroid gland Follow-up tests. CV examPulmonary examAbdominal examExtremities Auscultation Auscultation Auscultation, palpation, percussion Inspection, DTRs HEENT mouth and throat, lymph nodes; Inspected conjunctivae, examined Drug allergies None. Alcohol useCAGE questionsTobaccoIllicit drug use No (to all 4). Three glasses of wine every day. None. 30 years. I have been smoking a pack a day for the past Yes, Current medicationsFamily historyOccupation None. thyroid disease and my father had lung cancer. My mother had Retired teacher. Exam Component Question

Examinee asked if the SP had any other questions or concerns. Examinee discussed initial diagnostic impressions. Examinee discussed initial management plans: Examinee did not repeat painful maneuvers. Examinee washed his/her hands. exam. Examinee asked permission to start the Examinee used respectful draping. Examinee recognized the SP’s emotions and responded with PEARLS. Examinee recognized the SP’s ✓ ✓ neck. These tests will likely reveal the underlying problem. Since cigarette smoking is dangerous to your health, I advise you neck. These tests will likely reveal the underlying problem. Since cigarette smoking that you stop drinking, as alcohol and quit smoking; we have many ways to help you if you are interested. I also recommend Do you have any questions for me? smoking are associated with laryngeal cancer. Sample Closure Jefferson, there are a few things that could be causing your hoarseness, such as an infection or a benign or cancerous Mr. the inside of your throat, and a CT scan of your which is a procedure to view find out, I need to do a laryngoscopy, growth. To Closure Physical Examination Connecting with the Patient PRACTICE CASES 238 Physical Examination History USMLE STEP2CS Patient Note PRACTICE CASES 239 Patient Note Patient Diagnosis #3 History Finding(s): Physical Exam Finding(s): Diagnosis #2 History Finding(s): Physical Exam Finding(s): Diagnosis #1 History Finding(s): Physical Exam Finding(s): USMLE STEP 2 CS STEP USMLE Diagnostic Workup Differential Diagnosis Differential USMLE STEP 2 CS Patient Note

History HPI: 62 yo M c/o hoarseness × 3 months. Painless, gradually getting worse. Mild fever, fatigue, and “lump in my throat.” Poor appetite; lost 10 lbs in 3 months. History of flu 4 weeks ago. ROS: Negative except as above in addition to heartburn. Allergies: None. Medications: None. PMH: High cholesterol. PSH: None. SH: Drinks 3 glasses of wine/day/30 years; smoked 30 packs/year; CAGE (0/4). History of voice overuse (worked as a teacher for 20 years). FH: Mother with hypothyroidism, father with lung cancer.

Physical Examination Patient is in no acute distress. VS: WNL except for low-grade fever. HEENT: Nose, mouth, and pharynx WNL. Neck: Right anterior cervical chain with lymphadenopathy. No lymphadenopathy on the left. Chest: Nontender, bilateral clear BS. Heart: PMI not displaced, regular rhythm, no murmurs or rubs. Abdomen: BS, nondistended, no organomegaly. Extremities: DTRs are equal.

Differential Diagnosis Diagnosis #1: Laryngeal cancer History Findings(s): Physical Exam Finding(s) Cervical lymphadenopathy Temperature 99.9°F Worsening hoarseness over past 3 months Weight loss, decreased appetite, and low-grade fever History of cigarette smoking and alcohol use Advanced age PRACTICE CASES

240 USMLE STEP 2 CS Patient Note

Diagnosis #2: Laryngitis History Finding(s) Physical Exam Finding(s) History of flu 4 weeks ago Temperature 99.9°F Low-grade fever GERD History of cigarette smoking

Diagnosis #3: Vocal cord polyp/nodule History Finding(s) Physical Exam Finding(s) Vocal overuse from teaching for 20 years

Diagnostic Workup

Laryngoscopy ESR CT—chest and neck U/S—neck PRACTICE CASES

241 PRACTICE CASES 242 Diagnostic Workup Additional Differential Diagnoses Patient NoteDifferential Diagnoses CASE DISCUSSION Cardiac echocardiography: TSH: CBC: Esophageal pHmonitoring: U/S—neck: CT—chest andneck: ESR: also allowsbiopsyofsuspiciouslesionsforpathologicevaluation. Laryngoscopy: inflammation ofthevocalcords,leadingtohoarseness. Gastroesophageal reflux disease(GERD): murmur andtachycardia. fever. Othersymptomsincludepalpitationsandeasyfatigue.Thephysicalexamfindingsdiastolic the recurrentlaryngealnerve.MVSismorecommoninwomenandusuallypresentswithahistoryofrheumatic Mitral valvestenosis(MVS): some ofthepatient’s complaints,suchaslossofappetiteandfatigue, butdoesnotexplainallhissymptoms. Hypothyroidism: symptoms thatthepatientdescribes. can beeasilyidentifiedbymeansoflaryngoscopy. However, thisdiagnosisdoesnotexplaintheconstitutional Vocal cord polyps/nodules: patient hasalonghistoryofuntreatedGERD,soitcouldbesignchroniclaryngitis. viral andisself-limited.Commoncausesofthechronicformarecigarettesmoke,pollutedair, andGERD.This Laryngitis: fever, weightloss,fatigue,poorappetite)andlonghistoryofsmokingdrinking. Laryngeal cancer: Will beincreasedininfectiousandmalignantcauses. To diagnosethyroiddisease. Anemiacanbeassociatedwithhypothyroidism,andan elevated WBCcountiscommonininfections. Thisisacommonconditionofthelarynxandcanbeacuteorchronic.Theformmostlikely To identifythepresenceoflymphadenopathy. Thegoldstandardforevaluatingthelarynx;allowsdirectvisualizationofvocalcords.It Hoarsenessisoneofthemanifestationshypothyroidism.Hypothyroidismcanexplain Thisisthemostlikelydiagnosisgivenpatient’s constitutionalsymptoms(low-grade Canidentifythelocationandextentofmostlaryngeallesions. Essentialindiagnosingcardiacvalvulardiseases. Benignvocalfoldlesionsaresometimesrelatedtooveruseofthevoiceand To diagnoseGERDasacauseoflaryngitis. HoarsenessinMVSisduetoenlargementoftheleftatriumandcompression Longstandingacidrefluxcancausechronicirritationand PRACTICE CASES 243 98.0°F (36.7°C) Sit still with your back slightly hunched and head straight ahead; avoid turning your neck, and instead just Sit still with your back slightly hunched the examinee. move your eyes to make eye contact with when the examinee palpates your neck. Show pain when moving your neck and Pretend to have numbness in the back of your left forearm. 74/minute, regular 16/minute 115/75 mm Hg Examinee knocked on the door before entering. Examinee introducedself by name. 2. Perform a focused physical exam (do not perform rectal, genitourinary, or female breast exam). not perform rectal, genitourinary, 2. Perform a focused physical exam (do workup plan to the patient. 3. Explain your clinical impression and the patient note after leaving the room. 4. Write 1. Take a focused history. a focused 1. Take Challenging Questions to Ask I still be able to go?” “I’m supposed to visit my sister in Florida in 3 days. Will Sample Examinee Response like a broken bone have a serious injury, “Before I am comfortable with you traveling, I want to make sure you don’t tests first to make sure you’ll be safe.” or a nerve compression in your spine. I would like to see the results of some DOORWAY INFORMATION DOORWAY CASE 12 CASE Examinee Checklist Relationship Building the Doctor-Patient Entrance Checklist/SP Sheet Patient Description husband. Patient is a 67 yo F who lives with her Notes for the SP RR: HR: Examinee Tasks Carol Holland, a 67-year-old female, comes to the office complaining of neck pain. female, comes to the office Carol Holland, a 67-year-old Vital Signs BP: Temp: Opening Scenario PRACTICE CASES 244 Information Gathering Reflective Listening ✓ Examinee eliciteddataefficientlyandaccurately. Examinee summarizedtheSP’s concerns,oftenusingtheSP’s ownwords. Examinee askedtheSPtolisthis/herconcernsandlistenedresponsewithoutinterrupting. Examinee askedanopen-endedquestionandactivelylistenedtotheresponse. Examinee madeeyecontactwiththeSP. Examinee correctlyusedpatient’s name. Examinee identifiedhis/herroleorposition.

Question urn eiain Itakecalcium andvitaminDsupplements. Iamup to dateonmammogramsandhadanormalcolonoscopy None. None. Current medications Health maintenance Past surgicalhistory No. Past medicalhistory I’velostabout10poundsinthepast6months,andmyappetitehas nausea, vomiting No. No. Headaches, dizziness,photophobia, Ihavethrownmyneckoutbefore,butnotlikethis. Fever, nightsweats,weightloss Noweakness, butmyleftarmtingles. Sharp. Trouble breathing Turning myheadineitherdirection. Holdingmyheadstill. History ofneckpain/trauma Itradiatesdownmyleftarm. Thewholeneck,butworseontheleft. Recent heavylifting Recent trauma Weakness/numbness Ithasstayedthesame. 2/10atrest,8/10withmotion. Exacerbating factors Two daysago. Alleviating factors Quality Radiation Location Paininmyneck. Severity onascale SomeonecalledmynameandIturnedheadtotheleftlook. Progression Associated/precipitating events Onset Chief complaint Since thenithurtstomove. screening. last year. Iwasfoundtohaveosteopeniaatmylastosteoporosis No. decreased. Patient Response PRACTICE CASES 245 Maneuver Patient Response Patient Spurling’s test Spurling’s Follow-up tests. Neurologic exam signs and Brudzinski’s sensory exam, Kernig’s DTRs, Motor, Neck examExtremities sign, Inspection, palpation, stiffness, range of motion, Lhermitte’s Inspection, palpation of peripheral pulses, range of motion Alcohol useIllicit drug useTobaccoDrug allergies Just a glass of wine with dinner on weekends. Never. None. Never. Family historyOccupation sick contact history, Travel No. osteoporosis, and my father had a heart attack at 68. My mother had Retired magazine editor. Exam Component Question

Examinee asked if the SP had any other questions or concerns. Examinee discussed initial diagnostic impressions. Examinee discussed initial management plans: Examinee washed his/her hands. exam. Examinee asked permission to start the Examinee used respectful draping. Examinee did not repeat painful maneuvers. Examinee recognized the SP’s emotions and responded with PEARLS. Examinee recognized the SP’s ✓ ✓ of low bone density, I want to make sure your symptoms weren’t caused by a fracture. And although it’s unlikely, certain unlikely, caused by a fracture. And although it’s I want to make sure your symptoms weren’t of low bone density, I run some tests to rule out this possibility. cancers may spread to the neck and spine and cause similar symptoms. I want to for me? would like to start by getting an x-ray of your neck. Do you have any other questions Sample Closure nerve in your neck. Since you have a history Mrs. Holland, given your symptoms, I am concerned that you may have a pinched Closure Connecting with the Patient Connecting with Physical Examination PRACTICE CASES 246 Physical Examination History USMLE STEP2CS Patient Note PRACTICE CASES 247 Patient Note Patient Diagnosis #3 History Finding(s): Physical Exam Finding(s): Diagnosis #2 History Finding(s): Physical Exam Finding(s): Diagnosis #1 History Finding(s): Physical Exam Finding(s): USMLE STEP 2 CS STEP USMLE Diagnostic Workup Differential Diagnosis Differential USMLE STEP 2 CS Patient Note

History HPI: 67 yo F with 2 days of neck pain and left upper extremity numbness. Started after quick rotation to the left. Sharp pain 2/10 at rest, 8/10 with motion. Associated left arm numbness. Denies weakness. 10-lb weight loss in past 6 months attributed to poor appetite. No recent trauma or heavy lifting. No dyspnea, fevers, night sweats. Screenings up to date. ROS: Negative except as above. Allergies: NKDA. Medications: Calcium and vitamin D supplements. PMH: Osteopenia on last DEXA. PSH: None. SH: Social alcohol use, no tobacco or drugs. Retired magazine editor. FH: Mother with osteoporosis, father with MI at 68.

Physical Examination Patient sitting rigid and still, avoiding moving neck. VS: WNL. Neck: No scars or deformities, limited ROM 2/2 pain. Tenderness to palpation on cervical spinous processes. Lhermitte and Spurling tests. Extremities: No scars or deformities, brachial and radial pulses full. Full range of motion. Neuro: Motor: Strength 5/5 throughout upper extremities. DTRs: 2+ symmetric, Babinski bilaterally. Sensation: Loss of pinprick sensation noted on dorsum of left hand and posterior left arm and forearm; all other sensation normal.

Differential Diagnosis Diagnosis #1: Disk herniation History Finding(s) Physical Exam Finding(s) Neck pain that increases with movement Loss of pinprick sensation noted on dorsum of left hand and posterior left arm and forearm Radiculopathy (left arm numbness) PRACTICE CASES

248 USMLE STEP 2 CS Patient Note

Diagnosis #2: Cervical fracture History Finding(s) Physical Exam Finding(s) Rapid rotation of neck preceded pain Pain increases with movement Osteopenia on last DEXA

Diagnosis #3: Neck muscle strain History Finding(s) Physical Exam Finding(s) Rapid rotation of neck preceded pain

Diagnostic Workup

XR—C-spine MRI—C-spine Nerve conduction studies PRACTICE CASES

249 PRACTICE CASES 250 Diagnostic Workup Additional Differential Diagnoses Patient NoteDifferential Diagnoses CASE DISCUSSION Serum andurineprotein electrophoresis: underlying multiplemyeloma. CBC, calcium,BUN/Cr: necessarily sensitiveforcervicalpathology. a conductionissueintheperipheralnerve.Althoughtheyarespecific,nervestudiesnot Nerve conductionstudies: pathology. Becauseofitshighsensitivity, MRImaydetectclinicallyinsignificantabnormalities. withwhichtodiagnosedisk,spine,andspinalcord of plainfilmfindings.MRIisthemostsensitivemethod MRI—C-spine: narrowing orfractures. XR—C-spine: to theconstitutionalsymptomstypicallyfoundinmalignancy. and women.Associatedfindingsmayincludesymptomsofanemia,renalfailure,hypercalcemiainaddition Multiple myeloma: for metastaticdisease. compression. Apossiblespinallesioninconjunctionwithweightlossanolderwomanshouldraiseconcern Metastatic cancer: Cervical spondylosis: and fracture—compressionofthenerves. Osteoarthritis: radiculopathy suggeststhatthisismorethanasimplestrain. Neck musclestrain: is criticaltoincludethisinthedifferentialgivenpatient’s historyofosteopenia. diaphragm iftheyinterruptthephrenicnerve.Theexamwouldpresumablyshowtendernesstopalpation,butit Cervical fracture: nerve compressionsuggestsradiculopathycausedbydiskherniation. Disk herniation: Thefirsttesttoorderforpainthatraisesconcernfractureorradiculopathy. Checkforspace MRIisindicatedforpatientswhohaveneckpainwithneurologicsignsorsymptomsregardless Degenerativediseaseofthespinecouldcausefindingsseenbysameroutesasherniation Aswithotherareasofthespine,painatsitecompressionadditionsigns Cervicalfracturesaredangerous,acutefindingsthatcancompromiseinnervationtothe Breastandlungcancers,amongothers,canmetastasizetothebonecausecord Althoughararermalignancy, multiplemyelomaisacauseofspinallesionsinbothmen Manypeopleexperienceneckstrainscausedbyquickturningofthehead.Thepatient’s Aspondylosiswouldbecausedbythesamechannelsasdegenerativediskdisease. To detectanemia,hypercalcemia,andrenalfailure,allof whichmaybecluesto Nervestimulationwilldetermineifthepatient’s lossofsensationis dueto To detectamonoclonalparaproteininmyeloma. PRACTICE CASES 251 98.5°F (36.9°C) Sit up on the bed. abdomen that is exacerbated during inspiration. Show pain on palpation of the right upper Exhibit epigastric tenderness on palpation. If ultrasound is mentioned by the examinee, ask, “What does ‘ultrasound’ mean?” 76/minute, regular 16/minute 135/70 mm Hg Examinee knocked on the door before entering. Examinee introducedself by name. 2. Perform a focused physical exam (do not perform rectal, genitourinary, or female breast exam). not perform rectal, genitourinary, 2. Perform a focused physical exam (do workup plan to the patient. 3. Explain your clinical impression and the patient note after leaving the room. 4. Write 1. Take a focused history. a focused 1. Take Challenging Questions to Ask Could I have it too?” “My father had pancreatic cancer. Sample Examinee Response Regardless, some routine blood and as your symptoms are very unusual for pancreatic cancer. highly unlikely, “It’s x-ray tests should help us exclude that as a possibility.” DOORWAY INFORMATION DOORWAY CASE 13 CASE Examinee Checklist Relationship Building the Doctor-Patient Entrance Checklist/SP Sheet Patient Description married with 4 children. Patient is a 48 yo F, Notes for the SP RR: HR: Examinee Tasks Sharon Smith, a 48-year-old female, comes to the clinic complaining of abdominal pain. female, comes to the clinic Sharon Smith, a 48-year-old Vital Signs BP: Temp: Opening Scenario PRACTICE CASES 252 Information Gathering Reflective Listening ✓ Examinee eliciteddataefficientlyandaccurately. Examinee summarizedtheSP’s concerns,oftenusingtheSP’s ownwords. Examinee askedtheSPtolisthis/herconcernsandlistenedresponsewithoutinterrupting. Examinee askedanopen-endedquestionandactivelylistenedtotheresponse. Examinee madeeyecontactwiththeSP. Examinee correctlyusedpatient’s name. Examinee identifiedhis/herroleorposition.

Question atmdclhsoyIhadaurinary tractinfection1yearago,treatedwithamoxicillin, Maalox, ibuprofen(2pills2–3timesadayifasked). No. No. No. Past medicalhistory No. No. Current medications Change instoolcolor Itwas a sour, yellowishfluid. Appetite changes SometimesI feelnauseatedwhenIaminpain.Yesterday Ivomited Weight changes Diarrhea/constipation invomitus Blood Description ofvomitus No. Burning. Well, usuallythepainwilldecreaseorstopcompletely whenIeat, Heavymeals andhunger. Nausea/vomiting antacids,andmilk. Food, No. ofsimilarpain Previous episodes Itishere(pointstotheepigastrium). Heavy, fattymeals,likepizza. topain Relationship offood Types thatexacerbatepain offood Itisgettingworse. WhenIhave thepain,itis7/10,andthencangodownto0. Exacerbating factors Atleastonceeveryday. Alleviating factors Two weeksago. Quality Radiation Well, Idon’t havethepainalltime.Itcomesandgoes. Location Abdominalpain. Severity onascale Progression Frequency Constant/intermittent Onset Chief complaint Patient Response and arthritisinbothknees,for whichItakeibuprofen. for thefirsttime. but itcomesbackafter2–3hours. PRACTICE CASES 253 bly Maneuver percussion Patient Response Patient healthy. Follow-up tests: Examinee mentioned the need for a rectal exam. Pulmonary examAbdominal exam Auscultation sign), Inspection, auscultation, palpation (including Murphy’s CV exam Auscultation Drug allergies No. OccupationAlcohol useIllicit drug useTobaccoSexual activity Housewife. No. No. my husband (laughs). With No. Past surgical historyPast surgical Family history I had 2 C-sections. is alive and My mother at 55 of pancreatic cancer. My father died Exam Component Question

Examinee discussed initial diagnostic impressions. Examinee discussed initial management plans: Examinee asked if the SP had any other questions or concerns. Examinee did not repeat painful maneuvers. Examinee washed his/her hands. exam. Examinee asked permission to start the Examinee used respectful draping. Examinee recognized the SP’s emotions and responded with PEARLS. Examinee recognized the SP’s ✓ ✓ the diagnosis, we will be able to treat your condition and help alleviate your pain. Do you have any questions for me? the diagnosis, we will be able to treat your condition and help alleviate your pain. Sample Closure you have described. Pain of this type is most Mrs. Smith, there are a number of disorders that can cause pain similar to what will have to run some tests to confirm the diagnosis and an abdominal infection, or a gallstone. We commonly due to an ulcer, of your abdomen, blood tests, and possi to rule out more serious illness. These tests will include a rectal exam, an ultrasound made which examines your stomach by means of a tiny camera passed through your mouth. Once we have an upper endoscopy, Closure Physical Examination Connecting with the Patient PRACTICE CASES 254 Physical Examination History USMLE STEP2CS Patient Note PRACTICE CASES 255 Patient Note Patient Diagnosis #3 History Finding(s): Physical Exam Finding(s): Diagnosis #2 History Finding(s): Physical Exam Finding(s): Diagnosis #1 History Finding(s): Physical Exam Finding(s): USMLE STEP 2 CS STEP USMLE Diagnostic Workup Differential Diagnosis Differential USMLE STEP 2 CS Patient Note

History HPI: 48 yo F c/o intermittent, burning, nonradiating epigastric pain that started for the first time 2 weeks ago. The pain occurs at least once a day, usually 2–3 hours after meals. It is exacerbated by hunger and heavy, fatty foods and is alleviated by milk, antacids, and other food. It reaches 7/10 in severity and then diminishes to 0/10. It is sometimes accompanied by nausea. The patient vomited once yesterday: a sour, yellowish, nonbloody fluid. No diarrhea or constipation. No changes in weight or appetite. No changes in the color of the stool. ROS: Negative except as above. Allergies: NKDA. Medications: Maalox, ibuprofen. PMH: Arthritis in the knees, treated with ibuprofen. UTI last year, treated with amoxicillin. PSH: 2 C-sections. SH: No smoking, no EtOH, no illicit drugs. Sexually active with husband only. FH: Father died of pancreatic cancer at age 55.

Physical Examination Patient is in no acute distress. VS: WNL. Chest: No tenderness, clear breath sounds bilaterally. Heart: RRR; normal S1/S2; no murmurs, rubs, or gallops. Abdomen: Soft, nondistended, C-section scar, epigastric tenderness without rebound, Murphy’s sign, BS, no hepatosplenomegaly.

Differential Diagnosis Diagnosis #1: Cholecystitis History Finding(s): Physical Exam Finding(s): Pain is exacerbated by heavy, fatty foods Epigastric tenderness Associated with nausea and vomiting Positive Murphy’s sign Female gender, age in 40s

Diagnosis #2: Peptic ulcer disease History Finding(s): Physical Exam Finding(s): History of NSAID use Epigastric tenderness Epigastric pain 2−3 hours after meals Pain is exacerbated by hunger and fatty foods and is relieved by antacids PRACTICE CASES

256 USMLE STEP 2 CS Patient Note

Diagnosis #3: Gastritis History Finding(s): Physical Exam Finding(s): History of NSAID use Epigastric tenderness Epigastric pain associated with food Nausea and vomiting

Diagnostic Workup

Rectal exam, stool for occult blood U/S—abdomen Upper endoscopy H pylori antibody testing PRACTICE CASES

257 PRACTICE CASES 258 Diagnostic Workup Additional Differential Diagnoses etiologies: Although thecausesofabdominalpainaremany, this presentation shouldpromptyoutoponderthecommon Patient NoteDifferential Diagnoses CASE DISCUSSION required forgastriccancerdiagnosisandissometimesnecessary forthediagnosisof Upper endoscopy: Murphy’s sign). acute cholecystitis(itmayshowstones,pericholecysticfluid, athickenedgallbladderwall,andsonographic U/S—abdomen: or othercauses. Rectal exam,stoolforoccultblood: ischemia, andextra-abdominalcauses. Other etiologies: rare disease. mass (Virchow’s itshouldbenotedthatsignsandsymptomsareminimaluntillateinthecourseofthis node), Gastric cancer: and involuntaryguarding. Perforated ulcer: evaluation, noobviousorganicetiologyisdiscovered. Functional ornonulcerdyspepsia: more likelysignalsthepresenceofanulcer, truedifferentiationwould bestbemadeonupperendoscopy. the painassociatedwithgastritisistypicallymilderthanthatofpepticulcerdisease.Althoughepigastric Gastritis: to performit. easily causediscomfortinanypatientwithupperabdominalpainbecauseofthedeeppalpationthatisrequired peptic ulcers.AlthoughthepositiveMurphy’s signismoresuggestiveofcholecystitis,themaneuveritselfcould gastric ulcers).Inaddition,theabdominalexamrevealsepigastricpain,classiclocationforpainrelatedto from are consistentwiththisdiagnosis(althoughtheclinicalhistorycannotaccuratelydistinguishduodenal Peptic ulcerdisease: is sensitiveforcholecystitis,andthelocationofpainclassicallyRUQ. colic,” representingtransientobstructionofthecysticduct,usuallyduetogallstones.ThepositiveMurphy’s sign unremitting andisnotalleviatedbymilkorantacids.Thepatient’s intermittentpainmaybedueto“biliary and thepatient’s ageandgender(“femaleforty”).However, thepaininacutecholecystitisisusually Cholecystitis: Gastritisisacommoncauseofepigastricpain,nausea,andvomitinginpatientstakingNSAIDs,but Severalfeaturessuggestthisdiagnosis,includingpainfollowingfattymeals,nauseaandvomiting, Althoughthispatientdoesnothaveearlysatiety, anorexia,weightloss,oraleftsupraclavicular Aquick,inexpensiveimagingtechniquewithwhichto examine apatientwithsuspected Thesepatientsappeartoxicandhaveseverediffuseabdominalpainwithreboundtenderness Lesslikelypossibilitiesincludepancreatitis,atypicalGERD,choledocholithiasis,mesenteric Pepticulcer, gastritis,andgastriccancerhavelesions thatcanbevisualized(biopsyis The history of NSAID use and burning epigastric pain alleviated by antacids and food ThehistoryofNSAIDuseandburningepigastricpainalleviatedbyantacidsfood Thisisthemostcommoncauseofchronicdyspepsia.Afterthorough May document occult blood lossduetopepticulcer, Maydocumentoccultblood gastritis,cancer, H pylori ). PRACTICE CASES 259 eradication in peptic ulcer eradication are adequate for diagnosis but not to for diagnosis but are adequate H pylori H pylori To look for evidence of hepatocellular injury, biliary of hepatocellular injury, look for evidence To Serologic tests for antibodies Serologic to Uses scintigraphy with technetium-99m DISIDA (a bilirubin analog) to diagnose with technetium-99m DISIDA (a bilirubin Uses scintigraphy testing: H pylori obstruction, or pancreatitis. scan: HIDA (hepatobiliary) ordered if of the cystic duct and is usually HIDA can reveal obstruction acute and chronic cholecystitis. a diagnosis. ultrasound fails to establish Noninvasive cure, as antibodydocument exposure, not necessarily (indicating detectable after treatment levels often remain means of confirming breath test is a useful The urease active infection). disease. phosphatase, lipase: AST/ALT/bilirubin/alkaline PRACTICE CASES 260 Entrance Building theDoctor-Patient Relationship Examinee Checklist pain sothatIcanrecommendthebestmedicine.” “Yes, wehavemanyoptionsformedicinestorelieveyourpain,butfirstIneedlearnasmuchcanabout Sample ExamineeResponse “Do youhaveanythingthatwillmakemefeelbetter?Please,doctor, Iaminpain.” Challenging QuestionstoAsk Hold therightsideofyourheadduringencounterandlookasifyouareinseverepain. Notes fortheSP Patient isa35yoF, marriedwith3children. Patient Description Checklist/SP Sheet Examinee Tasks HR: RR: Temp: BP: Signs Vital Kelly Clark,a35-year-old female,comestotheEDcomplainingofheadache. Opening Scenario DOORWAY INFORMATION CASE 14 Examinee madeeyecontact with theSP. Examinee correctlyusedpatient’s name. Examinee identifiedhis/herroleorposition. selfbyname. Examinee introduced Examinee knockedonthedoorbeforeentering. 4. Write thepatientnoteafterleavingroom. 3. Explainyourclinicalimpressionandworkupplantothepatient. 2. Performafocusedphysicalexam(donotperformrectal,genitourinary, orfemalebreastexam). 1. Take afocusedhistory. 135/80mmHg 16/minute 76/minute,regular 98.6°F(37°C) PRACTICE CASES 261 Patient Response nausea. No. having one now. The headache may come at any time. I’m for the first time. Joint pain/fatigueaches and pains treated with ibuprofen. Occasional Runny nose during the attackSimilar episodes before changesWeight/appetite No. was accompanied by in college I had a similar headache that Yes, No. Headache wakes you up from sleep changes/tears/red eyeVisual No. Weakness/numbnessSpeech difficulties No. No. No. Relationship with mensesAlleviating factorsExacerbating factorsNausea/vomiting No. Resting in a quiet, dark room; sleep, aspirin. Stress, light, and noise. I vomited Sometimes I feel nauseated when I am in pain. Yesterday QualityAura (warning that the headache is about to come) day/ (the same time every Timing morning/evening) Sharp and pounding. Severity on a scaleLocationDurationRadiation (changes its location) When I have the pain, it is 9/10 and prevents me from working. No. It is here (points to the right side of the head). hours. One or two OnsetConstant/intermittentFrequencyProgression all the time. It comes and goes. have the pain I don’t Well, weeks ago. Two At least once a day. (2–3 times a day). It is getting worse Chief complaint Headache. Question

Examinee elicited data efficiently and accurately. Examinee elicited data Examinee asked an open-ended question and actively listened to the response. listened to the question and actively asked an open-ended Examinee interrupting. response without and listened to the his/her concerns asked the SP to list Examinee own words. SP’s concerns, often using the the SP’s Examinee summarized ✓ Reflective Listening Reflective Information Gathering PRACTICE CASES 262 Physical Examination Connecting withthePatient ✓ ✓ Examinee didnotrepeatpainfulmaneuvers. Examinee usedrespectfuldraping. Examinee askedpermissiontostarttheexam. Examinee washedhis/herhands. Examinee recognizedtheSP’s emotionsandrespondedwithPEARLS.

ExamComponent Question erlgcea Cranialnerves, musclestrength,DTRs Auscultation Auscultation Inspection,palpation Palpation(head,facialsinuses,temporomandibular joints), Neurologic exam Pulmonary exam CV exam Neck exam HEENT No. No,Ihadatuballigationaftermythirdchild8yearsago. With myhusband. Ilivewithmyhusband and3children. No. No. No. Drug allergies Engineer. Use ofOCPs Sexual activity Myfatherdied atage65ofabraintumor. Mymotherisaliveand Social history Tobacco use Tubal ligation8yearsago. Illicit druguse Alcohol use ofsinusitis4monthsago,treatedwithamoxicillin(but Anepisode Occupation Ibuprofen. No. Family history Yes, Iamworkingonanewprojectthathavetofinishthismonth. Two weeksago. Past surgicalhistory Past medicalhistory Current medications Last menstrualperiod Head trauma Stress funduscopic exam;inspectednose,mouth,teeth,andthroat Maneuver has migraines. the painwasdifferentfromwhatIhavenow). were rejected,andIhavetostartalloveragain. Last monthwasadisaster. Iworkedhardonmydesigns,butthey Patient Response PRACTICE CASES 263 T Follow-up tests. Examinee discussed initial diagnostic impressions. discussed initial diagnostic Examinee plans: discussed initial management Examinee SP had any other questions or concerns. Examinee asked if the problems other than migraine. Do you have any questions for me? migraine. Do you have any questions for problems other than Sample Closure do is prescribe some headache, so the first thing I will as if your symptoms are due to a migraine Mrs. Clark, it sounds like to get a C I would going on, however, something else ensure that there isn’t alleviate your pain. To medications that will also show if you have cause of your headache. A may rule out a mass or vascular problem as the scan of your head to Closure PRACTICE CASES 264 Physical Examination History USMLE STEP2CS Patient Note PRACTICE CASES 265 Patient Note Patient Diagnosis #3 History Finding(s): Physical Exam Finding(s): Diagnosis #2 History Finding(s): Physical Exam Finding(s): Diagnosis #1 History Finding(s): Physical Exam Finding(s): USMLE STEP 2 CS STEP USMLE Diagnostic Workup Differential Diagnosis Differential USMLE STEP 2 CS Patient Note

History HPI: 35 yo F c/o daily headaches for 2 weeks. These headaches occur 2–3 times a day and last for 1–2 hours. The pain is sharp and pounding. The pain is located on the right hemisphere of the head, with no radiation or preceding aura. The pain reaches 9/10 in severity and prevents the patient from continuing her activities. Headaches are exacerbated by stress, light, and noise and are alleviated by resting in a dark room, sleeping, and taking aspirin. The pain is sometimes accompanied by nausea and vomiting. No changes in weight or appetite. ROS: Occasional aches and pains. Allergies: NKDA. Medications: Ibuprofen, aspirin. PMH: Headaches at age 20, accompanied by nausea. One episode of sinusitis 4 months ago, treated with amoxicillin. PSH: Tubal ligation 8 years ago. SH: No smoking, no EtOH, no illicit drugs. Patient is an engineer, lives with husband and 3 children, and is sexually active with husband only. FH: Father died of a brain tumor at age 65. Mother has migraines.

Physical Examination Patient is in severe pain. VS: WNL. HEENT: NC/AT, nontender to palpation, PERRLA, EOMI, no papilledema, no nasal congestion, no pharyngeal erythema or exudates, dentition good. Neck: Supple, no lymphadenopathy. Chest: Clear breath sounds bilaterally. Heart: RRR; normal S1/S2; no murmurs, rubs, or gallops. Neuro: Mental status: Alert and oriented × 3, good concentration. Cranial nerves: 2–12 grossly intact. Motor: Strength 5/5 throughout. DTRs: 2+ intact, symmetric.

Differential Diagnosis Diagnosis #1: Migraine History Finding(s): Physical Exam Finding(s): Unilateral, sharp headaches Severe pain with lack of neurologic findings Associated with nausea and vomiting Photophobia PRACTICE CASES

266 USMLE STEP 2 CS Patient Note

Diagnosis #2: Tension headache History Finding(s): Physical Exam Finding(s): Chronic headaches Severe pain with lack of neurologic findings Associated with stress at work Improve with sleep

Diagnosis #3: Intracranial mass lesion History Finding(s): Physical Exam Finding(s): Headaches associated with nausea and vomiting Family history of brain tumor

Diagnostic Workup

CBC CT—head or MRI—brain LP CT—sinus PRACTICE CASES

267 PRACTICE CASES 268 Diagnostic Workup Additional Differential Diagnoses should stillbeconsidered: Headaches withoutneurologicfindingsonexamarecommonandhaveroutinecauses,butlesspathology Patient NoteDifferential Diagnoses CASE DISCUSSION CT—sinus: can beseeninsubarachnoidhemorrhage(performifsuspicion ishighdespiteanegativeCTscan). LP: exam. MRIprovidesgreateranatomicdetail,butCTispreferred toruleoutacutebleeds. reserved forpatientswithacutesevereheadache,chronic unexplainedheadache,orabnormalitiesonneurologic CT—head orMRI—brain: thrombocytosis mayalsobeseenintemporalarteritis. CBC: are nosignsorsymptomsofsinusrespiratoryinfectioninthiscase. Sinusitis: termed and oftenawakenpatientsatnight.However, thisrarelyoccursinwomen(asimilarentityseenis rhinorrhea, lacrimation,rednessoftheeye,and/orHorner’s ofdailypainoccurinclusters syndrome.Episodes Cluster headache: first fewdaysaftertheonsetofillness. diplopia andothervisualsymptoms.Thephysicalexamshouldrevealpapilledemabutmaybenormalduringthe Pseudotumor cerebri: depression. Thepatientalsoreportsstressandrejectionatwork. Depression: out thisdiagnosis. familial basis.Thepatient’s lackofweightlossorneurologicfindingsonexamcastsdoubtbutdoesnotrule headache. Headacheisnonspecificandmaymimicfeaturesofmigraine.Certainbraintumorshavea Intracranial masslesion: deprivation. hours todaysandworsensasthedayprogresses.Tension headachesareoftenassociatedwithstressandsleep Tension headache: vomiting, andphotophobia.Apositivefamilyhistorymakesthediagnosisevenmorelikely. common inwomenandtypicallyappearasaunilateralheadache.Theyareoftenassociatedwithaura,nausea, Migraine: To lookforelevatedopeningpressureinpseudotumor. CSFisotherwisenormal.RBCsandxanthochromia To lookforleukocytosis,anonspecificsignofinfectionorinflammation.Mildnormocyticanemiaand chronic paroxysmalhemicrania Thisisararecauseofheadache.Althoughthepatienthadsinusinfectionseveralmonthsago,there Despitelackinganaura,thepatient’s presentationisclassicforthisdiagnosis.Migrainesaremore To lookforsinusitis. Headachesmaybeworseonarisinginthemorningandareassociatedwithothersymptomsof Thisinvolvesunilateralperiorbitalpain,oftenaccompaniedbyipsilateralnasalcongestion, Thisisoftenassociatedwithstressbutusuallybilateralandsqueezing.Itlastsfrom Inpseudotumorcerebri,headachesmaybefocalbutareusuallyaccompaniedby One-thirdofpatientswithbraintumorspresentaprimarycomplaint Headachesyndromesarelargelyclinicaldiagnoses.Neuroimaging isgenerally ). PRACTICE CASES 269 98.0°F (36.7°C) 65/minute, regular 13/minute 120/85 mm Hg Examine correctly used patient’s name. Examine correctly used patient’s Examinee made eye contact with the SP. Examinee knocked on the door before entering. Examinee introducedself by name. Examinee identified his/her role or position. 2. Perform a focused physical exam (do not perform rectal, genitourinary, or female breast exam). not perform rectal, genitourinary, 2. Perform a focused physical exam (do workup plan to the patient. 3. Explain your clinical impression and the patient note after leaving the room. 4. Write 1. Take a focused history. a focused 1. Take DOORWAY INFORMATION DOORWAY CASE 15 CASE Entrance “I doubt it. It would be extremely unusual at your age. I need to learn more by asking you about other symptoms “I doubt it. It would be extremely unusual at your age. I need to learn more periods.”and doing an exam. Then we can discuss possible reasons you are not having Examinee Checklist Relationship Building the Doctor-Patient None. Challenging Questions to Ask “Am I going through menopause?” Sample Examinee Response Checklist/SP Sheet Patient Description Patient is a 36 yo F. Notes for the SP RR: HR: Examinee Tasks Patricia Garrison, a 36-year-old female, comes to the office complaining of not having menstrual periods office complaining of not having menstrual female, comes to the recently. 36-year-old Patricia Garrison, a Vital Signs BP: Temp: Opening Scenario PRACTICE CASES 270 Information Gathering Reflective Listening ✓ Examinee eliciteddataefficientlyandaccurately. Examinee summarizedtheSP’s concerns,oftenusingtheSP’s ownwords. Examinee askedtheSPtolisthis/herconcernsandlistenedresponsewithoutinterrupting. Examinee askedanopen-endedquestionandactivelylistenedtotheresponse.

Question rgace Ihave1child;he is10yearsold. Thesamepills for8years. Onceaweekon averagewithmyhusband. No. No. No. Pregnancies Contraceptives Yes, justlast weekInoticedsomemilkydischargefrommyleft Sexual activity No. Abdominal pain Headache No. Visual changes No. No. Nipple discharge Urinary frequency No. ing asleep,earlywaking,snoring) Sleeping problems(fallingasleep,stay- No,I’vebeen avegetarianfor10years. appetite. Ihaveagood Vaginal dryness/itching No. Hot flashes No. Depression/anxiety/stress Fatigue Actually, Irecentlynoticedsomehaironmychinthathavebeen Fad dietorpills No. Ihavegained15poundsoverthepastyear.Appetite changes Change inbowelhabits Age14. Voice change Skin/hair changes Cold intolerance everymonthlastingfor4–5days,but Iusedtohave regularperiods Itwas2–3aday, flowisbecomingless,andIuseonly buttheblood Weight changes Ihaven’t in3months. hadaperiod Age atmenarche Pads/tampons changedaday Menstrual history Chief complaint 1 adaynow. weeks, lastingfor7days. over thepastyearIstartedhavingthemlessfrequently—every5–6 breast. No. plucking. Patient Response PRACTICE CASES 271 Maneuver Patient Response Patient age 55. Follow-up tests: Examinee mentioned the need for pelvic and breast exams. Neurologic exam extraocular movements, checked DTRs fields, Visual Neck examCV examPulmonary examExtremities Examined thyroid gland Auscultation Auscultation Inspection TobaccoExerciseDrug allergies No. No. I run 2 miles 3 times a week. Family historyOccupationAlcohol useIllicit drug use My father and mother are healthy; my mother began menopause at Nurse. None. Never. History of abnormal Pap smearsHistory of abnormal Current medicationsPast medical history No. Past surgical history None. None. None. Problems during pregnancy/deliveryProblems Miscarriages/abortions healthy. and my child is delivery, No, it was a normal Last Pap smear No. months ago. It was normal. Ten Exam Component Question

Examinee discussed initial management plans: Examinee asked if the SP had any other questions or concerns. Examinee discussed initial diagnostic impressions. Examinee asked permission to start the exam. Examinee asked permission to start the Examinee used respectful draping. Examinee did not repeat painful maneuvers. Examinee recognized the SP’s emotions and responded with PEARLS. Examinee recognized the SP’s Examinee washed his/her hands. ✓ ✓ Closure Connecting with the Patient Physical Examination PRACTICE CASES 272 and wewillgofromthere.Doyouhaveanyquestionsforme? are menopausalorhaveahormonalimbalance.Thiswillgiveusgoodstartinfiguringoutwhyyouhaven’t hadyourperiod, at yourage,butonrareoccasionsitmayoccur. Abloodtesttomeasureyourhormonelevelswillalsohelpusdetermineifyou exams, especiallysinceyouhavehadsomenippledischarge,andlookforanysignsofmenopause.Menopauseishighlyunlikely whether youarepregnant.We candothatwithasimpleurinetest.Theotherthingweneedtoisconductbreastandpelvic Mrs. Garrison,thereareafewreasonsyoumaynotbehavingregularperiods.Thefirstthingweneedtodoisdetermine Sample Closure

PRACTICE CASES 273 Patient Note Patient USMLE STEP 2 CS STEP USMLE Physical Examination History USMLE STEP 2 CS Patient Note

Differential Diagnosis Diagnosis #1 History Finding(s): Physical Exam Finding(s):

Diagnosis #2 History Finding(s): Physical Exam Finding(s):

Diagnosis #3 History Finding(s): Physical Exam Finding(s):

Diagnostic Workup PRACTICE CASES

274 USMLE STEP 2 CS Patient Note

History HPI: 36 yo F c/o amenorrhea for 3 months. She recently noticed some milky discharge from her left breast as well as abnormal facial hair but denies visual changes or headache. She also describes oligomenorrhea, hypomenorrhea, and a 15-lb weight gain over the past year but denies dry skin, cold intolerance, voice change, constipation, depression, fatigue, or sleep problems. She also denies hot flashes and vaginal dryness or itching. OB/GYN: Menarche at age 14. For the past year, menses have cycled every 5–6 weeks and lasted for 7 days, with decreased blood flow. Before that, menses cycled every 4 weeks. G1P1; 1 uncomplicated vaginal delivery 10 years ago. Last Pap smear 10 months ago; no history of abnormal Pap smears. Sexually active with husband once a week on average; uses OCPs for contraception. ROS: Negative except as above. Allergies: NKDA. Medications: None. PMH/PSH: None. SH: Denies tobacco, alcohol, or illicit drug use. Exercises regularly. Vegetarian; hasn’t changed her diet recently. FH: Mother had menopause at age 55.

Physical Examination Patient is in no acute distress. VS: WNL. HEENT: EOMI without diplopia or lid lag; visual fields full to confrontation. Neck: No thyromegaly. Chest: Clear breath sounds bilaterally. Heart: RRR; normal S1/S2; no murmurs, rubs, or gallops. Abdomen: Soft, nontender, nondistended, BS, no hepatosplenomegaly. Extremities: No edema, no tremor. Neuro: See HEENT. Normal DTRs in lower extremities bilaterally.

Differential Diagnosis Diagnosis #1: Pregnancy History Finding(s): Physical Exam Finding(s): Change in menstrual cycles Regular sexual activity Previous successful pregnancy PRACTICE CASES

275 USMLE STEP 2 CS Patient Note

Diagnosis #2: Hyperprolactinemia History Finding(s): Physical Exam Finding(s): Galactorrhea Oligomenorrhea

Diagnosis #3: Polycystic ovary syndrome History Finding(s): Physical Exam Finding(s): Weight gain Hirsutism Oligomenorrhea

Diagnostic Workup

Urine hCG Pelvic and breast exams Prolactin, TSH LH/FSH PRACTICE CASES

276 PRACTICE CASES 277 To check renal and hepatic To is also useful if hyperthyroidism (or 4 This and virilization, infertility, as hirsutism, obesity, manifests variably This refers to primary hypogonadism that occurs before age 40. Causes include This refers to primary hypogonadism Required to check for genital virilization (ie, clitoromegaly), uterine or adnexal Required to check for genital virilization (ie, clitoromegaly), uterine or adnexal This describes amenorrhea due to endometrial scarring, which can occur following This describes amenorrhea due to endometrial To screen for hyperandrogenism when amenorrhea is accompanied by hirsutism and screen for hyperandrogenism when To This from It may result galactorrhea, and infertility. causes menstrual cycle disturbances, Both hyper- and hypothyroidism can cause menstrual irregularities, although amenorrhea and hypothyroidism can cause menstrual irregularities, although Both hyper- To screen for hyperprolactinemia and thyroid disease. FT screen for hyperprolactinemia and To To look for endometrial adhesions that are diagnostic for Asherman’s syndrome. for Asherman’s look for endometrial adhesions that are diagnostic To Required to evaluate the pituitary region in patients suspected of having amenorrhea due to a mass Required to evaluate the pituitary region in patients suspected of having amenorrhea Although this patient’s symptoms suggest a hormonal cause of oligomenorrhea, any change in of oligomenorrhea, any change in symptoms suggest a hormonal cause Although this patient’s To rule out pregnancy. To PCOS is a clinical diagnosis; an increased LH/FSH ratio is often seen but is neither necessary nor PCOS is a clinical diagnosis; an increased LH/FSH ratio is often seen but is neither central hypothyroidism) is suspected. MRI—brain: effect (eg, prolactinoma). Hysteroscopy: anovulation (and often amenorrhea). An elevated FSH (> 40 mIU/mL) is diagnostic for premature ovarian failure. anovulation (and often amenorrhea). An elevated FSH (> 40 mIU/mL) is diagnostic phosphatase: AST/ALT/bilirubin/alkaline glucose, BUN/Cr, Electrolytes, function and to screen for evidence of hypercortisolism (eg, high sodium and low potassium). DHEAS: Testosterone, due to ovarian or adrenal tumors. virilization. Mild elevations are often due to PCOS, but high levels may be Prolactin, TSH: Prolactin, LH/FSH: increased levels of estrone (derived from obesity) are believed to sufficient to make the diagnosis. Physiologically, of the ovary then results insuppress pituitary FSH, leading to a relative increase in LH. Constant LH stimulation Urine hCG: exams: Pelvic and breast elicit breast discharge. enlargement, and estrogen effects (via inspection of vaginal mucosa) and to autoimmunity against the ovary, pelvic radiation therapy, chemotherapy, surgical bilateral oophorectomy, and bilateral oophorectomy, surgical chemotherapy, pelvic radiation therapy, autoimmunity against the ovary, libido, lack of menopausal symptoms (eg, fatigue, insomnia, headache, diminished familial factors. The patient’s diagnosis unlikely. depression, and hot flashes) makes this syndrome: Asherman’s effect is normal. uterine infections. The vaginal estrogen Thyroid disease: Thyroid Except for galactorrhea and weight gain, the patient does not have is more commonly due to hypothyroidism. other signs or symptoms of thyroid disease. ovarian failure: Premature have hyperprolactinemia. (PCOS): syndrome Polycystic ovary patients have amenorrhea (due to chronic anovulation). The patient’s glucose intolerance. About one-half of of recent weight gain suggest this diagnosis. oligomenorrhea and hirsutism in the context amenorrhea in women of childbearing age and should be ruled out during the initial evaluation. Menstruation ruled out during the initial evaluation. of childbearing age and should be amenorrhea in women cease completely during pregnancy. may not necessarily Hyperprolactinemia: renal failure, and cirrhosis, or it pituitary lesions, hypothyroidism, including pregnancy, a variety of conditions, will with secondary amenorrhea and galactorrhea of medications. Roughly 70% of women can be a side effect Pregnancy: common cause of secondary Pregnancy is the most warrants consideration of pregnancy. the menstrual cycle Diagnostic Workup Diagnoses Additional Differential CASE DISCUSSION CASE Diagnoses Note Differential Patient PRACTICE CASES 278 Entrance Building theDoctor-Patient Relationship Examinee Checklist discharge tothelabcheckforinfection.” try tolookforcluesbyaskingyoumorequestionsandexamining you,andwewilldefinitelysendasampleofthe “There aremanycausesofvaginaldischarge,onlysomewhichduetosexuallytransmittedinfections.Iwill Sample ExamineeResponse When askedaboutvaginaldischarge,ask,“DoyouthinkIhaveasexuallytransmitteddisease?” Challenging QuestionstoAsk None. Notes fortheSP Patient isa28yoF. Patient Description Checklist/SP Sheet Examinee Tasks HR: RR: Temp: BP: Signs Vital Stephanie McCall,a28-year-old female,comestotheofficecomplainingofpainduringsex. Opening Scenario DOORWAY INFORMATION CASE 16 Examinee madeeyecontact with theSP. Examinee correctlyusedpatient’s name. Examinee identifiedhis/her role orposition. selfbyname. Examinee introduced Examinee knockedonthedoorbeforeentering. 4. Write thepatientnoteafterleavingroom. 3. Explainyourclinicalimpressionandworkupplantothepatient. 2. Performafocusedphysicalexam(donotperformrectal,genitourinary, orfemalebreastexam). 1. Take afocusedhistory. 120/85mmHg 13/minute 65/minute,regular 98.0°F(36.7°C) PRACTICE CASES 279 Patient Response smells like fish. relationship with my ex-boyfriend for 5 years. usually talk about it, but I was raped in college, and that was I don’t when I contracted gonorrhea. inside with deep thrusting. Depression/anxietyHot flashes No. No. tional abuse History of vaginal infections or STDsLast Pap smear I had gonorrhea 10 years ago in college. History of abnormal Pap smears No. Six months ago; it was normal. Sexual desireConflicts with partnerFeeling safe at homeHistory of physical, sexual, or emo- are pretty close. No, we Good. I have my own apartment. Yes, Painful periodsPostcoital or intermenstrual bleedingSexual partner No. Contraception they have started to be painful over the past year. Yes, the past year; before that, I had a I have had the same boyfriend for I am using the patch. DouchingLast menstrual periodFrequency of menstrual periodsPads/tampons changed a day every month; lasts for 3 days. Regular, ago. weeks Two Three. No. Vaginal dischargeVaginal Color/amount/smellItching recently. Yes, have to wear a pad); it (I don’t White, small amount every day a little bit. Yes, OnsetDescribe painTimingLocation Aching and burning. Three months ago. time I try to have sex. It happens every In the vaginal area. It starts on the outside, and I feel it on the Chief complaint I have been experiencing pain during sex. Question

Examinee elicited data efficiently and accurately. Examinee elicited data Examinee asked an open-ended question and actively listened to the response. listened to the question and actively asked an open-ended Examinee interrupting. response without and listened to the his/her concerns asked the SP to list Examinee own words. SP’s concerns, often using the the SP’s Examinee summarized ✓ Reflective Listening Reflective Information Gathering PRACTICE CASES 280 Closure Physical Examination Connecting withthePatient ✓ ✓ Examinee askediftheSPhad anyotherquestionsorconcerns. Examinee discussedinitialmanagement plans: Examinee discussedinitialdiagnosticimpressions. Examinee didnotrepeatpainfulmaneuvers. Examinee usedrespectfuldraping. Examinee askedpermissiontostarttheexam. Examinee washedhis/herhands. Examinee recognizedtheSP’s emotionsandrespondedwithPEARLS.

ExamComponent Question boia xmAuscultation, palpation,percussion Auscultation Auscultation Abdominal exam Pulmonary exam CV exam Iswimandrun3timesaweek. No. No. Marijuanaincollege, butIdon’t useanythingnow. No(toall4). Acoupleofbeersontheweekends;sometimesaglasswine Drug allergies Editorforafashionmagazine. Exercise Bothparentsare healthy. Tobacco Illicit druguse None. CAGE questions None. Ihaveneverbeenpregnant. Alcohol use None. Occupation Family history No. Past surgicalhistory Past medicalhistory Current medications Pregnancies No. Urinary frequency/painwithurination No. Sleeping problems Vaginal drynessduringintercourse Follow-up tests:Examineementioned theneedforapelvicexam. romantic dinner. Patient Response Maneuver PRACTICE CASES o 281 Sample Closure Sample causes less common are other, there cervix. However, in the vagina or diagnosis is an infection your most likely Ms. McCall, I will als find under a microscope. a look at what I a pelvic exam and take diagnosis until I do make a I can’t of your problem. testing. Do you have any questions for me? and send it for gonorrhea and chlamydia take a cervical swab PRACTICE CASES 282 Physical Examination History USMLE STEP2CS Patient Note PRACTICE CASES 283 Patient Note Patient Diagnosis #3 History Finding(s): Physical Exam Finding(s): Diagnosis #2 History Finding(s): Physical Exam Finding(s): Diagnosis #1 History Finding(s): Physical Exam Finding(s): USMLE STEP 2 CS STEP USMLE Diagnostic Workup Differential Diagnosis Differential USMLE STEP 2 CS Patient Note

History HPI: 28 yo F c/o pain during intercourse for 3 months, located both superficially and with deep thrusting. She also noticed a scant white vaginal discharge with a fishy odor, accompanied by mild vaginal pruritus. She denies postcoital or intermenstrual vaginal bleeding. She is sexually active with her boyfriend (only) for the past year, and her sexual desire is normal. She feels safe at home and denies any conflicts with her partner. She also denies vaginal dryness, hot flashes, hirsutism, depression, fatigue, sleep problems, dysuria, and urinary frequency. OB/GYN: G0P0. Last menstrual period 2 weeks ago; has regular menses but started to be painful over the past year. No history of abnormal Pap smears; most recent was 6 months ago. Uses patch for contraception. ROS: Negative except as above. Allergies: NKDA. Medications: None. PMH: History of rape 10 years ago; subsequently contracted gonorrhea. PSH: None. SH: No tobacco. Drinks a couple of beers on the weekends, occasional wine, CAGE 0/4; used marijuana in college. Exercises regularly. FH: Noncontributory.

Physical Examination Patient is in no acute distress. VS: WNL. Chest: Clear breath sounds bilaterally. Heart: RRR; normal S1/S2; no murmurs, rubs, or gallops.

Differential Diagnosis Diagnosis #1: Vulvovaginitis History Finding(s): Physical Exam Finding(s): White vaginal discharge Fishy odor of discharge Vaginal pruritus

Diagnosis #2: Cervicitis History Finding(s): Physical Exam Finding(s): White vaginal discharge Dyspareunia Sexual activity without barrier contraception PRACTICE CASES

284 USMLE STEP 2 CS Patient Note

Diagnosis #3: Endometriosis History Finding(s): Physical Exam Finding(s): Dysmenorrhea Dyspareunia

Diagnostic Workup

Pelvic exam Wet mount, KOH prep, “whiff test” Cervical cultures (chlamydia and gonorrhea DNA probes) Laparoscopy PRACTICE CASES

285 PRACTICE CASES 286 Diagnostic Workup Additional Differential Diagnoses Patient NoteDifferential Diagnoses CASE DISCUSSION pathology. U/S—pelvis: organs frompriorinfections or surgeries. Laparoscopy: characterized bythepresenceofvesiclesorulcerson cervix). Cervical cultures: exposure ofthedischargetoadroppotassiumhydroxide ischaracteristicofbacterialvaginosis. following spores indicatescandidalinfection.Motileorganismsare seenintrichomonalinfection.A“fishy”odor epithelial cellscoveredwithbacteria(cluecells)suggests bacterialvaginosis,andthepresenceofhyphae Wet mount,KOHprep, “whiff test”: exams. complete examincludesexternalgenitalinspectionandpalpation,aspeculumexam,bimanualrectal Pelvic exam: patient wasrapedinthepast,shedoesnotdescribemusclecontractionscharacteristicofvaginismus. pain, orsexualpsychologicaltrauma.Themusclecontractionsgenerallyprecludepenetration.Althoughthis Vaginismus: dysmenorrhea. However, pelvictumorsarenotassociatedwithvaginaldischargeandpruritus. Pelvic tumor: screening isrequired,asvictimsmaynotdisclosethishistoryinitially. Domestic violence: vestibulitis. making itadiagnosisofexclusion.However, termed vulvarerythemacanbe seeninasubsetofvulvodynia Pain maybeconstantorintermittent,focaldiffuse,andsuperficialdeep.Physicalfindingsareoftenabsent, Vulvodynia: her atriskforpelvicscarringandsubsequentdyspareunia(duetoimpairedmobilityoftheorgans). Pelvic inflammatorydisease(PID): so, couldalsocausedyspareuniawithdeepthrusting. scarring inthelowerpelvis.Endometriosismayaccountforpatient’s dysmenorrheaoverthepastyearand,if Endometriosis: could stillcontractanSTDifherpartnerweretoacquireone. of thecervix.Althoughpatientisinamonogamousrelationship,shedoesnotusebarriercontraceptionand Cervicitis: andpruritusmakesthisthemostlikelydiagnosis. accompanied byafishyodor Gardnerella Vulvovaginitis: ), allergicorcontactreactions,andfrictionfromintercourse.Thepresenceofavaginaldischarge Thepresenceofvaginaldischargeandpainwithdeepthrustingsuggestsinfectionorinflammation This is the leading cause of dyspareunia in premenopausal women but is not well understood. Thisistheleadingcauseofdyspareuniainpremenopausalwomenbutnotwellunderstood. Thisdescribessevereinvoluntaryspasmofmusclesaroundtheintroitusandoftenresultsfromfear, Canbeusedtoassessthesize andpositioningofpelvicorganstohelpruleoutmasses orother To thepainordiscomfortandtodetermineifanypathologyispresent.A localizeandreproduce Thiscouldaccountforthepatient’s painwithdeepthrustingandpossibly forherhistoryof Thegoldstandardforconfirming aclinicaldiagnosisofendometriosisorscarring the pelvic Thisdescribesinfectionorinflammationofthevagina.Etiologiesincludepathogens(eg, Thisdescribesabnormalectopicendometrialtissue,whichcancauseinflammationand To diagnosechlamydia,gonorrhea,andoccasionallyHSV infection(thelatteris Physiciansmustscreenforthisinanywomanpresentingwithdyspareunia.Serial Thepatient’s historyofgonorrheainfection(ifitcausedPID)also puts Thevaginaldischargeisexaminedmicroscopically. Thepresenceof vulvar PRACTICE CASES 287 98.4°F (36.9°C) Ask the examinee to speak up if he or she did not speak in a loud and clear manner. Ask the examinee to speak up if he or in both ears. Pretend that you have difficulty hearing test (ie, show that your hearing have no lateralization on the Weber On physical exam, demonstrate that you is equal in both ears). Pretend that you cannot hear when spoken to from behind. 84/minute, regular 18/minute 132/68 mm Hg Examinee knocked on the door before entering. Examinee introducedself by name. 2. Perform a focused physical exam (do not perform rectal, genitourinary, or female breast exam). not perform rectal, genitourinary, 2. Perform a focused physical exam (do workup plan to the patient. 3. Explain your clinical impression and the patient note after leaving the room. 4. Write 1. Take a focused history. a focused 1. Take more tests to figure out the cause of the problem, whether it is going to get worse, and whether we can halt its more tests to figure out the cause of the problem, whether it is going to get taking aspirin.” progression or improve your hearing. In the meantime, I would like you to stop Challenging Questions to Ask “Do you think I am going deaf?” Sample Examinee Response need to perform the results of my exam show that you have some kind of hearing deficit. We symptoms and “Your DOORWAY INFORMATION DOORWAY CASE 17 CASE Entrance Examinee Checklist Relationship Building the Doctor-Patient Checklist/SP Sheet Patient Description Patient is a 75 yo M. Notes for the SP RR: HR: Examinee Tasks Paul Stout, a 75-year-old male, comes to the office complaining of hearing loss. male, comes to the office complaining Paul Stout, a 75-year-old Vital Signs BP: Temp: Opening Scenario PRACTICE CASES 288 Information Gathering Reflective Listening ✓ Examinee eliciteddataefficientlyandaccurately. Examinee summarizedtheSP’s concerns,oftenusingtheSP’s ownwords. Examinee askedtheSPtolisthis/herconcernsandlistenedresponsewithoutinterrupting. Examinee askedanopen-endedquestionandactivelylistenedtotheresponse. Examinee madeeyecontactwiththeSP. Examinee correctlyusedpatient’s name. Examinee identifiedhis/herroleorposition.

Question edce Rarely. No. Yes. No. I wasinthearmy, anditwasalwaysloud. Sometimes, inbothears. Insertion offoreignbody Ihadaurinary infectionaboutayearago.Mydoctorgavemean Headaches No. Exposure toloudnoises No. No. Trauma totheears Ringing intheears No. Recent infections Feeling ofimbalance No. Isawmydoctor amonthago,andhecleanedoutsomewaxfrommy Sensation ofroomspinningaroundyou Ear discharge Ear pain Did thathelpyou? ItseemslikeI’mhavingtroublewithbothears,butnotsure. Treatments tried Thishasbeengoingonforayear. Ithasbeengettingworse. ing speech? Yes. Do youhaveanyproblemsunderstand- Yes, especiallyincrowdedplacesorwhenIwatchtelevision. Can youlocatethesourceofsound? MywifehastoldmethatIcan’t hearwell,andlatelyIhavenoticed Do wordssoundjumbledordistorted? anything specific? Is hearinglostforallsoundsor Ican’t hearaswellIusedto. Location Progression Onset Description Chief complaint that Ihavebeenreadinglips. antibiotic, butIdon’t remember itsname. was before. ears. Thatseemedtohelpforawhile,butnowit’s justasbadit No. Nothing specific. Patient Response PRACTICE CASES 289 Maneuver Patient Response Patient nose, mouth, and throat; funduscopic exam and otoscopy; assessed tests and whisper test hearing with Rinne and Weber nose, heel to shin No. heart. daily to protect my controlled. Follow-up tests. Neurologic exam reflexes, cerebellar—finger to Cranial nerves, sensation, motor, HEENT examCV/pulmonary exam hearing by speaking with back turned; inspected sinuses, Tested Auscultation TobaccoSexual activityDrug allergies Only with my wife. Never. I develop a rash when I take penicillin. Family history of hearing lossFamily history of hearing OccupationAlcohol use No. Illicit drug use Retired military veteran. Never. Never. Current medicationsPast medical historyPast surgical history past 25 years, I have also taken aspirin Hydrochlorothiazide. For the well and it’s Hypertension. I take my blood pressure every day, None. Nausea/vomitingsensation, problems, loss of Neurologic or tingling muscle weakness, numbness anywhere in the body No. Exam Component Question

Examinee discussed initial management plans: Examinee asked if the SP had any other questions or concerns. Examinee discussed initial diagnostic impressions. Examinee asked permission to start the exam. Examinee asked permission to start the Examinee used respectful draping. Examinee did not repeat painful maneuvers. Examinee recognized the SP’s emotions and responded with PEARLS. Examinee recognized the SP’s Examinee washed his/her hands. ✓ ✓ Closure Connecting with the Patient Physical Examination PRACTICE CASES 290 your hearingloss.Iwillreferyoutoanaudiometrist,whoassessforaaid.Dohaveanyquestionsme to runseveraltests,includingsomebloodtests.Iwouldalsolikeyoustoptakingaspirin,becausethismaybecontributin Mr. Stout,Iknowthatyouareconcernedaboutyourproblem.canconfirmdohavesomehearingloss.wouldlike Sample Closure g to ? PRACTICE CASES 291 Patient Note Patient USMLE STEP 2 CS STEP USMLE Physical Examination History USMLE STEP 2 CS Patient Note

Differential Diagnosis Diagnosis #1 History Finding(s): Physical Exam Finding(s):

Diagnosis #2 History Finding(s): Physical Exam Finding(s):

Diagnosis #3 History Finding(s): Physical Exam Finding(s):

Diagnostic Workup PRACTICE CASES

292 USMLE STEP 2 CS Patient Note

History HPI: 75 yo M c/o bilateral hearing loss for all sounds that started 1 year ago and is progressively worsening. He had cerumen removal 1 month ago with moderate improvement. He reports occasional tinnitus and rare headaches. He notes that words sound jumbled in crowded places or when he is watching TV. He denies inserting any foreign body into the ear canal. No ear pain, no ear discharge, no vertigo, no loss of balance. No history of trauma to the ears; no difficulty comprehending or locating the source of sounds. ROS: Negative. Allergies: Penicillin, causes rash. Medications: HCTZ, aspirin (for 25 years). PMH: Hypertension. UTI 1 year ago, treated with antibiotics. PSH: None. SH: No smoking, no EtOH, no illicit drugs. Retired veteran. Sexually active with wife only. FH: No history of hearing loss.

Physical Examination Patient is in no acute distress. VS: WNL. HEENT: NC/AT, PERRLA, EOMI, no nystagmus, no papilledema, no cerumen. TMs with light reflex, no stigmata of infection, no redness to ear canal, no tenderness of auricle or periauricle, no lymphadenopathy, oropharynx normal. Weber test without lateralization; Rinne test (revealed air conduction > bone conduction). Chest: Clear breath sounds bilaterally. Heart: RRR; S1/S2; no murmurs, rubs, or gallops. Neuro: Cranial nerves: 2–12 grossly intact except for decreased hearing. Motor: Strength 5/5 throughout. DTRs: 2+ throughout. Sensation: Intact. Gait: Normal; no past pointing and heel to shin.

Differential Diagnosis Diagnosis #1: Presbycusis History Finding(s): Physical Exam Finding(s): Bilateral, progressive hearing loss Positive Rinne test Advanced age Lack of lateralization on Weber test Hypertension

Diagnosis #2: Cochlear nerve damage History Finding(s): Physical Exam Finding(s): Prior exposure to loud noise Positive Rinne test PRACTICE CASES Bilateral hearing loss Lack of lateralization on Weber test

293 USMLE STEP 2 CS Patient Note

Diagnosis #3: Otosclerosis History Finding(s): Physical Exam Finding(s): Bilateral, progressive hearing loss Lack of lateralization on Weber test Advanced age

Diagnostic Workup

Audiometry Tympanography Brain stem auditory evoked potentials PRACTICE CASES

294 PRACTICE CASES 295 Used to diagnose auditory neuropathy. The cochlear nerve can become damaged as a result of loud noise. This patient is can become damaged as a result of loud The cochlear nerve This condition usually presents with hearing loss, tinnitus, and episodicThis condition usually presents with hearing It vertigo. It is unlikely that the patient has an intracranial lesion such as a brain tumor in the It is unlikely that the patient has an intracranial A graphic display that represents the conduction of sound in the middle ear. It may help A graphic display that represents the conduction of sound in the middle ear. This is a disease of the elderly that presents as gradual hearing loss resulting from abnormal This is a disease of the elderly that presents To assess hearing function and deafness to specific frequencies. To This is a process of the inner ear in which bone loss is greater than air loss, leading to a gradual than air loss, leading to a gradual the inner ear in which bone loss is greater This is a process of Hearing loss caused by antibiotics will become more pronounced and may even continue to Hearing loss caused by antibiotics will To rule out syphilis associated with Ménière’s disease. rule out syphilis associated with Ménière’s To Used to rule out any intracranial process, tumor, bleed, or CVA. An MRI of the brain would be An MRI of the brain would bleed, or CVA. Used to rule out any intracranial process, tumor, better for an acoustic neuroma or a schwannoma. VDRL/RPR: Audiometry: Tympanography: distinguish middle ear from inner ear dysfunction. Brain stem auditory evoked potentials: CT—head: Acoustic neuroma: is this diagnosis should be considered if evidence of focal neurologic deficits absence of any other signs. However, found. unilateral or bilateral. Ototoxicity: Any sensorineural hearing loss associated with these drugs is worsen for a time after the drug is discontinued. loss, but such loss is reversible with discontinuation of the drug. permanent. Aspirin can also cause hearing aspirin should be withheld. While workup is pending in this patient, bilateral, but in a minority of patients the disease can be unilateral or can affect one side more than the other. bilateral, but in a minority of patients disease: Ménière’s It can be Causes include head trauma and syphilis. in the inner ear. is caused by endolymphatic disruption a military veteran and admits to a history of exposure to loud noises. Cochlear nerve damage would present in loud noises. Cochlear nerve damage would admits to a history of exposure to a military veteran and referred to an with suspected damage should be presbycusis. As with presbycusis, patients a manner similar to an ENT specialist. Such patients will likely need hearing aids as well. audiologist working in conjunction with Otosclerosis: hearing loss, so air loss exceeds bone loss. Otosclerosis is usually temporal bone growth. It is a conductive by performing the Rinne test. Chronic hypertension can lead to vascular changes that reduce blood lead to vascular changes that reduce test. Chronic hypertension can by performing the Rinne flow to affect the as can other conditions that contribute to the development of presbycusis, the cochlea and can works in be referred to an audiologist who diabetes and smoking. This patient should vasculature, such as a hearing aid. ENT specialist. He will likely need conjunction with an Cochlear nerve damage: Presbycusis: be detected diagnosis as people age and can typically bilateral. Presbycusis is a common loss of hearing. It is Diagnostic Workup Additional Differential Diagnoses Additional Differential CASE DISCUSSION CASE Diagnoses Note Differential Patient PRACTICE CASES 296 Reflective Listening Entrance Building theDoctor-Patient Relationship Examinee Checklist should beabletogiveyouamoreaccurateassessmentofhiscondition.” severity ofyourchild’s tests.Afterseeinghim,I illness,Imustexaminehimintheofficeandobtainsomeblood pigment levels,knownasbilirubinmayrisetoohighandcausedamagetohisbrain.To determinethe and rarelyposesathreattothebaby. However, ifyournewbornhasamoreseveretypeofjaundice,hisyellow “Newborns oftendevelopamildcaseofnaturaljaundiceafterbirth.Thistypephysiologicwillresolve Sample ExamineeResponse “Can thisjaundicehurtmybaby?Whyishelikethis?” Challenging QuestionstoAsk because youdonothavetransportation. Show concernaboutyourchild’s health,butaddthatyoudonotwant tocometheofficeunlessyouhave Notes fortheSP The patient’s motheroffersthehistory. Patient Description Checklist/SP Sheet Examinee Tasks skin andeyes. The motherofDavidWhitestone,a5-day-oldmalechild,callstheofficecomplainingthatherchildhasyellow Opening Scenario DOORWAY INFORMATION CASE 18 Examinee summarizedtheSP’s concerns,oftenusingtheSP’s ownwords. Examinee askedtheSPtolist his/herconcernsandlistenedtotheresponsewithout interrupting. Examinee askedanopen-endedquestionandactivelylistened totheresponse. Examinee correctlyusedpatient’s nameandidentified callerandrelationshipoftopatient. Examinee identifiedhis/herroleorposition. selfbyname. Examinee introduced 3. Write thepatientnoteafterleavingroom. 2. Explainyourclinicalimpressionandworkupplantothemother. 1. Take afocusedhistory. PRACTICE CASES 297 Patient Response No. I’m B Rh positive and my husband is A Rh positive. My baby is also B Rh positive. healthy. antibiotics before delivery. during pregnancy First bowel movement of baby Soon after delivery. Birth historyComplications during pregnancyDelivery at term or premature I had a positive culture for some bacteria and received Yes, Smoking, alcohol, or recreational drugs At term. vaginal delivery. It was an uncomplicated Your own blood group and the blood Your groups of your husband and baby Ill contactsOther pregnancies and miscarriages daughter and have had no miscarriages. She is I have a 3-year-old Not to my knowledge. FeverBreathing fastDry mouthShaking (seizures) No. No. No. No. Sucking wellActivities and cry and responsiveAwake Recent URI cries occasionally. he is playful and active. He Yes, Yes. Yes. No. Blood in stoolUrinary frequencyNumber of wet diapersBreast-feeding and frequency soon after birth. Every 4–5 hours. Started No. hours. Every 3–4 7–8 diapers per day. About VomitingAbdominal distentionFrequency of bowel movementsColor of stool He has 2–3 bowel movements a day. No. None. Brown. OnsetProgressionParts of body involvedAge of child visible on his face and hands. It is mainly but I’m still concerned. It is not getting worse, I noticed it yesterday. Five days old. Chief complaint My baby has yellow skin and eyes. Question

Examinee elicited data efficiently and accurately. elicited data efficiently Examinee ✓ Information Gathering Information PRACTICE CASES 298 today. Doyouhaveanyconcernsorquestions? out. Isuggestthatyoubringyourchildtothemedicalcenterforfurtherevaluation.hopeunderstoodwhatwediscussed feeding, someotherpathologicconditions,andcertainbirthdefectscanalsocausejaundiceininfants,theseneedtober This conditionusuallypeaksonday4or5afterbirthandthengraduallydisappearswithin1–2weeks.However, breast- Mrs. Whitestone,giventheinformationyouhaveprovided,I’mconsideringpossibilityofphysiologicornaturaljaundice. Sample Closure Closure None. Physical Examination Connecting withthePatient ✓ Examinee askediftheSPhadanyotherquestionsorconcerns. Examinee discussedinitialmanagementplans: Examinee discussedinitialdiagnosticimpressions. Examinee recognizedtheSP’s emotionsandrespondedwithPEARLS.

Question rgalrisNone.Hehasn’t takenanymedications. Mydaughteralsohadjaundiceafterthefirstweekofbirth.Shewas None. None. None. Drug allergies Uneventful. Family history Past surgicalhistory Past medicalhistory Current medications Discharge fromhospital Follow-up tests. admitted tothehospital. Patient Response uled PRACTICE CASES 299 Patient Note Patient USMLE STEP 2 CS STEP USMLE Physical Examination History USMLE STEP 2 CS Patient Note

Differential Diagnosis Diagnosis #1 History Finding(s): Physical Exam Finding(s):

Diagnosis #2 History Finding(s): Physical Exam Finding(s):

Diagnosis #3 History Finding(s): Physical Exam Finding(s):

Diagnostic Workup PRACTICE CASES

300 USMLE STEP 2 CS Patient Note

History HPI: The source of information is the patient’s mother. The mother of a 5-day-old M c/o her child having yellow discoloration of the eyes and skin for 2 days. It has not worsened. The child is awake, responsive, playful, and active. He is breast-fed. His stomach is soft and he has 2–3 daily bowel movements. The color of his stools is brown. She denies any h/o recent fever, vomiting, seizure, URI, or breathlessness. There is no noticeable dryness of the mouth. He is wetting 7–8 diapers per day every 3–4 hours. He was delivered vaginally at full term. The mother did receive antibiotics for a positive culture before delivery. The blood group of both mother and neonate is B positive, while that of the father is A positive. ROS: Negative. Allergies: NKDA. Medications: None. PMH: None. PSH: None. FH: His elder sister was hospitalized after the first week of birth for jaundice.

Physical Examination None.

Differential Diagnosis Diagnosis #1: Physiologic jaundice History Finding(s): Physical Exam Finding(s): Infant in first week of life No changes in feeding, urination, or bowel movements

Diagnosis #2: ABO or Rh incompatibility History Finding(s): Physical Exam Finding(s): Infant in first week of life Mother and father with different ABO types

Diagnosis #3: Neonatal sepsis History Finding(s): Physical Exam Finding(s):

History of maternal infection PRACTICE CASES

301 USMLE STEP 2 CS Patient Note

Diagnostic Workup

Total and indirect bilirubin Blood typing Direct Coombs test CRP PRACTICE CASES

302 PRACTICE CASES 303 Look for a positive history of a sibling who had neonatal jaundice Although both the mother and father are Rh positive, the fact that they have mother and father are Rh positive, the Although both the This is a condition that results from poor breast-feeding, which in turn results in This is a condition that results from poor A condition that peaks between the third and seventh days of life. Underlying causes between the third and seventh days A condition that peaks This condition results from insufficient mechanisms in the neonatal digestive tract to This condition results from insufficient mechanisms in the neonatal digestive These include hypothyroidism, galactosemia, and hereditary hemolytic disorders such as These include hypothyroidism, galactosemia, and hereditary hemolytic disorders As this scalp hemorrhage reabsorbs, it can also serve as a source of increased bilirubin As this scalp hemorrhage reabsorbs, it Jaundice may be a manifestation of early-onset neonatal sepsis. A history of maternal Jaundice may be a manifestation of early-onset This condition would also present with jaundice but is considerably rarer than the others listed This condition would also present with jaundice but is considerably rarer than This condition may also lead to abnormally elevated levels of bilirubin resulting from increased This condition may also lead to abnormally Biliary atresia: be diagnostic. here. Labs show a direct hyperbilirubinemia, and an abdominal ultrasound may Metabolic disorders: spherocytosis or G6PD deficiency. Breast milk jaundice: Breast with this condition typically feed adequately excrete bilirubin. In contrast to breast-feeding jaundice, neonates well and therefore increase their bilirubin loads. requiring phototherapy. The patient’s sister had jaundice after birth, making this differential a possibility. sister had jaundice after birth, making this The patient’s requiring phototherapy. frequent bowel movements. Polycythemia: total RBC mass. Familial neonatal hyperbilirubinemia: Cephalohematoma: production. presentation. There is no mention of cephalohematoma in this jaundice: Breast-feeding mother reports good feeding as well as removal of bilirubin. This child’s slow bowel movements and insufficient toxoplasmosis, rubella, CMV, HSV, and others) could present with neonatal jaundice within the first week of and others) could present with neonatal HSV, toxoplasmosis, rubella, CMV, that may help reach the correct diagnosis, such as small size for life. These infants may exhibit other findings microphthalmia, and/or hepatosplenomegaly. cataracts, gestational age, rash, microcephaly, group incompatibility may also cause clinically significant jaundice in neonates within the first week of life. jaundice in neonates within the first may also cause clinically significant group incompatibility Neonatal sepsis: neonatal sepsis may be a clue to this diagnosis. However, infections, particularly with group B streptococcus, hypothermia, vomiting, poor feeding, fever, infection, such as lethargy, typically manifests with other signs of infections (commonly referred to as TORCH— intrauterine and/or abnormally colored urine. Additionally, Physiologic jaundice: and decreased activity of the excretory capacity, destruction of erythrocytes, decreased include accelerated commonly seen in preterm infants. enzymes in hepatic cells. It is most bilirubin-conjugating ABO or Rh incompatibility: from blood The hemolysis that results different bloodrisk for ABO incompatibility. types puts the neonate at Early-onset neonatal jaundice (within the first week of life): Early-onset neonatal jaundice (within Additional Differential Diagnoses: Additional Differential CASE DISCUSSION CASE Diagnoses Note Differential Patient onset) and those that in the first week of life (early can be divided into causes that predominate Neonatal jaundice makes early-onset causes more likely. age (five days) onset). The patient’s appear thereafter (late Late-onset neonatal jaundice (after the first week of life): PRACTICE CASES 304 Diagnostic Workup suspected intrauterine (TORCH) infections. suspected intrauterine(TORCH) Serology forCMV, toxoplasmosis,andrubella;RPRforsyphilis;urineculture forCMV: these canbemoresubtleininfantsthanadults,sincetheneonatalimmunesystemisimmature. hemolysis. Differentialcountsmayprovideadditionalcluesaboutinfectionscausingneonatalsepsis,although CBC: CRP: jaundice fromhemolysis. testing tocheckformaternal-fetalincompatibility. Suchchildren shouldalsobecloselyfollowedforevidenceof incompatibility. shouldroutinelyreceivedirectCoombs AllinfantswhoareborntomotherswithtypeOblood Blood typinganddirect Coombstesting: when totalserumbilirubinlevelsexceed15mg/dLinanotherwisewelltwo-day-oldterminfant). usually indicatedandismaintainedonthebasisofbilirubinmeasurements(eg,phototherapyshouldbeinitiated Total andindirect bilirubin: To monitorforsignsofinfection. To parameters suchashematocritandhemoglobinduetosuspectedunderlying evaluatethestatusofblood Thefirststepindeterminingtheseverityandtypeofjaundice.Phototherapyis To group evaluateforjaundicestemmingfromblood In PRACTICE CASES 305 “Is my baby going to be okay?” “Do I need to bring my baby to the hospital?” Examinee summarized the SP’s concerns, often using the SP’s own words. concerns, often using the SP’s Examinee summarized the SP’s Examinee made eye contact with the SP. Examinee asked an open-ended question and actively listened to the response. without interrupting. Examinee asked the SP to list his/her concerns and listened to the response Examinee knocked on the door before entering. Examinee introducedself by name. Examinee identified his/her role or position. name. Examinee correctly used patient’s 3. Write the patient note after leaving the room. the patient 3. Write 1. Take a focused history. 1. Take mother. impression and workup plan to the 2. Explain your clinical to the hospital.” Sample Examinee Response answers, but I will need to examine your child first. Although I suspect “I understand you are concerned and want make sure he does not have anything more serious that might require a trip that he has a viral infection, I need to DOORWAY INFORMATION DOORWAY CASE 19 CASE Entrance Examinee Checklist Relationship Building the Doctor-Patient The patient’s mother offers the history; she is a fair historian. mother offers The patient’s Notes for the SP situation. Show concern regarding your child’s Challenging Questions to Ask Checklist/SP Sheet Patient Description The mother of Josh White, a 7-month-old male child, comes to the office complaining that her child has a fever. to the office complaining that her White, a 7-month-old male child, comes The mother of Josh Examinee Tasks Opening Scenario Reflective Listening PRACTICE CASES 306 Information Gathering ✓ Examinee eliciteddataefficientlyandaccurately.

Question leighbt Lastnighthe did notsleepwellandcriedwhenIlaidhimdown. Iambreast-feeding him,andIgivehimallthevitaminsthathis Itwasa40-week vaginaldeliverywithnocomplications. Two weeksago, andeverythingwasperfectwithhim. Sleeping habits Uptodate. Eating habits Yes. development His3-year-old brotherhadanupperrespiratorytractinfectiona Child weight,height,andlanguage Birth history Idon’t know. Last checkup Heisnoteating anythingatall. Vaccinations Ill contactsindaycarecenter Day carecenter No. Ill contacts Appetite changes No. irritated, playful,etc.)? How hasthebabylooked(lethargic, No. Shaking (seizures) wet diapers) or color(changeinnormalnumberof Change inurinaryhabits,urinesmell, No. Idon’t know, buthehasn’t eatenanythingsinceyesterdayandis color orconsistency Ihavenotnoticedanybellybreathingorflaringofhisnostrils. Change inbowelhabitsorstool Ithinkso; heisbreathingquickly. Nausea/vomiting Rash Yes. Yesterday. Imeasurediton hisforehead,anditwas101. Difficulty swallowing No. Difficulty breathing Shortness ofbreath Mychildhas afever. Cough Ear pulling/eardischarge Runny nose Temperature Onset Chief complaint refusing todrinkfromhisbottleormybreast. Patient Response also gets baby food 3timesaday.also getsbabyfood pediatrician prescribes.Hehasrefusedmybreastsinceyesterday. He Normal. week ago,butheisfinenow. He haslookedtiredandirritatedsinceyesterday. No. No. PRACTICE CASES 307 of Patient Response Patient Follow-up tests. Drug allergies No. Current medications historyPast medical Past surgical history Tylenol. life. in the first week of Jaundice None. Question

Examinee discussed initial management plans: Examinee discussed initial management questions or concerns. Examinee asked if the SP had any other Examinee discussed initial diagnostic impressions. Examinee discussed initial diagnostic Examinee recognized the SP’s emotions and responded with PEARLS. emotions and the SP’s Examinee recognized ✓ Sample Closure respiratory tract infection, or it may be attributable to an ear fever may be due to a simple upper Mrs. White, your child’s of bacteria. I would like to examine him so that I can better determine the cause infection caused by a virus or certain types such as meningitis. In addition to a detailed physical exam, your baby may need some his fever and exclude more serious causes, for me? Do you have any questions x-ray. blood tests, a urinalysis, and possibly a chest Physical Examination None. Closure Connecting with the Patient Connecting with PRACTICE CASES 308 Physical Examination History USMLE STEP2CS Patient Note PRACTICE CASES 309 Patient Note Patient Diagnosis #3 History Finding(s): Physical Exam Finding(s): Diagnosis #2 History Finding(s): Physical Exam Finding(s): Diagnosis #1 History Finding(s): Physical Exam Finding(s): USMLE STEP 2 CS STEP USMLE Diagnostic Workup Differential Diagnosis Differential USMLE STEP 2 CS Patient Note

History HPI: History obtained from mother. The patient is a 7-month-old M with fever × 1 day. Temperature recorded by forehead thermometer at home reached 101°F yesterday. The child has been tired, irritated, and breathing rapidly for the past day. The mother denies any abdominal retractions or nasal flaring. The mother also notes rhinorrhea and refusal of breast and baby food. The child has a history of sick contact with his 3 yo brother, who had a URI 1 week ago that has since resolved. He attends day care. No cough, ear pulling, ear discharge, or rash. ROS: Negative except as above. Allergies: NKDA. Medications: Tylenol. PMH: Jaundice in the first week of life. PSH: None. Birth history: 40-week vaginal delivery with no complications. Dietary history: Breast-feeding and supplemental vitamins. Immunization history: UTD. Developmental history: Last checkup was 2 weeks ago and showed normal weight, height, and develop- mental milestones.

Physical Examination None.

Differential Diagnosis Diagnosis #1: Viral URI History Finding(s): Physical Exam Finding(s): Fever (101°F) Rhinorrhea Sibling with URI Day care attendance Increased breathing rate

Diagnosis #2: Pneumonia History Finding(s): Physical Exam Finding(s): Fever (101°F) Day care attendance Sibling with URI Increased breathing rate PRACTICE CASES

310 USMLE STEP 2 CS Patient Note

Diagnosis #3: Otitis media History Finding(s): Physical Exam Finding(s): Fever (101°F) Irritability Day care attendance

Diagnostic Workup

CBC with differential Blood culture UA and urine culture CXR Respiratory viral panel Pneumatic otoscopy PRACTICE CASES

311 PRACTICE CASES 312 Diagnostic Workup Additional Differential Diagnoses Patient NoteDifferential Diagnoses CASE DISCUSSION necessarily indicatedinthiscase,asfeveris<102°Fandthechildappropriatelyimmunized. culturethatcanprogresstosepsisifleftuntreated.Anextensiveworkup(seebelow)isnot a positiveblood high proportionofunimmunizedorincompletelyimmunizedchildrenwithnoidentifiablefeversourcewillhave media. Notroutinelyusedin primarypediatriccaresettings. Tympanometry: Pneumatic otoscopy: tract infectionthatmaypresentasfeverwithnolocalizing source. Respiratory viralpanel(rapidantigenorPCRtests): CXR: be investigated. UTI isaprominentcauseoffeverwithoutlocalizingsource, especiallyinfullyimmunizedchildren,andmust in childrenwithunexplainedhighfever. Notably, aWBCcount>15,000/μL suggestsoccultbacteremia.Occult CBC withdifferential, bloodculture, UA,andurineculture: vaccines against localizing sourcewhohavereceiveduniversalinfantimmunizationsintheUnitedStates,includingconjugate no obvioussource.However, occultbacteremiahassignificantlydeclined amongchildrenwithfeverwithouta Occult bacteremia: pain. or waterydiarrhea,whereasbacterialinfectionmaycausefever, diarrhea,andsevereabdominal tenesmus, bloody Gastroenteritis: pyelonephritis untilprovenotherwise. dribbling orcolicbeforeandduringvoiding.PatientswithhighfeverCVA tendernessarepresumedtohave UTI: infants (nuchalrigidityandfocalneurologicsignsaremorecommonlyseeninolderchildren). as seeninthiscase.Thephysicalexammayrevealabulgingfontanelle.Meningealsignsnotbeobvious Meningitis: tympanic membrane(TM),lossofTMlandmarks,anddecreasedmobilityonpneumaticotoscopy. are oftennotpresent(asinthiscase).Thephysicalexamisimportantandmayrevealahyperemic,bulging Otitis media: rales. present. Thephysicalexammayrevealretractions,nasalflaring,grunting,dullnessonchestpercussion,and Pneumonia: be ruledoutinlightofthechild’s apparentdyspneaandtachypnea. sibling withaURI.URIsareusuallyviral,self-limited,andbenign,butlowerrespiratorytractinfectionmustfirst URI: Viral InfantswithaUTImaynothavesymptomsreferabletotheurinarytract.whodo To diagnosepneumonia. Cluessuggestingthisdiagnosisasthesourceoffeverincluderhinorrheaandrecentexposuretoa Findingsareoftensubtleandnonspecificmaybelimitedtofever, irritability, andpoorfeeding, Fever, rhinorrhea,tachypnea,anddyspneasupportthisdiagnosiseventhoughcoughisnot Otalgiaandeardrainageinanill,febrilechildcansuggestadiagnosisofacuteotitismediabut Streptococcus pneumoniae Usefulininfantsolderthan sixmonthsofage;confirmsabnormalTMmobilityinotitis ThispatienthasfeverbutnoGIsymptoms.Viral gastroenteritistypicallycausesvomitingand/ Thisisanimportantconsiderationforchildrenwithhighfever(>102°F/38.9°C)and KeytolookfortheTMerythema anddecreasedmobilityseeninotitismedia. and Haemophilus influenzae Usedtodiagnosecommonviralcausesofrespiratory Theworkupforsepsisoroccultbacteremia typeb.Ontheotherhand,arelatively PRACTICE CASES 313 To identify causative viruses in pediatric infections (not commonly used). identify causative viruses in pediatric To Should be performed if there is any concern for meningitis. CSF analysis includes cell includes cell meningitis. CSF analysis is any concern for performed if there Should be Rarely used in the initial workup for fever without a localizing source. A diagnostic aid in cases source. A diagnostic aid in cases initial workup for fever without a localizing Rarely used in the Used mainly to rule out brain abscess or hemorrhage. Used mainly to rule of severe or refractory pneumonia. of severe or refractory titers: Serum antibody LP—CSF analysis: LP—CSF and occasionally latex for specific viruses, culture, PCR protein, Gram stain, differential, glucose, count and antigens. for common bacterial agglutination CT—head: Bronchoscopy: PRACTICE CASES 314 Notes fortheSP Patient isa26yoM. Patient Description Checklist/SP Sheet Examinee Tasks HR: RR: Temp: BP: Signs Vital Eric Glenn,a26-year-old male,comestotheofficecomplainingofcough. Opening Scenario DOORWAY INFORMATION CASE 20 sensitive toantibiotics.However, ifIfind thatyouhavebacterialpneumonia,antibioticswillbe needed.” “Possibly. Antibioticsdon’t helpwithbronchitisbecausethisconditionisprimarilycausedbyvirusesthatare not Sample ExamineeResponse “Do Ineedantibioticstogetbetter?” Challenging QuestionstoAsk 4. Write thepatientnoteafterleavingroom. 3. Explainyourclinicalimpressionandworkupplantothepatient. 2. Performafocusedphysicalexam(donotperformrectal,genitourinary, orfemalebreastexam). 1. Take afocusedhistory. 120/80mmHg 15/minute 75/minute,regular glass ofwateroratissue. During theencounter, pretendtohaveasevereattackofcoughing.Notewhethertheexamineeoffersyou If askedaboutsputum,asktheexaminee,“Whatdoes‘sputum’ mean?” right side,andsayitinacoarse,deepvoice. Chest palpation:Whentheexamineepalpatesyourrightchestandasksyoutosay“99,”turnface right chestbymovingyourshouldersup,butdonotactuallybreathein. Chest auscultation:Whenaskedtotakeabreath,pretendinhalewhiletheexamineeislisteningyour Continue coughingevery3–4minutesduringtheencounter. Cough astheexamineeentersroom. 99.9°F(37.7°C) PRACTICE CASES 315 but everything is better now. chills. better when I sleep on my right side. Patient Response Recent travelLast PPD None. Never had it. Severity on a scaleNight sweatsExposure to TBPet, animal exposure 8/10. No. None. None. QualityAlleviating/exacerbating factorsRadiation of pain It increases when I take a deep breath and when I cough. I feel take a deep breath. It feels like a knife. I can’t No. Sputum productionBlood in sputumChest painLocation Small amounts of white mucus. No. breath. I feel a sharp pain when I cough or take a deep Yes, Right chest. OnsetPreceding symptoms/eventsFever/chillsweek, and sore throat 2 weeks ago for a had a runny nose, fever, I One week ago. take my temperature; no but I didn’t I think I had a mild fever, Chief complaint Cough. Question

Examinee elicited data efficiently and accurately. Examinee elicited data efficiently and or a tissue during the severe bout of coughing. Examinee offered the SP a glass of water Examinee asked an open-ended question and actively listened to the response. open-ended question and actively listened Examinee asked an concerns and listened to the response without interrupting. Examinee asked the SP to list his/her own words. often using the SP’s concerns, Examinee summarized the SP’s Examinee introduced self by name. his/her role or position. Examinee identified name. used patient’s Examinee correctly contact with the SP. Examinee made eye Examinee knocked on the door before entering. Examinee knocked ✓ Examinee Checklist Examinee Relationship the Doctor-Patient Building Entrance Information Gathering Reflective Listening PRACTICE CASES 316 Physical Examination Connecting withthePatient ✓ ✓ Examinee didnotrepeatpainfulmaneuvers. Examinee usedrespectfuldraping. Examinee askedpermissiontostarttheexam. Examinee washedhis/herhands. Examinee recognizedtheSP’s emotionsandrespondedwithPEARLS.

ExamComponent Question xrmte Inspection Auscultation,palpation Auscultation, palpation,percussion Examinedmouth,throat,lymphnodes Extremities Pulmonary exam CV exam Head andneckexam No. Nope,Idon’t enjoyitwithacondom. Yes, Ismokeapackday. Istartedwhenwas15yearsold. Well, I’vehad manygirlfriends.EverySaturdaynight,Ipickupa Never. Idrinkalotontheweekends.nevercount. No(toall4). Drug allergies Pizzadeliveryperson. Use ofcondoms health. Myfatherandmotherarealiveingood Sexual activity Tobacco None. Illicit druguse Ihadgonorrhea2yearsagoandwastreatedwithantibiotics. CAGE questions Alcohol use Tylenol. Occupation No. Family history Past surgicalhistory Past medicalhistory Current medications Weight/appetite changes sea/vomiting, diarrhea/constipation) breath, wheezing,abdominalpain,nau- Associated symptoms(shortnessof Maneuver Patient Response new girlfromthenightclub. None. PRACTICE CASES 317 cing HIV testing (and discussed consent). HIV testing (and discussed Follow-up tests. Safe sex practices. SP had any other questions or concerns. Examinee asked if the Examinee discussed initial diagnostic impressions. discussed initial diagnostic Examinee plans: discussed initial management Examinee pregnancies. Do you have any questions for me? pregnancies. Do you have any questions Mr. Glenn, your cough is most likely due to an infection that can be either bacterial or viral. The chest pain you are experien can be either bacterial or viral. The chest pain is most likely due to an infection that Glenn, your cough Mr. common with HIV, Some of these infections can be more of your lung membranes by an infection. is probably due to irritation reason for your cough may be acid reflux, more I recommend that we test for it. Another and given your sexual history, a going to test your blood and sputum and will obtain a chest x-ray to help us make are commonly known as heartburn. We I to obtain a PPD to test for tuberculosis if your cough is persistent. In the meantime, may also need definitive diagnosis. We during intercourse to prevent STDs such as HIV as well as to prevent unwanted strongly recommend that you use condoms Sample Closure Closure PRACTICE CASES 318 Physical Examination History USMLE STEP2CS Patient Note PRACTICE CASES 319 Patient Note Patient Diagnosis #3 History Finding(s): Physical Exam Finding(s): Diagnosis #2 History Finding(s): Physical Exam Finding(s): Diagnosis #1 History Finding(s): Physical Exam Finding(s): USMLE STEP 2 CS STEP USMLE Diagnostic Workup Differential Diagnosis Differential USMLE STEP 2 CS Patient Note

History HPI: 26 yo M c/o cough × 1 week. 2 weeks ago: fever, rhinorrhea, sore throat. Persistent productive cough with small amount of white mucus but no hemoptysis. Sharp, stabbing 8/10 pain in right chest, exacerbated by cough and deep inspiration. Mild fever. Denies chills, night sweats, SOB, or wheezing. No recent travel. No TB exposure. No weight or appetite changes. ROS: Negative except as above. Allergies: NKDA. Medications: Tylenol. PMH: Gonorrhea 2 years ago, treated with antibiotics. SH: 1 PPD since age 15; drinks heavily on weekends. CAGE 0/4. Unprotected sex with multiple female partners. FH: Noncontributory.

Physical Examination Patient is in no acute distress. VS: WNL except for low-grade fever. HEENT: Nose, mouth, and pharynx WNL. Neck: No JVD, no lymphadenopathy. Chest: Increase in tactile fremitus and decrease in breath sounds on the right side. No rhonchi, rales, or wheezing. Heart: Apical impulse not displaced; RRR; normal S1/S2; no murmurs, rubs, or gallops. Extremities: No cyanosis or edema.

Differential Diagnosis Diagnosis #1: Pneumonia History Finding(s): Physical Exam Finding(s): Persistent cough Increased tactile fremitus Low-grade fever Decreased breath sounds on the right Temperature 99.9°F PRACTICE CASES

320 USMLE STEP 2 CS Patient Note

Diagnosis #2: URI-associated cough (postinfectious cough) History Finding(s): Physical Exam Finding(s): Recent URI Temperature 99.9°F Low-grade fever Persistent cough

Diagnosis #3: Acute bronchitis History Finding(s): Physical Exam Finding(s): Low-grade fever Increased tactile fremitus Persistent cough Temperature 99.9°F White sputum production

Diagnostic Workup

CXR CBC with differential Sputum Gram stain and culture PRACTICE CASES

321 PRACTICE CASES 322 Diagnostic Workup Additional Differential Diagnoses pneumonia. Rarely, severecoughingcanleadtoaribfracture, whichinturncancauseseverepleuritis. This youngman’s coughandpleuriticpainarelikelycausedbyaviralrespiratoryinfectionor acuteproductive Patient NoteDifferential Diagnoses CASE DISCUSSION acquired pneumonia. specific causesofatypicalpneumonia.Seldomusefulinthe initialevaluationofpatientswithcommunity- with riskfactorsforthisinfection. HIV antibody: Pulse oximetryorABG: suspected. community-acquired pneumonia,butagoldstandardthat isoftenusedearlywhen Bronchoscopy withbronchoalveolar lavage: Urine diagnosis inpneumonia. results betweenGramstainandcultureinpneumococcal pneumonia),butmayhelpidentifyamicrobiologic Sputum Gramstainandculture: CBC withdifferential: necessarily ruleitout. CXR: (eg, nothrush),andin history, heshouldbescreenedforHIVinfection.Notably, thereisnoevidenceofimmunosuppressiononexam risk factors),andpulmonaryedema(signssymptomsofheartfailurewouldbepresent).Giventhepatient’s neurologically impairedpatientsareatrisk),pulmonaryembolism(extremelyrareinayoungpatientwithno Other etiologies: or breathing.Mostpatientsrecoverwithinthreedaystooneweek. and earlyfallpresentswithacute,severeparoxysmalpainofthethoraxorabdomenthatworsenscough Pleurodynia: an acute,notchronic,bronchitis. Acute bronchitis: bronchitis. 1–2 weeks(orupto6–8inpatientswithunderlyingasthma).Causesrangefromrhinosinusitisacute URI-associated cough: rales tohelpsuggestafocalpneumonia.Theabsenceofdyspneaalsoarguesagainstthisdiagnosis. patient, increasedtactilefremitussuggestsairspaceconsolidation,buttherearenobronchialbreathsoundsor characteristic chestexamfindings,whichmaybedifficulttoelicitinanotherwisehealthypatient.Inthis Pneumonia: dyspnea. To helpdiagnosepneumonia(ie,toseeinfiltrates andeffusion),althoughanormalfilmdoesnot Legionella Pleuriticpainmaysignallowerrespiratorytractinfection.Thisdiagnosisisoftenconfirmedby Anuncommonacuteillnessusuallycausedbyoneofthecoxsackieviruses.Itoccursinsummer Although HIV is less likely in this scenario, an antibody testshouldbeofferedtoallpatients AlthoughHIVislesslikely in thisscenario,anantibody antigen,serum CoughcanalsoaccompanyacuteURI.Theonsetofthispatient’s symptomspointsto Othercausesofacutecoughincludeaspiration(forwhichalcoholic,elderly, and Pneumocystis jiroveci Inacuteinfection,canrevealleukopeniaorleukocytosis. AcutecoughfrequentlyfollowsURI(“postinfectious”)andcommonlypersistsfor Mayhelpdeterminetheneed forhospitalization. Oftenlowyield(duetocontaminationbyoralfloraand oftendiscordant Mycoplasma pneumonia, cough is usually nonproductive andaccompaniedby pneumonia,coughisusuallynonproductive Aninvasivetestthatisrarelynecessarytodiagnose PCR,coldagglutininmeasurement: P jiroveci infectionis To helpdiagnose PRACTICE CASES 323 98.3°F (36.8°C) Sit up on the bed. Show signs of scratching. Exhibit RUQ tenderness on palpation. ultrasound, or MRI is mentioned, ask for an explanation. If ERCP, 70/minute, regular 15/minute 130/80 mm Hg Examinee knocked on the door before entering. Examinee introducedself by name. 2. Perform a focused physical exam (do not perform rectal, genitourinary, or female breast exam). not perform rectal, genitourinary, 2. Perform a focused physical exam (do workup plan to the patient. 3. Explain your clinical impression and the patient note after leaving the room. 4. Write 1. Take a focused history. a focused 1. Take Challenging Questions to Ask Could I have it too?” “My father had pancreatic cancer. Sample Examinee Response family history does put you or skin. Your why we always rule it out in patients with yellow eyes possible; that’s “It’s know anything for certain until we run some tests.” we won’t at slightly increased risk. However, DOORWAY INFORMATION DOORWAY CASE 21 CASE Examinee Checklist Relationship Building the Doctor-Patient Entrance Checklist/SP Sheet Patient Description Patient is a 52 yo F. Notes for the SP RR: HR: Examinee Tasks Gail Abbott, a 52-year-old female, comes to the office complaining of yellow eyes and skin. female, comes to the office Gail Abbott, a 52-year-old Vital Signs BP: Temp: Opening Scenario PRACTICE CASES 324 Information Gathering Reflective Listening ✓ Examinee eliciteddataefficientlyandaccurately. Examinee summarizedtheSP’s concerns,oftenusingtheSP’s ownwords. Examinee askedtheSPtolisthis/herconcernsandlistenedresponsewithoutinterrupting. Examinee askedanopen-endedquestionandactivelylistenedtotheresponse. Examinee madeeyecontactwiththeSP. Examinee correctlyusedpatient’s name. Examinee identifiedhis/herroleorposition.

Question asavmtn SometimesI feelnauseatedwhenIaminpain,butnovomiting. No. Dull. None. No. None. Diarrhea/constipation Tylenol. Itake4pillseverydayjusttomakesuredonotfeelthe No. Nausea/vomiting Itishere(pointstotheRUQ). ofsimilarpain Previous episodes topain Relationship offood Itisthesame. Exacerbating factors Atleastonceeveryday. Alleviating factors WhenIhavethepain,itis3/10,andthenmaygodownto 0. ItwasaroundthesametimethatInoticedchange inthecolor Quality Radiation Well, Idon’t havethepainalltime.Itcomesandgoes. Location Severity ofpainonascale Sometimes. Istarteditching2monthsago;Benadrylusedtohelp,butnot Progression Frequency Constant/intermittent Dark. Threeweeksago. Sometimesit’s 7/10. Light. Onset Abdominal pain Severity ofpruritusonascale Yellow eyesandskin. Pruritus Color ofurine Color ofstool Onset Chief complaint Patient Response pain. of myeyesandskin. anymore. PRACTICE CASES 325 healthy. No. Patient Response Patient How much alcohol do you drink?CAGE questions I have had 1 or 2 glasses of wine every day for the past 30 years. Affecting job/relationships/legal prob- lems No (to all 4). Illicit drug useTobaccoSexual activityDrug allergies No. with my husband. Yes, No. Penicillin, causes rash. Past medical historyPast surgical historyFamily historyOccupation Hypothyroidism. I had 2 C-sections at ages 25 and 30 and a tubal ligation at age 35. My mother is alive and My father died at 55 of pancreatic cancer. I work in a travel agency. Travel historyTravel Immunization before travelCurrent medicationsSimilar episodes No. ago. I went to Mexico for a brief vacation about 2 months Synthroid. Tylenol, No. Fatigue changesWeight Appetite changesJoint pain No. I have no appetite. recently. Yes, No. ColonoscopyBlood transfusion night sweatsFever, Never. when I had a C-section 20 years ago. Yes, No. Question

Examinee washed his/her hands. Examinee asked permission to start the exam. Examinee used respectful draping. Examinee did not repeat painful maneuvers. Examinee recognized the SP’s emotions and responded with PEARLS. Examinee recognized the SP’s ✓ Connecting with the Patient Physical Examination PRACTICE CASES 326 questions forme? Until then,IrecommendthatyoustopdrinkingandlimityouruseofTylenol, asbothmayharmyourliver. Doyouhaveany better ideaofwhatisgoingon.Oncewefindthecauseyourproblem,cancomeupwithanappropriatetreatmentplan. physically closetoyourliver. We willhavetorunsomebloodtestsandconductimagingstudiessuchasultrasoundgeta Mrs. Abbott,thesymptomsyoudescribeareusuallyduetoadisordereitherinliveritselforbileducts,which are Sample Closure Closure ✓ Examinee askediftheSPhadanyotherquestionsorconcerns. Examinee discussedinitialmanagementplans: Examinee discussedinitialdiagnosticimpressions.

ExamComponent knLookedforspidernevi,cutaneoustelangiectasias,palmarerythema Checkedforasterixis,edema Inspection,auscultation,palpation(includingMurphy’s sign), Auscultation Auscultation Inspectedsclerae,undertongue Skin Extremities Abdominal exam Pulmonary exam CV exam HEENT Follow-up tests. Maneuver splenomegaly, fluidwaveforshiftingdullness percussion, measurementofliverspan,palpationorpercussionfor PRACTICE CASES 327 Patient Note Patient USMLE STEP 2 CS STEP USMLE Physical Examination History USMLE STEP 2 CS Patient Note

Differential Diagnosis Diagnosis #1 History Finding(s): Physical Exam Finding(s):

Diagnosis #2 History Finding(s): Physical Exam Finding(s):

Diagnosis #3 History Finding(s): Physical Exam Finding(s):

Diagnostic Workup PRACTICE CASES

328 USMLE STEP 2 CS Patient Note

History HPI: 52 yo F c/o yellow skin and eyes × 3 weeks. Light-colored stool and dark urine. 3/10 RUQ pain, dull, intermittent (daily), no radiation, unrelated to meals, relieved by Tylenol. Fatigue. Anorexia. Pruritus up to 7/10 in severity. Nausea. Recent travel to Mexico. History of blood transfusion 20 years ago. No diarrhea, constipation, or weight loss. ROS: Negative except as above. Allergies: Penicillin, causes rash. Medications: Tylenol, Synthroid. PMH: Hypothyroidism. PSH: 2 C-sections, tubal ligation. SH: No smoking, 1–2 glasses of wine/day for 30 years, CAGE 0/4, no illicit drugs. Sexually active with husband only. FH: Father died of pancreatic cancer at age 55. No other FH of GI cancer.

Physical Examination Patient is in no acute distress. VS: WNL. HEENT: Sclerae icteric. Chest: Clear breath sounds bilaterally. Heart: RRR; normal S1/S2; no murmurs, rubs, or gallops. Abdomen: Soft, nondistended, C-section scar. Mild RUQ tenderness without rebound or guarding, Murphy’s sign, BS, no organomegaly or masses. No evidence of fluid wave suggestive of ascites. Skin: Jaundice, excoriations due to scratching, no spiders/telangiectasias/palmar erythema. Extremities: No asterixis, no edema.

Differential Diagnosis Diagnosis #1: Extrahepatic biliary obstruction (eg, pancreatic cancer, cholangiocarcinoma, ampullary carcinoma, sphincter of Oddi dysfunction) History Finding(s): Physical Exam Finding(s): Light stools, dark urine Jaundice, scleral icterus PRACTICE CASES Pruritus RUQ tenderness Father with pancreatic cancer

329 USMLE STEP 2 CS Patient Note

Diagnosis #2: Viral hepatitis History Finding(s): Physical Exam Finding(s): History of blood transfusion Jaundice, scleral icterus Recent travel to Mexico RUQ tenderness

Diagnosis #3: Acetaminophen hepatotoxicity History Finding(s): Physical Exam Finding(s): Frequent acetaminophen use Jaundice, scleral icterus Concomitant alcohol use RUQ tenderness

Diagnostic Workup

AST/ALT/bilirubin/alkaline phosphatase U/S—RUQ abdomen Viral hepatitis serologies PRACTICE CASES

330 PRACTICE CASES 331 These levels can help differentiate a hepatocellular process These levels can help differentiate a hepatocellular The patient’s family history puts her at increased risk for pancreatic family history puts her at increased The patient’s This should be suspected in acute liver injury, as even moderate amounts of injury, This should be suspected in acute liver Hepatitis A IgM antibody should be checked to document recent infection. Other This usually occurs in women 40–60 years of age, often with pruritus as a presenting This usually occurs in women 40–60 Used to diagnose acetaminophen overdose. Used to diagnose biliary obstruction, stones, and intrahepatic tumors. The patient’s symptoms are consistent with this diagnosis. Hepatomegaly is often symptoms The patient’s A CT scan provides information similar to that above but is more expensive. The patient is at risk for hepatitis A (in light of her trip to Mexico) and chronic hepatitis C The patient is at risk for hepatitis A (in Can identify the cause, location, and extent of biliary obstruction. ERCP is invasive but has the Can identify the cause, location, and extent of biliary obstruction. ERCP is A coagulopathy is often seen in advanced liver disease and is attributable to synthetic dysfunction and A coagulopathy is often seen in advanced liver disease and is attributable to Patients with chronic liver disease often exhibit a low platelet count as a result of portal hypertension and Patients with chronic liver disease often exhibit a low platelet count as a result MRCP/ERCP: MRCP is a noninvasive MRI-based advantage of being both a diagnostic and a therapeutic tool in many cases. diagnostic substitute. subsequent splenomegaly. PT/PTT: subsequent clotting factor deficiencies. Acetaminophen level: CT—abdomen: U/S—RUQ abdomen: Viral hepatitis serologies: screening tests include hepatitis B surface antigen and hepatitis C antibody. CBC: case). However, jaundice is usually a late finding and is not associated with RUQ pain. jaundice case). However, Alcoholic hepatitis: patients often underreport alcohol only one or two glasses of wine daily, present. Although she reports drinking consumption. Primary biliary cirrhosis: with other autoimmune diseases, such as hypothyroidism (as in this symptom. It is commonly found in patients AST/ALT/bilirubin/alkaline phosphatase: AST/ALT/bilirubin/alkaline from a cholestatic process (primarily associated with (primarily associated with increased AST and ALT) increased bilirubin and alkaline phosphatase). Acetaminophen hepatotoxicity: capacity of a damaged liver (usually in alcoholics and in patients acetaminophen may overwhelm the metabolic with chronic hepatitis or cirrhosis). intermittent biliary obstruction) narrows the differential to choledocholithiasis (stone in the common bile duct), to choledocholithiasis (stone in the common obstruction) narrows the differential intermittent biliary of Oddi dysfunction. carcinoma of the ampulla, or sphincter cholangiocarcinoma, Viral hepatitis: the intermittent nature of her RUQ pain makes acute hepatitis (given her remote blood transfusion). However, less likely. Extrahepatic biliary obstruction: Extrahepatic biliary pain (suggesting her intermittent jaundice. However, which classically presents with painless cancer, Additional Differential Diagnoses Additional Differential Diagnostic Workup Gilbert’s syndrome). Thus, the predominantly conjugated hyperbilirubinemia suspected in this patient may be due hyperbilirubinemia suspected in this Thus, the predominantly conjugated syndrome). Gilbert’s or extrahepatic biliary such as Dubin-Johnson syndrome, drugs, sepsis, hereditary disorders to hepatocellular disease, obstruction.chills associated with episodes ruled out by the absence of fever and Cholangitis is of abdominal pain. CASE DISCUSSION CASE Diagnoses Note Differential Patient Jaundice of a may be hepatic or nonhepatic. The presence the cause of which results from hyperbilirubinemia, with hemolysis or hyperbilirubinemia (eg, that associated urine color excludes unconjugated change in stool and PRACTICE CASES 332 Entrance Building theDoctor-Patient Relationship Examinee Checklist Notes fortheSP Patient isa53yoM,marriedwith3children. Patient Description Checklist/SP Sheet Examinee Tasks HR: RR: Temp: BP: Signs Vital Edward Albright,a53-year-old male,comestotheEDcomplainingofdizziness. Opening Scenario DOORWAY INFORMATION CASE 22 for yourhearingproblems.” understand whyyouhavebeendizzyandyourhearing isaffected.Afterthat,wecandiscusspossiblereasons your symptoms,butbeforeIcanconfidentlyanswer question,Iwouldliketodoafewmoretestsbetter “I understandyourconcern,Mr. Albright.Avarietyofpermanentandnonpermanentconditionscancause Sample ExamineeResponse “I amreallyscaredaboutmyhearing,doctor. Doyouthinkthiswill bepermanent?” Challenging QuestionstoAsk Examinee introduced selfbyname. Examinee introduced Examinee knockedonthedoor beforeentering. 4. Write thepatientnoteafterleavingroom. 3. Explainyourclinicalimpressionandworkupplantothepatient. 2. Performafocusedphysicalexam(donotperformrectal,genitourinary, orfemalebreastexam). 1. Take afocusedhistory. 135/90mmHg 16/minute 76/minute,regular examinee explainswhyhe/shewouldliketoseeyourgait. Refuse towalkiftheexamineeasksyouto.Pretendthatareafraidoffallingdown.Walk onlyifthe better whentheexamineemovesclosertoyourrightear. Ask theexamineetospeakloudly. Pretendthatyouhavedifficultyhearinginyourleftearandhear 98.0°F(36.7°C) PRACTICE CASES 333 Patient Response When I get up from bed or lie down to sleep, but as I said, it can When I get up from bed or lie down to happen anytime. yesterday. fall. don’t is much better today. HeadachesHead trauma No. No. FeverDescription of stoolAbdominal painURI (runny nose, sore throat, cough) No. It was a watery diarrhea with no blood. No. No. FallsNausea/vomitingRecent infections I feel nauseated, and I vomited several times. Yes, I’ve had it for the past 3 days, but it I have had really bad diarrhea. No, sometimes I feel unsteady as if I were going to fall down, but I Hearing loss (which ear, when)Hearing loss (which ear, Fullness or pressure in the ears This started I have difficulty hearing you in my left ear. Yes, Discharge from the ears No. No. TimingPositions that can elicit the dizziness (lying down, sitting, standing up) Positions that can relieve the dizziness None. Tinnitus It can happen anytime. No. OnsetProgressionConstant/intermittentDuration It comes and goes. It is getting worse. days ago. Two It lasts for 20–30 minutes. Chief complaintDescribe the meaning of dizziness around me. I feel as if the room were spinning Well, I feel dizzy. Question

Examinee asked the SP to list his/her concerns and listened to the response without interrupting. SP to list his/her concerns and listened Examinee asked the own words. SP’s concerns, often using the the SP’s Examinee summarized efficiently and accurately. Examinee elicited data Examinee identified his/her role or position. identified his/her Examinee name. correctly used patient’s Examinee with the SP. made eye contact Examinee to the response. open-ended question and actively listened Examinee asked an ✓ Information Gathering Reflective Listening PRACTICE CASES 334 have anyquestionsforme? be carefulwhenyoustandup quickly orwalkunaccompanied,andyoushouldusehandrailings wheneverpossible.Doyou evaluation, andanMRIthatwill providedetailedimagesofyourbrain.Untilwefindthecause ofyourproblem,youshould pressure. We willhavetorun someteststopinpointthesourceofyoursymptoms.These mayincludebloodtests,ahearing Mr. Albright,thedizzinessyouareexperiencingmaybedue toaprobleminyourearsorbrain,itmayresultfromlowblood Sample Closure Closure Physical Examination Connecting withthePatient ✓ ✓ Examinee askediftheSPhadanyotherquestionsorconcerns. Examinee discussedinitialmanagementplans: Examinee discussedinitialdiagnosticimpressions. Examinee didnotrepeatpainfulmaneuvers. Examinee usedrespectfuldraping. Examinee askedpermissiontostarttheexam. Examinee washedhis/herhands. Examinee recognizedtheSP’s emotionsandrespondedwithPEARLS.

ExamComponent Question Vea Auscultation,orthostaticvitalsigns Cranialnerves, motorexam,DTRs,gait,Romberg’s sign,tilttest Inspectedfornystagmus,funduscopicexam,otoscopy, assessed Neurologic exam CV exam HEENT No. No. Yes, withmywife. No. Yes, Idrink2–3beersaweek. Executivedirectorofaninsurancecompany. Nosimilarprobleminthefamily. Drug allergies Sexual activity Tobacco Appendectomy. Illicit druguse pressure,diagnosed7yearsago. Highblood Alcohol use Furosemide,captopril. Occupation Family history Past surgicalhistory Past medicalhistory Current medications Follow-up tests. (eg, Dix-Hallpikemaneuver) hearing, RinneandWeber tests,inspectedmouthand throat Maneuver Patient Response

PRACTICE CASES 335 Patient Note Patient USMLE STEP 2 CS STEP USMLE Physical Examination History USMLE STEP 2 CS Patient Note

Differential Diagnosis Diagnosis #1 History Finding(s): Physical Exam Finding(s):

Diagnosis #2 History Finding(s): Physical Exam Finding(s):

Diagnosis #3 History Finding(s): Physical Exam Finding(s):

Diagnostic Workup PRACTICE CASES

336 USMLE STEP 2 CS Patient Note

History HPI: 53 yo M c/o intermittent dizziness × 2 days. Sensation of room spinning around him. Occurs during day when getting up or lying down. Episodes last 20–30 minutes and are progressively getting worse. Left-sided hearing loss since yesterday. Nausea and vomiting. Watery, nonbloody diarrhea × 3 days that has since resolved. No tinnitus, fullness in ear, ear discharge, headache, or head trauma. No recent URI. ROS: Negative except as above. Allergies: NKDA. Medications: Furosemide, captopril. PMH: Hypertension, diagnosed 7 years ago. PSH: Appendectomy. SH: No smoking, 2–3 beers/week, no illicit drugs. FH: Noncontributory.

Physical Examination Patient is in no acute distress. VS: WNL, no orthostatic changes. HEENT: NC/AT, PERRLA, EOMI without nystagmus, no papilledema, no cerumen, TMs normal, mouth and oropharynx normal. Heart: RRR; normal S1/S2; no murmurs, rubs, or gallops. Neuro: Cranial nerves: 2–12 grossly intact except for decreased hearing acuity in the left ear. Rinne (air conduction > bone conduction on the left), Weber no lateralization, tilt test. Motor: Strength 5/5 throughout. DTRs: 2+ intact, symmetric, Babinski bilaterally. Cerebellar: Romberg, finger to nose normal. Gait: Normal.

Differential Diagnosis Diagnosis #1: Ménière’s disease History Finding(s): Physical Exam Finding(s): Sensation of room spinning Decreased hearing acuity on the left Left-sided hearing loss Positive Rinne test PRACTICE CASES

337 USMLE STEP 2 CS Patient Note

Diagnosis #2: Benign paroxysmal positional vertigo History Finding(s): Physical Exam Finding(s): Sensation of room spinning Onset with positional changes Duration 20−30 minutes

Diagnosis #3: Orthostatic hypotension causing dizziness History Finding(s): Physical Exam Finding(s): History of diarrhea Taking antihypertensive medication

Diagnostic Workup

Dix-Hallpike maneuver Audiometry MRI—brain PRACTICE CASES

338 PRACTICE CASES 339 This describes transient vertigo following changes in head This describes transient vertigo following Risk factors for dehydration in this case include diarrhea Risk factors for dehydration in this case Used to help diagnose central vestibular disease. Used to document characteristics of nystagmus that may differentiate central from Used to document characteristics of nystagmus that may differentiate central Used to diagnose BPPV (look for nystagmus and reproduction of vertigo). This is a rare cause of vertigo and sensorineural hearing loss that usually results from This is a rare cause of vertigo and sensorineural Acoustic neuroma more commonly causes continuous dysequilibrium rather than episodic Acoustic neuroma more commonly causes This classically presents with episodic This classically presents (usually lasting 1–8 hours) and low- vertigo This frequently follows a viral infection (usually URI) and is accompanied by hearing loss and This frequently follows a viral infection Used to assess hearing function. Required for the evaluation of central vestibular lesions. To rule out syphilis, which can cause Ménière’s disease. Ménière’s rule out syphilis, which can cause To Electronystagmography: peripheral vestibular system lesions. Audiometry: MRI—brain: VDRL/RPR: Brain stem auditory evoked potentials: normal neurologic exam. However, an intracranial mass lesion must be ruled out in any patient with unilateral an intracranial mass lesion must be ruled out in any patient with normal neurologic exam. However, hearing loss. Dix-Hallpike maneuver: tinnitus, but vertigo is usually continuous and lasts several days to a week. tinnitus, but vertigo is usually continuous Perilymphatic fistula: Episodeshead trauma or extensive barotrauma. lasting seconds. of vertigo are fleeting, generally Acoustic neuroma: are unlikely in patients with vertigo, hearing loss, and an otherwise vertigo. As noted above, central lesions and loop diuretic use. However, the patient does not complain of lightheadedness and is not objectively the patient and loop diuretic use. However, orthostatic. Labyrinthitis: fullness. Symptoms result from distention of the endolymphatic compartment of the inner ear. Syphilis and head compartment of the inner ear. result from distention of the endolymphatic fullness. Symptoms causes. trauma are two known (BPPV): positional vertigo Benign paroxysmal loss. position, but it is not associated with hearing Orthostatic hypotension due to dehydration: Ménière’s disease: Ménière’s of aural this case, such as tinnitus and a sensation as well as with features not seen in frequency hearing loss Diagnostic Workup Additional Differential Diagnoses Additional Differential this patient given the presence of hearing loss and an otherwise normal neurologic exam. Vertigo syndromes due to syndromes due normal neurologic exam. Vertigo presence of hearing loss and an otherwise this patient given the and vertigo may by nausea and vomiting, These cases are often accompanied discussed below. peripheral lesions are patient is unable to walk or stand. be so severe that the CASE DISCUSSION CASE Diagnoses Note Differential Patient A are usually nonvestibular in origin. lightheadedness and dysequilibrium signals vestibular disease, whereas Vertigo in and cerebellar tumors, MS) is unlikely brain stem lesion (eg, vertebrobasilar insufficiency, central vestibular system PRACTICE CASES 340 Entrance Building theDoctor-Patient Relationship Examinee Checklist Notes fortheSP Patient isa33yoF, divorcedwith2daughters. Patient Description Checklist/SP Sheet Examinee Tasks HR: RR: Temp: BP: Signs Vital Kathleen Moore,a33-year-old female,comestothecliniccomplainingofkneepain. Opening Scenario DOORWAY INFORMATION CASE 23 physical examinationbeforewecanfigureoutanappropriate courseoftreatment.” “Most likely, butthatdependsontheunderlyingproblemandyourresponsetotreatment.Ineedperforma Sample ExamineeResponse “Do youthinkIwillbeabletowalkonmykneelikebefore?” Challenging QuestionstoAsk Examinee identifiedhis/her role orposition. selfbyname. Examinee introduced Examinee knockedonthedoorbeforeentering. 4. Write thepatientnoteafterleavingroom. 3. Explainyourclinicalimpressionandworkupplantothepatient. 2. Performafocusedphysicalexam(donotperformrectal,genitourinary, orfemalebreastexam). 1. Take afocusedhistory. 130/80mmHg 16/minute 76/minute,regular Paint yourleftkneeredtomakeitlookinflamed. Do notallowtheexamineetofullyflexorextendyourleftknee. Pretend tohavepainwhentheexamineemovesyourleftkneeinalldirections. 99.9°F(37.7°C) PRACTICE CASES 341 Patient Response No. had a painful, swollen big toe on my left foot, but the swelling went had a painful, swollen big toe on my left gave me some medicine. away after the doctor at the urgent clinic seem to come and go. the past 6 months. Sometimes my fingers become pale and then blue when they are exposed to cold weather or cold water. No. breath, cough) Cold temperature causing problems with the fingers Heart symptoms (chest pain, palpita- tions) Pulmonary complaints (shortness of FatigueFever/chillsHair loss I’ve had no energy to work and have felt tired all the time for Yes, but I have no chills. I feel hot now, No. Stiffness in the morning/durationPhotosensitivity for an hour. Yes, RashesOral ulcers No. No. They I had many in my mouth last month, but they’ve resolved now. History of trauma to the kneeOther joint pain No. Duration of the pain in the fingers Six months. painful and stiff. Five years ago I my wrists and fingers are always Yes, RednessSwelling of the jointAlleviating factorsExacerbating factors Yes. bit. help a little Rest and Tylenol Yes. Moving my knee and walking. Chief complaintOnsetFunction Left knee pain. days ago. Two move it. I use a cane to walk. I can’t Question

Examinee summarized the SP’s concerns, often using the SP’s own words. SP’s concerns, often using the the SP’s Examinee summarized efficiently and accurately. Examinee elicited data Examinee correctly used patient’s name. correctly used patient’s Examinee with the SP. made eye contact Examinee to the response. open-ended question and actively listened Examinee asked an to the response without interrupting. SP to list his/her concerns and listened Examinee asked the ✓ Information Gathering Reflective Listening Reflective PRACTICE CASES 342 Connecting withthePatient ✓ Examinee recognizedtheSP’s emotions andrespondedwithPEARLS.

Question rgalrisNo. Yes, Ihadgonorrhea ayearago.Itookantibioticsandwasfine. No. Drug allergies Occasionally. History ofSTDs Yes, apackdayforthepast20years. Menonly. Iamsexuallyactivewithanewboyfriendwhommet2months Vaginal discharge Active withmen,women,orboth No. past year Number ofsexualpartnersduringthe Use ofcondoms No(toall4). Fourdaysago. Idon’t drinkalot,usually2–4beersweekexceptforweekends, Sexual activity Waitress. Tobacco Illicit druguse Mymotherhas rheumatoidarthritisandislivinginanursinghome. Last alcoholingestion CAGE questions Two C-sectionsatages23and25. None. Alcohol use Occupation IusedTylenol torelievemypain,butitisnotworkingaswell Idon’t appetite. haveagood Family history I’velostabout10poundsoverthepast6months. Past surgicalhistory Past medicalhistory Ihave2daughters. BothweredeliveredbyC-section. Two weeks ago. Ihad2spontaneousabortionsalongtimeago. Current medications Appetite changes No. Weight changes No. Last menstrualperiod Miscarriages/abortions Pregnancies No. History ofrecenttickbite Abdominal pain Urinary problems(hematuria) ness, numbness) Neurologic complaints(seizures,weak- anymore. No. Four. ago. when Idon’t count. I don’t knowmyfather. Patient Response PRACTICE CASES

343 ely motion); examined other joints (shoulders, elbows, wrists, hands, other joints (shoulders, elbows, wrists, motion); examined fingers, hips, ankles) Maneuver Follow-up tests: Examinee mentioned the need for a pelvic exam. Follow-up tests: Examinee mentioned Examinee discussed safe sex practices. Abdominal exam Auscultation, palpation, percussion Hair and skin examCV examPulmonary exam Inspection Auscultation Auscultation Mouth examMusculoskeletal exam both knees, including range of Inspection and palpation (compared Inspection Exam Component

Examinee discussed initial management plans: Examinee discussed initial management questions or concerns. Examinee asked if the SP had any other Examinee discussed initial diagnostic impressions. Examinee discussed initial diagnostic Examinee washed his/her hands. Examinee start the exam. asked permission to Examinee draping. Examinee used respectful painful maneuvers. Examinee did not repeat ✓ use regularly, but they are important in helping control the spread of STDs. Do you have any questions for me? use regularly, Sample Closure To be causing your knee pain, such as gout, an infection, or rheumatoid arthritis. Ms. Moore, there are a few things that could your knee and then draw some blood. Sometimes infections from the pelvis can spread find out, I would like to obtain fluid from lik such as your knee, and for that reason I would also like to do a pelvic exam. These tests will to other parts of your body, to always use condoms. I know condoms may be difficult mentioned earlier that you don’t reveal the source of your pain. You Closure Physical Examination Physical PRACTICE CASES 344 Physical Examination History USMLE STEP2CS Patient Note PRACTICE CASES 345 Patient Note Patient Diagnosis #3 History Finding(s): Physical Exam Finding(s): Diagnosis #2 History Finding(s): Physical Exam Finding(s): Diagnosis #1 History Finding(s): Physical Exam Finding(s): USMLE STEP 2 CS STEP USMLE Diagnostic Workup Differential Diagnosis Differential USMLE STEP 2 CS Patient Note

History HPI: 33 yo F c/o left knee pain that started 2 days ago and is causing difficulty walking. She has swelling and redness in her left knee and mild fever but no chills. She denies trauma. She has a history of fatigue and painful wrists and fingers and has experienced 1-hour morning stiffness over the past 6 months. She also recalls multiple oral ulcers that resolved last month. She describes Raynaud’s phenomenon but denies rash, photosensitivity, hair loss, or recent tick bites. She recalls a 10-lb weight loss over the past 6 months and has no appetite. ROS: Negative except as above. Allergies: NKDA. Medications: Tylenol. PMH: Episode of acute left big toe arthritis 5 years ago; gonorrhea 1 year ago. PSH: Two C-sections, 2 spontaneous abortions. SH: 1 PPD for 20 years. Usually drinks 2–4 beers/week; on weekends drinks more; last ingestion 4 days ago; CAGE 0/4. No illicit drugs. Sexually active with multiple partners; inconsistent condom use. FH: Mother has rheumatoid arthritis and lives in a nursing home.

Physical Examination Patient is in no acute distress but favors the left knee. VS: WNL except for low-grade fever. HEENT: No oral lesions. Chest: Clear breath sounds bilaterally. Heart: RRR; normal S1/S2; no murmurs, rubs, or gallops. Abdomen: Soft, nondistended, BS, no hepatosplenomegaly. Extremities: Erythema, tenderness, pain, and restricted range of motion on flexion and extension of left knee compared to right knee. swelling at left knee. Fingers and hands with stiffness bilaterally. Shoulder, elbow, wrist, hip, and ankle joints WNL bilaterally.

Differential Diagnosis Diagnosis #1: Gout History Finding(s): Physical Exam Finding(s): Monoarticular joint pain and tenderness Joint tenderness and stiffness History of swollen toe Swelling at left knee Occasional alcohol use PRACTICE CASES

346 USMLE STEP 2 CS Patient Note

Diagnosis #2: Rheumatoid arthritis History Finding(s): Physical Exam Finding(s): Morning joint stiffness Joint tenderness and stiffness Family history of rheumatoid arthritis Temperature 99.9°F Systemic symptoms (anorexia, weight loss, fatigue, fever)

Diagnosis #3: Systemic lupus erythematosus History Finding(s): Physical Exam Finding(s): Systemic symptoms (anorexia, weight loss, Joint tenderness and stiffness fatigue) History of multiple oral ulcers History of 2 spontaneous abortions Raynaud’s phenomenon

Diagnostic Workup

CBC with differential Immunologic testing (eg, ANA titer, anti- dsDNA, RF, anti-CCP) Knee aspiration with Gram stain, culture, and inspection for crystals XR—left knee and both hands PRACTICE CASES

347 PRACTICE CASES 348 Diagnostic Workup Additional Differential Diagnoses Patient NoteDifferential Diagnoses CASE DISCUSSION suspected asthediagnosis. hashighspecificity(upto96%)forRAandisfrequently sentwhenRAis peptide (anti-CCP)antibody to helpconfirmthediagnosis ofSLE.RFispresentin>75%patientswithRA.Anti−cyclic citrullinated andlupusanticoagulantshouldbeinvestigated againsttheSmithantigen,anticardiolipinantibodies, antibody againstdouble-strandedDNA(anti-dsDNA), essentially excludesthedisease.IfANAispositive,antibody Immunologic tests: septic arthritis. CBC: associated withconstitutionalsymptoms. incongruent withthispatient’s presentation.Also,osteoarthritissparesthewristandMCPjointsisnot Osteoarthritis: commonly causedby Nongonococcal septicarthritis: characteristic rash,whichconsistsofsmallnecroticpustulesontheextremities(includingpalmsandsoles). tenosynovitis) isusuallyprecededbyafewdaysofmigratorypolyarthralgias.Also,thispatientdoesnothavethe bacteria tomutateanddisseminate).Thekneeisthemostfrequentlyinvolvedjoint,butmonoarthritis(or much morelikelythanmentohaveasymptomaticgenitourinarygonococcalinfection,whichallowsthe Gonococcal septicarthritis: usually seeninpatientsolderthan60yearsofage. Pseudogout: testing needstobedonebeforeSLEcandiagnosedinthiscase. orANApositivity.of spontaneousabortionsmaysignalthepresenceantiphospholipidantibodies), More involvement, hematologicdisorders(herfatiguemaybeduetoanemia),immunologicabnormalitieshistory (“butterfly”) rash,discoidsymmetricarthritis,photosensitivity, oralulcers,serositis,renaldisease,CNS knee involvementisnottypical.Thediagnosisrequiresatleastfourofthefollowing11criteria:malar constitutional symptoms,andRaynaud’s phenomenonmaybeearlymanifestationsofthisdisease.Unilateral Systemic lupuserythematosus(SLE): exam. Monoarthritisisalsouncommonbutoccasionallyseenearlyinthecourseofdisease. weight loss,fatigue,andweakness).However, thispatient’s handjointswerenotred,warm,swollen,ortenderon arthritis (eg,fingers,wrists),prolongedmorningstiffness,andsystemicsymptoms(low-gradefever, anorexia, Rheumatoid arthritis(RA): explain herhandarthralgias,butosteoarthritisiscommonandmaycoexist. and mayprecipitateanacuteattack.Foot,ankle,kneejointsarealsocommonlyaffected.Goutdoesnot the patient’s isclassicforgout.Alcoholingestioncauseshyperuricemia historyoffirstMTParthritis(“podagra”) Gout: use arekeyriskfactorsnotpresentinthiscase. To lookforanemia,leukopenia,and/orthrombocytopeniainSLEorleukocytosisacutegout and Ti acute,usuallymonoarticular, crystal-inducedarthritisrarelyoccursinpremenopausalwomen,but This Anothercrystal-inducedarthritis,pseudogoutfrequentlyinvolvesthekneesandwristsbutis Onsetisinsidious,jointstiffnessbrief,andinflammationminimal,allofwhichare S aureus. ANAisahighlysensitivebutnonspecificscreeningtest forSLE.Anegativetest Thisissuggestedinapatientwithpositivefamilyhistory, symmetricsmalljoint However, itisadiseaseofanabnormalhost;previousjointdamage andIVdrug Thisoccursinhealthyhosts,mostcommonlyyoungwomen(womenare Thisoccurssuddenly, usuallyaffectsthekneeorwrist,andismost Joint symptoms(usuallysymmetricperipheralarthralgias), PRACTICE CASES 349 In most cases of acute monoarthritis, of acute monoarthritis, In most cases Necessary to investigate gonococcal infection and often positive in the gonococcal infection and often Necessary to investigate Specific changes in RA include symmetric joint space narrowing, marginal RA include symmetric joint space narrowing, Specific changes in An important test in septic arthritis if systemic symptoms are present. An important test in septic arthritis if Pelvic exam and cervical cultures: Pelvic exam and may also be necessary). (urine, anorectal, and throat cultures absence of symptoms Blood culture: XR—left knee and both hands: XR—left knee and during the first six months x-rays are usually normal periarticular demineralization. However, bony erosions, and look for and a sclerotic joint margin. In pseudogout, for punched-out cortical erosions of illness. In gout, look marginal look for joint space narrowing, (“chondrocalcinosis”). In osteoarthritis, calcified articular cartilage subchondral cysts. osteosclerosis, and occasionally osteophytes, subchondral Knee aspiration, Gram stain, culture, and inspection for crystals: and inspection culture, Gram stain, Knee aspiration, fluid contains joint synovial arthritis. Inflammatory to rule out septic must be performed joint aspiration > of needle-shaped, The demonstration > 50,000 cells/μL. fluid often contains and septic joint 3000 WBCs/μL, or confirms gout birefringent crystals weakly positively or rhomboid-shaped, birefringent crystals negatively pseudogout, respectively. PRACTICE CASES 350 Entrance Building theDoctor-Patient Relationship Examinee Checklist Notes fortheSP Patient isa31yoM. Patient Description Checklist/SP Sheet Examinee Tasks HR: RR: Temp: BP: Signs Vital Will Foreman,a31-year-old male,comestohisprimarycarephysiciancomplainingofheelpain. Opening Scenario DOORWAY INFORMATION CASE 24 of yourtreatment.” “First weneedtodoacompleteevaluationdetermine thecauseofyourpain.Thenwecandiscussnature Sample ExamineeResponse “Doctor, canyoujustgivemesomepowerfulpainmeds sothatIcankeeprunning?amtrainingforamarathon.” Challenging QuestionstoAsk Examinee introduced selfbyname. Examinee introduced Examinee knockedonthedoor beforeentering. 4. Write thepatientnoteafterleavingroom. 3. Explainyourclinicalimpressionandworkupplantothepatient. 2. Performafocusedphysicalexam(donotperformrectal,genitourinary, orfemalebreastexam). 1. Take afocusedhistory. 125/80mmHg 14/minute 69/minute,regular Give theappearanceofpainwithfirstfewstepsyoutakeaftersitting. Exhibit painwhentheexamineepalpatesarchofyourrightfootandbottomheel. extends yourtoes(movesthemup). Pretend tohavepainonthebottomofyourrightheelandintoarchfootwhenexaminee 99.0°F(37.2°C) PRACTICE CASES 351 Patient Response No. No. time. No. Past medical history (be sure to address diabetes, rheumatologic disorders, and cancer) Previous injury to your feet or ankles.Constitutional symptoms (nausea/vom- No. iting, weight/appetite changes, fever/ chills, diarrhea/constipation, fatigue) Current medications Occasionally I take ibuprofen for the pain. Exacerbating factorsOther joint painPrevious episodes of similar pain barefoot or walking after sitting for a prolonged period of Walking No. No. down from the back) QualityBurning, tingling, numbnessAlleviating factors No. Massaging the arch of my foot and applying ice. Stretching/tearing pain. When does it hurt in the morning?Progression It hurts the most with the first few steps I take after I get out of bed. Severity on a scaleRadiationRadiation proximally (up the leg or a 7/10. When it hurts, it can get up to It has stayed about the same. It occasionally radiates into the arch of my foot. OnsetPrecipitating eventsConstant/intermittentFrequency but I have been training for a marathon. Not really, Intermittent. It came on gradually over the past 2 weeks. It seems to be worse in the morning. hurts every day. It usually Chief complaintLocation Right heel pain. It hurts the most at my heel. Question

Examinee asked the SP to list his/her concerns and listened to the response without interrupting. SP to list his/her concerns and listened Examinee asked the own words. SP’s concerns, often using the the SP’s Examinee summarized efficiently and accurately. Examinee elicited data Examinee identified his/her role or position. identified his/her Examinee name. correctly used patient’s Examinee with the SP. made eye contact Examinee to the response. open-ended question and actively listened Examinee asked an ✓ Information Gathering Reflective Listening PRACTICE CASES 352 Physical Examination Connecting withthePatient ✓ ✓ Examinee didnotrepeatpainfulmaneuvers. Examinee usedrespectfuldraping. Examinee askedpermissiontostarttheexam. Examinee washedhis/herhands. Examinee recognizedtheSP’s emotionsandrespondedwithPEARLS.

ExamComponent Question boia xmAuscultation,palpation Auscultation Auscultation,distalpulses(posteriortibialis,dorsalis pedis), Extremities Abdominal exam Pulmonary exam CV exam No. No. Iamsexuallyactive withmywifeof10years. No. Ihaveapproximately1–2beersaweek. Runner. Iworkasanaccountant. Drug allergies Sexual activity Tobacco Illicit druguse Alcohol use None. Avocation Occupation cancer) diabetes, rheumatologicdisorders,and Family history(besuretoaddress Past surgicalhistory erlgcea Checkedsensation tolighttouchfordermatomesoffootandankle; Neurologic exam My fatherhasarthritis. capillary refillofthetoes assessed Achillestendonreflex Patient Response Maneuver motion; strengthtestingofankledorsiflexionandplantar flexion Ankle dorsiflexionandgreattoeextensionpassiverange of and hip Passive rangeofmotionandgeneralstrengthipsilateral knee fascia, retrocalcanealbursae Palpation ofmedialcalcanealtuberosity, Achillestendon,plantar bearing, andwithambulation Inspection ofbothfeetandankles—non–weightbearing,weight PRACTICE CASES

353 on Follow-up tests: X-ray of right ankle. Follow-up tests: X-ray Examinee discussed initial diagnostic impressions. discussed initial diagnostic Examinee plans: discussed initial management Examinee SP had any other questions or concerns. Examinee asked if the barefoot on hard surfaces until this improves. We will get an x-ray today to help confirm that there is no obvious fracture or that there is no obvious fracture or will get an x-ray today to help confirm until this improves. We barefoot on hard surfaces spurs. If you would like, I can send you to physical therapy to help you get started foreign body and to look for possible bone to responsive to this treatment over the next 2 months, we may consider a bone scan these exercises. If your symptoms are not questions for me? rule out a stress fracture. Do you have any Sample Closure of pain on the bottom fasciitis, which is the most common cause likely cause of your heel pain is plantar Foreman, the most Mr. massage, NSAIDs, treatment consisting of stretching, resolves over a few months, with conservative of the heel. It typically do and avoid walking you decrease the amount of running you activities. I would highly suggest that and avoidance of painful Closure PRACTICE CASES 354 Physical Examination History USMLE STEP2CS Patient Note PRACTICE CASES 355 Patient Note Patient Diagnosis #3 History Finding(s): Physical Exam Finding(s): Diagnosis #2 History Finding(s): Physical Exam Finding(s): Diagnosis #1 History Finding(s): Physical Exam Finding(s): USMLE STEP 2 CS STEP USMLE Diagnostic Workup Differential Diagnosis Differential USMLE STEP 2 CS Patient Note

History HPI: 31 yo M c/o pain on the plantar surface of his right heel. The pain started gradually about 2 weeks ago and has not progressed. The patient denies trauma or a specific inciting event but admits to training for a marathon. He describes the pain as intermittent and states that it is worse after getting out of bed in the morning and after prolonged sitting. He reports that the pain has a tearing/stretching quality and that it can get as high as 7/10. He has used ice, massage, and occasional ibuprofen for the pain, with limited relief. The patient denies any tingling, burning, or numbness. He denies proximally radiating symptoms but does report occasional pain radiating into his arch. ROS: Denies nausea/vomiting, weight/appetite changes, fever/chills, diarrhea/constipation, or fatigue. Allergies: NKDA. Medications: Occasional ibuprofen. PMH: None. Denies cancer, rheumatologic disorders, or diabetes. PSH: None. SH: No smoking, 1–2 beers/week, no illicit drugs. Works as an accountant; sexually active with wife of 10 years. Marathon runner. FH: Father with arthritis. Denies FH of cancer, rheumatologic disorders, or diabetes.

Physical Examination Patient is pleasant and in no acute distress. VS: WNL. Chest: Clear to auscultation bilaterally. Heart: RRR; normal S1/S2; no murmurs, rubs, or gallops. Abdomen: Soft, nontender, nondistended, BS. Extremities: Posterior tibialis and dorsalis pedis pulses 2+ bilaterally; mild bilateral rear/midfoot pronation; range of motion of hip/knee/ankle and foot WNL. Tender to palpation over medial calcaneal tuberosity and plantar fascia; plantar heel and arch pain with dorsiflexion of toes. Neuro: Motor: Strength 5/5 in hip/knee/ankle and foot. Sensation: Intact to light tough in saphenous, sural, and deep/superficial peroneal nerve distributions (dermatomes L4−S1). DTRs: 1+ in Achilles tendon. Gait: Non-antalgic gait pattern.

Differential Diagnosis Diagnosis #1: Plantar fasciitis History Finding(s): Physical Exam Finding(s): Training for a marathon Tenderness over medial calcaneal tuberosity Pain is gradual Pain with toe dorsiflexion Pain worsens with first few steps in morning and after prolonged sitting PRACTICE CASES

356 USMLE STEP 2 CS Patient Note

Diagnosis #2: Calcaneal stress fracture History Finding(s): Physical Exam Finding(s): Training for a marathon Tenderness over plantar heel and arch Diffuse pain over heel Refractory to conservative management

Diagnosis #3: Achilles tendinitis History Finding(s): Physical Exam Finding(s): Training for a marathon Pain with toe dorsiflexion

Diagnostic Workup

XR—right ankle/foot Bone scan MRI—right ankle/foot PRACTICE CASES

357 PRACTICE CASES 358 Additional Differential Diagnoses the preciselocationofsymptoms,asthisisfirststepindeterminingmostlikelydiagnosis. Heel paininadultscanbecausedbyseveraldistinctentities.Forthisreason,itisessentialthattheexaminerascertain Patient NoteDifferential Diagnoses CASE DISCUSSION over theinvolvedligaments and increasedlaxityonstresstesting.Significantedema and ecchymosisareoften “rolling” theankle,oftenin the plantarflexedandinvertedposition.Examinationreveals tendernesstopalpation ligament complexmostcommonlyinvolved.Patientstypically describeaninjurypatternconsistentwith Ankle sprain: topical antimicrobials,andappropriatedressing. removal, induration, orafluctuantmassshouldalsobesought.Conservative managementincludesforeignbody patient mayornotdescribeamechanismofinjury. Signsoflocalinfectionsuchaswarmth,erythema,pain, Foreign body: Conservative managementincludesanalgesicsandcorrection offootmechanicswithorthotics. Symptoms maybeexacerbatedbypercussionofthetarsal tunnelorwithdorsiflexionandeversionofthefoot. pain, burning,tingling,ornumbnessthatcanradiatetotheplantarheelandevendistalsoletoes. retinaculum traversingoverthetalusandcalcaneus.Compressionoftibialnerveintunnelcanleadto Tarsal tunnelsyndrome: includes analgesics,propershoewear, andheelpadding. on theposterosuperioraspectofcalcaneus),whichmayexacerbatecondition.Conservativemanagement footwear thathasapoorlyfitting,rigidheelcup.ItcanalsobeassociatedwithHaglund’s deformity(abonyspur Achilles tendonandbetweentheskin.Theconditioniscommonlycausedbyill-fitting chronic irritationoftheunderlyingbursae.Thebursaearelocatedbetweenposteriorcalcaneusand Retrocalcaneal bursitis: Conservative managementincludesrest,analgesics,andstretching/strengtheningexercises. passive dorsiflexionoftheankle.Again,thisconditioniscommonlyduetooveruseorpoorbiomechanics. alongtheAchillestendonarecommon.Painmayalsoincreasewith to palpation,swelling,andnodules Achilles tendoninsertionsiteorontheitselfduringrunning,jumping,andharshactivities.Tenderness Achilles tendinitis: to conservativemanagement.Follow-updiagnostictesting(eg,x-ray, bonescan)maythenbewarranted. lateral compression.Acalcanealstressfracturemaybeconsideredinthispatientifhissymptomsproverefractory who haveriskfactorsforosteopenia.Patientstypicallydiffuseheelpainthatismadeworsebymedialand foot. Stressfracturesarecommoninathleteswhoinvolvedrunningorjumpingsportsaswellpatients Calcaneal stress fracture: Conservative managementincludesanalgesics,stretching,exercise,orthotics,andnightsplinting. marked tendernessoverthemedialcalcanealtuberosityandincreasedpainwithpassivedorsiflexionoftoes. in onsetandworsewiththefirstfewstepsmorningafterprolongedsitting.Examinationreveals joint mobility, andatightAchillestendoncanallpredispose toplantarfasciitis.Thepainistypicallygradual from repetitiveuseorexcessiveloading(eg,trainingforamarathon).Pesplanus,pescavus,decreasedsubtalar Plantar fasciitis: Ankleligamentinjuriesarethemostcommonmusculoskeletal injury, withthelateralcollateral If a foreign body issuspected,thefootshouldbeinspected forsignsofanentrancewound.The Ifaforeignbody Themostcommoncauseofplantarheelpaininadults,fasciitistypicallyresults PatientswithAchillestendinitistypicallycomplainofposteriorheelpaineitheronthe Patientswiththisconditionusuallycomplainofposteriorheelpainsecondaryto Thetarsaltunnelisonthemedialaspectofheelandformedbyflexor Thecalcaneusissecondonlytothemetatarsalsintermsofstressfractures PRACTICE CASES 359 Reserved for suspected soft tissue involvement, which could include the degree of Reserved for suspected soft tissue involvement, X-rays in this region may demonstrate calcaneal spur formation (calcification) at the may demonstrate calcaneal spur formation X-rays in this region If conservative treatment fails in this patient, follow-up with a bone scan is recommended in two with a bone scan is recommended in fails in this patient, follow-up If conservative treatment months to rule out calcaneal stress fracture, as would be demonstrated by an increased area of uptake. demonstrated by an increased area of calcaneal stress fracture, as would be months to rule out MRI—right ankle/foot: of the Achilles tendon, or articular cartilage defects. Achilles tendon degeneration, rupture proximal plantar fascia (as in this patient) or at the Achilles tendon insertion. Care must be taken to correlate tendon insertion. Care must be taken (as in this patient) or at the Achilles proximal plantar fascia be seen in as such calcification can also symptoms and with the physical examination, these findings with may also deformity) calcaneus (Haglund’s Increased prominence of the posterosuperior asymptomatic patients. be demonstrated. Bone scan: present in the acute/subacute stages. Conservative treatment involves rest, ice, compression, elevation, NSAIDs, elevation, rest, ice, compression, treatment involves stages. Conservative the acute/subacute present in and bracing. XR—right ankle/foot: Diagnostic Workup Diagnostic PRACTICE CASES 360 Reflective Listening Entrance Building theDoctor-Patient Relationship Examinee Checklist Challenging QuestionstoAsk Show concernregardingyourchild’s situation. Notes fortheSP The patient’s motheroffersthehistory;childisathome. Patient Description Checklist/SP Sheet Examinee Tasks a fever. The motherofMariaSterling,an18-month-oldfemalechild,comestotheofficecomplainingthatherchildhas Opening Scenario DOORWAY INFORMATION CASE 25 need toexamineherhereintheofficeandthendecidewhethersheneedsanytestsand/ortreatment.” “You certainlydidtherightthingbycomingintoday. Mariamayhave aninfectionthatiscausingherfever, sowe Sample ExamineeResponse Examinee summarizedtheSP’s concerns,oftenusingtheSP’s ownwords. Examinee askedtheSPtolist his/herconcernsandlistenedtotheresponsewithout interrupting. Examinee askedanopen-endedquestionandactivelylistened totheresponse. Examinee madeeyecontactwiththeSP. Examinee correctlyusedpatient’s name. Examinee identifiedhis/herroleorposition. selfbyname. Examinee introduced Examinee knockedonthedoorbeforeentering. 3. Write thepatientnoteafterleavingroom. 2. Explainyourclinicalimpressionandworkupplantothemother. 1. Take afocusedhistory. “Is mychildgoingtobeokay?” “Do youthinkthatIdidtherightthingbycominghereandtellingaboutmychild’s fever?” PRACTICE CASES 361 and belly. No. No. her toys todayShe looks tired. She is not playing with and is not watching TV the way she usually does. Normal. Patient Response week ago. face. There is no rash on her arms or legs. Sleeping habitsHearing problems She has not slept well for 2 days. No. Birth historyChild weight, height, and language development Eating habits It was a 40-week vaginal delivery with no complications. solid food; Whole milk and I did not breast-feed my child. Day care centerIll contacts in day care centerVaccinationsLast checkup know. I don’t Yes. Up to date. One month ago, and everything was normal. Shaking (seizures)How has the child looked (lethargic, irritated, playful, etc.)? Appetite changesIll contacts No. She is not eating much but is able to drink milk. No. Nausea/vomitingChange in bowel habits or in stool color or consistency Change in urinary habits or in urine smell or color she had an episode of vomiting last night Yes, Description of the rashOnset of rash and progression and red dots, some slightly elevated, over the chest, back, belly, Tiny ago on her face and then spread to her chest, back, It started 2 days Ear pulling/ear dischargeCoughShortness of breathDifficulty swallowing she has been pulling at her right ear for 2 days. Yes, Rash No. not sure. She seems to have trouble swallowing, but I’m but she was coughing for a few days about a week ago. Not currently, she has a rash on her face and chest. Yes, OnsetTemperatureRunny nose I measured it, and it was 101°F on her forehead. days ago. Two but she did have a runny nose for a few days about a Not currently, Chief complaint My child has a fever. Question

Examinee elicited data efficiently and accurately. elicited data efficiently Examinee ✓ Information Gathering Information PRACTICE CASES 362 infections requireantibiotics;however, suchinfectionsgenerallyrespond well totreatment.Doyouhaveanyquestionsforme? of infectionandthetypevirusorbacteriainvolved.Althoughviralinfectionsgenerallyclearontheirown,mostbacterial from anearinfectionorsomethingmoreserious.Aphysicalexamandsomebloodtestswillbeneededtoidentifythesource Mrs. Sterling,itappearsthatyourchildissufferingfromaninfectionmaybeviralorbacterial.She Sample Closure Closure None. Physical Examination Connecting withthePatient ✓ Examinee askediftheSPhadanyotherquestionsorconcerns. Examinee discussedinitialmanagementplans: Examinee discussedinitialdiagnosticimpressions. Examinee recognizedtheSP’s emotionsandrespondedwithPEARLS.

Question rgalrisNo. None. Threemonthsagoshehadanearinfectionthatwastreated Tylenol. No. Drug allergies Past surgicalhistory Past medicalhistory Current medications Vision problems Follow-up tests. successfully withamoxicillin. Patient Response PRACTICE CASES 363 Patient Note Patient USMLE STEP 2 CS STEP USMLE Physical Examination History USMLE STEP 2 CS Patient Note

Differential Diagnosis Diagnosis #1 History Finding(s): Physical Exam Finding(s):

Diagnosis #2 History Finding(s): Physical Exam Finding(s):

Diagnosis #3 History Finding(s): Physical Exam Finding(s):

Diagnostic Workup PRACTICE CASES

364 USMLE STEP 2 CS Patient Note

History HPI: History obtained from mother. Patient is 18-month-old F with fever × 2 days. Temperature recorded at home, 101°F. Tired and not playing with toys or watching TV as usual. Pulling at right ear. Difficulty swallowing and sleeping × 2 days. Loss of appetite. One episode of vomiting. Maculopapular facial rash that spread over the chest, back, and abdomen, sparing the arms and legs. Attends day care center, no known history of sick contacts. No ear discharge. History of cough and runny nose for a few days last week. ROS: Negative except as above. Allergies: NKDA. Medications: Tylenol. PMH: Otitis media 3 months ago, treated with amoxicillin. Birth history: 40-week vaginal delivery with no complications. Dietary history: Formula milk and solid food. She was not breast-fed. Immunization history: UTD. Developmental history: Last checkup was 1 month ago and showed normal weight, height, hearing, vision, and developmental milestones.

Physical Examination None.

Differential Diagnosis Diagnosis #1: Acute otitis media History Finding(s): Physical Exam Finding(s): Fever (101°F) Pulling at right ear; fatigued and not watching TV as usual History of otitis media Runny nose and cough that have subsided PRACTICE CASES

365 USMLE STEP 2 CS Patient Note

Diagnosis #2: Meningococcal meningitis History Finding(s): Physical Exam Finding(s): Maculopapular facial rash that spread to the chest, back, and abdomen Fever (101°F) Difficulty sleeping for 2 days Recent episode of vomiting

Diagnosis #3: Scarlet fever History Finding(s): Physical Exam Finding(s): Maculopapular facial rash that spread to the chest, back, and abdomen Fever (101°F) Difficulty swallowing for 2 days

Diagnostic Workup

Pneumatic otoscopy LP—CSF analysis CBC with differential, blood culture, UA and urine culture Throat culture Platelets, PT/PTT, D-dimer, fibrin split products, fibrinogen PRACTICE CASES

366 PRACTICE CASES 367 and to screen infection can lead to Neisseria meningitidis To isolate To Streptococcus pyogenes In children, viruses commonly present with low-grade fever and In children, viruses commonly present which causes scarlet fever. The rash is pathognomonic for this diagnosis. which causes scarlet fever. Fever, lethargy, and a possible petechial rash suggest meningococcemia. Patients petechial rash suggest meningococcemia. and a possible lethargy, Fever, S pyogenes, Key to look for the tympanic membrane (TM) erythema and decreased mobility seen in Key to look for the tympanic membrane (TM) erythema and decreased mobility Infections of the middle ear are more common in younger children because of their ear are more common in younger Infections of the middle Should be performed if there is any concern for meningitis. CSF analysis includes cell count Should be performed if there is any concern for meningitis. CSF analysis includes To isolate To This patient has fever, difficulty swallowing (possible pharyngitis), and a rash that started on difficulty swallowing This patient has fever, Fever and rash, along with day care attendance, are consistent with this infection. In varicella, lesions Fever and rash, along with day care attendance, are consistent with this infection. and the rash is intensely pruritic. Because the patient’s immunizations are up to date, it is unlikely that she has and the rash is intensely pruritic. Because the patient’s varicella. are present in various stages of development at any given time (eg, red macules, vesicles, pustules, crusting), are present in various stages of development at any given time (eg, red macules, for occult bacteremia or UTI. culture: Throat otitis media. LP—CSF analysis: common bacterial antigens, and and differential, glucose, protein, Gram stain, culture, latex agglutination for occasionally PCR for specific viruses. UA and urine culture: blood culture, CBC with differential, Pneumatic otoscopy: accompanied by rash in a pediatric patient, and it is not always necessary to ascertain which virus is causing accompanied by rash in a pediatric patient, particularly troublesome, viral cultures, molecular tests (PCR), and/or the illness. If the illness is prolonged or antibody titers can be ordered to determine the exact etiology. Varicella: Fifth disease or other viral exanthem: in their appearance and are usually maculopapular and rash. In general, viral exanthems are nonspecific disease, usually presents as intense red facial flushing (a “slapped diffuse. Parvovirus B19 infection, or fifth can be almost any virus the trunk and becomes more diffuse. However, cheek” appearance) that spreads over her face and spread to her trunk. However, the history does not indicate whether the rash consists of a diffuse her face and spread to her trunk. However, around the mouth. In addition, scarlet elevations that spare the area erythema with punctate, sandpaper-like she may the patient does attend day care, where children. However, fever is more common among school-age contacts. Left untreated, potentially have been exposed to sick would aid in identifying this illness. rheumatic heart disease. A throat culture may also have headache, vomiting, photophobia, neck stiffness, and seizures. This patient had a single episode and seizures. This patient had vomiting, photophobia, neck stiffness, may also have headache, not given meningococcal vaccinations are typically her immunizations are up to date, of vomiting. Although is 18 months, the patient would not yet have been immunized. Treatment until 11−12 years of age; therefore, at is a severe, rapidly progressive, and sometimes fatal infection. critical, as meningococcal meningitis Scarlet fever: irritability suggest this diagnosis but are not always present. Upper respiratory viral infection is a common risk Upper respiratory viral infection is diagnosis but are not always present. irritability suggest this both of a recent history of cough and runny nose, acute otitis media. This patient has factor for developing more prone to with a prior history of otitis media are of a viral URI. In addition, patients which are suggestive having another episode. Meningococcal meningitis: Acute otitis media: hearing loss, and general otalgia, loss of appetite, temporary Eustachian tubes. Fever, shorter and more horizontal Diagnostic Workup Additional Differential Diagnoses Additional Differential CASE DISCUSSION CASE Diagnoses Note Differential Patient PRACTICE CASES 368 acute infection). Varicella antibodytiter: cells invaricellainfection. lesions. ATzancksmear(moreofahistorictestandnolongerrecommended)mayshowmultinucleatedgiant Skin lesionscrapings: Parvovirus B19IgMantibody: media. Tympanometry: Platelets, PT/PTT, meningococcemia. Usefulininfantsolderthansixmonthsofage;confirmsabnormalTMmobilityotitis D -dimer, fibrinsplitproducts, fibrinogen: Varicella antigensareidentifiedbyPCRordirectimmunofluorescence (DFA) ofskin May be useful in uncertain cases (look for a fourfold rise in antibody titerfollowing Maybeusefulinuncertaincases(lookforafourfoldriseantibody Thebestmarkerofacuteorrecentinfectioninsuspectedfifthdisease. EvidenceofDICisoftenseenin PRACTICE CASES 369 99.5°F (37.5°C) Cough as the examinee enters the room. during the encounter. Continue coughing every 3–4 minutes to the examinee unless he/she asks show it to simulate blood.Hold a red-stained tissue in your hand Don’t you. pretend to have a severe attack of coughing. Note whether the examinee offers you a During the encounter, glass of water or a tissue. 75/minute, regular 15/minute 120/80 mm Hg 2. Perform a focused physical exam (do not perform rectal, genitourinary, or female breast exam). not perform rectal, genitourinary, 2. Perform a focused physical exam (do workup plan to the patient. 3. Explain your clinical impression and the patient note after leaving the room. 4. Write 1. Take a focused history. a focused 1. Take “Well, we still have to sort out exactly what is causing your cough. If you stop smoking, your chronic cough should “Well, significantly decrease your cancer improve. But regardless of what is causing your cough, smoking cessation will risk in the long term.” Challenging Questions to Ask I get better if I stop smoking?” “Will Sample Examinee Response DOORWAY INFORMATION DOORWAY CASE 26 CASE Checklist/SP Sheet Patient Description Patient is a 54 yo F. Notes for the SP RR: HR: Examinee Tasks Marilyn McLean, a 54-year-old female, comes to the office complaining of persistent cough. female, comes to the 54-year-old Marilyn McLean, a Vital Signs BP: Temp: Opening Scenario PRACTICE CASES 370 Information Gathering Reflective Listening Entrance Building theDoctor-Patient Relationship Examinee Checklist ✓ Examinee offeredtheSPaglassofwaterortissueduringsevereboutcoughing. Examinee eliciteddataefficientlyandaccurately. Examinee summarizedtheSP’s concerns,oftenusingtheSP’s ownwords. Examinee askedtheSPtolisthis/herconcernsandlistenedresponsewithoutinterrupting. Examinee askedanopen-endedquestionandactivelylistenedtotheresponse. Examinee madeeyecontactwiththeSP. Examinee correctlyusedpatient’s name. Examinee identifiedhis/herroleorposition. selfbyname. Examinee introduced Examinee knockedonthedoorbeforeentering.

Question hs anNo. Yes. Mildfever, especially atnight.Ididn’t takemytemperature.Ihave Yes, recently. Streaks. None. Chest pain Thickandviscous. None. Yellowish mucus. Night sweats Two teaspoonfuls,stable. Fever/chills Preceding symptoms/events Amount ofblood Blood Yes. Consistency Odor Yes, sometimes Ican’t sleepbecauseofit. Color I’vehadacoughforyears,especiallyinthemorning. Thispast Nothing. Amount Sputum production Alleviating/exacerbating factors Persistentcough. Do youcoughatnight? past month No. Progression ofthecoughduring Changes inthecoughduringday Onset Chief complaint had nochills. It isgettingworse. month, thecoughhasgottenworse,anditisreallyannoyingme. Patient Response PRACTICE CASES 371 Patient Response Patient treatment for TB. negative. None. DurationAmountSexual activityDrug allergies for the past 35 years. I’ve smoked my husband. With One to two packs a day. No. OccupationAlcohol useIllicit drug useTobacco aide. Nurse’s None. Never. No, I stopped smoking 2 weeks ago. Current medicationsPast medical historyPast surgical historyFamily history albuterol inhaler. multivitamins, “over the counter,” Cough syrup bronchitis. Chronic age 11. and adenoidectomy, Tonsillectomy of old age. My mother is alive and has Alzheimer’s. My father died Appetite changes changesWeight FatigueSince when I no longer have an appetite. Yes, without intending to. I’ve lost 6 pounds in the past 2 months have the energy that I had before. I don’t months ago. Yes, Two Recent travelLast PPD (wheezing, Associated symptoms diar- abdominal pain, nausea/vomiting, rhea/constipation) None. before I started working in the nursing home. It was Last year, Shortness of breathShortness to TBExposure when I walk up the stairs. Yes, a nursing home, and several of our residents are under I work in Yes, Question

Examinee washed his/her hands. Examinee asked permission to start the exam. Examinee used respectful draping. Examinee did not repeat painful maneuvers. Examinee recognized the SP’s emotions and responded with PEARLS. Examinee recognized the SP’s ✓ Connecting with the Patient Physical Examination PRACTICE CASES 372 questions forme? that youadheretostandardrespiratoryprecautionswhileworkingwithpatientswhoareinfectedTB.Dohave any a nursinghomeputsyouatriskforacquiringtuberculosis,sowearegoingtotestthataswell.Iwouldalsorecommen of yourcough.Inaddition,wemayfinditnecessarytoconductmoresophisticatedtestsinthefuture.Thefactthatyouwork i more serious,suchascancer. We willneedtoobtainsomebloodandsputumtestsaswellachestx-rayidentifythesourc Mrs. McLean,yourcoughmaybeduetoalunginfectionthatcantreatedwithantibiotics,oritresultfromsomething Sample Closure Closure ✓ Examinee askediftheSPhadanyotherquestionsorconcerns. Examinee discussedinitialmanagementplans: Examinee discussedinitialdiagnosticimpressions.

ExamComponent xrmte Inspection Auscultation,palpation Auscultation Auscultation,palpation,percussion Inspectedmouth,throat,lymphnodes Extremities Abdominal exam Pulmonary exam CV exam Head andneckexam Follow-up tests. Maneuver n d e PRACTICE CASES 373 Patient Note Patient USMLE STEP 2 CS STEP USMLE Physical Examination History USMLE STEP 2 CS Patient Note

Differential Diagnosis Diagnosis #1 History Finding(s): Physical Exam Finding(s):

Diagnosis #2 History Finding(s): Physical Exam Finding(s):

Diagnosis #3 History Finding(s): Physical Exam Finding(s):

Diagnostic Workup PRACTICE CASES

374 USMLE STEP 2 CS Patient Note

History HPI: 54 yo F with PMH of chronic bronchitis c/o worsening cough × 1 month. Chronic cough for years. 2 teaspoons of yellowish phlegm with streaks of blood. Dyspnea on exertion. Fever and sweats at night. Fatigue. Decreased appetite, 6-lb unintentional weight loss over 2 months. Exposure to TB as nurse’s aide working in nursing home. Last PPD: 1 year ago and negative. No chest pain, chills, or wheezing. No recent travel. ROS: Negative except as above. Allergies: NKDA. Medications: OTC cough syrup, multivitamins, albuterol inhaler. PMH: Per HPI. PSH: Tonsillectomy and adenoidectomy, age 11. SH: 1–2 PPD for 35 years; stopped smoking 2 weeks ago. No EtOH. Sexually active with husband only. FH: Noncontributory.

Physical Examination Patient is in no acute distress. VS: WNL. HEENT: Mouth and pharynx WNL. Neck: No JVD, no lymphadenopathy. Chest: Clear breath sounds bilaterally; no rhonchi, rales, or wheezing; tactile fremitus normal. Heart: Apical impulse not displaced; RRR; normal S1/S2; no murmurs, rubs, or gallops. Abdomen: Soft, nontender, BS, no hepatosplenomegaly. Extremities: No clubbing, cyanosis, or edema.

Differential Diagnosis Diagnosis #1: Pulmonary tuberculosis History Finding(s): Physical Exam Finding(s): Fever and night sweats with fatigue; worsening Blood-tinged mucus cough of 1 month’s duration

Close contact with patients with active TB PRACTICE CASES Decreased appetite with unintentional weight loss of 6 lbs over 2 months

375 USMLE STEP 2 CS Patient Note

Diagnosis #2: Lung cancer History Finding(s): Physical Exam Finding(s): Fever and night sweats with fatigue Blood-tinged mucus Decreased appetite with unintentional weight loss of 6 lbs over 2 months History of heavy smoking and chronic bronchitis

Diagnosis #3: Typical pneumonia History Finding(s): Physical Exam Finding(s): Fever Sputum production Mucus production History of heavy smoking and chronic bronchitis

Diagnostic Workup

PPD or QuantiFERON Gold CBC Blood cultures Sputum Gram stain, AFB smear, routine and mycobacterial sputum cultures, and cytology CXR—PA and lateral CT—chest Bronchoscopy Lung biopsy PRACTICE CASES

376 PRACTICE CASES 377 To Legionella and M tuberculosis Mycobacterium 5 mm of induration. 5 mm of induration. ≥ The PPD test is a screening tool for Mycoplasma pneumoniae, Chlamydia pneumoniae, Clinical suspicion is high for this diagnosis given the patient’s constitutional is high for this diagnosis given the patient’s Clinical suspicion infection, and severe ear pain due to bullous myringitis may complicate up to 5% infection, and severe ear pain due to Refers to infection by This patient’s baseline productive bronchitis secondary cough is due to COPD/chronic This patient’s Classic bacterial pneumonia begins with abrupt onset of fever, chills, pleuritic chest pain, chills, pleuritic fever, begins with abrupt onset of Classic bacterial pneumonia Other common, benign causes of chronic cough include postnasal drip, GERD, asthma, and Other common, benign causes of chronic cough include postnasal drip, GERD, May be useful in severe pneumonia to identify causative pathogenic bacteria. Legionella A lung abscess due to anaerobic bacteria is usually associated with gradual onset of fatigue, fever, is usually associated with gradual onset of fatigue, fever, A lung abscess due to anaerobic bacteria As noted above, constitutional symptoms and hemoptysis in a long-time smoker are worrisome in a long-time smoker are worrisome symptoms and hemoptysis As noted above, constitutional infections. The presence of weight loss, night sweats, and productive infections. The presence of weight loss, cough makes atypical infection. The QuantiFERON Gold test is a newer and more specific test for prior infection. The QuantiFERON Gold test is a newer and more specific test for To identify leukocytosis in infection (nonspecific). To Mycoplasma infection, but its availability varies depending on the testing center. CBC: Blood cultures: and cytology: and mycobacterial sputum cultures, routine Sputum Gram stain, AFB smear, identify a causative agent of infection or to help detect malignancy. ACE inhibitors. skin test) or QuantiFERON Gold: PPD (tuberculin tuberculosis COPD exacerbation: chronic bronchitis are more acute and involve increased sputum to tobacco exposure. Exacerbations of production and weight loss are not typical features of this and/or increased wheezing and dyspnea. Night sweats diagnosis. Other etiologies: of pneumonia less likely in this case. Lung abscess: night sweats, and cough producing evolve over a period a foul-smelling expectoration. Symptoms of weeks or abscess over uncomplicated pneumonia). Other bacterial causes of months (the time course in this case favors lung abscess typically present more acutely. Atypical pneumonia: Typical pneumonia: Typical and productive two-thirds of on physical exam are absent in up to Signs of pulmonary consolidation cough. time course seen here makes this diagnosis less likely. documented cases. The more subacute to active TB. The patient should be placed in respiratory isolation immediately. In those who have had recent isolation immediately. should be placed in respiratory to active TB. The patient it shows a PPD is considered positive if contact with TB patients, Lung cancer: be found in COPD patients with physical exam, clubbing can Although not found on this patient’s for cancer. underlying lung malignancy. Pulmonary tuberculosis: exposure loss) coupled with hemoptysis and recent night sweats, unintentional weight symptoms (fever and species. These can all present similarly with an insidious onset of fever, malaise, headache, myalgia, sore malaise, headache, myalgia, with an insidious onset of fever, species. These can all present similarly throat, hoarseness, chest pain, and nonproductive may be blood-streaked. cough. Sputum GI symptoms may be prominent in Diagnostic Workup Diagnoses Additional Differential CASE DISCUSSION CASE Diagnoses Note Differential Patient PRACTICE CASES 378 the tumor. Lung biopsy: Bronchoscopy: in staging. if available.AchestCTcanalsoguidediagnosticprocedures(eg,percutaneoustransthoracicbiopsies)andassist composition oflungandmediastinalpathology. foundonCTrequirecomparisontoapreviousscan Anynodules CT—chest: lobar consolidationintypicalpneumonia. cancer, acavitywithanair-fluid levelinlungabscess, apatchyinfiltrativepatterninatypicalpneumonia,and CXR—PA andlateral: MaydemonstratelesionsunseenonCXR,andaidsincharacterizingthesize,shape, Canleadtodefinitivediagnosis.Arangeoftechniquescanbeuseddependingonthelocation Usefulindiagnosingandstaginglungcanceraswellinfections. To inlung lookforapicalcavitarydiseaseinTBreactivation,noncalcifiednodules PRACTICE CASES 379 98.6°F (37°C) Look weak and sad, and lean forward while seated. Look weak and sad, and lean forward when you lie on your back. Exhibit abdominal discomfort that increases area. Show pain on palpation of the epigastric 70/minute, regular 13/minute 135/85 mm Hg Examinee introducedself by name. Examinee knocked on the door before entering. 2. Perform a focused physical exam (do not perform rectal, genitourinary, or female breast exam). not perform rectal, genitourinary, 2. Perform a focused physical exam (do workup plan to the patient. 3. Explain your clinical impression and the patient note after leaving the room. 4. Write 1. Take a focused history. a focused 1. Take “I want to go on a trip with my wife. Can we do the tests after I come back?” Sample Examinee Response do some initial blood tests, a trip. Let’s sound as though you’re feeling well enough to be able to enjoy “It doesn’t letting you go away.” and then we can see how you’re feeling and decide whether we’re comfortable Challenging Questions to Ask DOORWAY INFORMATION DOORWAY CASE 27 CASE Examinee Checklist Relationship Building the Doctor-Patient Entrance Checklist/SP Sheet Patient Description Patient is a 61 yo M, married with 3 children. Notes for the SP RR: HR: Examinee Tasks William Jordan, a 61-year-old male, comes to the office complaining of fatigue. male, comes to the a 61-year-old Jordan, William Vital Signs BP: Temp: Opening Scenario PRACTICE CASES 380 Information Gathering Reflective Listening ✓ Examinee eliciteddataefficientlyandaccurately. Examinee summarizedtheSP’s concerns,oftenusingtheSP’s ownwords. Examinee askedtheSPtolisthis/herconcernsandlistenedresponsewithoutinterrupting. Examinee askedanopen-endedquestionandactivelylistenedtotheresponse. Examinee madeeyecontactwiththeSP. Examinee correctlyusedpatient’s name. Examinee identifiedhis/herroleorposition.

Question esnfrfeigsdIdon’t know, really. Ifeelthediscomfortreachingmyback. Yes, Ifeel sad. SometimesI feelnauseated. No. Suicidal thoughts/plans/attempts No. Vague, deep. Reason forfeelingsad 4/10. Feeling ofdepression Nausea/vomiting Nothingmakes itworse,butIfeelbetterwhenleanforward. Radiation Fourmonths ago;itincreasedgradually. No. Relationship tofood Alleviating/exacerbating factors Severity onascale Yes, Idofeelsomediscomforthere(pointstotheepigastricarea). Quality Onset ofdiscomfort Ihaveabowelmovement2−3timesweek.Ithasbeenlikethis Ihavelost8poundsduringthepast6months.didnotintendto Abdominal painordiscomfort Ihaveapoorappetite. instool Blood Change inbowelhabits Sixmonthsago. Yes, Idon’t have energyformydaily30-minutewalkwithdog, Weight None. changes Appetite changes Feelingtired,weak,lowenergy. Affecting job/performance day Progression ofthefatigueduring Associated events Onset Chief complaint The samethroughouttheday. greasy-looking stools. for thepast10years.RecentlyI’venoticedmorefoul-smellingand do so. and evenatworkIamnotasenergeticwasbefore. Patient Response PRACTICE CASES 381 Patient Response Patient years now. It helps me relax. years now. me feel sick). (because I felt disgusted, and smoking made I wake up unusually early in the morning. It has been like this for morning. It has unusually early in the I wake up the past 2 months. friends. playing cards with my is boring. None. died of breast cancer. ExerciseDietSexual activityDrug allergies I walk 30 minutes every day. Sexually active with my wife. food. Regular; I like junk No. Illicit drug useTobacco Never. a pack a day I stopped it 6 months ago after 30 years of smoking Past surgical historyFamily historyOccupationAlcohol use at age 16. Appendectomy in a car accident and had diabetes, and my mother My father died retired 1 year ago. Police officer, been like this for many beers daily and 3−4 on weekends. It’s Two cold intolerance, skin/hair changes) cold intolerance, skin/hair Current medicationsPast psychiatric historyPast medical history but it is not helping. Tylenol, No. No. Loss of concentrationLoss of interest (fever/chills, Associated symptoms of breath, chest pain, cough, shortness the news or while watching concentrate anymore I can’t Yes, friends anymore. I feel that life enjoy playing cards with my I don’t Feelings of blame, guilt, worthlessnessFeelings of stay- (falling asleep, Sleeping problems No. snoring) ing asleep, early waking, Question

Examinee washed his/her hands. Examinee asked permission to start the exam. Examinee used respectful draping. Examinee did not repeat painful maneuvers. Examinee recognized the SP’s emotions and responded with PEARLS. Examinee recognized the SP’s ✓ Connecting with the Patient Physical Examination PRACTICE CASES 382 with thestressyouhavebeenexperiencinginyourlife.Doanyquestionsforme? give usagoodideawhatiswrongwithyou.Inthemeantime,oursocialworkercanmeetyoutohelpfindwayscope you areconcernedaboutyourupcomingvacation,buttheresultsoftestsshouldbebackwithinafewdays,andthey to scheduleyouforanabdominalCTscanthatmayrevealthesourceofyourpain,andIwillalsorunsomebloodtests.know with acourseofantibioticsandacidsuppressors,ortheymayhavemoreseriouscause,suchaspancreaticcancer. Iam going Mr. Jordan,yoursymptomsareconsistentwithafewdifferentdiagnoses.Theymaybecausedbyanulcerthatwould resolve Sample Closure Closure ✓ Examinee askediftheSPhadanyotherquestionsorconcerns. Examinee discussedinitialmanagementplans: Examinee discussedinitialdiagnosticimpressions.

ExamComponent xrmte Inspection,palpation Auscultation,percussion,palpation(includingreboundtenderness Auscultation Auscultation Extremities examined Inspectedconjunctivae,mouthandthroat,lymphnodes; Abdominal exam Pulmonary exam CV exam Head andneckexam Coping skills:Exercise,relaxationtechniques,spendingmoretimewithfamilyandfriends. Depression counseling: Follow-up tests. Support systemsinthehospitalandcommunity. Support systemathome(friends,family). and Murphy’s sign) thyroid gland Maneuver

PRACTICE CASES 383 Patient Note Patient USMLE STEP 2 CS STEP USMLE Physical Examination History USMLE STEP 2 CS Patient Note

Differential Diagnosis Diagnosis #1 History Finding(s): Physical Exam Finding(s):

Diagnosis #2 History Finding(s): Physical Exam Finding(s):

Diagnosis #3 History Finding(s): Physical Exam Finding(s):

Diagnostic Workup PRACTICE CASES

384 USMLE STEP 2 CS Patient Note

History HPI: 61 yo M c/o fatigue and weakness. The patient notes that the fatigue and weakness started 6 months ago. He feels tired all day. He has poor appetite and unintentionally lost 8 lbs in the past 6 months. He also complains of occasional nausea and of a vague, deep epigastric discomfort that radiates to the back. This discomfort started 4 months ago and has gradually increased to a severity of 4/10. The discomfort decreases when he leans forward and increases when he lies on his back. There is no relationship of the pain to food. No changes in bowel movement regularity, but he has recently noticed more foul-smelling, greasy-looking stools. He denies blood in the stool. He feels sad sometimes, has lost interest in things that he used to enjoy, wakes up unusually early in the morning, and complains of low energy and concentration that have affected his daily activities and work. The patient denies suicidal ideation or plans. No feelings of guilt or worthlessness. ROS: Negative except as above. Allergies: NKDA. Medications: Tylenol. PMH: None. PSH: Appendectomy at age 16. SH: 1 PPD for 30 years; stopped 6 months ago. Drinks 2 beers daily and 3−4 beers on weekends. Sexually active with his wife. FH: Father with diabetes, died accidentally. Mother died from breast cancer.

Physical Examination Patient is in no acute distress, looks sad. VS: WNL. HEENT: No conjunctival pallor, mouth and pharynx normal. Neck: Supple, no JVD, no lymphadenopathy, thyroid normal. Chest: Clear breath sounds bilaterally. Heart: RRR; normal S1/S2; no murmurs, rubs, or gallops. Abdomen: Soft, nondistended, mild epigastric tenderness, no rebound tenderness, Murphy’s sign, BS, no hepatosplenomegaly. Extremities: No edema.

Differential Diagnosis Diagnosis #1: Pancreatic cancer History Finding(s): Physical Exam Finding(s): History of smoking and eating foods that are Mild epigastric tenderness high in fat content Unintentional weight loss of 8 lbs over past 6 months PRACTICE CASES Foul-smelling, greasy-looking stools

385 USMLE STEP 2 CS Patient Note

Diagnosis #2: Depression History Finding(s): Physical Exam Finding(s): Feelings of sadness Loss of interest in activities; early awakening; impaired concentration; low energy Decreased appetite and unintentional weight loss

Diagnosis #3: Chronic pancreatitis History Finding(s): Physical Exam Finding(s): History of alcohol use Mild epigastric tenderness Worsening epigastric discomfort that radiates to the back Foul-smelling, greasy-looking stools

Diagnostic Workup

CBC, stool for occult blood Glucose Fecal fat studies Amylase, lipase AST/ALT/bilirubin (direct, indirect, and total)/ alkaline phosphatase CT—abdomen PRACTICE CASES

386 PRACTICE CASES 387 ; see Case 33). Although it ; see Case 33). Although To look for evidence of obstructive To SIG E CAPS A fecal occult blood blood test is a useful means of screening for potential loss. The pattern and location of pain are consistent with this diagnosis, but usually there is of pain are consistent with this The pattern and location Suspect this diagnosis in any patient with epigastric pain, although the complaint is Suspect this diagnosis in any patient To diagnose ulcer disease. To The pattern and location of the patient’s pain are worrisome for pancreatic disease, and pain are worrisome of the patient’s The pattern and location Ordered in suspected cases of pancreatic insufficiency. Fecal elastase and chymotrypsin Ordered in suspected cases of pancreatic insufficiency. Nonspecific symptoms such as fatigue and weakness may suggest this common diagnosis. Nonspecific symptoms such as fatigue Nonspecific, but can be elevated in chronic pancreatitis or malignancy. To diagnose gallstones as the underlying cause of pancreatitis. This test is particularly useful diagnose gallstones as the underlying cause of pancreatitis. To To diagnose pancreatic cancer or other pathology and to look for pancreatic calcifications diagnose pancreatic cancer or other To The patient has many classic symptoms of depression ( The patient has many To screen for pancreatic endocrine dysfunction (eg, diabetes mellitus, which is a risk factor for screen for pancreatic endocrine dysfunction To Thyroid disease must be ruled out in a patient with symptoms of depression. Upper endoscopy: medical evaluation. may be a somatic symptom of depression, his abdominal pain is of significant concern and warrants a thorough pain is of significant concern and warrants of depression, his abdominal may be a somatic symptom U/S—abdomen: with acute pancreatitis to help if acute pancreatitis is suspected. Ultrasound is routinely performed on patients determine if gallstones are the cause. obstruction, whereas AST and ALT are generally normal unless the liver is involved. are generally normal obstruction, whereas AST and ALT CT—abdomen: suggestive of chronic pancreatitis. TSH: Fecal fat studies: would likely be decreased in the setting of pancreatic insufficiency. Amylase, lipase: and total)/alkaline phosphatase: indirect, (direct, AST/ALT/bilirubin levels would be elevated in jaundice (often seen in pancreatic cancer). Alkaline phosphatase and bilirubin CBC, stool for occult blood: which, when compared to a known baseline level, can confirm the A CBC can determine hemoglobin levels, presence of significant blood loss. Glucose: pancreatic cancer). may suggest gastric malignancy. Hypothyroidism: Abdominal pain is unusual. Peptic ulcer disease: make a reliable diagnosis. It is important to note that many patients neither sensitive nor specific enough to disease and is unusual in uncomplicated ulcer loss, however, Weight deny any relationship of the pain to meals. Chronic pancreatitis: Chronic as alcoholism alcohol use should be explored further, a history of recurrent episodes of similar pain. The patient’s pancreatitis (the patient consumes more than 14 drinks a week, which accounts for 70–80% of cases of chronic his history of foul-smelling, greasy-looking stools may suggest is considered the limit for males). Moreover, of chronic pancreatitis. which is a manifestation pancreatic insufficiency, cancer; others include chronic pancreatitis, diabetes mellitus, and a high-fat diet. Depression may be the initial and a high-fat diet. Depression may chronic pancreatitis, diabetes mellitus, cancer; others include early due to malabsorption—is an occasional and diarrhea—presumably cancer, manifestation of pancreatic stools. foul-smelling, greasy-looking is suggested by the patient’s finding. Malabsorption Depression: Pancreatic cancer: Pancreatic risk factors for pancreatic Smoking is among the most significant concern for malignancy. his weight loss raises Diagnostic Workup Additional Differential Diagnoses Additional Differential CASE DISCUSSION CASE Diagnoses Note Differential Patient PRACTICE CASES 388 Entrance Building theDoctor-Patient Relationship Examinee Checklist decrease withage,andwewillcheckforthat.” “No, Idon’t thinkit’s becauseofyourage.Iworry moreaboutyourmedications.However, testosteronelevelscan Sample ExamineeResponse “I thinkitismyage.Isn’t thatright,doctor?” Challenging QuestionstoAsk or whetheryouhaveanyotherconcerns. Don’t mentionimpotenceunlesstheexamineeaskswhetheryouarehavinganysideeffectsfromyourmedications Notes fortheSP Patient isa54yoMwhoappearsanxious. Patient Description Checklist/SP Sheet Examinee Tasks HR: RR: Temp: BP: Signs Vital James Miller, a54-year-old male,comestotheclinic forhypertensionfollow-up. Opening Scenario DOORWAY INFORMATION CASE 28 Examinee madeeyecontact with theSP. Examinee correctlyusedpatient’s name. Examinee identifiedhis/her role orposition. selfbyname. Examinee introduced Examinee knockedonthedoorbeforeentering. 4. Write thepatientnoteafterleavingroom. 3. Explainyourclinicalimpressionandworkupplantothepatient. 2. Performafocusedphysicalexam(donotperformrectal,genitourinary, orfemalebreastexam). 1. Take afocusedhistory. 135/88mmHg 16/minute 70/minute,regular 98.0°F(36.7°C) PRACTICE CASES 389 Patient Response I’ve been losing more hair than usual from my head. I think I’m I’ve been losing more hair than usual from starting to go bald. it is the propranolol, but I my sexual performance. A friend told me right, doctor? that think it is my age. Isn’t About a 4. be. No. pressure has remained high. He added propranolol 6 months ago. pressure has remained high. He added regularly. Appetite changesDiabetes No. No. Feelings of anxiety or stressAny leg or buttock pain while walking or resting changesWeight No. No. Early-morning or nocturnal erectionsLibido No. Marital or work problemsFeelings of depression No, my wife is great, and I am very happy in my job. No. weak, too, doc. I’m just not as interested in sex as I used to That’s Description of the problemSeverity on 1–10 scale, where 1 is flac- cid and 6 is adequate for penetration get an erection at all. I have a weak erection. Sometimes I can’t aches, dizziness, blurred vision, nausea, aches, dizziness, blurred vision, nausea, palpitations, chest pain, shortness of breath, urinary changes, weakness, bowel movement changes, sleep prob- lems, hair loss) Medication side effects Over the past 4 months I have started to experience problems with Last blood pressure checkupHow he is feeling todayHome monitoring of blood pressureAny other symptoms (fatigue, head- Six months ago. No. Good. OnsetTreatmentCompliance with medicationsthe pills, but in general I take them sometimes I forget to take Well, but my blood started me on hydrochlorothiazide, The doctor Last year I found out that I have hypertension. Chief complaint I am here to check on my blood pressure. Question

Examinee elicited data efficiently and accurately. Examinee elicited data Examinee asked an open-ended question and actively listened to the response. listened to the question and actively asked an open-ended Examinee interrupting. response without and listened to the his/her concerns asked the SP to list Examinee own words. SP’s concerns, often using the the SP’s Examinee summarized ✓ Reflective Listening Reflective Information Gathering PRACTICE CASES 390 Physical Examination Connecting withthePatient ✓ ✓ Examinee didnotrepeatpainfulmaneuvers. Examinee usedrespectfuldraping. Examinee askedpermissiontostarttheexam. Examinee washedhis/herhands. Examinee recognizedtheSP’s emotionsandrespondedwithPEARLS.

ExamComponent Question xrmte Inspection,palpation ofperipheralpulses DTRs,Babinski’s sign,sensationandstrengthinbilaterallower Auscultation, palpation Palpation,auscultation Auscultation Neurologic exam Funduscopic exam,carotidauscultation Extremities Abdominal exam Pulmonary exam CV exam Head andneckexam No. Ihadawonderfulsexlifewithmywifeuntil4monthsago,when No. Iammarriedandlivewithmywife. No. Yes, 3−4beersaweekforthepast10years. No. Drug allergies Ieatalotofjunkfood. Sexual activity Schoolteacher. Social history Tobacco Myfatherdied atage50ofaheartattack.Mymotherishealthy, but Illicit druguse Alcohol use None. Exercise None. Diet Propranolol,hydrochlorothiazide,lovastatin. Occupation Family history No. Past surgicalhistory No. Past medicalhistory Current medications Yes, itwasdiagnosedlastyear. History ofTIAorstroke Previous heartproblems History ofhypercholesterolemia she hasAlzheimer’s disease.Sheisinanursinghomenow. extremities getting old. I startedhavingthisproblemthattoldyouabout.thinkam Patient Response Maneuver PRACTICE CASES 391 I Changing propranolol to another antihypertensive medication that does not cause erectile dysfunction. to another antihypertensive medication Changing propranolol Follow-up tests: Examinee mentioned the need for genital and rectal exams. mentioned the need for genital Follow-up tests: Examinee Lifestyle modification cessation). (diet, exercise, alcohol SP had any other questions or concerns. Examinee asked if the Examinee discussed initial diagnostic impressions. discussed initial diagnostic Examinee plans: discussed initial management Examinee exam as well as a rectal exam to assess your prostate. Do you have any questions for me? exam as well as a rectal exam to assess your Mr. Miller, your blood pressure level was 135/88 when we measured it earlier today, which is close to our target of 130/80. which is close to our target of 130/80. when we measured it earlier today, your blood pressure level was 135/88 Miller, Mr. that should be feasible with lifestyle get it down to around 120/80. Fortunately, it would be even better if we could However, fat intake and increasing the amount of exercise you are doing. As for your problems changes such as decreasing your salt and side effect of one of the blood pressure medications you are taking. For this reason, with your erection, this is a very common than propranolol to control your blood pressure. I am also going to order some blood would like to give you a medication other due to any other medical condition. In addition, I would like to perform a genital tests to make sure that your problem is not Sample Closure Closure PRACTICE CASES 392 Physical Examination History USMLE STEP2CS Patient Note PRACTICE CASES 393 Patient Note Patient Diagnosis #3 History Finding(s): Physical Exam Finding(s): Diagnosis #2 History Finding(s): Physical Exam Finding(s): Diagnosis #1 History Finding(s): Physical Exam Finding(s): USMLE STEP 2 CS STEP USMLE Diagnostic Workup Differential Diagnosis Differential USMLE STEP 2 CS Patient Note

History HPI: 54 yo M presents for follow-up of his hypertension that was diagnosed last year. He was initially started on HCTZ; propranolol was added 6 months ago. He is fairly compliant with his medications. He does not monitor his blood pressure at home. His last blood pressure checkup was 6 months ago. He is feeling well except for erectile dysfunction and decreased libido noted 4 months ago. No leg claudication or any previous history of heart problems, stroke, TIA, or diabetes. No marital or work problems. No depression, anxiety, appetite or weight changes, or history of trauma. ROS: Negative except as above. Allergies: NKDA. Medications: HCTZ, propranolol, lovastatin. PMH: Hypertension, hypercholesterolemia diagnosed 1 year ago. PSH: None. SH: No smoking, 3–4 beers/week, no illicit drugs. Works as a schoolteacher; married and lives with his wife. FH: Father died of a heart attack at age 50. Mother is in a nursing home due to Alzheimer’s disease.

Physical Examination Patient is in no acute distress. VS: WNL. HEENT: No funduscopic abnormalities. Neck: No carotid bruits, no JVD. Chest: Clear breath sounds bilaterally. Heart: Apical impulse not displaced; RRR; normal S1/S2; no murmurs, rubs, or gallops. Abdomen: Soft, nondistended, nontender, BS, no bruits, no organomegaly. Extremities: No edema, no hair loss or skin changes. Radial, brachial, femoral, dorsalis pedis, and posterior tibialis 2+ and symmetric. Neuro: Motor: Strength 5/5 in bilateral lower extremities. Sensation: Intact to pinprick and soft touch in lower extremities. DTRs: Symmetric 2+ in lower extremities, Babinski bilaterally.

Differential Diagnosis Diagnosis #1: Medication-induced erectile dysfunction History Finding(s): Physical Exam Finding(s): Taking propranolol Onset of ED coincides with propranolol use No early-morning or nocturnal tumescence PRACTICE CASES

394 USMLE STEP 2 CS Patient Note

Diagnosis #2: Erectile dysfunction secondary to vascular disease History Finding(s): Physical Exam Finding(s): History of hypertension History of hyperlipidemia No early-morning or nocturnal tumescence

Diagnosis #3: Hypogonadism History Finding(s): Physical Exam Finding(s): Loss of libido and ED Hair loss No early-morning or nocturnal tumescence

Diagnostic Workup

Genital and rectal exams Serum glucose Testosterone level Prolactin, TSH, LH/FSH Ferritin MRI—brain Doppler U/S—penis Dynamic cavernosography PRACTICE CASES

395 PRACTICE CASES 396 Diagnostic Workup Additional Differential Diagnoses Patient NoteDifferential Diagnoses CASE DISCUSSION lowering medication,andfor complicationsofmedicaltherapy(eg,diuretic-induced hypokalemia). hyperlipidemia. Canbeused toscreenforkidneydisease,LVH orpriorsilentMIs,forresponse tocholesterol- BUN/Cr, electrolytes, cholesterol, UA,ECG: normal arterialinflow). Dynamic cavernosography: Doppler U/S—penis: hypogonadism. MRI—brain: deposition inthepituitaryglandcausinghypogonadotropic hypogonadism. Ferritin: the settingofpituitaryorhypothalamicdisease. are elevated(“hypergonadotropic”)inthesettingoftesticularpathologyandlow(“hypogonadotropic”) LH/FSH: hypogonadism. Prolactin, TSH: Testosterone level: Serum glucose: Rectal exam: Genital exam: Peyronie’s disease: symptoms, butfurtherexplorationofhisfeelingsaboutnursing-home-boundmothermaybemorerevealing. nocturnal orearly-morningerectionsarepreserved(notseeninthiscase).Thispatientdeniesotherdepressive Depression: testosterone deficiency. symptoms suchashotflashes,fatigue,hairloss,anddepression.Thispatienthaswhichissuggestiveof (pathology inthetestesthemselves).InadditiontodiminishedlibidoandpossibleED,thereareoftenassociated is attributabletoeithercentral(dueinsufficientgonadotropinsecretionbythepituitary)orend-organdisease Hypogonadism: claudication, diminishedpulses,hairlossinthelegs,orthin,shinyskin). vascular disease,buttherearenohistoricalorphysicalfindingstosuggestitspresenceinthiscase(eg,angina,leg ED secondarytovasculardisease: commonly associatedwithED. Medication-induced erectile dysfunction(ED): suggests anorganicratherthanapsychologicaletiology. months afterhewasstartedonpropranolol.Inaddition,hislackofearly-morningandnocturnaltumescence To screenforhemochromatosis,acommoncondition;ED canbeanearlymanifestationduetoiron Gonadotropinlevelsshouldbecheckedinpatientswithloworborderlinetestosteronelevels.Levels To ruleoutapituitaryorhypothalamiclesioninpatients presentingwithhypogonadotropic Psychogeniccausescanleadtolossoflibidoanderectionsaresuggestedwhen To detectmassesorprostaticabnormalities. To ruleoutPeyronie’s disease(eg,tolookforpenilescarringorplaqueformation). To screenforotherabnormalitiesofpituitaryfunctioninpatientswithhypogonadotropic To screenfordiabetes,apossiblecontributor toED. Testosterone deficiencyhasmanyunderlyingetiologiesbut,aswithotherendocrineproblems, To screenforhypogonadism. FibrousplaqueofthetunicaalbugineacanleadtopenilescarringandED. To flowinthecavernousarteries. assessblood To determinethesiteandextentofvenousleak(suspectedinpatientswith β -blockers canoftencauselossoflibidoandED.Thispatient’s EDbegantwo Hypertensionandhyperlipidemiaareriskfactorsforatherosclerotic Usefulinthelongitudinalcare ofhypertensionand Antihypertensives(butrarelydiuretics)andalcoholare PRACTICE CASES 397 each night. It would help if you went to sleep around the . M . P 98.6°F (37°C) Look anxious and irritable. well in college. Pretend that you are worried about performing fingertips and brisk reflexes. Exhibit a fine tremor on outstretched 102/minute 18/minute 120/80 mm Hg Examinee knocked on the door before entering. Examinee introducedself by name. 2. Perform a focused physical exam (do not perform rectal, genitourinary, or female breast exam). not perform rectal, genitourinary, 2. Perform a focused physical exam (do workup plan to the patient. 3. Explain your clinical impression and the patient note after leaving the room. 4. Write 1. Take a focused history. a focused 1. Take Sample Examinee Response In the meantime, I recommend some “First we need to run some tests to rule out underlying medical problems. could on your caffeine intake. You lifestyle changes. If you drink coffee, I strongly recommend that you cut down you should get into bedtime. Finally, also benefit from exercising, preferably during the day and not right before the habit of going to bed early—for example, at 10 Challenging Questions to Ask I ever be able to sleep well again, doctor?” “Will DOORWAY INFORMATION DOORWAY CASE 29 CASE same time each night and woke up around the same time each morning. I would also encourage you to abstain from same time each night and woke up around the same time each morning. I would drinking alcohol several hours before bedtime.” Entrance Examinee Checklist Relationship Building the Doctor-Patient Patient Description and weight. Patient is a 20 yo F of average height Notes for the SP Checklist/SP Sheet RR: HR: Examinee Tasks Gwen Potter, a 20-year-old female, comes to the clinic complaining of sleeping problems. female, comes to the clinic complaining a 20-year-old Gwen Potter, Vital Signs BP: Temp: Opening Scenario PRACTICE CASES 398 Information Gathering Reflective Listening ✓ Examinee eliciteddataefficientlyandaccurately. Examinee summarizedtheSP’s concerns,oftenusingtheSP’s ownwords. Examinee askedtheSPtolisthis/herconcernsandlistenedresponsewithoutinterrupting. Examinee askedanopen-endedquestionandactivelylistenedtotheresponse. Examinee madeeyecontactwiththeSP. Examinee correctlyusedpatient’s name. Examinee identifiedhis/herroleorposition.

Question ieyufl sepIusuallygetinbedaroundmidnight,butdon’t fallasleepuntil Ithasbeengoingonformorethan6monthsnowbutworsened Difficultyfallingasleep. Isleeparound4hoursanight.Afterwakeup,havetrouble Time youfallasleep Total hoursofsleeppernight Duration Chief complaint le nerpin Yes, Iwakeupacoupleoftimesduringthenight. No,thealarmgoesoffandwakesmeupat6 Iwatch TV. Early spontaneousawakening Sleep interruptions Activities beforesleep nrn Idosnore.Myboyfriendtoldmeaboutmysnoring afewmonths Snoring atm leiesIfeelvery sleepyduringclassandwhiledrivingtoschoolat7 Daytime sleepiness rmr None. Idrinkatleast 5–6cupsofcoffeeorenergydrinkseverydaytostay BeforeIstartedcollege,workedoutanhouraday everyevening, Tremors Myboyfriendis very understandingbuthasahardtimewakingme Caffeine intake Exercise hobbies Sadness, depression,lossofinterestin Ifeeltheneedtotakenapsbuthavenotimeforthem.Myfinal Well, gradesincollege.Ihave Iamstressedoutaboutgettinggood Relationship Recent stressfulevents/illnesses Daytime naps falling backasleep.UsuallyIneed8hoursofsleeptofeelrefreshed. over thepastmonth. Patient Response around 2 ago, buthesaidthatisfinewithit. awake. hit thegym. but latelyithasbecomeharderandformetofindthe timeto No. up inthemorningsforclass.We relationship. haveagood heavier courseloadthissemestertofinishschoolontime. been workinghardtogetanAinallofmyclasses.I’mtaking a how I’lldoonthem. exams arecomingupsoon,andIneedtostudy. I’mworried about A . M . A . M . A . M . PRACTICE CASES 399 All I take are multivitamins and oral contraceptive pills. All I take are multivitamins and oral contraceptive immediately before bed. Patient Response Patient coffee. the time. more than usual. appetite and eating I’ve been going 2 or 3 times each but lately I used to go once a day, I have no loose stools or blood in my stool. day. Drug allergies None. OccupationAlcohol useIllicit drug useTobacco College student. 2 beers a week, and only on the weekends, never Occasionally 1 or None. None. Past medical historyPast surgical historyFamily history None. I had a tonsillectomy when I was 12. None. Skin changesAny pain in joints/muscleHair loss/thinningCurrent medications (antidepressants, antihistamines, pain medication) No. No. No. Change in bowel habits or in stool color or consistency Urinary habitsNeck pain Normal. No. Frequency of menstrual periodContraceptivesFever been on oral contraceptive pills for the past 2 years. I have Regular. Condoms and oral contraceptive pills. No. SweatingIrritabilityIntolerance to heat/cold changesWeight No. are wet most of but lately I have noticed that my palms Not really, Yes. month despite having a good I have lost 6 pounds over the past Shortness of breathShortness Palpitations No. racing most of the time, especially after I drink I feel my heart Yes, Question

Examinee recognized the SP’s emotions and responded with PEARLS. Examinee recognized the SP’s ✓ Connecting with the Patient PRACTICE CASES 400 will seeyouforfollow-uptofindouthowaredoing.Do you haveanyquestionsorconcerns? future toruleoutsleepapnea.Atthispoint,Iencourageyouproceedwiththelifestylechangeshaverecommended,and I the onlywaytorulethisoutisthroughabloodtest.Inlightofyourhistorysnoring,wemayneeddosleepstudyin with yourthyroidfunction.Sometimeshyperactivityoftheglandcancausesomesymptomsyoudescribe, and your problemscouldstemfromcaffeineuse,whichIurgeyoutoreduceorstopcompletely. Anotherpossibilityhastodo perfectly understandable,youmaynotbeabletoperformatyourbestifdon’t getagoodnight’s sleep.Ontheotherhand, sleeping problems.Thefirstistheanxietyandstressyou’vebeenexperiencingoverperformingwellincollege.Althoughthis i Ms. Potter, onthebasisofyourhistoryandmyexamination,Ithinkthere areafewfactorsthatmightbecontributingtoyour Sample Closure Closure Physical Examination ✓ Examinee askediftheSPhadanyotherquestionsorconcerns. Examinee discussedinitialmanagementplans. Examinee discussedinitialdiagnosticimpressions. Examinee didnotrepeatpainfulmaneuvers. Examinee usedrespectfuldraping. Examinee askedpermissiontostarttheexam. Examinee washedhis/herhands.

ExamComponent knea Inspection Lookedforbrisk reflexes Checkedfortremoronoutstretchedfingertips;lookededema Inspection, auscultation, palpation Auscultation Auscultation Inspection,palpation, auscultationofthyroidforlymphadenopathy Neurologic exam Skin exam Extremities Abdominal exam Pulmonary exam CV exam HEENT exam Follow-up tests. Maneuver the s

PRACTICE CASES 401 Patient Note Patient USMLE STEP 2 CS STEP USMLE Physical Examination History USMLE STEP 2 CS Patient Note

Differential Diagnosis Diagnosis #1 History Finding(s): Physical Exam Finding(s):

Diagnosis #2 History Finding(s): Physical Exam Finding(s):

Diagnosis #3 History Finding(s): Physical Exam Finding(s):

Diagnostic Workup PRACTICE CASES

402 USMLE STEP 2 CS Patient Note

History HPI: 20 yo F college student c/o inability to sleep. She has difficulty falling asleep until 2 A.M. and also has difficulty staying asleep. She used to get 8 hours of sleep, but for the past month she has been getting a total of only 4 hours per night. She has difficulty getting up after hearing the alarm and feels tired while at school. She notes inability to concentrate during classes and while driving. The patient appears to be stressed about her coursework and about her performance at school. She has also been snoring for the past few months and has had palpitations, especially after drinking caffeine. She has a history of drinking 4–5 cups of coffee per day. She has lost weight (6 lbs in 1 month) and has sweaty palms. There is an increase in the frequency of her bowel movements. She lives with her boyfriend, and they use condoms and OCPs for contraception. There is no history of sexual abuse, recent infection, or recent tragic events in her life. ROS: Negative except as above. Allergies: NKDA. Medications: Multivitamins, OCPs. PMH: None. PSH: Tonsillectomy at age 12. SH: No smoking, 1–2 beers/week, no illicit drugs. FH: Not significant.

Physical Examination Patient appears anxious and restless. VS: HR 102/minute. Chest: Clear breath sounds bilaterally. Heart: Tachycardic; normal S1/S2; no murmurs, rubs, or gallops. Abdomen: Soft, nontender, nondistended, BS, no guarding, no hepatosplenomegaly. Skin: Normal, no rashes, palms moist. Neuro: Brisk reflexes.

Differential Diagnosis Diagnosis #1: Anxiety History Finding(s): Physical Exam Finding(s): Impaired concentration, irritability, difficulty Tachycardia (HR 102/minute) sleeping, muscle tension, sweating, and palpitations Anxiety over academic achievement No history of substance use PRACTICE CASES

403 USMLE STEP 2 CS Patient Note

Diagnosis #2: Caffeine-induced insomnia History Finding(s): Physical Exam Finding(s): Drinks 4−5 cups of caffeine per day Tachycardia (HR 102/minute) Spends 2 hours awake before falling asleep History of palpitations that are more pronounced after drinking caffeine

Diagnosis #3: Hyperthyroidism History Finding(s): Physical Exam Finding(s): Anxiety Tachycardia (HR 102/minute) History of unintentional weight loss, fatigue, Brisk reflexes sweating, palpitations, and increased bowel movements

Diagnostic Workup

TSH, FT3,FT4 PRACTICE CASES

404 PRACTICE CASES 405 with suppressed TSH is diagnostic. 4 Any significant life event, such as a change of occupation, Any significant life event, such as a change More than 50% of patients evaluated for OSA complain of symptoms of Several mood can be disorders are associated with insomnia. Depression The most common pharmacologic cause of insomnia, caffeine use produces pharmacologic cause of insomnia, caffeine The most common Clinical hyperthyroidism is associated with anxiety, tremor, palpitations, sweating, frequent tremor, with anxiety, Clinical hyperthyroidism is associated Although this patient denies illicit drug use, a toxicology screen will help rule out the use of Although this patient denies illicit drug use, a toxicology screen will help rule Drugs such as cocaine and amphetamine increase sympathetic activity and can thus cause Drugs such as cocaine and amphetamine increase sympathetic activity and The patient gives a history of weight loss, increased frequency of bowel movements, palpitations, The patient gives a history of weight loss, increased frequency of bowel movements, : 4 , FT Fatigue and sleep disturbances are common in anxiety states. The clinical manifestations of are common in anxiety states. Fatigue and sleep disturbances 3 and sweaty palms, all of which suggest hyperthyroidism. An elevated FT Urine toxicology: CNS stimulants that can cause insomnia (eg, cocaine, amphetamine). TSH, FT insomnia. Obstructive sleep apnea (OSA): awakening. OSA has a insomnia, including difficulty in initiating and maintaining sleep and early-morning it is given that this patient has had a tonsillectomy, higher association with obesity and large tonsils. However, unlikely that enlarged tonsils secondary to OSA are the cause of her disorder. loss of a loved one, illness, or examinations, can be a significant stressful event in people’s lives. Behavioral or can be a significant stressful event in people’s loss of a loved one, illness, or examinations, mood start within three months of the stressful event, changes associated with adjustment disorder typically frequent life. Increased sleep latency, and cause significant impairment in one’s end six months after the stressor, awakening can all result. Recovery is rapid, usually occurring within a awakenings from sleep, and early-morning few weeks. Illicit drug use: Insomnia due to depression: sleep maintenance insomnia, or early-morning wakefulness. Hypersomnia associated with sleep onset insomnia, adolescents and those with either bipolar or seasonal (fall/winter) occurs in some depressed patients, especially depression. disorder: Insomnia secondary to adjustment bowel movements, fatigue, menstrual irregularities, unintentional weight loss, and heat intolerance. The patient bowel movements, fatigue, menstrual increased bowel movements, fatigue, and weight loss, palpitations, sweating, presented in this case has anxiety, suggesting the need to rule out hyperthyroidism. Caffeine-induced insomnia: Caffeine-induced time several sleep, and a reduction in total sleep sleep onset, more frequent arousals during increased latency to of caffeine can significantly disturb sleep in some patients. This hours after ingestion. Even small amounts caffeine-induced insomnia a possible diagnosis. high intake of coffee makes patient’s Hyperthyroidism: sweating, hyperventilation, palpitations, tremor). This patient describes irritability, trouble concentrating, and describes irritability, palpitations, tremor). This patient sweating, hyperventilation, The disorder. a diagnosis of generalized anxiety more than six months, which supports difficulty sleeping for diagnosis of Although not required for an official is likely her desire to excel in college. source of her anxiety tachycardia, sweating, of anxiety are many and include somatic manifestations disorder, generalized anxiety and tremor. hyperventilation, palpitations, Anxiety: tachycardia, concentrating) and somatic (eg, psychological (eg, tension, fears, difficulty anxiety can be both Diagnostic Workup Additional Differential Diagnoses Additional Differential CASE DISCUSSION CASE Diagnoses Note Differential Patient PRACTICE CASES 406 ECG: well asanycomorbiditieswithwhichitmightbeassociated. Polysomnography: loss seeninthispatient. CBC: Canhelpdetectanemia,hiddeninfection,ormalignancy, allofwhichcancausethefatigueandweight Nonspecificchangescanbeseenwithhyperthyroidismandanxietydisorders. AdiagnostictestforOSAsyndromethatcanalsohelpassesstheseverityofdiseaseas PRACTICE CASES 407 “How will I be able to get a ride to the office?” “How will I be able to get a ride to the “Can you explain to me exactly what is going on with my child and what can be done for it?” “Can you explain to me exactly what Examinee summarized the SP’s concerns, often using the SP’s own words. concerns, often using the SP’s Examinee summarized the SP’s Examinee correctly used patient’s name and identified caller and relationship of caller to patient. Examinee correctly used patient’s Examinee asked an open-ended question and actively listened to the response. without interrupting. Examinee asked the SP to list his/her concerns and listened to the response Examinee introducedself by name. Examinee identified his/her role or position. 2. Explain your clinical impression and workup plan to the mother. impression and workup plan to the 2. Explain your clinical the patient note after leaving the room. 3. Write 1. Take a focused history. 1. Take “It is hard for me to give you an accurate answer over the phone. I would like you to bring your child here so that I answer over the phone. I would like you to bring your child here so that I “It is hard for me to give you an accurate tests. After that, I will be able to give you a more accurate assessment of her can examine her and perhaps run some worker to speak with you about arranging transportation to the office.” will arrange for the social condition. We Sample Examinee Response DOORWAY INFORMATION DOORWAY CASE 30 CASE Building the Doctor-Patient Relationship Building the Doctor-Patient Entrance Examinee Checklist Challenging Questions to Ask Patient Description the history over the phone. mother offers The patient’s Notes for the SP want to come to the office unless you have to but add that you don’t health, Show concern about your child’s because you do not have transportation. Checklist/SP Sheet The mother of Angelina Harvey, a 2-year-old female child, calls the office complaining that her child has noisy complaining that her child has noisy female child, calls the office a 2-year-old Harvey, The mother of Angelina and strange breathing. Examinee Tasks Opening Scenario Reflective Listening PRACTICE CASES 408 Information Gathering ✓ Examinee eliciteddataefficientlyandaccurately.

Question acntosUptodate. Yes. Nottomyknowledge. Vaccinations No. Ill contactsindaycarecenter Thereisoccasional hoarseness. Day carecenter No. recent past No. Psychological orsocialstressinthe No. History ofallergiesinthefamily Snoring atnight No. No. No. Ican’t tell,but itseemsasthoughshe’s tryinghardtobreathe. Hoarseness ofvoice inthepast Similar episodes Yes. Difficulty inswallowingfood Blueness ofskinorfingers Drooling Weak withoccasionalmuffling. No. No. Nausea/vomiting Breathing fast Is hercryingmuffledorweak? Is shecrying? No. incough? Any blood Is itproductive? Is thecoughbarkinginnature? Yes, thereissomecoughing,butitwaspresentearlier. Shehada Thesoundisalwaysthesame. Associated problems(cough,fever) ing, crying,supineposition,sleep) Alleviating/exacerbating factors(feed- Itstartedsuddenlyaboutanhourago. Itisgettingworse. Itisanoisysound,asifsheswallowedwashingmachine. have causedit? Can youidentifyanythingthatmay Oninhalation. Best heardoninhalationorexhalation Consistency Mybabyhas noisyandstrangebreathing. Description ofthesound ceded theevent Description oftheactivitythatpre- Progression Onset Chief complaint None. None. She wasplayingwithtoys. Patient Response No. normal. Itwas101.2°Fatitsworst. was low-grade feverforthepastweek,buthertemperaturetoday PRACTICE CASES

409 Patient Response Patient Follow-up tests. Family history None. Eating habitsCurrent medicationsPast medical historyPast surgical history None. Normal. Nothing of note. None. Last checkup and milestonesGrowth, development, fine. She met all milestones in a timely manner. All were Birth history normal. and everything was weeks ago, Two It was an uncomplicated spontaneous vaginal delivery. Question

Examinee discussed initial management plans: Examinee discussed initial management questions or concerns. Examinee asked if the SP had any other Examinee discussed initial diagnostic impressions. Examinee discussed initial diagnostic Examinee recognized the SP’s emotions and responded with PEARLS. Examinee recognized the SP’s ✓ deeply lodged if it is actually present. If you observe significant respiratory compromise or choking, perform the Heimlich deeply lodged if it is actually present. If you observe significant respiratory compromise tummy with sudden pressure. I hope you understood what we have discussed. Do you maneuver by thrusting your daughter’s I will see you once you get to the hospital. have any questions or concerns? Okay, Sample Closure from you, I’m considering the possibility that your daughter on the basis of the information I have gathered Mrs. Harvey, might be causing her problem needs to be ruled the possibility that an infection However, might have swallowed a foreign body. I feel that your daughter needs emergency medical attention. Since you do not have access to transportation, out. Right now, In the meantime, I suggest that you avoid I strongly suggest that you call 911 immediately and bring her to the medical center. cause the foreign body to become more putting a finger in her mouth or performing any blind finger sweep, as doing so may Physical Examination None. Closure Connecting with the Patient PRACTICE CASES 410 Physical Examination History USMLE STEP2CS Patient Note PRACTICE CASES 411 Patient Note Patient Diagnosis #3 History Finding(s): Physical Exam Finding(s): Diagnosis #2 History Finding(s): Physical Exam Finding(s): Diagnosis #1 History Finding(s): Physical Exam Finding(s): USMLE STEP 2 CS STEP USMLE Diagnostic Workup Differential Diagnosis Differential USMLE STEP 2 CS Patient Note

History HPI: The source of information is the patient’s mother. The mother of a 2 yo F c/o her child suddenly developing noisy breathing that is getting progressively worse. The child was playing with her toys when she developed the noisy breathing. The sound is consistent, best heard on inhalation, and similar to that of a washing machine. There is no relation to posture. It is associated with a nonproductive cough without any associated hemoptysis, tachypnea, drooling, or bluish discoloration of the skin. Her vaccinations are up to date. ROS: Negative. Allergies: NKDA. Medications: None. PMH: Uncomplicated spontaneous vaginal delivery. PSH: None. FH: Noncontributory.

Physical Examination None.

Differential Diagnosis Diagnosis #1: Foreign body aspiration History Finding(s): Physical Exam Finding(s): Sudden onset while playing with toys Noisy breathing

Diagnosis #2: Croup History Finding(s): Physical Exam Finding(s): Noisy breathing Difficulty breathing Fever for the past week

Diagnosis #3: Epiglottitis History Finding(s): Physical Exam Finding(s): Occasional voice hoarseness Occasional muffling PRACTICE CASES

412 USMLE STEP 2 CS Patient Note

Diagnostic Workup

ABG CXR—PA and lateral XR—neck, AP and lateral CBC with differential Bronchoscopy Direct laryngoscopy PRACTICE CASES

413 PRACTICE CASES 414 to it; Diagnostic Workup Additional Differential Diagnoses obstruction inthetrachea.Stridorthatpresentswithhoarsenesssuggestsinvolvementofvocalcords. There arethreetypesofstridor: Patient NoteDifferential Diagnoses CASE DISCUSSION result oftheball-and-valveeffect. obstructs thelowerairwayand causesairtrapping,theexpiratoryfilmmaysometimes revealairtrappingasa films. Therefore,anormalradiographcannotruleoutan aspiratedforeignbody. However, whenaforeignbody CXR—PA andlateral: exchange inthelung. ABG: children threemonthstoyearsofage. Laryngeal papilloma: of asorethroatandtonsillitis.Thereisnostridor. Peritonsillar abscess: is laborious. immune compromise.Onsetissudden,andtheclinicalfeaturesofstridorfacialedemaarefound.Respiration angioedema doesnotappeartoapplythispatient,asshewouldlikelyhaveexhibitedsomemanifestationof Angioedema: muffled, anddroolingisoftenpresent. Retropharyngeal abscess: voice arecharacteristic. Laryngitis: the causeofherdisorder. Haemophilus influenzae patient. However, thepatienthasexperiencedvoicehoarseness.The mostcommonetiologyofepiglottitisis hallmark feature,significantdroolingwithsymptomaticreliefwhilebendingforward,isnotpresentinthis Epiglottitis: present incroup. is suggestiveofaviralinfection.Althoughnotfoundinthispatient,characteristicbarkingcoughoften The mostlikelyculpritforcroupisparainfluenza.Thispatienthashadalow-gradefeverthepastweek,which Croup: aspiration ofaforeignbody. The patientisbreathingnoisilyandexperiencingsomeshortnessofbreath,bothwhichareconsistentwith (usually atoyorpeanuts)isinthevicinitybeforepatientdevelopssymptoms,helpssupportthisdiagnosis. Foreign bodyaspiration: expiratory stridor, It is essential to determine blood gasconcentrationsinordertoindirectlyassessventilationandgaseous Itisessentialtodetermineblood Croupiscommoninchildrensixmonthstothreeyearsofage,usuallydevelopinginsidiouslyasaURI. Occursinchildrenolderthanfiveyearsofage.Theabsencestridorandthepresenceahoarse Occurs more frequently in children 2−6 years of age, and begins with a short prodrome. Its Occursmorefrequentlyinchildren2−6yearsofage,andbeginswithashortprodrome. Canoccuratanyage,andmaybeanallergicresponseorhereditary(congenital).Congenital whichpointstoobstructioninferiorthelarynx;and typeb,butgiventhatthispatient’s immunizationsareuptodate, it isunlikelythatthis Typically occursinchildrenolderthan10 yearsofage.Onsetisgradual,withahistory Achronicconditioncharacterizedbyahoarsevoice;most commonlydiagnosedin It is noteworthy that the majority of foreign bodies arenotvisibleonCXRPA Itisnoteworthythatthemajorityofforeignbodies plain The sudden and dramatic onset of symptoms, especially when a foreign body Thesuddenanddramaticonsetofsymptoms,especiallywhenaforeignbody inspiratory stridor, Patientsareusuallyyoungerthansixyearsofage.Theylackstridor, theirvoiceis whichindicatesobstructionattheleveloflarynxorsuperior biphasic stridor, whichsuggests PRACTICE CASES 415 May show narrowing of the trachea (steeple sign) in croup, extrinsic pressure, or a in croup, extrinsic trachea (steeple sign) narrowing of the May show To rule out or rule in an underlying infective pathology. underlying infective out or rule in an rule To Useful when differentials of laryngomalacia or laryngeal lesions such as papilloma are of laryngomalacia or laryngeal Useful when differentials Used as a diagnostic and therapeutic modality Used as a diagnostic in cases of foreign body aspiration. suspected. XR—neck, AP and lateral: XR—neck, sign) in epiglottitis. glottis (thumbprint classic swollen differential: CBC with Bronchoscopy: laryngoscopy: Direct PRACTICE CASES 416 Notes fortheSP Patient isa21yoF, singlewith1child. Patient Description Checklist/SP Sheet Examinee Tasks HR: RR: Temp: BP: Signs Vital Jessica Anderson,a21-year-old female,comestotheEDcomplainingofabdominalpain. Opening Scenario DOORWAY INFORMATION CASE 31 taken careof,aswelltoaddressanyfinancialconcerns youmayhave.” your illnessisn’t lifethreatening.Oursocialworker wouldbehappytoworkwithyouensurethatyourchildis “Ms. Anderson,Iunderstandyourconcernforchild’s safety. However, itismostimportantthatwemakesure Sample ExamineeResponse Challenging QuestionstoAsk 4. Write thepatientnoteafterleavingroom. 3. Explainyourclinicalimpressionandworkupplantothepatient. 2. Performafocusedphysicalexam(donotperformrectal,genitourinary, orfemalebreastexam). 1. Take afocusedhistory. 120/80mmHg 20/minute 88/minute,regular “I can’t affordtostayinthehospital.Pleasegiveme aprescriptionforantibioticssothatIcanleave.” “My childisinthehousealone.Imustleavenow.” Manifest painwhentheexamineeextendsyourrighthip(psoassign). Experience painintheRLQwhenexamineepressesonLLQ(Rovsing’s sign). Demonstrate guarding(contractionoftheabdominalmuscleswhenpalpatingRLQ). Show reboundtenderness(painwhentheexamineeremoveshispalpatinghand). Exhibit rightlowerabdominaltendernessonpalpation. 100.5°F(38.1°C) PRACTICE CASES 417 Patient Response Blood in stool No. Description of vomitusBlood in vomitusDiarrhea/constipationDescription of stool yellowish fluid. It was a sour, Loose bowel movements this morning. No. Brown. Pain with ride to hospitalPrecipitating eventsFever/chillsNausea/vomiting Yes. None. and vomited once 2 hours ago. I feel nauseated since this morning, but no chills. I’ve been a little hot RadiationQualityAlleviating factorsExacerbating factors No. None. Movement. Cramping. OnsetFrequencyProgressionSeverity on a scaleLocation This morning. Strong, steady pain. 7/10. It is getting worse. It is here (points to the right lower abdomen). Chief complaint Abdominal pain. Question

Examinee elicited data efficiently and accurately. Examinee elicited data efficiently and Examinee asked an open-ended question and actively listened to the response. open-ended question and actively listened Examinee asked an concerns and listened to the response without interrupting. Examinee asked the SP to list his/her own words. often using the SP’s concerns, Examinee summarized the SP’s Examinee introduced self by name. his/her role or position. Examinee identified name. used patient’s Examinee correctly contact with the SP. Examinee made eye Examinee knocked on the door before entering. Examinee knocked ✓ Examinee Checklist Examinee Relationship the Doctor-Patient Building Entrance Information Gathering Reflective Listening PRACTICE CASES 418 Connecting withthePatient ✓ Examinee recognizedtheSP’s emotionsandrespondedwithPEARLS.

Question oac Onepackadayforthepast6years. No. No. Two orthreebeersaweek. Waitress. No. Drug allergies None. Tobacco NoneexceptforwhatI’vementioned. Illicit druguse Alcohol use Occupation Herefusedthetreatment. Yes, Ihadsomekindofinfection6monthsago,butcan’t Past surgicalhistory Past medicalhistory Ihad3sexualpartners. Onepartner; I methim6monthsago. HIV test Oralcontraceptivepills.Myboyfriendrefusestousecondoms. Treatment ofthepartner Yes. History ofSTDs Ibuprofen. Over thepastyear Threeyearsago. None. Sexual partners Contraceptives Sexual activity Age13. Current medications No,itwasanormaldelivery, andmychildishealthy. Miscarriages/abortions One,butusually 2–3aday. No. Problems duringpregnancy/delivery Pregnancies Brownish. Yes, isthefirstdayofmymenstrualperiod. today Every4weeks;lastsfor7days. Pads/tampons changedthisday Started menses Fiveweeksago. Frequency ofmenstrualperiods Vaginal discharge No. Color ofthespotting Vaginal spotting Last menstrualperiod Urinary frequency/burning Patient Response for 1week,andthenitwasover. remember thenameofit.Thedoctorgavemeashotandsomepills PRACTICE CASES h 419 Maneuver tenderness sign, CVA sign, Rovsing’s Assistance of social workers to help the patient identify available financial resources. Assistance of social workers to help the Follow-up tests: Examinee mentioned the need for rectal and pelvic exams. Follow-up tests: Examinee mentioned Safe sex practices. Help with smoking cessation. Abdominal exam Inspection, auscultation, palpation, percussion, psoas sign, obturator CV examPulmonary exam Auscultation Auscultation Exam Component

questions or concerns. Examinee asked if the SP had any other Examinee discussed initial management plans: Examinee discussed initial management Examinee discussed initial diagnostic impressions. Examinee discussed initial diagnostic Examinee washed his/her hands. Examinee start the exam. asked permission to Examinee draping. Examinee used respectful painful maneuvers. Examinee did not repeat ✓ can meet with you to discuss your social situation, and she can offer you a variety of resources. Do you have any questions can meet with you to discuss your social situation, and she can offer you a variety for me? tube or a cyst on your ovary. They might also result from a complicated pregnancy, which could be indicated if your pregnancy which could be indicated if your They might also result from a complicated pregnancy, tube or a cyst on your ovary. an ensure To is an infection in your appendix, which could require surgery. test comes back positive. Another possibility CT tests, including a blood test, a urinalysis, a pregnancy test, and possibly a accurate diagnosis, we will need to run some Since cigarette smoking is associated wit scan of your abdomen and pelvis. I will also need to perform rectal and pelvic exams. if you are interested. I also recommend that a variety of diseases, I advise you to quit smoking; we have many ways to help you Our social worker to avoid pregnancy. and you use a condom every time you have intercourse to prevent STDs, including HIV, Sample Closure to a problem with your reproductive organs, such as an infection in your fallopian Ms. Anderson, your symptoms may be due Closure Physical Examination Physical PRACTICE CASES 420 Physical Examination History USMLE STEP2CS Patient Note PRACTICE CASES 421 Patient Note Patient Diagnosis #3 History Finding(s): Physical Exam Finding(s): Diagnosis #2 History Finding(s): Physical Exam Finding(s): Diagnosis #1 History Finding(s): Physical Exam Finding(s): USMLE STEP 2 CS STEP USMLE Diagnostic Workup Differential Diagnosis Differential USMLE STEP 2 CS Patient Note

History HPI: 21 yo G1P1 F c/o right lower abdominal pain that started this morning. The pain is 7/10, crampy, nonradiating, and constant. It is exacerbated by movement and accompanied by fever, nausea, vomiting, and loose stools. The patient noticed some brownish spotting this morning. No urinary symptoms; no abnormal vaginal discharge. OB/GYN: LMP 5 weeks ago. Regular periods every 4 weeks lasting 7 days. Menarche at age 13. Uncomplicated NSVD at full term 3 years ago. ROS: Negative except as above. Allergies: NKDA. Medications: Ibuprofen. PMH: STD 1 month ago, possibly treated with ceftriaxone and doxycycline. PSH: None. SH: 1 PPD for 6 years, 2–3 beers/week, no illicit drugs. Unprotected sex with multiple partners over the past year.

Physical Examination Patient is in pain. VS: WNL except for temperature of 100.5°F. Chest: No tenderness, clear breath sounds bilaterally. Heart: RRR; normal S1/S2; no murmurs, rubs, or gallops. Abdomen: Soft, nondistended, hypoactive BS, no hepatosplenomegaly. Direct and rebound RLQ tenderness, RLQ guarding, psoas sign, Rovsing’s sign, obturator sign, no CVA tenderness.

Differential Diagnosis Diagnosis #1: Appendicitis History Finding(s): Physical Exam Finding(s): Right lower abdominal pain RLQ direct and rebound tenderness Pain is exacerbated by movement RLQ guarding Nausea and vomiting Temperature 100.5°F Low-grade fever Positive Rovsing’s sign Positive psoas sign PRACTICE CASES

422 USMLE STEP 2 CS Patient Note

Diagnosis #2: Pelvic inflammatory disease History Finding(s): Physical Exam Finding(s): STD 6 months ago with untreated partner RLQ tenderness Nausea and vomiting Temperature 100.5°F Spotting Unprotected sex with multiple partners Low-grade fever

Diagnosis #3: Ruptured ectopic pregnancy History Finding(s): Physical Exam Finding(s): Last menstrual period 5 weeks ago and spotting RLQ rebound tenderness Crampy lower abdominal pain RLQ guarding Pain is exacerbated by movement Nausea and vomiting Pain is of recent onset

Diagnostic Workup

Urine hCG Pelvic exam Cervical cultures U/S—abdomen/pelvis CT—abdomen/pelvis CBC PRACTICE CASES

423 PRACTICE CASES 424 Diagnostic Workup Additional Differential Diagnoses presentation ofgynecologicdiseasescommonlymimicsappendicitis. This caseiswrittenprimarilytoelicitthedifferentialdiagnosisofRLQpaininawomanchildbearingage.The Patient NoteDifferential Diagnoses CASE DISCUSSION but quantitativehCGlevels (availableonlyviaserumtest)mayhelpdiagnoseandtreat ectopicpregnancy. Urine hCG: an endometrioma(“chocolatecyst”). this presentationinapatientwithacute,severepain,including reboundtenderness,couldbeduetoruptureof dysmenorrhea, dyspareunia,orinfertility, whichareoftenassociated.Inthesettingofestablishedendometriosis, Endometriosis: fever suggestspossiblesepticabortion. likely, butthecrampyabdominalpainandvaginalspottingmaysignalanabortion.Furthermore,presenceof Abortion: in thiscasegiventhepresenceofreboundtenderness. fever, anddiarrhea.Itcanbedifficulttodistinguishfromappendicitisgynecologicetiologiesbutislesslikely Gastroenteritis: often associatedwithovarianenlargementduetoabenignmass. Adnexal torsion: would notbeassociatedwithalatemenstrualperiod. five weeksago,rupturedectopicpregnancymustbeplacedhigheronthedifferential,asaovariancyst common thanappendicitisandPID.Inaddition,giventhepatient’s historyofhavingherlastmenstrualperiod and symptomsmayresemblearupturedectopicpregnancyasdescribedabove.However, thisdiagnosisisless and guardingareconsistentwiththisdiagnosis.Rupturemayoccuratanytimeduringthemenstrualcycle, Ruptured ovariancyst: Rovsing’s signsarenottypicalofanectopicpregnancy. ofamenorrheasuggestthisdiagnosis.However,she isexperiencingafterafive-weekperiod positivepsoasand previous tubalpregnancy),thecrampylowerabdominalpain,nauseaandvomiting,vaginalspottingthat Ruptured ectopicpregnancy: menstrual bleeding,nauseaandvomiting,ahistoryofmultiplesexpartners. Left untreated,theseinfectionscanprogresstoPID.OtherfindingssuggestiveofPIDincludeabnormal untreated partner. Thestandardtreatmentforgonorrheaandchlamydiaconsistsofceftriaxonedoxycycline. onset lowerabdominalpainandlow-gradefeverinthesettingofarecentSTDunprotectedsexwithan Pelvic inflammatorydisease(PID): sign—all ofwhichareassociatedwithappendicitis.However, theonsetofpaininappendicitisisusuallygradual. revealed directandreboundRLQtenderness,guarding,apositivepsoassign,Rovsing’s signs (painexacerbatedbymovement),appendicitisshouldcertainlybeinthedifferential.Theabdominalexam Appendicitis: Thefactthatthepatient’s wasonlyfiveweeksagomakesthisdiagnosisless lastmenstrualperiod Positiveinbothectopicand intrauterinepregnancies.Urineandserumtestsareequally sensitive, InapatientpresentingwithRLQpain,low-gradefever, nauseaandvomiting,peritoneal Thisisanunlikelydiagnosis,inpartbecausethepatient hasnohistoryofchronicpelvicpain, Viral gastroenteritispresentswithcrampyabdominalpain,nausea andvomiting,low-grade Thispresentationmaybeduetoadnexaltorsion,anuncommoncomplicationthatismost Thepatient’s sudden-onset,unilaterallowerabdominalpain,reboundtenderness, AlthoughthispatientdoesnothavepreviouslydocumentedPID(ora Suspicionishighforthisdiagnosisinapatientwhopresentswithrecent- PRACTICE CASES 425 ± the main causes of PID, are detected by causes of PID, are the main Chlamydia trachomatis, Chlamydia and Can help diagnose appendiceal or ovarian pathology. Transvaginal ultrasound can ultrasound Transvaginal appendiceal or ovarian pathology. Can help diagnose Can detect the presence of appendiceal inflammation, abscess in appendicitis, or signs of appendiceal inflammation, abscess Can detect the presence Neisseria gonorrhoeae -hCG of approximately 1500 mIU/mL); fluid in the cul-de-sac is nonspecific and may suggest 1500 mIU/mL); fluid in the cul-de-sac -hCG of approximately β Can diagnose ectopic pregnancy (gold standard), ruptured ovarian cyst, ovarian torsion, PID Can diagnose ectopic pregnancy (gold Look for cervical motion tenderness and discharge, uterine size, and adnexal masses or tenderness. adnexal masses uterine size, and and discharge, cervical motion tenderness Look for Findings are nonspecific, but leukocytosis may be seen in infection or appendicitis. but leukocytosis may be seen in Findings are nonspecific, To rule out UTI. To UA: Laparoscopy: the like. tubo-ovarian abscess, appendicitis, and corresponds to a or a ruptured ovarian cyst. ectopic pregnancy CT—abdomen/pelvis: pathology. of other GI or gynecologic CBC: Pelvic exam: cultures: Cervical DNA probes. means of U/S—abdomen/pelvis: since the last menstrual period gestational sac when the time elapsed identify an intrauterine is 35 days (this PRACTICE CASES 426 Entrance Building theDoctor-Patient Relationship Examinee Checklist Notes fortheSP Patient isa65yoF, widowedwith1daughter. Patient Description Checklist/SP Sheet Examinee Tasks HR: RR: Temp: BP: Signs Vital Virginia Black,a65-year-old female,comestotheclinic complainingofforgetfulnessandconfusion. Opening Scenario DOORWAY INFORMATION CASE 32 “At thistimeIdon’t know;westillneedtorunsometests.Whatmakesyouconcernedabouthaving Alzheimer’s?” Sample ExamineeResponse “Do youthinkIhaveAlzheimer’s disease?” Challenging QuestionstoAsk Examinee introduced selfbyname. Examinee introduced Examinee knockedonthedoor beforeentering. 4. Write thepatientnoteafterleavingroom. 3. Explainyourclinicalimpressionandworkupplantothepatient. 2. Performafocusedphysicalexam(donotperformrectal,genitourinary, orfemalebreastexam). 1. Take afocusedhistory. 135/85mmHg 16/minute 76/minute,regular Show anincreaseinDTRsoftheleftarmandleg. Pretend thatyouhavesomeweaknessinyourleftarm. hydrochlorothiazide, andaspirin”writtenonit). If asked,givetheexamineealistofyourcurrentmedications(apiecepaperwith“nitroglycerinpatch, are unabletodoso. The examineewillname3objectsforyouandasktorecallthemafterafewminutes.Pretendthat 98.0°F(36.7°C) PRACTICE CASES 427 don’t remember what happened anymore. don’t the names of my friends. help sometimes. I have some trouble with these, and I need times. My daughter shops for me. accidentally started a fire once. this for me now. forgetfulness makes me more upset. Patient Response forgetting last year. Did you see a doctor for that fall? No, it was just a bruise. Lightheadedness or feeling faintPassing outFalls Only if I stand up too quickly. Head trauma No. side of my head a while back. I I think so; I had a large bruise on the sometimes. Yes, Urinary incontinenceFeelings of sadness or depressionDifficulty sleeping died a year ago, I sometimes get sad. My Since my husband Headaches No. No. No. HouseworkPaying the billsGait problems and she does most of it. I live with my daughter, keep up. My daughter does I used to do my own bills, but I couldn’t No. out of chairs and bed) ShoppingCooking I stopped shopping, since I’ve lost my way home so many Well, I stopped cooking because I often leave the stove on and ProgressionThings that are difficult to remember my keys, the way home, off the stove, my phone number, Turning Daily activities (bathing, feeding, toi- leting, dressing, transferring into and me that it is getting worse. My daughter has told Chief complaintOnset Difficulty remembering things. I started but my daughter told me that remember exactly, I can’t Question

Examinee asked the SP to list his/her concerns and listened to the response without interrupting. SP to list his/her concerns and listened Examinee asked the own words. SP’s concerns, often using the the SP’s Examinee summarized efficiently and accurately. Examinee elicited data Examinee identified his/her role or position. identified his/her Examinee name. correctly used patient’s Examinee with the SP. made eye contact Examinee to the response. open-ended question and actively listened Examinee asked an ✓ Information Gathering Reflective Listening PRACTICE CASES 428 Physical Examination Connecting withthePatient ✓ Examinee didnotrepeatpainfulmaneuvers. Examinee usedrespectfuldraping. Examinee askedpermissiontostarttheexam. Examinee washedhis/herhands. Examinee recognizedtheSP’s emotionsandrespondedwithPEARLS.

Question rgalrisNo. Notsincethe death ofmyhusbandayearago. No. Ilivewithmydaughter. Ihavemanyfriendswhocareaboutme,besidesmydaughter. No. No. Drug allergies Iretiredafterthedeathofmyhusband. Support systems(family, friends) Myfatherandmotherdiedhealthyalongtimeago. Sexual activity Social history Tobacco Ihadabowel obstructionalongtimeago,andtheyremovedpartof Illicit druguse Ithinkthat’s enough,isn’t it? Alcohol use Idon’t knowtheirnames.(Showsthelisttoexaminee.) Occupation Yes, fora longtime. Family history Idon’t haveanappetite. I’velostweight.Idon’t knowhowmuch. Past surgicalhistory Past medicalhistory Current medications Ihadaheartattacklongtimeago. pressure High blood Appetite changes No. Weight changes rhea/constipation No. abdominal pain,nausea/vomiting,diar- Chest pain,shortnessofbreath, Heart problems No. Yes, Ihaveweaknessinmyleftarmfromastrokehadlongtime Speech difficulties Weakness/numbness/paresthesias Visual changes Any shakingorseizures No. ago. Patient Response my intestine.Idon’t rememberhowlongagoitwas. PRACTICE CASES 429 Romberg’s sign, sensory exam Romberg’s Maneuver Need to obtain history directly from other family members. Need to obtain history directly from other Need to evaluate home safety and supervision. to help the patient at home. Need to obtain community resources Follow-up tests. Pulmonary examAbdominal examNeurologic exam Auscultation Palpation Mini-mental status exam, cranial nerves, motor exam, DTRs, gait, Eye examNeck examCV exampupils, fundus Inspected Carotid auscultation Auscultation, orthostatic vital signs Exam Component

Examinee asked if the SP had any other questions or concerns. Examinee asked if the SP had any other the illness. Examinee offered support throughout Examinee discussed initial diagnostic impressions. Examinee discussed initial diagnostic plans: Examinee discussed initial management ✓ daughter. She can help me with your diagnosis, and I can answer any questions she might have about what is happening to you with your diagnosis, and I can answer any questions she might have about what She can help me daughter. and and your family to meet with the social worker to assess at-home supervision and how she can help. I would also like you you of resources that are available in the community to help you. If you would safety measures. The social worker will inform and your family to provide additional help and support. Do you have any questions like, I can remain in close contact with you for me? Sample Closure to a number of disorders that can affect the brain, many of which are treatable. Mrs. Black, your symptoms may be due to speak with your to identify the cause of your problem. I would also like to ask your permission need to run some tests We Closure PRACTICE CASES 430 Physical Examination History USMLE STEP2CS Patient Note PRACTICE CASES 431 Patient Note Patient Diagnosis #3 History Finding(s): Physical Exam Finding(s): Diagnosis #2 History Finding(s): Physical Exam Finding(s): Diagnosis #1 History Finding(s): Physical Exam Finding(s): USMLE STEP 2 CS STEP USMLE Diagnostic Workup Differential Diagnosis Differential USMLE STEP 2 CS Patient Note

History HPI: 65 yo F c/o difficulty remembering × 1 year, after death of husband. Progressively worsening memory. Affects daily activities (bathing, feeding, toileting, dressing, transferring into and out of chairs and bed, shopping, cooking, managing money, using the telephone, cleaning the house). Transient orthostatic lightheadedness with frequent falls, 1 head injury without medical attention. Upset due to memory difficulty. Weight loss, no appetite. No headache, visual changes, gait problems, difficulty sleeping, or urinary incontinence. ROS: Residual weakness in left arm after a stroke. Allergies: NKDA. Medications: HCTZ, aspirin, transdermal nitroglycerin. PMH: Hypertension, stroke, MI. The patient cannot remember exactly when she had them. PSH: Partial bowel resection due to obstruction many years ago. Patient does not remember how long ago this occurred. SH: No smoking, no EtOH, no illicit drugs. She is a widow (husband died 1 year ago), is retired, lives with her daughter, and has a good support system (family, friends). FH: Noncontributory.

Physical Examination Patient is in no acute distress. VS: WNL, no orthostatic changes. HEENT: Normocephalic, atraumatic, PERRLA, no funduscopic abnormalities. Neck: Supple, no carotid bruits. Chest: Clear breath sounds bilaterally. Heart: RRR; normal S1/S2; no murmurs, rubs, or gallops. Abdomen: Soft, nondistended, nontender, no hepatosplenomegaly. Neuro: Mental status: Alert and oriented × 3, spells backward but can’t recall 3 items. Cranial nerves: 2–12 intact. Motor: Strength 5/5 in all muscle groups except 3/5 in left arm. DTRs: Asymmetric 3+ in left upper and lower extremities, 1+ in the right, Babinski bilaterally. Cerebellar: Romberg. Gait: Normal. Sensation: Intact to pinprick and soft touch.

Differential Diagnosis Diagnosis #1: Alzheimer’s disease History Finding(s): Physical Exam Finding(s): Steady cognitive decline Failed 3-item recall Memory impairment Impaired executive functioning Decline in activities of daily living PRACTICE CASES

432 USMLE STEP 2 CS Patient Note

Diagnosis #2: Vascular (“multi-infarct”) dementia History Finding(s): Physical Exam Finding(s): Previous stroke Decreased strength in left upper extremity History of coronary artery disease (MI) DTRs 3+ in left upper and lower extremities Hypertension Positive Babinski bilaterally Impaired executive functioning Failed 3-item recall

Diagnosis #3: Dementia syndrome of depression History Finding(s): Physical Exam Finding(s): Dysphoria after husband’s death Failed 3-item recall Impaired executive functioning Memory impairment

Diagnostic Workup

CT—head or MRI—brain EEG or SPECT CBC

Serum B12, TSH, RPR Electrolytes, calcium, glucose, BUN/Cr PRACTICE CASES

433 PRACTICE CASES 434 search forcausessuchaselectrolyte disturbances,neoplasms,orinfarcts. The goalofthediagnosticworkup forcognitivedeclineistoruleoutpotentiallyreversible causesofdementiaand Diagnostic Workup Additional Differential Diagnoses Patient NoteDifferential Diagnoses further complicatingtreatment. of partiallyreversiblecausesdementia.Moreover, thetopthreediagnosesforthispatientencountermaycoexist, are primarilyclinicaldiagnoses,andthereforetheinitialdiagnosticworkupshouldbedirectedtowardexclusion sought fromotherfamilymemberstoestablishanaccuratetimecourseofcognitivedecline.Thedementiasyndromes degree ofaphasia,apraxia,agnosia,and/orimpairedexecutivefunction.Additionalhistoricalinformationmustbe isanacquired,progressiveimpairmentincognitivefunctionthatincludesamnesiaaccompaniedbysome Dementia CASE DISCUSSION precede thecharacteristicmegaloblasticanemia. associated withotherneurologicsymptoms,suchasparesthesias andlegweakness.Onoccasion,dementiamay at riskforthisdeficiency. Itcancausedepression,irritability, paranoia,confusion,anddementiabutisusually patients withdementia.However, therearenoclassicsignsorsymptomstosuggesthypothyroidisminthiscase. Hypothyroidism: B Vitamin exacerbated hermentalstatuschangesinrecentweeksormonths. her cognitivedeclinespansatleastayear, itispossiblethatacomorbid chronicsubduralhematomacouldhave Subdural hematoma: but becamemorenoticeabletoherchildrenafterhusbanddied. conducted. However, itismorelikelythatthispatient’s cognitive declinehasbeenprogressiveforseveralyears dysexecutive syndromeandisareversiblecauseofdementia.Athoroughscreeningfordepressionshouldbe or, morecommonly, coexistwithdementia.IncontrasttoAlzheimer’s disease,DSDpresentsprimarilyasa of neurocognitivedecline(vs.youngpatients,inwhomdysphoriapredominates).Thesesymptomsmaymimic the patient’s husbandmayindicatedepression.Intheelderly, depressioncanpresentatypicallywithsymptoms Dementia syndrome ofdepression (DSD): earlier lossofexecutivefunctionandpersonalitychanges. is moreconsistentwiththatofAlzheimer’s disease.Inaddition,vasculardementiamaybecharacterizedbyan temporally relatedtoarecentstroke.Thispatient’s strokeisnotrecent,andthepatternofhercognitivedecline this case.Invasculardementia,thereisclassicallymoreofafluctuating,stepwisecognitivedeteriorationthat the patient’s historyofatheroscleroticvasculardisease(eg,stroke, MI), itcouldcertainlybecontributingin Vascular (“multi-infarct”) dementia: are impairmentinmemoryandvisuospatialabilities.Alzheimer’s diseaseisaclinicaldiagnosis. characterized byasteady, ofyears.Theearliestfindings progressivedeclineincognitive functionoveraperiod with Alzheimer’s disease,themostcommoncauseofdementia.Alzheimer’s diseaseusuallyhasaninsidiousonset Alzheimer’s disease: 12 deficiency: Thiscancauseneuropsychiatricsymptoms(oftenalate finding) andmustberuledoutin Thispatientpresentswithasteadydeclineincognitivefunctionthatismostconsistent Thisshouldberuledoutgiventhepatient’s historyoffallsandhead trauma.Although Apriorbowelresection(eg,oftheterminalileum) mayputthepatient Vascular dementiaoftencoexistswithAlzheimer’s disease,andgiven Thetimecourseofcognitivedeclinefollowingthedeath PRACTICE CASES 435 deficiency. 12 To screen for medical conditions that can present with cognitive conditions that can present with cognitive screen for medical To To screen for partially reversible causes of dementia (RPR can be restricted to patients causes of dementia (RPR can be restricted screen for partially reversible To Used in rare cases to help differentiate delirium from depression or dementia. to help differentiate delirium from depression Used in rare cases The most sensitive exam with which to look for focal CNS lesions or atrophy. focal CNS lesions which to look for sensitive exam with The most Used to look for a crescent-shaped, hyperdense extra-axial mass in subdural hematoma, subdural hematoma, extra-axial mass in hyperdense for a crescent-shaped, Used to look , TSH, RPR: 12 Used to look for macrocytic anemia in vitamin B Used to look for macrocytic who manifest signs of neurosyphilis). who manifest signs Serum B BUN/Cr: calcium, glucose, Electrolytes, uremia). hypercalcemia, hyperglycemia, dysfunction (eg, hypernatremia, CT—head: hydrocephalus). (as in normal pressure or dilated ventricles masses, strokes, intracerebral MRI—brain: EEG or SPECT: CBC: PRACTICE CASES 436 Notes fortheSP Patient isa46yoM. Patient Description Checklist/SP Sheet Examinee Tasks HR: RR: Temp: BP: Signs Vital Gary Mitchell,a46-year-old male,comestotheofficecomplainingoffatigue. Opening Scenario DOORWAY INFORMATION CASE 33 confidential andwillnotleavethisroom.” yourself ortriedtodoso?”Or“Tell memoreaboutyourconcernAIDS.Everythingthatyou tellmeis continue. Alternatively, youcansay, “Itsoundsasthough you’re losinghope.Haveyouthoughtabouthurting This patientclearlyhasmoretosay. Silenceisappropriatehere,orthepatientshouldbesubtlyencouragedto Sample ExamineeResponse Challenging QuestionstoAsk 4. Write thepatientnoteafterleavingroom. 3. Explainyourclinicalimpressionandworkupplantothepatient. 2. Performafocusedphysicalexam(donotperformrectal,genitourinary, orfemalebreastexam). 1. Take afocusedhistory. 120/85mmHg 12/minute 65/minute,regular “I amafraidthatImighthaveAIDS.” “I thinkthatlifeisfullofmisery. Whydowehavetolive?” Start yawningastheexamineeentersroom. Speak andmoveslowly. Look sad,anddon’t smile. 98.2°F(36.8°C) PRACTICE CASES 437 Patient Response for help, and the car blowing up. I feel so scared and helpless. I wake up multiple times at night and feel sleepy all day. from the car before it blew up. Same throughout the day. energy to work. couldn’t. sometimes have problems falling asleep, but I wake up I don’t Well, because of nightmares. I always see the accident, my friend calling Sleeping problems (falling asleep, stay- ing asleep, early waking) of stimuliAvoidance No. Weight changesWeight Feeling of depressionSuicidal thoughts/plans/attemptsFeelings of blame or guilt I think of suicide sometimes but have had no plans or attempts. I feel sad all the time. Yes, I have gained 6 pounds over the past 3 months. It was an accident. I tried to help my friend but know. I don’t Progression of the fatigue during the day Affecting job/performanceAppetite changes have the concentrate on my work anymore. I don’t I can’t Yes, Loss of appetite. OnsetAssociated eventsInjuries related to the accident No. I was in a car accident 3 months ago, and I failed to save my friend Three months ago. Chief complaint Feeling tired, no energy. Question

Examinee elicited data efficiently and accurately. Examinee elicited data efficiently and me more about that.”). about AIDS (eg, “Tell concern Examinee explored the SP’s Examinee asked an open-ended question and actively listened to the response. open-ended question and actively listened Examinee asked an concerns and listened to the response without interrupting. Examinee asked the SP to list his/her own words. often using the SP’s concerns, Examinee summarized the SP’s Examinee introduced self by name. his/her role or position. Examinee identified name. used patient’s Examinee correctly contact with the SP. Examinee made eye Examinee knocked on the door before entering. Examinee knocked ✓ Examinee Checklist Examinee Relationship the Doctor-Patient Building Entrance Information Gathering Reflective Listening PRACTICE CASES 438 Physical Examination Connecting withthePatient ✓ Examinee didnotrepeatpainfulmaneuvers. Examinee usedrespectfuldraping. Examinee askedpermissiontostarttheexam. Examinee washedhis/herhands. Examinee recognizedtheSP’s emotionsandrespondedwithPEARLS.

Question rgalrisNo. Theusual.Ihaven’t changedanythinginmydietmore than10 Notinterested anymore.Ihaveagirlfriend,andwebeen No. Onepackadayfor25years. Drug allergies Never. Ihave2or3beers amonth. Sexual activity Accountant. health. Myparentsarealiveandingood Diet Exercise Tobacco None. Illicit druguse Alcohol use Well, Ihadsomeburningduringurination.don’t reallyremember Occupation None. Family history Myhairisfallingoutmorethanusual. Yes. Past surgicalhistory Past medicalhistory Current medications Yes, Ican’t concentrateonmywork. Skin/hair changes Cold intolerance dominal pain,diarrhea/constipation) chest pain,shortnessofbreath,ab- Associated symptoms(fever, chills, Mygirlfriendandparentsareverysupportive.TheyknowI’vebeen Loss ofconcentration Support system(friends,family) C. Itookantibioticsforaweek.Thiswas5monthsago. the diagnosisthatdoctorreached,butitstartedwithletter No. having ahardtimeandsuggestedIcomeseeyoutosortitout. the past. make mefeeluncomfortable.Ihavehadseveralsexualpartnersin together forthepast6months.Idon’t usecondoms becausethey years. Patient Response PRACTICE CASES p 439 at d. Maneuver thyroid gland Sources of support (eg, trusted friends and loved ones) and information about community groups. Sources of support (eg, trusted friends Possible need for referral to a psychiatrist. or go to the ED for any suicidal thoughts or plans). Suicide contract (ie, contact your physician Safe sex practices. HIV testing and consent. Depression counseling: Follow-up tests. Lifestyle modification smoking cessation). (diet, exercise, relaxation techniques, Pulmonary examAbdominal examExtremities Auscultation Auscultation, palpation, percussion DTRs Inspection, checked Head and neck examHead and CV exam nodes; and throat, lymph Inspected conjunctivae, mouth examined Auscultation Exam Component

Examinee asked if the SP had any other questions or concerns. Examinee asked if the SP had any other Examinee discussed initial diagnostic impressions. Examinee discussed plans: Examinee discussed initial management ✓ Mr. Mitchell, it appears that your life has been very stressful lately, and my suspicion is that you may be clinically depresse and my been very stressful lately, Mitchell, it appears that your life has Mr. as you have risk I would like to order some blood tests, including one for HIV, Before I make a definitive diagnosis, however, Once we have completed these tests, we should have a better idea of what is causing factors for sexually transmitted diseases. in activities th and participate regularly, your fatigue. In the meantime, I strongly recommend that you quit smoking, exercise yourself, you will call someone who can hel you find relaxing. I would also like you to promise me that if you feel like hurting me? you or go immediately to an emergency department. Do you have any questions for Sample Closure Closure PRACTICE CASES 440 Physical Examination History USMLE STEP2CS Patient Note PRACTICE CASES 441 Patient Note Patient Diagnosis #3 History Finding(s): Physical Exam Finding(s): Diagnosis #2 History Finding(s): Physical Exam Finding(s): Diagnosis #1 History Finding(s): Physical Exam Finding(s): USMLE STEP 2 CS STEP USMLE Diagnostic Workup Differential Diagnosis Differential USMLE STEP 2 CS Patient Note

History HPI: 46 yo M c/o fatigue × 3 months. Fatigue began after unsuccessful attempt to save his friend after a car accident. Constant fatigue throughout the day. Low energy. Decreased concentration that is negatively affecting job as accountant. Decreased appetite, but gained 6 lbs over 3 months. Multiple awakenings and difficulty staying asleep due to recurrent nightmares about accident. Feels sleepy throughout the day. Feelings of being depressed and helpless. Passive suicidal ideation but no suicide plans/attempts. Cold intolerance. Hair loss. Loss of interest in sex. No constipation. ROS: Negative except as above. Allergies: NKDA. Medications: None. PMH: Urethritis (possibly chlamydia), treated 5 months ago. PSH: None. SH: 1 PPD for 25 years, 2 beers/month. History of unprotected sex with multiple female partners. FH: Noncontributory.

Physical Examination Patient is in no acute distress, looks tired with a flat affect, speaks and moves slowly. VS: WNL. HEENT: No conjunctival pallor, mouth and pharynx WNL. Neck: No lymphadenopathy, thyroid normal. Chest: Clear breath sounds bilaterally. Heart: RRR; normal S1/S2; no murmurs, rubs, or gallops. Abdomen: Soft, nondistended, nontender, BS, no hepatosplenomegaly. Extremities: No edema, normal DTRs in lower extremities. PRACTICE CASES

442 USMLE STEP 2 CS Patient Note

Differential Diagnosis Diagnosis #1: Major depressive disorder History Finding(s): Physical Exam Finding(s): Dysphoria, anhedonia Loss of appetite Passive suicidal ideation Decreased energy/fatigue Impaired concentration Early awakening

Diagnosis #2: Hypothyroidism History Finding(s): Physical Exam Finding(s): Fatigue for 3 months Cold intolerance Hair loss Weight gain

Diagnosis #3: Posttraumatic stress disorder History Finding(s): Physical Exam Finding(s): Nightmares about the trauma Negative mood/anhedonia Decreased concentration Difficulty staying asleep

Diagnostic Workup

TSH CBC HIV antibody PRACTICE CASES

443 PRACTICE CASES 444 such ascancer. Fatigue isacommon,nonspecificcomplaintwithmanyetiologiesrangingfromsimpleoverexertiontoseriousdiseases Patient NoteDifferential Diagnoses CASE DISCUSSION Diagnostic Workup Additional Differential Diagnoses accident andthereforedoesnotmeetthefullcriteriaatthistime. anddifficultystayingasleep),hedoesnotavoidstimulirelatedtothe concentration, anhedonia,negativemood, reactivity. AlthoughthispatienthasmanyofthesymptomsPTSD(nightmaresabouttrauma,decreased andalterationsinarousal clusters: intrusion,avoidance,negativealterationsincognitionandmood, exposure toatraumaticeventthatmeetsspecificrequirementsandsymptomsfromeachoffoursymptom experience, andthedurationofsymptomsislongerthanamonth.DSM-5criteriaincludehistory Posttraumatic stress disorder (PTSD): hair loss,andweightgainareadditionalnonspecificsymptomsthatsuggestthisdiagnosis. Hypothyroidism: the criteriaoutlinedin many classicsymptoms.Themnemonic Major depressive disorder (MDD): HIV antibody: CBC: TSH: there arefrontallobelesionsduetoinfectionormalignancy). be testedforHIV. However, itishighlyunlikelythatHIVinfectionaccountsforhiscurrentdepression(unless HIV infection: symptoms mustsignificantlyimpairdailyfunctioning. oranhedonia asoneofthefive,fortwoweeks,and least fiveoftheabovesymptoms,includingdepressedmood I change in nterest, feelingsof Ascreeningtestforhypothyroidism. To ruleoutanemia. A ppetite/weight changes, GivenhishistoryofSTDsandunprotectedsexwithmultiplepartners,thispatientshouldalso To ruleoutHIVinfection. Thisshouldberuledoutinapatientwithfatigueformonths.Thepatient’s coldintolerance, G uilt (worthlessness),decreased Diagnostic andStatisticalManualofMentalDisorders P sychomotor agitationorslowing,and ThispatientmeetsthecriteriafordiagnosisofMDD,exhibiting SIG ECAPS PTSDusuallyoccurswithinthreemonthsofthetraumatic E helpsrecallthesesymptoms: nergy (fatigue),decreased S uicidal ideation.Inordertomeet (DSM-5),apatientmustreportat C oncentration/ S leep disturbance,decreased C ognition, PRACTICE CASES 445 98.2°F (36.8°C) Look anxious and pale. elicit pain when touched. Exhibit bruises on the face and arms that 80/minute, regular 13/minute 120/85 mm Hg Examinee identified his/her role or position. name. Examinee correctly used patient’s Examinee made eye contact with the SP. Examinee knocked on the door before entering. Examinee introducedself by name. 2. Perform a focused physical exam (do not perform rectal, genitourinary, or female breast exam). not perform rectal, genitourinary, 2. Perform a focused physical exam (do workup plan to the patient. 3. Explain your clinical impression and the patient note after leaving the room. 4. Write 1. Take a focused history. a focused 1. Take Sample Examinee Response a lot of water could be the first sign of know for sure, but I want to run some tests. Drinking “At this point I don’t diabetes, and we will need to check for that.” Challenging Questions to Ask reason is?” What do you think the doctor. “I am drinking a lot of water, DOORWAY INFORMATION DOORWAY CASE 34 CASE Building the Doctor-Patient Relationship Building the Doctor-Patient Entrance Examinee Checklist Checklist/SP Sheet Patient Description married with 2 children. Patient is a 32 yo F, Notes for the SP RR: HR: Examinee Tasks Jessica Lee, a 32-year-old female, comes to the office complaining of fatigue. female, comes Jessica Lee, a 32-year-old Vital Signs BP: Temp: Opening Scenario PRACTICE CASES 446 Information Gathering Reflective Listening ✓ Examinee eliciteddataefficientlyandaccurately. Examinee summarizedtheSP’s concerns,oftenusingtheSP’s ownwords. Examinee askedtheSPtolisthis/herconcernsandlistenedresponsewithoutinterrupting. Examinee askedanopen-endedquestionandactivelylistenedtotheresponse.

Question oso ocnrto Yes, Ican’t concentrateonmywork. No. Loss ofconcentration ing asleep,earlywaking,snoring) No. Sleeping problems(fallingasleep,stay- Yes, Ithinkambeingawkward.Itismyfault. Emergency plan the abuse Any familymemberswhoknowabout No. Presence ofgunsathome Feelings ofblameorguilt Suicidal thoughts/plans/attempts threatened? SometimesIfeelafraid,especiallywhenmyhusbandgetsdrunk. Are thechildrenbeingabusedor Ifelldownthestairsandhurtmyself(looksanxious).Itismyfault. husband? trauma oraccidentsasaresultofyour Have youeverexperiencedanyhead No. SometimesIfeelsad. Feeling safe/afraidathome appetite. Ihaveaverygood abused byanybody Being physicallyoremotionallyhurt Cause ofbruises Yes, Idon’t haveenergytowork. Feeling ofdepression Fivemonthsago. Weight changes Appetite changes None. Affecting job/performance Feelingtired,weak,noenergy. ing theday Change invision(doublevision)dur- day Progression ofthefatigueduring Associated events Onset Chief complaint No. more tiredandweak. I feelokayinthemorning;thengraduallystartfeelingmoreand No. No. He shouldbemoreattentive. Well, heslappedmyyoungersontheotherdayforbreakingaglass. No. me verymuch,andhepromisesnottodoitagain. Well, sometimeswhenmyhusbandgetsangrywithme,butheloves don’t alwayspayattention. Patient Response PRACTICE CASES 447 Patient Response Patient nursing home with Alzheimer’s. nursing home with Alzheimer’s. eat healthier foods. I am trying to change because my dad had diabetes. No. None. DietSexual activityDrug allergies one, but I know that I am overweight and should really have I don’t feel any desire for sex, but we do it when my husband wants. I don’t No. Alcohol useIllicit drug useTobaccoExercise No. Never. No. No. Past surgical historyFamily historyOccupation I fell and broke my arm a year ago. My father had diabetes and died of a heart attack. My mother is in a Nurse. chest pain, shortness of breath, ab- chest pain, shortness dominal pain, diarrhea/constipation, cold intolerance, skin/hair changes) Current medicationsPast medical history None. None. Polyuria or change in the Pain during urination color of urine Polydipsia chills, (fever, Associated symptoms to go to the bathroom more often during the day. I have Yes, water. I feel thirsty all the time, and I drink a lot of Yes, Menstrual periodMenstrual periodLast menstrual Urinary symptoms ago. weeks heavy; lasts 7 days. Regular and Two night to urinate. I recently started to wake up at Question

Examinee washed his/her hands. Examinee asked permission to start the exam. Examinee used respectful draping. Examinee did not repeat painful maneuvers. Examinee recognized the SP’s emotions and responded with PEARLS. Examinee recognized the SP’s ✓ Connecting with the Patient Physical Examination PRACTICE CASES 448 problems withyourbloodsugarorhormones.Doyouhaveanyquestions? to talkto.Iamalsoconcernedaboutyourfrequenturinationandthirst.willrunasimplebloodtestseeifyouhave any contact informationforyouregardingwheretogohelpiforyourchildrenareinacrisisjustwantsomeone protective servicesifIhavereasontobelievethatyourchildrenarebeingabused.willbringbacksometelephonenumbers a available forhelpandsupportwheneveryouneedit.Althougheverythingwediscussisconfidential,Imustinvolvechild Ms. Lee,Iamconcernedaboutyoursafetyandrelationshipwithhusband.wouldlikeyoutoknowthat am Sample Closure Closure ✓ Examinee askediftheSPhadanyotherquestionsorconcerns. Examinee discussedinitialmanagementplans: Examinee discussedinitialdiagnosticimpressions.

ExamComponent xrmte Inspection,motorexam,DTRs Auscultation,palpation,percussion Auscultation Auscultation Extremities examined Inspectedconjunctivae,mouthandthroat,lymphnodes; Abdominal exam Pulmonary exam CV exam Head andneckexam Domestic violencecounseling: Follow-up tests. Safety planning. Support groupinformation,includingcontactnumbersorWeb sites. being harmed.” “Everything wediscussisconfidential,butImustinvolvechildprotectiveservicesifyourchildrenare “I careaboutyoursafety, andIamalwaysavailableforhelpsupport.” thyroid gland Maneuver nd PRACTICE CASES 449 Patient Note Patient USMLE STEP 2 CS STEP USMLE Physical Examination History USMLE STEP 2 CS Patient Note

Differential Diagnosis Diagnosis #1 History Finding(s): Physical Exam Finding(s):

Diagnosis #2 History Finding(s): Physical Exam Finding(s):

Diagnosis #3 History Finding(s): Physical Exam Finding(s):

Diagnostic Workup PRACTICE CASES

450 USMLE STEP 2 CS Patient Note

History HPI: 32 yo F c/o fatigue and weakness × 5 months. Fatigue increases throughout the day. Loss of energy and concentration, which is affecting job as nurse. Patient admits that husband, who is an alcoholic, has beaten her. At least 1 episode of physical abuse directed at youngest son. Patient attempts to defend husband’s actions. Feels guilty. Self-blame. Has not reported abuse. No head trauma or accidents due to husband. No emergency plan. Feels sad but denies suicidal ideation. Polyuria, polydipsia, nocturia × 5 months. LMP 2 weeks ago, menstrual period is regular, q28 days, lasting 7 days of heavy flow. No dysuria or change in color of urine. No constipation, cold intolerance, or change in appetite or weight. No sleep problems. ROS: Negative except as above. Allergies: NKDA. Medications: None. PMH/PSH: None. SH: No smoking, no EtOH. Sexually active with her husband; decreased sexual desire. FH: Diabetic father died from a heart attack; mother is in a nursing home with Alzheimer’s disease.

Physical Examination Patient is obese, in no acute distress, looks anxious. VS: WNL. HEENT: Pale conjunctivae. Neck: No lymphadenopathy, thyroid normal. Chest: Clear breath sounds bilaterally. Heart: RRR; normal S1/S2; no murmurs, rubs, or gallops. Abdomen: Soft, nondistended, nontender, BS, no hepatosplenomegaly. Extremities: Muscle strength 5/5 throughout; DTRs 2+; symmetric, painful bruises on both arms. PRACTICE CASES

451 USMLE STEP 2 CS Patient Note

Differential Diagnosis Diagnosis #1: Domestic violence History Finding(s): Physical Exam Finding(s): Admits to physical abuse Symmetrical bruises on extremities Exhibits self-blame Attempts to defend husband Episode of abuse directed at child

Diagnosis #2: Diabetes mellitus History Finding(s): Physical Exam Finding(s): Polyuria, polydipsia Obesity Family history of diabetes

Diagnosis #3: Anemia History Finding(s): Physical Exam Finding(s): Fatigue/weakness Conjunctival pallor Heavy menstrual flow

Diagnostic Workup

Serum glucose, HbA1c CBC

Serum iron, ferritin, TIBC, serum B12 UA Electrolytes PRACTICE CASES

452 PRACTICE CASES 453 deficiency anemia. 12 To confirm a clinical suspicion of central DI. To levels should also be ordered to check for B 12 This patient does not currently meet the criteria for MDD. However, the criteria for MDD. However, This patient does not currently meet To look for mass lesions in central DI. To Aside from domestic violence issues, many of the patient’s symptoms can be violence issues, many of the patient’s Aside from domestic This is an uncommon disease characterized by polyuria (of low specific gravity) and This is an uncommon disease characterized To screen for DM. To : 1c The patient is clearly a victim of domestic violence and of her husband’s alcoholism. This of her husband’s a victim of domestic violence and The patient is clearly Increasing fatigue as the day progresses is highly nonspecific. By contrast, this disease Increasing fatigue as the day progresses Nonspecific symptoms such as fatigue and weakness may suggest this common diagnosis. Nonspecific symptoms such as fatigue Hypernatremia may be seen in DI. This may also help explain her fatigue and weakness. Menstruating females often have an iron females often have an iron explain her fatigue and weakness. Menstruating This may also help To investigate anemia. If the CBC is suggestive of iron deficiency anemia, the next step would be to order investigate anemia. If the CBC is suggestive of iron To Glucose or protein may be present in DM. MRI—brain (pituitary protocol): challenge test”): nasal spray test (“vasopressin DDAVP CBC: a serum iron level, ferritin, and TIBC. Serum B UA: Electrolytes: involves fluctuating muscle weakness and presents with ptosis, diplopia, difficulty chewing or swallowing, involves fluctuating muscle weakness of which the patient has denied. respiratory difficulties, and/or limb weakness—all Serum glucose, HbA Diabetes insipidus (DI): is caused by a deficiency of or resistance to vasopressin. Central diabetes polydipsia. It has many etiologies and family obesity, trauma, benign tumors, or surgery). The patient’s can be idiopathic or acquired (eg, post−head of DI support DM as a more probable explanation for her symptoms. history of DM, and lack of acquired causes Myasthenia gravis: her history of intimate partner violence increases her risk of developing a mental disorder, with the degree of risk with the increases her risk of developing a mental disorder, her history of intimate partner violence episodes.directly related to the frequency of violent Hypothyroidism: weight/appetite changes, or cold intolerance. Hypothyroidism does the patient denies constipation, However, admitted physical abuse. not explain polyuria, polydipsia, or the deficiency anemia. Conjunctival pallor on exam has a high likelihoodConjunctival pallor on exam has a high deficiency anemia. < 30% ratio for predicting a hematocrit (Hb < 10 g/dL). (MDD): disorder Major depressive Diabetes mellitus (DM): Diabetes mellitus also be asked family history put her at risk. She should diabetes. Her obesity and positive explained by new-onset (and may be its complication of hyperglycemia yeast infections, which are a frequent about any recent vaginal initial presenting symptom). Anemia: Domestic violence: polydipsia. her symptoms but not the polyuria or can explain many of Diagnostic Workup Additional Differential Diagnoses Additional Differential CASE DISCUSSION CASE Diagnoses Note Differential Patient PRACTICE CASES 454 Entrance Building theDoctor-Patient Relationship Examinee Checklist Notes fortheSP Patient isa27yoM. Patient Description Checklist/SP Sheet Examinee Tasks HR: RR: Temp: BP: Signs Vital Jack Edwards,a27-year-old male,comestotheEDcomplainingofseeingstrangewritingonwall. Opening Scenario DOORWAY INFORMATION CASE 35 are causingyoutoseethiswriting.Inanycase,wegoing todosometeststryfigureoutwhatisgoingon.” “I don’t thinkanyoneistryingtogiveyouinstructions. Ifyouhavebeentakingillicitdrugs,itmaybethatthedrugs Sample ExamineeResponse “Do youthinksomeoneistryingtogivemeinstructionsthroughthewritingIseeonwall?” Challenging QuestionstoAsk Examinee madeeyecontact with theSP. Examinee correctlyusedpatient’s name. Examinee identifiedhis/her role orposition. selfbyname. Examinee introduced Examinee knockedonthedoorbeforeentering. 4. Write thepatientnoteafterleavingroom. 3. Explainyourclinicalimpressionandworkupplantothepatient. 2. Performafocusedphysicalexam(donotperformrectal,genitourinary, orfemalebreastexam). 1. Take afocusedhistory. 140/80mmHg 15/minute 110/minute,regular Give theimpressionthatyouarestaringatwall. Sit uponthebed. 98.3°F(36.8°C) PRACTICE CASES 455 Patient Response No, but sometimes I feel very sleepy during the day. No. No. No. Sometimes. Fever No. Do you ever have these symptoms without drug use? Sleeping problemsDo you fall asleep suddenly during the day? No, but sometimes I find it difficult to wake up in the morning. harm yourself or others? Do you think about harming yourself or others? Enjoyment of daily activitiesMental illness in family Yes. No. Hearing lossContent of the voicesFeeling of being controlledDo the voices/writing order you to No. understand them; the voices seem distant. I can’t No. Major life changes or stressorsHeadache changes or vision lossVisual Not really. Hearing changes None. None. I feel as though I hear strange voices when I see the writing. Frequency Do you see the writing while your eyes are closed? Alleviating factorsExacerbating factors 3–4 times since yesterday. It has happened None. None. OnsetContentDurationConstant/intermittent and goes. It comes It started yesterday. read it most of the time. and I can’t It is not clear, than a minute. It lasts less Chief complaint I have been seeing strange writing on the wall. Question

Examinee elicited data efficiently and accurately. Examinee elicited data Examinee asked an open-ended question and actively listened to the response. listened to the question and actively asked an open-ended Examinee interrupting. response without and listened to the his/her concerns asked the SP to list Examinee own words. SP’s concerns, often using the the SP’s Examinee summarized ✓ Reflective Listening Reflective Information Gathering PRACTICE CASES 456 Physical Examination Connecting withthePatient ✓ ✓ Examinee didnotrepeatpainfulmaneuvers. Examinee usedrespectfuldraping. Examinee askedpermissiontostarttheexam. Examinee washedhis/herhands. Examinee recognizedtheSP’s emotionsandrespondedwithPEARLS.

ExamComponent Question erlgcea Mini-mental statusexam,cranialnerves,motorDTRs,gait, Palpation Auscultation,vitalsigns Auscultation Inspectedpupils;checkedforreactivity Neurologic exam Abdominal exam Pulmonary exam CV exam Eye exam No. Yes, Iusethem. No. Yes, withmygirlfriend. Yes, Ihavesmokedapackdayfor6years. Drug allergies Yesterday atapartymyfriend’s house. Occasionally. Use ofcondoms No. Sexual activity Iworkasabartender. Exercise Angeldust;sometimesEcstasy. pressure. Myfatherhad high blood Tobacco Last useofillicitdrugs No. Which illicitdrugsdoyouuse? None. Illicit druguse Alcohol use None. Occupation None. None. Family history Past surgicalhistory Head trauma Past medicalhistory Current medications Weight changes sensory exam Patient Response Maneuver PRACTICE CASES t 457 Follow-up tests. Examinee discussed initial diagnostic impressions. discussed initial diagnostic Examinee plans. discussed initial management Examinee SP had any other questions or concerns. Examinee asked if the Sample Closure mental problem or even a drug use, or they may be the result of a symptoms could be caused by your illicit Edwards, your Mr. I recommend that you stop using illici try to clarify your condition. In addition, will run some tests to medical condition. We Do you have any questions for me? drugs and quit smoking. Closure PRACTICE CASES 458 Physical Examination History USMLE STEP2CS Patient Note PRACTICE CASES 459 Patient Note Patient Diagnosis #3 History Finding(s): Physical Exam Finding(s): Diagnosis #2 History Finding(s): Physical Exam Finding(s): Diagnosis #1 History Finding(s): Physical Exam Finding(s): USMLE STEP 2 CS STEP USMLE Diagnostic Workup Differential Diagnosis Differential USMLE STEP 2 CS Patient Note

History HPI: 27 yo M c/o episodes of seeing strange writing on the wall since yesterday. These episodes last less than a minute and have happened 3–4 times. The patient states that the writing is not clear and he cannot read the messages, but he thinks he might be getting instructions from them. He denies any other visual changes or visual loss. The patient also mentions hearing strange voices associated with the writing, adding that he cannot understand them either. He admits to having used illicit drugs 1 day before these events. He denies any headache, seizures, head trauma, or previous similar episodes. No appetite or weight changes, fever, or sleep problems. ROS: Negative except as above. Allergies: NKDA. Medications: None. PMH: None. PSH: None. SH: 1 PPD for 6 years; uses PCP (“angel dust”) and MDMA (Ecstasy) occasionally; no EtOH. Works as a bartender. FH: Noncontributory.

Physical Examination Patient seems anxious and in mild distress. VS: HR 110, BP 140/80 HEENT: Pupils dilated, vertical gaze nystagmus. Chest: Clear breath sounds bilaterally. Heart: Tachycardic; normal S1/S2; no murmurs, rubs, or gallops. Abdomen: Soft, nontender, nondistended, no hepatosplenomegaly. Neuro: Mental status: Alert and oriented × 3, spells backward and recalls 3 objects. Cranial nerves: 2–12 intact. Motor: Strength 5/5 in all muscle groups. DTRs: Symmetric. Gait: Normal.

Differential Diagnosis Diagnosis #1: PCP intoxication History Finding(s): Physical Exam Finding(s): Drug use 1 day before presentation Tachycardia (HR 110/minute) Visual hallucinations Hypertension (BP 140/80) Noncommand auditory hallucinations Vertical gaze nystagmus Delusions PRACTICE CASES

460 USMLE STEP 2 CS Patient Note

Diagnosis #2: Substance-induced psychosis History Finding(s): Physical Exam Finding(s): Drug use 1 day before presentation Pupils dilated Visual hallucinations Noncommand auditory hallucinations Delusions No history of non-drug-related psychosis Does not associate drug use with presentation

Diagnostic Workup

Urine toxicology Electrolytes CPK Urine myoglobin Mental status exam PRACTICE CASES

461 PRACTICE CASES 462 Diagnostic Workup Additional Differential Diagnoses Patient NoteDifferential Diagnoses CASE DISCUSSION cases oflargeingestions.SerumCPKandurinemyoglobinshouldbemeasuredtoruleoutthiscomplication. E prominent thanothersymptoms. or withdrawal.Incontrasttointoxicationwithperceptualdisturbances,hallucinationsanddelusionsaremore drugs) becapableofcausingpsychosisandthatthesymptomsmoreseverethanexpectedforintoxication this diagnosis.Substance-inducedpsychosisrequiresthatthesubstanceingested(medications,alcohol,orillicit the insighttoidentifytheirrecentdruguseasacauseofsymptoms.Thepresentationisconsistentwith Substance-induced psychosis: common symptomsofPCPintoxication: substance exposure,thediagnosis ofpsychopathologyisnotpossible. Mental statusexam: CPK andurinemyoglobin: Electrolytes: cannabinoids, cocaine,opioids,andphencyclidine(PCP). Urine toxicology: usually lastforafewseconds.Thisdiagnosisisunlikelybecausethepatienthasnoknownhistoryofseizures. Seizure: or withoutanemotionaltrigger)isclassifiedasmajorsomnolencedisorderinDSM-5. symptoms arenotsevereenoughtomeritthisdiagnosis.Narcolepsywithoutcataplexy(muscularweaknesswith associated withvisualhallucinationsaswell.Althoughthispatientcomplainsofdaytimesleepiness,his falling asleep(hypnagogic)orjustafterwakingup(hypnopompic).Auditorytactilesensationscanbe Narcolepsy: history tosupportasecondarymedicalcondition. parathyroid, oradrenalabnormalities;andhepaticrenaldisorders.However, thereisnothinginthispatient’s These includeneurologicproblemssuchasCNSinfectionsandneoplasms;endocrineconditionsthyroid, Psychosis secondarytoamedicalcondition: contemplated herebecausethepatienthasrecentlyingestedasubstanceknowntoinducepsychosis. Diagnostic andStatisticalManualofMentalDisorders hallucinations. ThispatientdescribesbothvisualandauditoryHowever, accordingtothe with removalofthestressor. Auditoryhallucinationsaremorecommonandtypicallyaccompanyvisual Brief psychoticdisorder: tachycardia, andhypertensionarecommoninPCPintoxication.Themnemonic PCP intoxication: xcitation, and Visual hallucinationsofepilepticorigincanbesimpleorcomplex.Theyarevariableinfrequencyand Thevisualhallucinationsofnarcolepsyarecomplex,generallyoccurringimmediatelybefore To detectanymedicalconditionthatmaycauseneurologic ormentalchanges. S kin dryness.Thispatientdoesnotcomplainofmyalgias,althoughrhabdomyolysiscanoccurin ThispatientclearlyshowssignsofPCPintoxication.Hallucinations,delusions,nystagmus, To detectcommonlyusedillicitdrugs,suchasamphetamines, barbiturates,benzodiazepines, To evaluateforapossiblepsychiatricdisorder, althoughinthesettingofarecent Symptomsofpsychosismaybeinducedbystressfuleventsandresolve To evaluateforrhabdomyolysis. Itisimportanttonotethatpatientswithsubstance-inducedpsychosislack R age, E rythema, (DSM-5),adiagnosisofbriefpsychoticdisordercannotbe Avarietyofmedicalconditionscanleadtohallucinations. D ilated pupils, D elusions, RED DANES A mnesia, N helps recall ystagmus, PRACTICE CASES 463 98.3°F (36.8°C) Sit up on the bed. your hand. Hold the physical exam request form in 70/minute, regular 15/minute 130/85 mm Hg 1. Take a focused history. a focused 1. Take or female breast exam). not perform rectal, genitourinary, 2. Perform a focused physical exam (do workup plan to the patient. 3. Explain your clinical impression and the patient note after leaving the room. 4. Write order further tests. Hopefully everything will be fine.” “Do you think they are going to give me the job?” Sample Examinee Response employees are fit for the job, as well as “Employers routinely request medical examinations to ensure that potential to others in the work environment. I to determine if they have any medical conditions that may prove hazardous basis of what I find, I may or may not will ask you a few questions and perform a physical examination, and on the Challenging Questions to Ask DOORWAY INFORMATION DOORWAY CASE 36 CASE Notes for the SP Checklist/SP Sheet Patient Description Patient is a 32 yo M. Temp: RR: HR: Examinee Tasks Frank Emanuel, a 32-year-old male, comes to the office for a preemployment medical checkup as requested by his for a preemployment medical checkup male, comes to the office Frank Emanuel, a 32-year-old prospective employer. Vital Signs BP: Opening Scenario PRACTICE CASES 464 Information Gathering Reflective Listening Entrance Building theDoctor-Patient Relationship Examinee Checklist ✓ Examinee eliciteddataefficientlyandaccurately. Examinee summarizedtheSP’s concerns,oftenusingtheSP’s ownwords. Examinee askedtheSPtolisthis/herconcernsandlistenedresponsewithoutinterrupting. Examinee askedanopen-endedquestionandactivelylistenedtotheresponse. Examinee madeeyecontactwiththeSP. Examinee correctlyusedpatient’s name. Examinee identifiedhis/herroleorposition. selfbyname. Examinee introduced Examinee knockedonthedoorbeforeentering.

Question drNone. Whitemucus. No. Iamnotsure. Aroundhalfateaspoonful;stable. Yes. No. insputum Blood Odor No. Color Yes. Amount ofsputum Itisthesame. I’vehadthiscoughforyears. Sputum production Do youcoughatnight? No. Wheezing No. None. Progression ofthecough Changes inthecoughduringday No. Onset ofcough Cough No. Weakness/numbness Headache No. Loss ofconsciousness/seizures Swelling inlegs No. Palpitations orslowheartrate No. No. Shortness ofbreath(currentandpast) Chest pain(currentandpast) Medical complaintsorproblems Patient Response PRACTICE CASES 465 Patient Response Patient None. fax them to you. job. Sexual activityDrug allergies with my wife. Yes, None. Alcohol useIllicit drug useTobacco No. No. a pack a day for 10 years. Yes, Medical problems or diseases in your family VaccinationsOccupation have my papers at home; I can My immunizations are up to date. I I used to work in a coal mine back home. I am applying for a new Change in stool colorCurrent medicationsPast medical historyPast surgical history No. None. None. None. Abdominal painDiarrhea/constipation changesWeight Appetite changes No. No. No. No. Recent travelLast PPDJoint pain or swellingNausea/vomiting I emigrated from Africa a month ago. No. had this test. I have never No. Fever/chillsNight sweatsExposure to TB None. No. No. Question

Examinee used respectful draping. Examinee did not repeat painful maneuvers. Examinee washed his/her hands. Examinee asked permission to start the exam. Examinee recognized the SP’s emotions and responded with PEARLS. Examinee recognized the SP’s ✓ Physical Examination Connecting with the Patient PRACTICE CASES 466 you toquit.Dohaveanyquestions? issue Iwanttotalkyouaboutisyoursmoking.Itputsatincreasedriskofheartandlungdisease,stronglyurge tuberculosis, notonlybecauseitisharmfultoyoubutalsomaytransmityourfuturecoworkers.Theother it rightaway. SinceyoujustcameherefromAfricaandhaveneverbeentestedforTB,weneedtoruleoutpulmonary We needtoordersometestsmakesureyouarefreeofanyseriousmedical conditions,andifwefindanything,willtreat Mr. Emanuel,yourphysicalexaminationisnormal,butcoughmayraiseconcernforsomepossiblemedicalproblems. Sample Closure Closure ✓ Examinee askediftheSPhadanyotherquestionsorconcerns. Examinee discussedinitialmanagementplans: Examinee discussedinitialdiagnosticimpressions.

ExamComponent xrmte Inspection Cranialnerves,motorexam,DTRs,gait Auscultation,palpation Auscultation Auscultation,palpation,percussion Neurologic exam Inspected mouth,throat;palpatedlymphnodes Extremities Abdominal exam Pulmonary exam CV exam Head andneckexam Follow-up tests. Maneuver PRACTICE CASES 467 Patient Note Patient USMLE STEP 2 CS STEP USMLE Physical Examination History USMLE STEP 2 CS Patient Note

Differential Diagnosis Diagnosis #1 History Finding(s): Physical Exam Finding(s):

Diagnosis #2 History Finding(s): Physical Exam Finding(s):

Diagnosis #3 History Finding(s): Physical Exam Finding(s):

Diagnostic Workup PRACTICE CASES

468 USMLE STEP 2 CS Patient Note

History HPI: 32 yo M with no PMH presents for a preemployment medical examination. He has no medical complaints or problems. Nevertheless, he mentioned having a chronic cough for many years with no recent change in frequency or severity. The cough is productive of half a teaspoonful of white mucus with no blood. The patient denies any dyspnea, fever or chills, chest pain, or wheezing and has had no appetite or weight changes. The patient is an African immigrant who came to the United States 1 month ago and reports no TB exposure. He has never had a PPD test. However, he states that his immunizations are up to date, and he will be faxing us the report to review. ROS: Negative except as above. Allergies: NKDA. Medications: None. PMH: Per HPI. PSH: None. SH: 1 PPD for 10 years, no EtOH, no illicit drugs. Sexually active with wife only. FH: Noncontributory.

Physical Examination VS: WNL. HEENT: Mouth and pharynx WNL. Neck: No JVD, no lymphadenopathy. Chest: Clear breath sounds bilaterally; no rhonchi, rales, or wheezing; tactile fremitus normal. Heart: RRR; normal S1/S2; no murmurs, rubs, or gallops. Abdomen: Soft, nontender, nondistended, BS, no hepatosplenomegaly. Extremities: No clubbing, cyanosis, or edema. Neuro: Cranial nerves: 2–12 intact. Motor: Strength 5/5 in all muscle groups. DTRs: Symmetric. Gait: Normal.

Differential Diagnosis Diagnosis #1: COPD/chronic bronchitis History Finding(s): Physical Exam Finding(s): Chronic cough Sputum production History of smoking 1 PPD × 10 years Worked as coal miner PRACTICE CASES

469 USMLE STEP 2 CS Patient Note

Diagnosis #2: Pneumoconiosis History Finding(s): Physical Exam Finding(s): Worked as coal miner Chronic cough

Diagnosis #3: Pulmonary tuberculosis History Finding(s): Physical Exam Finding(s): Recent emigration from Africa Chronic cough

Diagnostic Workup

CXR—PA and lateral PPD or QuantiFERON Gold CBC PRACTICE CASES

470 PRACTICE CASES 471 To identify a causative identify a To However, its availability is variable and its availability However, A QuantiFERON Gold test can also be considered A QuantiFERON Gold test can also The PPD test is a screening tool for determining if a The PPD test is a screening tool for determining M tuberculosis. Mycobacterium tuberculosis. This patient’s chronic cough and sputum production chronic might be due to COPD/ This patient’s May distinguish obstructive from restrictive disease but is not diagnostic for May distinguish obstructive from restrictive disease but is not diagnostic for Active TB infection is unlikely, as the patient denies systemic symptoms, blood- as the is unlikely, Active TB infection A good cavitary lesions in initial test in evaluating chronic cough. It may demonstrate Considering his occupational history as a coal miner, this patient has been exposed to coal this history as a coal miner, Considering his occupational To identify leukocytosis in infection (nonspecific). identify leukocytosis To in this case, as it is more specific for prior infection with in this case, as it is more specific for prior based on the testing center. Pulmonary function tests: pneumoconiosis. More often used as a test to determine the severity of disease. CBC: sputum cultures: and mycobacterial routine Sputum Gram stain, AFB smear, as a a PPD or QuantiFERON Gold test would typically be ordered agent of possible infection. However, setting. screening test before the collection of a sputum sample or culture in an outpatient or GERD. Gold: skin test) or QuantiFERON PPD (tuberculin patient has been infected with CXR—PA and lateral: CXR—PA TB or may show nodular normal in benign causes of cough, such as asthma calcification in silicosis. It is usually tinged sputum, dyspnea, chest pain, or exposure to TB. However, TB infection should be ruled out in this patient TB infection chest pain, or exposure to TB. However, tinged sputum, dyspnea, should be never been tested for TB. Latent infection job, as he is an immigrant and has before he starts a new to active TB. treated to decrease the risk of progression work in coal mines are more likely to develop COPD in addition to inhalant-induced restrictive lung diseases. addition to inhalant-induced restrictive are more likely to develop COPD in work in coal mines their severity. these causes of lung pathology and assess function tests can help distinguish CXR and pulmonary Pneumoconiosis: and pulmonary silicosis. pneumoconiosis worker’s silica and is at increased risk of coal dust and crystalline Pulmonary tuberculosis: COPD/chronic bronchitis: COPD/chronic are smokers and occupational exposure. Patients who secondary to his smoking history and chronic bronchitis There such as GERD and asthma. chronic cough that may be benign, causes of the patient’s are other possible Diagnostic Workup Additional Differential Diagnoses Additional Differential CASE DISCUSSION CASE Diagnoses Note Differential Patient PRACTICE CASES 472 Entrance Building theDoctor-Patient Relationship Examinee Checklist “It isapossibility. Tell memoreaboutthesymptomsyou’re havingthatconcernyouwithregardtocancer.” Sample ExamineeResponse “My fatherhadcoloncancer. CouldIhaveittoo?” Challenging QuestionstoAsk If colonoscopyismentionedbytheexaminee,ask,“Whatdoesthatwordmean?” Notes fortheSP Patient isa55yoM,marriedwith2children. Patient Description Checklist/SP Sheet Examinee Tasks HR: RR: Temp: BP: Signs Vital Kenneth Klein,a55-year-old inhisstool. male,comestothecliniccomplainingofblood Opening Scenario DOORWAY INFORMATION CASE 37 Examinee madeeyecontact with theSP. Examinee correctlyusedpatient’s name. Examinee identifiedhis/herroleorposition. selfbyname. Examinee introduced Examinee knockedonthedoorbeforeentering. 4. Write thepatientnoteafterleavingroom. 3. Explainyourclinicalimpressionandworkupplantothepatient. 2. Performafocusedphysicalexam(donotperformrectal,genitourinary, orfemalebreastexam). 1. Take afocusedhistory. 130/80mmHg 16/minute 76/minute,regular 98.5°F(36.9°C) PRACTICE CASES 473 Do you think I should stay home? laxatives. At first I got some relief from them, but now they are of laxatives. At first I got some relief from no help to me at all. A little. Patient Response The blood is mixed in with the brown stool. Contact with people with diarrheaExercise No. an hour every day. I walk for half Weight changesWeight Appetite changesRecent travel I have lost about 10 pounds over the past 6 months. My appetite has been the same. a trip with my family next week. No, but I am thinking of going on MelenaFever/chillsAbdominal painNausea/vomitingDiet No. No. No. No. eat vegetables at all. I eat a lot of junk food. I don’t empty the bowel) Frequency of diarrheaDescription of the diarrheaMucus in stool brown, mixed with blood. Watery, Three times a day. No. Frequency of bowel movementsDiarrheaUrgency the past 6 months. I have had 2 bowel movements a week for (ineffectual spasms of the Tenesmus rectum accompanied by the desire to diarrhea for the past 2 days. I have had No. Pain during defecationConstipation No. long time, and I keep taking I have had constipation for a Well, OnsetFrequencyDescription (blood before, during, or after defecation) Bright red or dark blood red. Bright One month ago. I have a bowel movement, I see some blood Every time mixed in. Chief complaint Bloodstool. in my Question

Examinee elicited data efficiently and accurately. Examinee elicited data Examinee asked an open-ended question and actively listened to the response. listened to the question and actively asked an open-ended Examinee interrupting. response without and listened to the his/her concerns asked the SP to list Examinee own words. SP’s concerns, often using the the SP’s Examinee summarized ✓ Reflective Listening Reflective Information Gathering PRACTICE CASES 474 Closure Physical Examination Connecting withthePatient ✓ ✓ Examinee askediftheSPhadanyotherquestionsorconcerns. Examinee discussedinitialmanagementplans: Examinee discussedinitialdiagnosticimpressions. Examinee didnotrepeatpainfulmaneuvers. Examinee usedrespectfuldraping. Examinee askedpermissiontostarttheexam. Examinee washedhis/herhands. Examinee recognizedtheSP’s emotionsandrespondedwithPEARLS.

ExamComponent Question boia xmAuscultation, palpation,percussion Auscultation Auscultation Abdominal exam Pulmonary exam CV exam None. No. With mywife. No. No. Lawyer. Drug allergies Myfatherdiedat55ofcoloncancer. Mymotherisaliveand Sexual activity Tobacco Hemorrhoidsresected4yearsago. Illicit druguse Alcohol use Occupation butIstopped No.Iusedtotakemanylaxatives,suchasbisacodyl, No. Family history Past surgicalhistory use) Past medicalhistory(recentantibiotic Current medications Urinary problems Follow-up tests:Examineementionedtheneedforarectal exam. healthy. I hadbronchitis3weeksago;itwastreatedwithamoxicillin. all ofthemwhenthediarrheastarted. Patient Response Maneuver PRACTICE CASES 475 at your problem. Do you have any questions for me? your problem. Do you Sample Closure Sample in your hemorrhoids, an infection problems, such as to readily treatable describe may be due the symptoms you Klein, Mr. th It is crucial cancer. such as colorectal of more serious disease, they may also be a sign However, colon, or diverticulosis. colon through a thin tube which involves looking at your a stool exam, and probably a colonoscopy, we run some blood tests, we should be able to treat Once we make a diagnosis, today. I will also need to perform a rectal exam that contains a camera. PRACTICE CASES 476 Physical Examination History USMLE STEP2CS Patient Note PRACTICE CASES 477 Patient Note Patient Diagnosis #3 History Finding(s): Physical Exam Finding(s): Diagnosis #2 History Finding(s): Physical Exam Finding(s): Diagnosis #1 History Finding(s): Physical Exam Finding(s): USMLE STEP 2 CS STEP USMLE Diagnostic Workup Differential Diagnosis Differential USMLE STEP 2 CS Patient Note

History HPI: 55 yo M c/o bright red blood per rectum. History of constipation 6 months ago, 2 bowel movements a week. 1 month ago noticed blood mixed with stool with each bowel movement. 2 days ago, tenesmus and watery brown diarrhea mixed with blood. 10-lb weight loss in 6 months despite good appetite. Diet of junk food and no vegetables. No urgency, mucus in stool, or pain with defecation. Denies fevers, chills, nausea, vomiting, abdominal pain, recent history of travel, or contact with ill persons. ROS: Negative except as above. Allergies: NKDA. Medications: Used to take many laxatives (bisacodyl), but stopped after the onset of diarrhea 2 days ago. PMH: Bronchitis 3 weeks ago, treated with amoxicillin. PSH: Hemorrhoids resected 4 years ago. SH: No smoking, no EtOH, no illicit drugs. Sexually active with wife only. FH: Father died of colon cancer at age 55.

Physical Examination Patient is in no acute distress. VS: WNL. Chest: Clear breath sounds bilaterally. Heart: RRR; normal S1/S2; no murmurs, rubs, or gallops. Abdomen: Soft, nondistended, nontender, BS, no hepatosplenomegaly.

Differential Diagnosis Diagnosis #1: Colorectal cancer History Finding(s) Physical Exam Finding(s) Blood mixed with stool for 1 month Family history of colon cancer Unintentional weight loss of 10 lbs

Diagnosis #2: Hemorrhoids History Finding(s) Physical Exam Finding(s) History of hemorrhoids Hematochezia PRACTICE CASES

478 USMLE STEP 2 CS Patient Note

Diagnosis #3: C difficile colitis History Finding(s) Physical Exam Finding(s) Acute diarrhea Recent antibiotic exposure

Diagnostic Workup

Rectal exam, stool for occult blood Colonoscopy Stool for C difficile PCR Fecal leukocytes CBC Anoscopy Flexible proctosigmoidoscopy PRACTICE CASES

479 PRACTICE CASES 480 Additional Differential Diagnoses Diagnostic Workup Patient NoteDifferential Diagnoses CASE DISCUSSION specificity higherthan97%. institutions withPCR.The useful intheevaluationofGI bleeding. CT—abdomen/pelvis: contraindicated. polyps andcancer, Usedprimarilywhen colonoscopyisunavailableor andcannotdiagnoseangiodysplasia. Double-contrast (aircontrast)bariumenema: Flexible proctosigmoidoscopy: Anoscopy: CBC: present in Fecal leukocytes: Stool for older than40yearsofagepresentingwithhematochezia. age atwhichafirst-degreefamilymemberwasfirstdiagnosed).Itshouldbetheinitialtestperformedinpatients Colonoscopy: witheachbowelmovement. instool,especiallyapatientcomplainingofvisibleblood blood Rectal exam,stoolforoccultblood: month-long hematochezia. recent onsetofdiarrheaandtenesmusmakeinflammatoryboweldiseasealesslikelyetiologyforthispatient’s Ulcerative colitis: cannot explaintheotherfeaturesofthispatient’s presentation. Angiodysplasia: volume bleedsoccurringindiscrete,self-limitedepisodes. Diverticulosis: (and otherformsofinfectiouscolitis)lesslikely. Theabsenceoffeverandlowerabdominal crampingalsomakesthisdiagnosis stool rarelycontainsgrossblood. antibiotic exposure,assymptomsofantibiotic-associatedcolitismaybedelayedforupto6–8weeks.However, Pseudomembranous onthetoiletpaperorinbowl. hemorrhoids arefreshblood Hemorrhoids: should besenttostartthenecessaryworkup. bowel habits,andweightlossisconsistentwiththisdiagnosis.Arectalexamstooltestedforoccultblood Colorectal cancer: To investigateanemia.Leukocytosiscouldalsosuggestinfectionorinflammatoryboweldisease. C difficile C difficile Canidentifybleedinginternalhemorrhoids,rectalulcers, andtraumaticlesions. Ascreeningcolonoscopyshouldhavebeenofferedtothepatientatage45(10yearsbefore Recurrenthemorrhoidsmayexplainthepatient’s hematochezia,butmoretypicalfindingsin ThisisthemostcommoncauseofmajorlowerGIbleeding,butitusuallypresentswithlarger- ThisisanothercommoncauseoflowerGItractbleeding,butaswithdiverticulardisease,it colitis. Usuallypresentininvasivebacterialinfectionandinflammatory boweldisease.Variably Althoughthepatienthaschronicconstipation,absenceofabdominalpainand A positive family history coupled with the presence of blood inthe stool,achangein Apositivefamilyhistorycoupledwiththepresenceofblood PCR: (C difficile) Contrast-enhancedexamscan detectdiverticulosisormassesbutgenerallyarenot Astool C difficile Ifnondiagnostic,followupwithabariumenemaorcolonoscopy. C difficile colitis: PCRtesthasaturnaroundtimeoftwohourswithsensitivityand Usefulfordetectingmassesandhemorrhoids.Alwaystestoccult Itisimportanttoaskallpatientswithacutediarrheaaboutrecent toxinassayhaslowsensitivityandbeenreplacedatmost Notasaccuratecolonoscopyforthediagnosisof PRACTICE CASES 481 98.6°F (37°C) Exhibit mild muscle rigidity in your wrists and arms—that is, when the examinee tries to move your wrists Exhibit mild muscle rigidity in your wrists slowly. and arms, stiffen them and move them in small, shuffling steps. Lean your back forward slightly and walk Exhibit a resting hand tremor (pill rolling) that disappears with movement. 70/minute, regular 16/minute 135/85 mm Hg Examinee knocked on the door before entering. Examinee introducedself by name. 2. Perform a focused physical exam (do not perform rectal, genitourinary, or female breast exam). not perform rectal, genitourinary, 2. Perform a focused physical exam (do workup plan to the patient. 3. Explain your clinical impression and the patient note after leaving the room. 4. Write 1. Take a focused history. a focused 1. Take evaluations to explore that possibility.” Challenging Questions to Ask “Do you think I will get better?” Sample Examinee Response long the improvement will last. The know how “I think your tremor will improve with medication, but I don’t disease, and we need to do some additional tremor may be a sign of a larger movement disorder called Parkinson’s DOORWAY INFORMATION DOORWAY CASE 38 CASE Entrance Examinee Checklist Relationship Building the Doctor-Patient Checklist/SP Sheet Patient Description Patient is a 66 yo M. Notes for the SP RR: HR: Examinee Tasks Charles Andrews, a 66-year-old male, comes to the clinic complaining of a tremor. male, comes to the 66-year-old Charles Andrews, a Vital Signs BP: Temp: Opening Scenario PRACTICE CASES 482 Information Gathering Reflective Listening ✓ Examinee eliciteddataefficientlyandaccurately. Examinee summarizedtheSP’s concerns,oftenusingtheSP’s ownwords. Examinee askedtheSPtolisthis/herconcernsandlistenedresponsewithoutinterrupting. Examinee askedanopen-endedquestionandactivelylistenedtotheresponse. Examinee madeeyecontactwiththeSP. Examinee correctlyusedpatient’s name. Examinee identifiedhis/herroleorposition.

Question xrieNo,I’mreallynotveryactiveanymore. Retiredchemistry professor. Iammarriedand livewithmywife. Myparentsdied inacaraccidenttheir40s,andmysisteris Exercise No. None.Bothofmy parentswerealcoholics,soInevertouchit. Occupation Highcholesterol,treatedwithdiet.Asthma,an Social history Onecupofcoffeeeverymorning.Iusedtodrink3cupsaday, but Family history History ofheadtrauma Past medicalhistory Alcohol use Caffeine intake Well, backincollegeIoccasionallyhad ahandtremorafterpulling ItseemsmoreseverewhenIamreallytired. ItshakeswhenI’mjustsittingarounddoingnothing.usuallystops Prior historyofsimilarsymptoms None. rash, etc.) ADLs/IADLs, depression,constipation, Inoticeditabout6monthsago,butlatelyseems tobegetting voice orhandwriting,difficultywith Onlyintherighthand. TIA symptoms,drooling,changesin Associated symptoms(falls,headaches, Exacerbating factors Ihaveatremorinthishand(pointstorighthand). Alleviating factors Context Duration Location Chief complaint worse. healthy. Ithinkmyfathermayhavehadatremor, but I’mnotsure. albuterol inhalerasneeded. I’ve cutbackoverthepastfewmonths. have now. hands, butitwasworseintheright.ItseemedfasterthanoneI an all-nighteranddrinkinglotsofcoffee.Thetremorwasinboth just becauseIretiredlastyear. keep upwithherwhenwegogroceryshopping,butIthinkthat’s No, Idon’t thinkso.MywifesaysI’vesloweddownbecauseIcan’t when Iholdouttheremotecontroltochangechannel. Patient Response PRACTICE CASES t e 483 Patient Response Patient Maneuver DTRs, cerebellar, gait, sensory exam DTRs, cerebellar, Follow-up tests. Possible need to compare an old handwriting sample with a present sample. Neurologic exam muscle tone), Mental status, cranial nerves, motor exam (including CV examPulmonary exam Auscultation Auscultation Drug allergies No. Tobacco useIllicit drug Current medications I have not used it in more than a year. inhaler as needed. Albuterol No. No. Exam Component Question

Examinee asked if the SP had any other questions or concerns. Examinee discussed initial management plans: Examinee discussed initial management illness. Examinee offered support throughout the patient’s Examinee discussed initial diagnostic impressions. Examinee discussed initial diagnostic Examinee did not repeat painful maneuvers. Examinee washed his/her hands. exam. Examinee asked permission to start the Examinee used respectful draping. Examinee recognized the SP’s emotions and responded with PEARLS. emotions and the SP’s Examinee recognized ✓ ✓ to help you every step of the way. Do you have any questions for me? to help you every step of the way. of disease progression involves looking closely at your handwriting. Do you think you could bring an old sample of your of disease progression involves looking closely at your handwriting. Do you think should also know that about 25% of the time, patients with your symptoms do not handwriting with you on your next visit? You For this reason, I would like to run a few tests, including some imaging studies of your head and som disease. have Parkinson’s I will print out a comprehensive patient pamphlet tha have those results before you leave today, blood tests. Although we won’t to know that I will be here to treat you and will give you resources to help answer your questions as they come up. I want you Sample Closure Andrews, I am sorry to have to tell you this, but on the basis of your history and physical exam, it would appear that Mr. symptoms may improve with medications, but eventually they will return. One indicator disease. Your you have Parkinson’s Closure Physical Examination Connecting with the Patient Connecting with PRACTICE CASES 484 Physical Examination History USMLE STEP2CS Patient Note PRACTICE CASES 485 Patient Note Patient Diagnosis #3 History Finding(s): Physical Exam Finding(s): Diagnosis #2 History Finding(s): Physical Exam Finding(s): Diagnosis #1 History Finding(s): Physical Exam Finding(s): USMLE STEP 2 CS STEP USMLE Diagnostic Workup Differential Diagnosis Differential USMLE STEP 2 CS Patient Note

History HPI: 66 yo M c/o right hand tremor for 6 months. It occurs at rest and seems to be getting worse. The tremor is exacerbated by fatigue. There are no alleviating factors (he does not drink alcohol). Reducing his caffeine intake to 1 cup of coffee daily did not seem to help. He denies associated symptoms but does say that his wife complains that he has “slowed down” since retiring last year. Specifically, he seems to be walking more slowly recently (time course unspecified, but within the past year). He had a hand tremor when very fatigued back in college, but it was bilateral and faster than his present tremor. ROS: Negative except as above. Allergies: NKDA. Medications: Albuterol MDI prn (no use in past year). PMH: High cholesterol, treated with diet. Mild asthma. SH: No smoking, no EtOH, no illicit drugs. He is a retired chemistry professor, married and lives with his wife. FH: Father may have had a tremor.

Physical Examination Patient is in no acute distress. VS: WNL. Chest: Clear breath sounds bilaterally. Heart: RRR; normal S1/S2; no murmurs, rubs, or gallops. Neuro: Mental status: Alert and oriented × 3. Cranial nerves: 2–12 grossly intact. Motor: Right hand resting tremor with “pill-rolling” movement that improves or disappears during purposeful action or posture. Mild muscle rigidity in both wrists and arms, but no frank cogwheeling. Strength 5/5 throughout. DTRs: Symmetric 2+ in all extremities. Cerebellar: Romberg, rapid alternating movements and heel-to-shin test normal and symmetric. Gait: Bradykinetic, takes small steps. Walks with back slightly bent forward. Sensation: Intact to soft touch and pinprick.

Differential Diagnosis Diagnosis #1: Parkinson’s disease History Finding(s) Physical Exam Finding(s) Resting tremor Low-frequency tremor in upper extremity Bradykinetic gait Upper extremity rigidity

Diagnosis #2: Essential tremor History Finding(s) Physical Exam Finding(s) Possible family history of tremor Tremor in distal upper extremity PRACTICE CASES

486 USMLE STEP 2 CS Patient Note

Diagnosis #3: Physiologic tremor History Finding(s) Physical Exam Finding(s) Resting tremor Tremor in distal upper extremity

Diagnostic Workup

MRI—brain PRACTICE CASES

487 PRACTICE CASES 488 Diagnostic Workup Additional Differential Diagnoses Patient NoteDifferential Diagnoses CASE DISCUSSION Heavy metalscreen: TSH: MRI—brain: Hyperthyroidism: older than40yearsofage. Wilson’s disease: limb. subsides entirelywhenthepatientisaskedtoperformacomplex,repetitivemotortaskwithcontralateral Psychogenic tremor: often absentinthesecases. also contributetotremor. Neurolepticsandmetoclopramidecancausedrug-inducedparkinsonism,buttremor is nicotine, theophylline,TCAs,lithium,valproicacid,andcorticosteroids.Mercuryarsenicexposuremay Drug-induced tremor: asymmetric restingtremor. Midbrain lesion: withdrawal, thyrotoxicosis,fever, andpheochromocytoma. physiologic tremorincludeanxiety, excitement,sleepdeprivation/fatigue,hypoglycemia,caffeineintake,alcohol conditions, itisthemostcommoncauseofposturalandactiontremors.Conditionsthatcanenhance in normalindividuals.Thetremorisoftennotvisible,butwhenenhancedbymedicationsorothermedical Physiologic tremor: this case. small amountsofalcohol.DifferentiationfromtheclassicrestingtremorPDisusuallystraightforward,asin can alsobeinvolved.ETisnotassociatedwithotherneurologicsignsandimprovesfollowingtheingestionof frequency andoftenasymmetricallyinvolvesthedistalupperextremity. Thehead,voice,chin,trunk,andlegs directed activitysuchasfinger-to-nose testing).Approximately50%ofcasesarefamilial.Tremor isusuallyhigh apparent whenthearmsareheldoutstretched)oractiontremor(ie,thatincreasesatendofgoal- Essential tremor (ET): bradykinesia andrigiditysuggestPD. be involved,butincontrasttoET, headtremor. PDdoesnotproduce Alongwiththetremor, thepatient’s the otherextremitiesaswell.LegtremorismorecommonlyduetoPDthanET. Theface,lips,andjawmay (ET, seebelow).Tremor isusuallylowfrequency(4–6Hz),beginsinoneupperextremity, andmaylaterinvolve although somepatientswithPDalsohaveapostural/actiontremorthatisindistinguishablefromessential partsupportedandcompletelyatrestbutimproves orsubsideswithvoluntaryactivity), with theaffectedbody Parkinson’s disease(PD): To screenfor hyperthyroidism. To ruleoutastructurallesion,particularlyinthemidbrain orbasalganglia. Midbraininjuryduetostroke,trauma,ordemyelinatingdiseaseisararecauseofsolitary Thiscancauserestingtremor(amongothermanifestations) butisnotconsideredinpatients Thisisassociatedwithfinetremoralongavarietyof otherclassicsignsandsymptoms. Thisreferstoaverylow-amplitude,high-frequency(10-12-Hz)tremorpresent To tests. screenformercuryandarsenic toxicityviaurineorblood Thisoftenmanifestswithvaryingfrequencyandeitherbecomesmoreirregularor Thisisthemostcommonneurologiccauseofposturaltremor(ie,that Manymedicationscanenhancephysiologictremor, notably Thisisthemostcommoncauseofrestingtremor(ie,athatevident β -agonists (eg,albuterol), PRACTICE CASES 489 These tests constitute the screening tests (and diagnostic tests, in the case of liver diagnostic tests, in tests (and constitute the screening These tests Ceruloplasmin, slit lamp examination for Kayser-Fleischer rings, AST/ALT, CBC, 24-hour urinary CBC, 24-hour AST/ALT, rings, Kayser-Fleischer examination for slit lamp Ceruloplasmin, liver biopsy: copper, advanced age precludes the patient’s previously, disease. As noted Wilson’s to evaluate for suspected biopsy) used disease. of Wilson’s consideration PRACTICE CASES 490 Entrance Building theDoctor-Patient Relationship Examinee Checklist if so,discussstrategiestodealwithit.” associated withweightgain.We alsoneedtodetermineifsomethingelseiscontributingyourweightgainand, “I understandthatcontrollingyourweightisimportantto you,butthehealthrisksofsmokingfaroutweighthose Sample ExamineeResponse “I wanttogobacksmokingbecauseIhavestartedgainingweightsincequit.” Challenging QuestionstoAsk None. Notes fortheSP Patient isa30yoF. Patient Description Checklist/SP Sheet Examinee Tasks BMI: HR: RR: Temp: BP: Signs Vital Kristin Grant,a30-year-old female,comestotheofficecomplainingofweightgain. Opening Scenario DOORWAY INFORMATION CASE 39 Examinee identifiedhis/her role orposition. selfbyname. Examinee introduced Examinee knockedonthedoorbeforeentering. 4. Write thepatientnoteafterleavingroom. 3. Explainyourclinicalimpressionandworkupplantothepatient. 2. Performafocusedphysicalexam(donotperformrectal,genitourinary, orfemalebreastexam). 1. Take afocusedhistory. 120/85mmHg 13/minute 65/minute,regular 30 98.0°F(36.7°C) PRACTICE CASES 491 Patient Response pregnancies. No. No. it every 6 weeks or more. The period 7 days. lasts 1 a day now. become dry. Miscarriages/abortions None. Age at menarchePregnanciesProblems during pregnancy/delivery and my child is healthy. No, it was a normal delivery, Age 13. is 10 years old. I have not had any other I have 1 child; he Frequency of menstrual periodsStart of change in cycle my period I used to get every 4 weeks, but recently I’ve been getting Pads/tampons changed a day but the blood flow is becoming less, and I use only It was 2–3 a day, Six months ago. ing asleep, early waking, snoring) Associated symptoms (fever/chills, chest pain, shortness of breath, ab- dominal pain, diarrhea) Last menstrual period One week ago. Appetite changesFatigueDepressionSleeping problems (falling asleep, stay- I have a good appetite. No. No. Cold intoleranceSkin/hair changes changeVoice Constipation Yes. my skin has My hair is falling out more than usual, and I feel that No. No. Chief complaintOnset gainedWeight I am gaining weight. I’ve gained 20 pounds over the last 3 months. Three months ago. Question

Examinee summarized the SP’s concerns, often using the SP’s own words. SP’s concerns, often using the the SP’s Examinee summarized efficiently and accurately. Examinee elicited data Examinee correctly used patient’s name. correctly used patient’s Examinee with the SP. made eye contact Examinee to the response. open-ended question and actively listened Examinee asked an to the response without interrupting. SP to list his/her concerns and listened Examinee asked the ✓ Information Gathering Reflective Listening Reflective PRACTICE CASES 492 Physical Examination Connecting withthePatient ✓ ✓ Examinee didnotrepeatpainfulmaneuvers. Examinee usedrespectfuldraping. Examinee askedpermissiontostarttheexam. Examinee washedhis/herhands. Examinee recognizedtheSP’s emotionsandrespondedwithPEARLS.

ExamComponent Question xrmte Inspectedforedema, checkedDTRs Auscultation, palpation,percussion Auscultation Auscultation thyroidgland Palpatedlymphnodes, Inspectedconjunctivae, mouth,andthroat Extremities Abdominal exam Pulmonary exam CV exam Neck exam Head exam No. Myhusbandhadavasectomy2yearsago. With myhusband. Theusual.Ihaven’t changedanythinginmydietmorethan10 No. Iquitsmoking3monthsago.hadsmoked2packsadayfor10 Drug allergies Contraceptives Never. Sexual activity None. Housekeeper. Diet Exercise Tobacco Mymotherandsisterareobese. None. Illicit druguse No. Ihavebipolardisorder. Iwasstartedonlithium6monthsago; Alcohol use Lithium. Occupation Family historyofobesity Past surgicalhistory Past medicalhistory Current medications Hirsutism eat out. years. Coffeeduringtheday, Iusually chicken,steak,Chinesefood. years. haven’t hadanyproblemssincethen. Patient Response Maneuver PRACTICE CASES u 493 ure cising Follow-up tests. Lifestyle modification support). techniques, smoking cessation (diet, exercise, relaxation Examinee asked if the SP had any other questions or concerns. Examinee asked if the Examinee discussed initial diagnostic impressions. discussed initial diagnostic Examinee plans: discussed initial management Examinee might incur from excessive weight gain. In addition, there may be other reasons for your weight gain; for example, it may be be other reasons for your weight gain; weight gain. In addition, there may might incur from excessive a side effect of the lithium you’re taking. I would like to draw some blood to meas related to your thyroid gland, or it may be a In the meantime, in addition to stopping smoking, you should continue to pursue your thyroid function and lithium levels. ones, such as fruits and vegetables. Exer to decrease the fatty foods you eat and increase the healthy healthier lifestyle. Try improve your health. Do you have any questions for me? only 30 minutes 3 times a week can also Sample Closure gained 20 pounds over 3 months. This may have they quit. You gain an average of 5 pounds when Mrs. Grant, most smokers worse than the risk yo the health risk posed by smoking is far smoking cessation, but bear in mind that have resulted from your Closure PRACTICE CASES 494 Physical Examination History USMLE STEP2CS Patient Note PRACTICE CASES 495 Patient Note Patient Diagnosis #3 History Finding(s): Physical Exam Finding(s): Diagnosis #2 History Finding(s): Physical Exam Finding(s): Diagnosis #1 History Finding(s): Physical Exam Finding(s): USMLE STEP 2 CS STEP USMLE Diagnostic Workup Differential Diagnosis Differential USMLE STEP 2 CS Patient Note

History HPI: 30 yo F c/o weight gain of 20 lbs over the past 3 months after she stopped smoking. She has a good appetite and reports no change in her diet. For 6 months she has experienced oligomenorrhea and hypomenorrhea, dry skin, and cold intolerance. The patient denies voice change, constipation, hirsutism, depression, fatigue, or sleep problems. OB/GYN: Last menstrual period last week. See HPI for other. ROS: Negative except as above. Allergies: NKDA. Medications: Lithium, started 6 months ago. PMH: Bipolar disorder, diagnosed 6 months ago. PSH: None. SH: 2 PPD for 10 years; stopped 3 months ago. No alcohol, no illicit drugs. Sexually active with husband only. Doesn’t exercise. FH: Mother and sister are obese. Diet: Consists mainly of lots of coffee during the day, chicken, steak, and Chinese food.

Physical Examination Patient is in no acute distress. VS: WNL. HEENT: No conjunctival pallor, mouth and pharynx WNL. Neck: No lymphadenopathy, thyroid normal. Chest: Clear breath sounds bilaterally. Heart: RRR; normal S1/S2; no murmurs, rubs, or gallops. Abdomen: Soft, nontender, nondistended, BS, no hepatosplenomegaly. Extremities: No edema, normal DTRs in lower extremities bilaterally.

Differential Diagnosis Diagnosis #1: Hypothyroidism History Finding(s) Physical Exam Finding(s) Oligo- and hypomenorrhea Chronic dry skin Chronic cold intolerance

Diagnosis #2: Smoking cessation History Finding(s) Physical Exam Finding(s) Weight gain following smoking cessation PRACTICE CASES

496 USMLE STEP 2 CS Patient Note

Diagnosis #3: Lithium-related weight gain History Finding(s) Physical Exam Finding(s) Ongoing lithium therapy

Diagnostic Workup

TSH Serum lithium level Fasting glucose, cholesterol, PRACTICE CASES

497 PRACTICE CASES 498 Diagnostic Workup Additional Differential Diagnoses Patient NoteDifferential Diagnoses CASE DISCUSSION hypercortisolism. 24-hour urinefree cortisol: bedtime dexamethasoneadministrationexcludesCushing’s syndromewith98%certainty. Dexamethasone suppression test: Urine hCG: or hyperlipidemia. Fasting glucose,cholesterol, triglycerides: index, andserumlevelsshouldbecheckedevery3−6months. Serum lithiumlevel: TSH: thin limbs,andabdominalstriae). exam (eg,mayrevealhypertension,moonfacies,plethora,supraclavicularfatpads,truncalobesitywith Cushing’s syndrome: woman ofchildbearingagewhohasunexplainedweightgain. Pregnancy: history doesnotaccountforacuteweightgain. Familial obesity: and fatigue. this case.Othersymptomsincludecoldintolerance,dryskin,confusion,dizziness,headache,lethargy, hairloss, Lithium-related weightgain: increased appetiteandcalorieconsumption. 4.5 lbs(2kg).However, majorweightgainsuchasthatseeninthiscasemayoccur. Patientsgenerallyreport Smoking cessation: cause ofherweightgain. voice, constipation,depression,andfatiguearealsosymptomsofhypothyroidism.Itneedstoberuledoutasa skin, coldintolerance,elevatedserumcholesterol,andchangesinmenstruationpatterns.Deepeningofthe Hypothyroidism: To diagnosesuspectedhypothyroidism. To ruleoutpregnancy. Regardlessofthemenstrualhistorygivenbypatient,pregnancyshouldbesuspectedina Therearestronggeneticinfluencesonthedevelopmentofobesity, butapositivefamily Thispatienthasclassicearlysymptomsofhypothyroidism,whichincludeweightgain,dry Weight gainoccursinmostpatientsfollowingsmokingcessationbutusuallyaveragesonly To checkifthepatient’s lithiumdosageisappropriate.Lithiumhasanarrowtherapeutic Thisisararecauseofunexplainedweightgainandcanusuallybediagnosedbyphysical Performedifthedexamethasonesuppressiontestisabnormal. Helpsconfirm Weight gainisacommonsideeffectoflithiumtherapyandmaycontributein To screenforhypercortisolism.Asuppressedmorningcortisol following To screenformedicalcomplicationsofobesitysuchasdiabetes PRACTICE CASES 499 Examinee asked an open-ended question and actively listened to the response. without interrupting. Examinee asked the SP to list his/her concerns and listened to the response own words. concerns, often using the SP’s Examinee summarized the SP’s Examinee introducedself by name. Examinee identified his/her role or position. name and identified caller and relationship of caller to patient. Examinee correctly used patient’s 2. Explain your clinical impression and workup plan to the mother. impression and workup plan to the 2. Explain your clinical the patient note after leaving the room. 3. Write 1. Take a focused history. 1. Take DOORWAY INFORMATION DOORWAY CASE 40 CASE can examine her and perhaps run some tests. After that, I should be able to give you a more accurate assessment.” can examine her and perhaps run some Examinee Checklist Relationship Building the Doctor-Patient Entrance Challenging Questions to Ask “How sick is my baby?” Sample Examinee Response answer over the phone. I would like you to bring your baby here so that I “It is hard for me to give you an accurate Patient Description the history. mother offers The patient’s Notes for the SP want to come to the office unless you have to but add that you don’t health, Show concern about your child’s because you do not have transportation. Checklist/SP Sheet The mother of Theresa Wheaton, a 6-month-old female child, calls the office complaining that her child has child, calls the office complaining Wheaton, a 6-month-old female The mother of Theresa diarrhea. Examinee Tasks Opening Scenario Reflective Listening PRACTICE CASES 500 Information Gathering ✓ Examinee showedcompassionfortheSPandherchild. Examinee eliciteddataefficientlyandaccurately.

Question ne Itstartedyesterdayat2 Mybabyhasdiarrhea. Onset Chief complaint iooscyNo,hercryisweak. ItriedsomeTylenol, butitdidnothelp. Notasplayfulshewasearlier. Nonesinceyesterday. Vigorous cry Sheislessresponsiveandlooksdrowsy. Treatment tried Sheisnotashungrysheusedtobe. head Dry mouthorsunkensoftspotoverthe No. Number ofwetdiapers No. Awake andresponsive Activities None. Appetite changes Yes. Itisgettingworse. Abdominal distention Previous regularbowelmovements Lightbrown,watery, largeamounts. Relationship tooralintake Shehasabout6bowelmovementsperday. instool Blood Description ofbowelmovements Frequency ofbowelmovements Progression aighbt Formulawithiron; ricecerealatnight;occasionallyjuice. Itwasanuncomplicated spontaneousvaginaldelivery. Two weeksago, andeverythingwasnormal. Uptodate. Yes. Eating habits No. Birth history Nottomyknowledge. Last checkup No. Vaccinations Yes; Itookhertemperature,anditwas100.5°F. Ill contactsindaycarecenter No. No. Day carecenter urine ispassed No. Cough, pullingear, orcryingwhen Shaking (seizures) Rash Nausea/vomiting Breathing fast Fever Recent URI No. Yes, hermouthisdry. Patient Response P . M . PRACTICE CASES 501 ent, Patient Response Patient Follow-up tests. Family historyDrug allergies None. None. Current medications historyPast medical Past surgical history None. Nothing of note. None. Question

Examinee discussed initial diagnostic impressions. Examinee discussed initial diagnostic plans: Examinee discussed initial management questions or concerns. Examinee asked if the SP had any other Examinee recognized the SP’s emotions and responded with PEARLS. emotions and the SP’s Examinee recognized ✓ and we will then proceed according to what we find on the exam. I understand that you may have problems with transportation, we find on the exam. I understand that you may have problems with transportation, and we will then proceed according to what will here who can help you handle these issues. After we are done on the phone, I but we are fortunate to have a social worker you. Do you have any questions for me? transfer your call to him, and he can help Sample Closure urinated have given me, I am concerned that your child may be dehydrated. She hasn’t Ms. Wheaton, from the information you not want to It is very hard for me to assess her over the telephone, and I do and she is weak and drowsy. since yesterday, her in for a physical exam and a full assessm For this reason, I am going to ask you to bring jeopardize her health in any way. Physical Examination None. Closure Connecting with the Patient Connecting with PRACTICE CASES 502 Physical Examination History USMLE STEP2CS Patient Note PRACTICE CASES 503 Patient Note Patient Diagnosis #3 History Finding(s): Physical Exam Finding(s): Diagnosis #2 History Finding(s): Physical Exam Finding(s): Diagnosis #1 History Finding(s): Physical Exam Finding(s): USMLE STEP 2 CS STEP USMLE Diagnostic Workup Differential Diagnosis Differential USMLE STEP 2 CS Patient Note

History HPI: The source of information is the patient’s mother. The mother of a 6-month-old F c/o her child having 1 day of diarrhea, weakness, and drowsiness. The child has had 6 watery brown bowel movements per day. There was no blood in her stool, but she has not urinated since yesterday. She received Tylenol without improvement. The mother reports the child’s temperature as 100.5°F and adds that her mouth is dry. The child has no known sick contacts but is in day care. The mother denies any vomiting, lethargy, excessive sleeping, abnormal behavior, or recent URIs. The child had a normal checkup with her pediatrician 2 weeks ago and is up to date on her immunizations. She has a diet of formula with iron and rice cereal at night with occasional juice. ROS: Negative. Allergies: NKDA. Medications: None. PMH: Uncomplicated spontaneous vaginal delivery. PSH: None. FH: Noncontributory.

Physical Examination None.

Differential Diagnosis Diagnosis #1: Viral gastroenteritis History Finding(s) Physical Exam Finding(s) Acute watery diarrhea Low-grade fever (100.5°F) Day care attendance

Diagnosis #2: Bacterial diarrhea History Finding(s) Physical Exam Finding(s) Acute diarrhea Day care attendance Low-grade fever (100.5°F)

Diagnosis #3: Malabsorption History Finding(s) Physical Exam Finding(s) Watery diarrhea Dry mouth PRACTICE CASES

504 USMLE STEP 2 CS Patient Note

Diagnostic Workup

Rotavirus enzyme immunoassay/norovirus PCR Electrolytes Stool leukocytes, culture, ova and parasitology, and pH PRACTICE CASES

505 PRACTICE CASES 506 Diagnostic Workup Additional Differential Diagnoses Patient NoteDifferential Diagnoses CASE DISCUSSION Blood cultures: AXR: UA: of diarrheabyrevealingapH>6or<5,respectively. infection thatiscommonamongdaycareattendees.Stool pHcandistinguishasecretoryfromanosmoticcause etiology, andculturemayrevealabacterialpathogen.Microscopyovaorparasitessuchas Stool leukocytes,culture, ovaandparasitology, andpH: as hyponatremia. Electrolytes: Serum titersfornoroviruscanbepositivewithintwoweeksofinitialsymptoms. immunoassay. Norovirus,previouslyknownastheNorwalkvirus,canbedetectedthroughPCRamplification. Rotavirus enzymeimmunoassay/norovirus PCR: output. Bacteremia: illness andlow-gradefever. patients withintussusceptionhaveonlytwoofthesefindings.Intussusceptionisalsoassociatedrecentviral (“currantjelly”)stools.Some75%of The classicpresentationincludesabdominalpain,vomiting,andbloody Intussusception: UTI: probably notpresentasacutelyisseenhere. babies inthisagegroup.Somechildrenmayhavemilkintoleranceaswell.However, milkintolerancewould current patient’s intothedietof case.Itisimportanttocounselparentsthatjuiceshouldnotbeintroduced Malabsorption: strains arecapableofcausingbacterialdiarrhea. Aeromonas, Bacterial diarrhea: gastroenteritis casesarenowcausedbyotherviruses(primarilynorovirus). ofrotavirusvaccineintothe routineinfantimmunizationschedule.Viralmost likelycauseuntiltheintroduction gastroenteritis:Viral To assessforpyelonephritisorUTI. DiarrheaininfantsmaybeanonspecificresponsetoaninfectionsuchasUTIorpyelonephritis. Aplainfilmabdominalradiographshouldpickupcharacteristics ofbowelobstructioninintussusception. and Bacteremia/sepsisshouldberuledoutinanychildwithhighfever, drowsiness,andnourine Childrenwithdiarrheafrequentlyhavemetabolicacidosisorotherelectrolyteabnormalities,such Yersinia enterocolitica.Ecoli To ruleoutbacteremia. Thisconditionmayresultfromababy’s consumptionofjuiceandmaybetheculpritin Giventheseverenatureofthisdisease,intussusceptionmustbeconsideredindifferential. Themostcommontypesofbacterialdiarrheaare Thisisthemostcommoncauseofpediatricacuteinfectiousdiarrhea.Rotaviruswas and Clostridium Rotaviruscanbedetectedthroughtherotavirusenzyme species arenormalintestinalflora,butpathogenic WBCsinthestoolwouldsuggestaninfectious Shigella, Salmonella,Campylobacterjejuni, Giardia, an PRACTICE CASES 507 “Did I do something wrong to cause this problem?” “Did I do something wrong to cause this “Is my child going to get better?” Examinee asked an open-ended question and actively listened to the response. without interrupting. Examinee asked the SP to list his/her concerns and listened to the response own words. concerns, often using the SP’s Examinee summarized the SP’s Examinee identified his/her role or position. name. Examinee correctly used patient’s Examinee made eye contact with the SP. Examinee knocked on the door before entering. Examinee introducedself by name. 2. Explain your clinical impression and workup plan to the mother. impression and workup plan to the 2. Explain your clinical the patient note after leaving the room. 3. Write 1. Take a focused history. 1. Take symptom. Bed-wetting is much more common than most people believe, and there is no reason for you or your common than most people believe, and there is no reason for you or your symptom. Bed-wetting is much more treatment options available for this condition, and after There are a number of child to feel embarrassed or guilty. physiologic abnormalities, I will discuss them with you.” we have run a few tests to rule out any Sample Examinee Response just as likely to be an isolated condition, but it’s can lead to your child’s “There are a few medical problems that DOORWAY INFORMATION DOORWAY CASE 41 CASE Building the Doctor-Patient Relationship Building the Doctor-Patient Entrance Examinee Checklist The patient’s mother offers the history; her son is in the waiting room. mother offers The patient’s Notes for the SP None. Challenging Questions to Ask Checklist/SP Sheet Patient Description The mother of Adam Davidson, an 8-year-old male child, comes to the office concerned that her son continues child, comes to the office concerned male Davidson, an 8-year-old The mother of Adam to wet the bed. Examinee Tasks Opening Scenario Reflective Listening PRACTICE CASES 508 Information Gathering ✓ Examinee eliciteddataefficientlyandaccurately.

Question aiyhsoyo nrssActually, hisfatherhadthesametroubleasakid.Frommy No,hedoes wellinschoolandhasgreatfriends.Ithinktheonly No. No. No. Family historyofenuresis No. Any majorstresses? No. No. ting Yellow. Environmental changesrelatedtowet- Nighttime awakening I’mnotsure.Sometimeshedoescomplainofpain. Snoring No. Constipation Abdominal pain Hematuria Idon’t think it’s alargeamount,butI’mnotsure.Thebediswetall Urine color Fever Urinary urgency No. Dysuria Volume ofurine Late-night eatingordrinking NonethatIcanthinkof. stress? Does theproblemincreaseintimesof Alleviating/exacerbating factors Iguesshehasalwayshadtroubleatnight.don’t thinkhehasever him? Have youeverpunishedorrewarded Itbothersme.I’mafraidhehas someunderlyingdiseaseor Onlyatnight. How hasthebehavioraffectedyou? Two orthreetimesaweek. child? How hasthebehavioraffected Mychildwetshisbed. drugs inthepast? Have youtriedanyinterventionsor Onset Time ofday Frequency Chief complaint encourage himbyrewardingondrynights. I feelirritatedsometimes,butI’veneverpunishedhim.tryto abnormality. because ofit. He isashamedofhimself.avoidsovernighttripsandsleepovers house couldhearit,sowedidn’t useitforlong. We orderedoneofthosenighttimealarms,buteveryoneinthe gone morethanafewnightswithoutanaccident. 10 yearsold. understanding, hisfatherdidn’t gainfullcontroluntilhewasabout hard thingforhimisnotbeingabletoattendsleepovers. problems. No, Ican’t thinkofanything. We haven’t movedorhadanyfamily over. I’m notsure,butitprobablydoes. Patient Response PRACTICE CASES 509 Patient Response Patient He was always on time with his development. He walked early, with his development. He walked early, He was always on time is reading at a third-grade level. talked on time, and Further examination. Follow-up tests. development Current medicationsPast medical historyPast surgical historyDrug allergies None. None. None. No. Neurologic historyNeurologic Birth history and language Child weight, height, had any problems of this kind. he has never know, As far as I Normal. Question

Examinee discussed initial management plans: Examinee discussed initial management questions or concerns. Examinee asked if the SP had any other Examinee discussed initial diagnostic impressions. Examinee discussed initial diagnostic Examinee recognized the SP’s emotions and responded with PEARLS. Examinee recognized the SP’s ✓ Sample Closure symptom, but I would still like to examine him and run some tests condition is probably an isolated Mrs. Davidson, your son’s can then discuss his treatment infection or a more serious medical problem. We to make sure he does not have an underlying options. Do you have any questions for me? Physical Examination None. Closure Connecting with the Patient PRACTICE CASES 510 Physical Examination History USMLE STEP2CS Patient Note PRACTICE CASES 511 Patient Note Patient Diagnosis #3 History Finding(s): Physical Exam Finding(s): Diagnosis #2 History Finding(s): Physical Exam Finding(s): Diagnosis #1 History Finding(s): Physical Exam Finding(s): USMLE STEP 2 CS STEP USMLE Diagnostic Workup Differential Diagnosis Differential USMLE STEP 2 CS Patient Note

History HPI: The source of the information is the patient’s mother. The mother of an 8 yo M c/o her child continuing to wet the bed several times a week. The child has never had a significant period of continence at night. He has no hematuria, fever, or urgency. There is possible dysuria, although the mother is not sure. The mother denies that the child c/o abdominal pain or constipation. The child does not snore or wake up multiple times during the night. There are no exacerbating factors, and there have been no major lifestyle changes or stresses in the family. The problem is causing distress for the child, who has been avoiding sleepovers, as well as for the mother, who is worried about the possibility of an underlying medical condition. ROS: Negative. Allergies: NKDA. Medications: None. PMH: None. PSH: None. Birth history: Normal. Developmental history: Normal. FH: Positive family history of male nocturnal enuresis.

Physical Examination None.

Differential Diagnosis Diagnosis #1: Monosymptomatic primary nocturnal enuresis History Finding(s) Physical Exam Finding(s) Chronic nocturnal enuresis Family history of enuresis

Diagnosis #2: Urinary tract infection History Finding(s) Physical Exam Finding(s) Enuresis Possible dysuria

Diagnosis #3: Secondary enuresis History Finding(s) Physical Exam Finding(s) Nocturnal enuresis PRACTICE CASES

512 USMLE STEP 2 CS Patient Note

Diagnostic Workup

Genital exam UA Urine culture PRACTICE CASES

513 PRACTICE CASES 514 Diagnostic Workup Additional Differential Diagnoses Patient NoteDifferential Diagnoses CASE DISCUSSION draws in children, blood forBUN/Crtesting shouldnotbedrawnuntilresultsfromaurinesample areobtained. draws inchildren,blood BUN/Cr: or recurrentUTIsconfirmedbyUAandurineculture. U/S—renal: morning ADHsurges,whichmaypredictapositiveresponse topharmacologictherapywithDDAVP. 1.015mayindicatealackofnighttimeandearly- nocturnal enuresis.Anearly-morningurineconcentration of< urinespecificgravity: First-morning Urine culture: have anegativepredictivevaluebetween95%and98%. UA: Genital exam: normal voidingpatternsduringthedayandremainscontinentamajorityofnights. the lastmoment,andwetsmallvolumesalmosteverynight,sometimesmultipletimesanight.Thispatienthas Functional bladderdisorder: indication ofapneathatmightwarrantfurtherevaluation. and narcolepsy. Thispatientdoesnotpresentwithsnoringorupperairwayobstruction,andthusthereisno Sleep apnea: examination canrevealimpactedstoolontheleftside. urinary bladderanddecreaseitscapacity. Thismaydelaycontinenceandlooklikeaprimarydisorder. Physical Constipation: diagnosis isfurtherunlikelybecausethechildhasnotbeencontinentforanysignificantperiod. such asthedivorceofparents,amajorillness,orabusethatmightresultinregressiontoincontinence.This Secondary enuresis: treatment. dysuria. ApositiveUAispresumptiveofUTI,andculturingurinecanestablishadefinitivediagnosisdirect urinary incontinencealoneshouldtriggeranevaluation.Additionally, accordingtothemother, hemayhave enuresis.Thispatientdoesnothavefrequencyorurgency,should bepartoftheworkupforchildhood but Urinary tractinfection(UTI): infection, andoldrecordsshouldbeevaluatedtoensurethatthepresentationisnotpartofaglobaldelay. related signsorsymptoms,suggestingamonosymptomaticpathology. Aurinesamplemustbetakentoruleout indicates aprimaryproblemasopposedtosecondaryone.Thehistoryandphysicalexamdonotprovideany enuresis:Monosymptomatic primarynocturnal To evaluateforaUTI.Clearurine,negative dipstick,andanegativemicroscopicexaminationcombined Shouldbeobtainedbeforerenalultrasoundtoevaluate renal function.To avoidunnecessaryblood Shouldbepursuedifbed-wettingcontinueswithmultiple treatments,abnormalvoidingpatterns, Wetting occursinallstagesofsleepbutisassociatedwithparticulardisorders,suchasapnea Infrequentorhardstoolsmayindicatechronicconstipation,whichcanputpressureonthe Theonly100%specifictestforUTI. To evaluatefordisorderssuchasabnormalities ofthemeatus,epispadias,andphimosis. Thepatient’s motherdoesnotreportamajortraumaorlifeenvironmentalchange EnuresismaybetheonlysymptomofUTIinchildren,andscreeningfor Childrenwithfunctionaldisordersvoidseveraltimesaday, holdurineuntil To evaluateforinsufficientADHlevelsasthecauseofpatient’s Thisisadiagnosisofexclusion.patient’s history PRACTICE CASES 515 Examinee summarized the SP’s concerns, often using the SP’s own words. concerns, often using the SP’s Examinee summarized the SP’s Examinee made eye contact with the SP. Examinee asked an open-ended question and actively listened to the response. without interrupting. Examinee asked the SP to list his/her concerns and listened to the response Examinee knocked on the door before entering. Examinee introducedself by name. Examinee identified his/her role or position. name. Examinee correctly used patient’s 2. Explain your clinical impression and workup plan to the mother. impression and workup plan to the 2. Explain your clinical the patient note after leaving the room. 3. Write 1. Take a focused history. 1. Take DOORWAY INFORMATION DOORWAY CASE 42 CASE sure one of those is not the cause.” Examinee Checklist Relationship Building the Doctor-Patient Entrance “Is my child going to have permanent brain damage from this?” “Is my child going to have permanent Sample Examinee Response no permanent in which case there should be seizure is her fever, daughter’s “The most likely explanation for your tests to make will run all the necessary that are more serious, though. We damage. There are some causes of seizures Patient Description the history; she is a good mother offers historian. The patient’s Notes for the SP condition. Express anxiety about your daughter’s Challenging Questions to Ask Checklist/SP Sheet The mother of Michaela Weber, an 11-month-old female child, comes to the emergency department after her female child, comes to the an 11-month-old Weber, The mother of Michaela daughter has a seizure. Examinee Tasks Opening Scenario Reflective Listening PRACTICE CASES 516 Information Gathering ✓ Examinee eliciteddataefficientlyandaccurately.

Question ne Thismorningat11 Mychildhadaseizure. Onset Chief complaint a ugn No. No. No. IgavehersomeChildren’s Tylenol lastnight—ithelpedalittle.I Hertemperaturewas102.9°Flastnight;shehasn’t hadanychills. Change inbowelhabitsorstool Yes, shehashadatemperatureandrunnynosethepast2days. Nausea/vomiting Ear tugging Medication forfever None. Yes. Rash Itlastedaboutaminutetotal. Fevers/chills No. No. Recent illness Sheseemedsleepyafterward. Family historyofseizures shaking Whole-body We juststarted were layingherdownfornapandbody Has thishappenedbefore? Tongue/head trauma Postictal symptoms Duration Description ofevent it itr A38-weekvaginal deliverywithnocomplications. Two monthsago for9-monthcheckup. Uptodate. Shelives withme,myhusband,andher3-year-old brother. No. No. ment Weight, height,andlanguagedevelop- Birth history Last checkup Vaccinations ShehashadPedialyteandsomebreastmilkbutnotmuchelse. Home environment Day carecenter Ill contacts ritated, playful,etc.) Appearance/demeanor (lethargic,ir- Appetite changes wet diapers smell orcolor;changeinnumberof Change inurinaryhabitsorurine color orconsistency shaking. Normal. She hasbeenmorefussythepastcoupledays,butconsolable. Fewer wetdiapersthanusual. No. didn’t takehertemperatureagain,butforeheadstillfelthot. Patient Response A . M . PRACTICE CASES 517 Patient Response Patient table food that takes iron supplements of days. She the past couple us. our pediatrician gave Follow-up tests. Current medicationsPast medical historyPast surgical historyDrug allergies Just the Tylenol. None. None. No. Eating habitsSleeping habits been eating the She is breast-fed and eats some table food but hasn’t She has not slept well the past 3 nights. Question

Examinee discussed initial management plans: Examinee discussed initial management questions or concerns. Examinee asked if the SP had any other Examinee discussed initial diagnostic impressions. Examinee discussed initial diagnostic Examinee recognized the SP’s emotions and responded with PEARLS. Examinee recognized the SP’s ✓ Mrs. Weber, it sounds as though your child has indeed had a seizure. The most likely cause is her high fevers; seizures caused as though your child has indeed had a seizure. The most likely cause is her high fevers; it sounds Mrs. Weber, like to examine your because there are many types of seizures, I would However, by fevers happen in many young children. Do that the seizures are not being caused by something more serious, like meningitis. child and also do some tests to make sure you have any questions for me? Sample Closure None. Closure Connecting with the Patient Physical Examination PRACTICE CASES 518 Physical Examination History USMLE STEP2CS Patient Note PRACTICE CASES 519 Patient Note Patient Diagnosis #3 History Finding(s): Physical Exam Finding(s): Diagnosis #2 History Finding(s): Physical Exam Finding(s): Diagnosis #1 History Finding(s): Physical Exam Finding(s): USMLE STEP 2 CS STEP USMLE Diagnostic Workup Differential Diagnosis Differential USMLE STEP 2 CS Patient Note

History HPI: The source of information is the patient’s mother. Patient is an 11-month-old F with a tonic-clonic seizure. Witnessed this A.M. by parents, lasted approx. 1 minute. No tongue or body trauma. Postictal drowsiness noted. No history of prior seizures. Patient has had rhinorrhea for past 2 days, fevers to 102.9°F with decreased PO intake, difficulty sleeping, and fewer wet diapers. No rash, nausea/vomiting, lethargy, or inconsolability. No sick contacts. ROS: Negative except as above. Allergies: NKDA. Medications: Tylenol. PMH/PSH: None. Birth history: Term uncomplicated vaginal delivery. Dietary history: Breast milk, table foods, and supplemental vitamins. Immunization history: Up to date. Developmental history: Last checkup was 2 months ago and showed normal weight, height, and development.

Physical Examination None.

Differential Diagnosis Diagnosis #1: Simple febrile seizure History Finding(s) Physical Exam Finding(s) Seizure duration < 15 minutes No prior history of seizures

Fever (Tmax 102.9°F)

Diagnosis #2: Meningitis History Finding(s) Physical Exam Finding(s) Tonic-clonic seizure

Fever (Tmax 102.9°F) Decreased appetite Decreased urine output PRACTICE CASES

520 USMLE STEP 2 CS Patient Note

Diagnosis #3: Hyponatremia History Finding(s) Physical Exam Finding(s) Seizure

Diagnostic Workup

LP—CSF analysis CBC Electrolytes PRACTICE CASES

521 PRACTICE CASES 522 Diagnostic Workup Additional Differential Diagnoses Patient NoteDifferential Diagnoses CASE DISCUSSION warrant anEEG. Electroencephalogram (EEG): CT—head: children withunexplainedhighfever. UTImaybeoccultandmust beinvestigated. Blood culture, UAandurineculture: assessing adrenalfunction.AWBCcount>15,000/μLmightbesuggestiveofoccultbacteremia. CBC, electrolytes: differentials, glucose,protein,Gramstain,culture,viralculturesandPCR,openingpressure. the mostdefinitivetestwithwhichtodiagnoseorruleoutmeningitis.ItallowsCSFanalysisofcellcountand LP—CSF analysis: and decreaseswithage. sepsis maypresentinthismannerchildrenyoungerthanoneyearofage;theincidenceishighestneonates Occult bacteremia: fever andgender, aUTIwithsubsequentpyelonephritisorbacteremia isapossibility. UTI: causes. congenital adrenalhyperplasia.Theoccurrenceinthesettingoffevermakesthislesslikelythaninfectious a poorfamilywatersdowntheirinfant’s formula.Therearealsocongenitalcausesofhyponatremia,suchas Hyponatremia: output. Aseizuresuggestsneurologicinvolvement,andthemostimportantcausetoruleoutismeningitis. infants andcouldbelikelyinthispatientconsideringthebaby’s poorappetite,highfever, anddecreasedurine clinical symptoms.Theinfantmightbeirritablebutisusuallyeasilyconsolable.Viral meningitisiscommonin Meningitis: Theydonotrequiretreatmentandcausepermanentneurologicdamage. adulthood. is afebrileseizure.Thesetendtobefamilialandmayrecurwithsubsequentillnessesbutdisappearbefore Simple febrileseizure: DecreasedurineoutputisnotlikelysecondarytoaUTIinthispatient.However, giventhepatient’s high Usedmainlytoruleoutbrainabscess,encephalitis,orhemorrhage. Inchildrenyoungerthanoneyearofage,meningitisfindingscanoftenbelimitedtofeverand Hyponatremiafromvariouscausescanresultinpediatricseizures;theclassiccaseoccurswhen Afeverwithseizuresmaybebenign,oritsuggestmeningitis.lumbarpunctureis To testforhyponatremiacausedbyanysource.Potassiumandglucoselevelshelpin Thischildhasafever>102°F(38.9°C)andnoclearsourceofinfection.Bacteremia Themostfrequentcauseofanisolatedseizureinachildwithcommonfebrileillness Usedtoidentifyepileptiformactivity, althoughasinglefebrileseizuredoesnot Thesetestsconstitutethesepsisoroccultbacteremiaworkupin PRACTICE CASES 523 99.5°F (37.5°C) Be rude and defensive. or “no.” Make most of your answers a curt “yes” on abdominal palpation. Pretend that you have LUQ tenderness 75/minute, regular 15/minute 120/80 mm Hg Examinee introducedself by name. Examinee identified his/her role or position. Examinee knocked on the door before entering. 2. Perform a focused physical exam (do not perform rectal, genitourinary, or female breast exam). not perform rectal, genitourinary, 2. Perform a focused physical exam (do workup plan to the patient. 3. Explain your clinical impression and the patient note after leaving the room. 4. Write 1. Take a focused history. a focused 1. Take “Do you think I have AIDS?” Sample Examinee Response have been exposed to HIV? It is a “What makes you think you might have AIDS? Do you believe that you not be able to tell until I have ordered some blood but I will tests.” possibility, Challenging Questions to Ask DOORWAY INFORMATION DOORWAY CASE 43 CASE Examinee Checklist Relationship Building the Doctor-Patient Entrance Checklist/SP Sheet Patient Description Patient is a 21 yo M. Notes for the SP RR: HR: Examinee Tasks Brian Davis, a 21-year-old male, comes to the office complaining of a sore throat. male, comes to the office Brian Davis, a 21-year-old Vital Signs BP: Temp: Opening Scenario PRACTICE CASES 524 Information Gathering Reflective Listening ✓ Examinee eliciteddataefficientlyandaccurately. Examinee summarizedtheSP’s concerns,oftenusingtheSP’s ownwords. Examinee askedtheSPtolisthis/herconcernsandlistenedresponsewithoutinterrupting. Examinee askedanopen-endedquestionandactivelylistenedtotheresponse. Examinee madeeyecontactwiththeSP. Examinee correctlyusedpatient’s name.

Question egtcagsYes, Ifeelthat Iamlosingweight,butdon’t knowhowmuch. Myex-girlfriendhad thesamething2monthsago.Idon’t know Ihavebeenfeelingtiredforthepast2weeks. No. Weight changes No. No. No. Ill contacts Fatigue Headache No. 4/10. Change inurinaryhabits None. No. Change inbowelhabits Nausea/vomiting I’vehadsome discomforthere(pointstotheLUQ)constantlysince No. Alleviating/exacerbating factors No. No. topain Relationship offood Severity onascale No. Radiation Abdominal pain No. Shortness ofbreath Chest pain Mildfeveroverthepast2weeks,butIdidn’t takemytemperature. Two weeksago. Yes, inmyneck(ifasked);alittlepainfulasked). Jaundice No. No. Rash beforeorafteronsetofsymptoms Swollen glandsandlymphnodes Cough Sorethroat. Night sweats Fever/chills Runny nose Onset Chief complaint No chills. Patient Response what happenedtoherbecause webrokeuparoundthattime. yesterday. PRACTICE CASES 525 course of antibiotics. Maneuver tenderness Patient Response Patient Two. CV examPulmonary examAbdominal examSkin/lymph node exam Auscultation Auscultation Inspected for rashes, lesions, lymphadenopathy Auscultation, palpation, percussion Head and neck exam Examined nose, mouth, throat, lymph nodes; checked for sinus Drug allergies No. Active with men, women, or bothNumber of sexual partners during the past year Men and women. History of STDs I told you, I had gonorrhea 4 months ago, and I was cured after a Illicit drug useTobaccoSexual activityUse of condoms No. I have a new girlfriend. I started when I was 15 years old. a pack a day. I smoke Yes, Yes. Past surgical historyFamily historyOccupationAlcohol use None. mother are alive and in good My father and health. Last year in college. on the weekends. Yes, Appetite changesCurrent medicationsPast medical history at all. feel like eating anything I don’t Tylenol. I took some antibiotics. 4 months ago. I had gonorrhea Exam Component Question

Examinee did not repeat painful maneuvers. Examinee washed his/her hands. exam. Examinee asked permission to start the Examinee used respectful draping. Examinee recognized the SP’s emotions and responded with PEARLS. Examinee recognized the SP’s ✓ ✓ Physical Examination Connecting with the Patient PRACTICE CASES 526 interested. Doyouhaveanyquestionsforme? smoking isassociatedwithavarietyofdiseases,Iadviseyoutoquitsmoking;wehavemanywayshelpif are for atleast3weeksbecauseofthepossiblerisktraumaticruptureyourspleen,whichcouldbefatal.Also,sincecigare mononucleosis isoneofthediseasesthatmightaccountforyoursymptoms,Ialsorecommendyouavoidcontactsports In themeantime,IstronglyrecommendusingcondomstoavoidanunwantedpregnancyandpreventSTDs.Sinceinfectious I recommendthatweobtainanHIVtest,andwillalsoneedtoathroatswabultrasoundofyourabdomen. infection, oritmayrepresentamoreseriousillnesssuchasHIV. We willneedtorunafewtestshelpusmakethediagnosi Mr. Davis,itislikelythatyouhaveacquiredthesameinfectionyourgirlfriendhad.Thismaybenomorethanatransientvir Sample Closure Closure Examinee askediftheSPhadanyotherquestionsorconcerns. Examinee discussedinitialmanagementplans: Examinee discussedinitialdiagnosticimpressions. Recommendation toavoidcontactsportsbecauseofthepossibleincreasedrisktraumaticsplenicrupture. Help withsmokingcessation. Safe sexpractices. Follow-up tests(includingconsentforHIVtesting). tte al s. PRACTICE CASES 527 Patient Note Patient USMLE STEP 2 CS STEP USMLE Physical Examination History USMLE STEP 2 CS Patient Note

Differential Diagnosis Diagnosis #1 History Finding(s): Physical Exam Finding(s):

Diagnosis #2 History Finding(s): Physical Exam Finding(s):

Diagnosis #3 History Finding(s): Physical Exam Finding(s):

Diagnostic Workup PRACTICE CASES

528 USMLE STEP 2 CS Patient Note

History HPI: 21 yo M c/o sore throat for the past 2 weeks. Two weeks ago he had a mild fever and fatigue, but he denies any chills, runny nose, cough, night sweats, shortness of breath, or wheezing. The patient also notes LUQ abdominal pain since yesterday. The pain is 4/10 and constant with no radiation, no relation to food, and no alleviating or exacerbating factors. He has poor appetite and subjective weight loss. His ex- girlfriend had the same symptoms 2 months ago. ROS: Negative except as above. Allergies: NKDA. Medications: Tylenol. PMH: Gonorrhea 4 months ago, treated with antibiotics. PSH: None. SH: 1 PPD since age 15; drinks heavily on weekends. Multiple female and male partners; uses condoms. FH: Noncontributory.

Physical Examination Patient is in no acute distress. VS: WNL. HEENT: Nose, mouth, and pharynx WNL. Neck: Supple, bilateral cervical lymphadenopathy. Chest: Clear breath sounds bilaterally. Heart: RRR; normal S1/S2; no murmurs, rubs, or gallops. Abdomen: Soft, nondistended, BS, no hepatosplenomegaly, mild LUQ tenderness on palpation. Skin: No rash.

Differential Diagnosis Diagnosis #1: Infectious mononucleosis History Finding(s): Physical Exam Finding(s): Sore throat for 2 weeks LUQ tenderness LUQ pain Lymphadenopathy Recent history of ill contact

Diagnosis #2: Acute HIV infection History Finding(s): Physical Exam Finding(s): Sore throat for 2 weeks Two sexual partners over past year, active with men and women PRACTICE CASES Treated for gonorrhea 4 months ago

529 USMLE STEP 2 CS Patient Note

Diagnosis #3: Streptococcal pharyngitis History Finding(s): Physical Exam Finding(s): Sore throat for 2 weeks Lymphadenopathy Low-grade fever History of cigarette smoking

Diagnostic Workup

CBC with peripheral smear Monospot test Anti-EBV antibodies HIV antibody and viral load Throat culture PRACTICE CASES

530 PRACTICE CASES 531 Neisseria Chlamydia Usually becomes positive in EBV-associated mononucleosis Usually becomes positive in EBV-associated Clinical features in patients with sore throat that predict group A Clinical features in patients with sore To check for CMV infection. To Has high negative predictive value (ie, it can accurately confirm the absence of Has high negative predictive value (ie, it can accurately confirm the absence Check antibody via ELISA and Western blot to exclude preexisting HIV Check antibody via ELISA and Western Other infections that can present with nonspecific symptoms include Other infections that can present with The differential diagnosis for “sore throat” includes many pathogens. This for “sore throat” includes many The differential diagnosis (although lower respiratory symptoms usually predominate), rubella, and (although lower respiratory symptoms Antibodies can be detected, such as IgM antibody to various EBV antigens to viral Acute HIV infection can be associated with fever, lymphadenopathy, sore throat, and a lymphadenopathy, can be associated with fever, Acute HIV infection Can reveal atypical lymphocytes in infectious mononucleosis. CMV can mimic infectious mononucleosis or acute HIV infection. Patients can present CMV can mimic infectious mononucleosis The gold standard for diagnosing bacterial pharyngitis. Findings are nonspecific, but leukocytosis may be seen in bacterial infection, and a lymphocytosis may be Findings are nonspecific, but leukocytosis Throat culture: Throat antigen: Rapid streptococcal group A streptococcal pharyngitis). CMV antibody titers/CMV PCR: within four weeks of onset of illness. Anti-EBV antibodies: to detect EBV in serum. capsid antigen (VCA) and to nuclear antigen (EBNA). There is also a PCR HIV antibody and viral load: infection, and check viral load to document acute infection. CBC: seen in viral infection. Peripheral smear: agglutination test): Monospot test (heterophil asymptomatic, requiring a high index of suspicion for clinical testing. asymptomatic, requiring a high index Other infectious etiologies: gonorrhoeae, Mycoplasma trachomatis. acute rheumatic fever. CMV infection: patients may also be and fatigue. However, lymphadenopathy, fever, with mild flulike symptoms, including generalized maculopapular rash. This stage of disease typically occurs within one month of exposure to the virus occurs within one month of exposure rash. This stage of disease typically generalized maculopapular on their own. weeks. Symptoms eventually resolve and can last up to several pharyngitis: A streptococcal Group a history of fever exudates, tender anterior cervical lymphadenopathy, streptococcal pharyngitis include tonsillar of cough. “Strep throat” must be recognized and treated to prevent (temperature > 100.4°F/38°C), and absence infectious mononucleosis caused by EBV or, less commonly, by CMV infection. The physical exam is notoriously by CMV infection. The physical exam less commonly, caused by EBV or, infectious mononucleosis presents with as in this case. This patient also splenomegaly and may be misleading, insensitive for detecting he does not exhibit exudative of infectious mononucleosis. However, a typical feature cervical lymphadenopathy, is commonly associated. feature with which infectious mononucleosis pharyngitis, another Acute HIV infection: Infectious mononucleosis: (for a unifying diagnosis) to an which could limit the differential suggests splenomegaly, LUQ pain patient’s Diagnostic Workup Additional Differential Diagnoses Additional Differential CASE DISCUSSION CASE Diagnoses Note Differential Patient PRACTICE CASES 532 Entrance Building theDoctor-Patient Relationship Examinee Checklist the brain.Thatwillhelpusseestructureofbrain andruleoutanybleedingortumor.” “I thinkit’s unlikely. To makeabsolutelysure,however, wewilldoaCTscan,whichisspecialimagingstudyof Sample ExamineeResponse “Do youthinkIhaveabraintumor?” Challenging QuestionstoAsk None. Notes fortheSP Patient isa49yoM,marriedwith3children. Patient Description Checklist/SP Sheet Examinee Tasks HR: RR: Temp: BP: Signs Vital Jay Keller, a49-year-old male,comestotheEDcomplaining ofpassingoutafewhoursearlier. Opening Scenario DOORWAY INFORMATION CASE 44 Examinee madeeyecontact with theSP. Examinee correctlyusedpatient’s name. Examinee identifiedhis/herroleorposition. selfbyname. Examinee introduced Examinee knockedonthedoorbeforeentering. 4. Write thepatientnoteafterleavingroom. 3. Explainyourclinicalimpressionandworkupplantothepatient. 2. Performafocusedphysicalexam(donotperformrectal,genitourinary, orfemalebreastexam). 1. Take afocusedhistory. 135/85mmHg 16/minute 76/minute,regular 98.0°F(36.7°C) PRACTICE CASES 533 Patient Response fell down. No. No. No. when I suddenly fell down and blacked out. when I suddenly fell down on the ground. I think I lost consciousness and then fell No. Weight changesWeight No. Head traumaSimilar falls, lightheadedness, or pass- ing out before Gait abnormality No. No. ness HeadachesChest pain, shortness of breathAbdominal pain, nausea/vomiting, diarrhea/constipation No. No. Lost control of the bladderWeakness/numbnessSpeech difficultiesConfusion after regaining conscious- No. No. No. Shaking (seizure)Duration of shakingBit tongue and legs started shaking after I my wife told me that my arms Yes, She said around 30 seconds. No. Palpitations before the fallSensing something unusual before losing consciousness (sounds, lights, smells, etc.) Spinning/lightheadedness just before I fell down, my heart started racing. Yes, before the fall. I felt lightheaded right Describe what happened Loss of consciousness before, during, or after the fall Duration of loss of consciousness morning I was taking the groceries to the car with my wife This several minutes. My wife told me that I did not respond to her for Chief complaint I passed out. Question

Examinee elicited data efficiently and accurately. Examinee elicited data Examinee asked an open-ended question and actively listened to the response. listened to the question and actively asked an open-ended Examinee interrupting. response without and listened to the his/her concerns asked the SP to list Examinee own words. SP’s concerns, often using the the SP’s Examinee summarized ✓ Reflective Listening Reflective Information Gathering PRACTICE CASES 534 Physical Examination Connecting withthePatient ✓ ✓ Examinee didnotrepeatpainfulmaneuvers. Examinee usedrespectfuldraping. Examinee askedpermissiontostarttheexam. Examinee washedhis/herhands. Examinee recognizedtheSP’s emotionsandrespondedwithPEARLS.

ExamComponent Question xrmte Palpatedperipheral pulses Mentalstatus, cranialnerves(includingfunduscopicexam),motor Palpation,auscultation,orthostaticvitalsigns Auscultation Neurologic exam Inspection(head,mouth),carotidauscultationandpalpation, Extremities Pulmonary exam CV exam Head andneckexam No. Yes, withmywife. No,Istoppedayearago.hadsmokedpackdayfortheprevious No. Yes, No(toall4). Idrink3–4beersaweek. Drug allergies Clerkinavideostore. Sexual activity Myfatherdiedfromaheartattackatage55,andmymotherin Tobacco Illicit druguse Appendectomy. CAGE questions pressureforthepast15years;heartattack1yearago. Highblood Alcohol use Occupation Hydrochlorothiazide,captopril,aspirin,atenolol. No. Family history Past surgicalhistory Past medicalhistory Current medications Appetite changes 25 years. health. good exam, DTRs,cerebellar, Rombergtest,gait,sensoryexam thyroid exam Patient Response Maneuver PRACTICE CASES r 535 . You . You Follow-up tests. Examinee discussed initial diagnostic impressions. discussed initial diagnostic Examinee plans: discussed initial management Examinee SP had any other questions or concerns. Examinee asked if the Doing so is just like having a constant ECG, and it will allow us to detect any abnormal heartbeats you might have. We will will might have. We us to detect any abnormal heartbeats you having a constant ECG, and it will allow Doing so is just like Do you have any questions for me? start with these tests and then go from there. Sample Closure so I am going to get a CT scan of you the reason you passed out this morning, I need to run some tests to determine Keller, Mr. abnormalities blood tests to look for infections or electrolyte or masses, and I will then order some head to look for bleeding monitor for 24 hours. out, so I will also ask you to wear a heart heart was racing just before you passed mentioned that your Closure PRACTICE CASES 536 Physical Examination History USMLE STEP2CS Patient Note PRACTICE CASES 537 Patient Note Patient Diagnosis #3 History Finding(s): Physical Exam Finding(s): Diagnosis #2 History Finding(s): Physical Exam Finding(s): Diagnosis #1 History Finding(s): Physical Exam Finding(s): USMLE STEP 2 CS STEP USMLE Diagnostic Workup Differential Diagnosis Differential USMLE STEP 2 CS Patient Note

History HPI: 49 yo M c/o 1 episode of syncope that occurred a few hours ago. He was taking the groceries to the car with his wife when he suddenly felt lightheaded, had palpitations, lost consciousness, and fell down. He was unconscious for several minutes. His wife recalls that his arms and legs started shaking for 30 seconds after he fell down. He denies subsequent confusion, weakness or numbness, speech difficulties, tongue biting, or incontinence. ROS: Negative except as above. Allergies: NKDA. Medications: HCTZ, captopril, aspirin, atenolol. PMH: Hypertension for the past 15 years; MI 1 year ago. PSH: Appendectomy. SH: 1 PPD for 25 years; quit smoking 1 year ago. Drinks 3–4 beers/week, CAGE 0/4, no illicit drugs. FH: Father died from an MI at age 55.

Physical Examination Patient is in no acute distress. VS: WNL, no orthostatic changes. HEENT: NC/AT, PERRLA, no funduscopic abnormalities, no tongue trauma. Neck: Supple, no carotid bruits, 2+ carotid pulses with good upstroke bilaterally, thyroid normal. Chest: Clear breath sounds bilaterally. Heart: Apical impulse not displaced; RRR; normal S1/S2; no murmurs, rubs, or gallops. Extremities: Symmetric 2+ brachial, radial, and dorsalis pedis pulses bilaterally. Neuro: Cranial nerves: 2–12 grossly intact. Motor: Strength 5/5 throughout. Sensation: Intact to pinprick and soft touch bilaterally. DTRs: Symmetric 2+ in upper and lower extremities, Babinski bilaterally. Cerebellar: Romberg, finger to nose normal. Gait: Normal.

Differential Diagnosis Diagnosis #1: Convulsive syncope History Finding(s): Physical Exam Finding(s): Loss of consciousness lasting several minutes Arms and legs shaking for 30 seconds No subsequent confusion or weakness PRACTICE CASES

538 USMLE STEP 2 CS Patient Note

Diagnosis #2: Cardiac arrhythmia History Finding(s): Physical Exam Finding(s): Loss of consciousness preceded by palpitations and lightheadedness Taking a β-blocker (atenolol) No subsequent confusion or weakness History of MI

Diagnosis #3: Seizure History Finding(s): Physical Exam Finding(s): Loss of consciousness lasting several minutes Arms and legs shaking for 30 seconds Sudden onset

Diagnostic Workup

CBC Electrolytes ECG and Holter or event monitor CT—head or MRI—brain EEG PRACTICE CASES

539 PRACTICE CASES 540 Diagnostic Workup Additional Differential Diagnoses Patient NoteDifferential Diagnoses CASE DISCUSSION Must becomparedtobaseline prolactinlevels. Prolactin: CXR: LVH withsmallresidualcavitysize,andhypertrophicobstructivecardiomyopathy). Echocardiography: EEG: other brainstemsigns). present. MRAishelpfulwhenvertebrobasilarinsufficiency issuspected(ie,whensyncopeaccompaniedby MRI—brain: abscess. CT—head: ECG andHolteroreventmonitor: arrhythmia orothercausesofsyncope. CBC, electrolytes: other physicalfindingsmakesthisunlikelyincase. myxoma) arecommonlyexertionalorpostexertionalandoccurwithoutwarning.Thelackofamurmur Aortic stenosis: were normal. postural (ie,theyoccurwhengettingupfromalyingorseatedposition),andthispatient’s orthostaticvitalsigns orthostatic hypotensionandsyncope.However, lightheadednessandsyncopeinthisconditionareusually Drug-induced orthostatichypotension: feeling “faint.”Thisisalsothemechanismofsyncopeinpostmicturitionsyncope. vagal tonewithresultinghypotension.Syncopeisoftenheraldedbynausea,sweating,tachycardia,pallor, and Vasovagal syncope: are oftenaccompaniedbytonguebiting,incontinence,andprolongedconfusionordrowsinesspostictally. a varietyofcauses,includingmetabolicfactors,trauma,vascularandbraintumors.Tonic-clonic seizures Seizure: ventricular tachycardia,and indicate thepresenceofanunderlyingarrhythmia.Thispatient’s historyofMIincreaseshisriskdeveloping Cardiac arrhythmia: hypoperfusion. ThereisnoEEGcorrelate,andaseizureworkupnotrequired. Convulsive syncope: To ruleoutlungmass,cardiomyopathy, or other pathology. To evaluatesuspectedseizureactivity. Seizuresusuallyoccurunpredictablyinamannerunrelatedtopostureorexertion.Theymaystemfrom Oftenelevatedwithin30–60minutesofageneralizedseizure (itisuselessafterthattimeinterval). Thetestofchoicetoexcludeintracranialhemorrhage.Alsorulesouttumor, trauma,priorstroke,or ProvidesbetteranatomicdetailthanCT. Indicatedwhenfocalneurologicsignsandsymptomsare Thisandothermechanicalcauses(eg,hypertrophicobstructivecardiomyopathy, atrial To ruleoutanemia,evidenceofhyperviscosity, orelectrolyteimbalancethatcouldleadto To ruleoutmechanicalcausesofsyncope(eg,severeaortic stenosis,atrialmyxoma,severe Thisoftenoccursinthesettingofemotionalstressorpainandmaybeduetoexcessive Cardiacsyncopetypicallyoccurswithoutwarning,althoughahistoryofpalpitationsmay Seizure-likeactivityoftenoccursaftersyncopeandisduetoglobalcerebral β -blocker therapymaycontributetobradyarrhythmia. To evaluatepossiblearrhythmia. Thepatient’s antihypertensivemedicationsincreasehisriskfor SECTION 5

Top-Rated Review Resources REVIEW RESOURCES 542 following: USMLE Step2CSexamination.Thisisbasedonanumberoffactors,includingthe The B B B A A evaluations. Eachbookreceivesoneofthefollowingratings: The ratingscaleiscomposedofsixlettergradesthatreflectthedetailedstudent and utilityforstudying.Finally, eachbookreceivesa of thebook.Mostentriesalsoinclude the First Author This sectionisadatabaseofrecommendedclinicalsciencereviewbooks,sample A studying fortheUSMLEStep2CS.Foreachbook,welist examination books,andcommercialreviewcoursesmarketedtomedicalstudents been alargerangeofopinionsorlimitedstudentfeedback onanyparticularbook. comments andoverallratingsrepresentaconsensusopinion, buttheremayhave hundreds ofmedicalstudentsfromschoolsacross thecountry. Thesummary Evaluations arebasedonthecumulativeresultsofformal andinformalsurveysof available intheclinicalsciences. have alsoavoidedlistingorcommentingonthewidevariety ofgeneraltextbooks ings arewellwrittenandinformativebutnotideal forboardpreparation.We other thanreviewingfortheUSMLEStep2CSexam. Please notethat HOW TOUSETHEDATABASE − + − + Number ofPages The importanceofthedisciplineonUSMLEStep2CSexamination The qualityandnumberofotherbooksavailableinthesamediscipline The lengthofthetext(longerisnotnecessarilybetter) graphs) The qualityandappropriatenessoftheillustrations(eg,graphs,diagrams,photo- The qualityofwrittenanswerstosamplequestions The qualityandnumberofsamplequestions The appropriatenessandaccuracyofthebook The readabilityofthetext The costofthebook Rating Good, butuseonlyafterexhaustingbettersources. Good, Very forboardsreview;chooseamongthegroup. good Excellent forboardsreview. ismeanttoreflecttheoverallusefulnessofbookinpreparingfor (oreditor),the the ratingdoesnotreflectqualityofbookfor purposes , the ISBN Code Current Publisher Summary Comments , the Approximate ListPrice , the Rating. Copyright Year Manybookswithlowrat- thatdescribetheirstyle Title , andthe ofthebook, , the Edition Format , REVIEW RESOURCES 543 Publishers’ prices change frequently. Publishers’ markup. an additional bookstores often charge Individual the quality of updating varies. and out frequently, New editions come be reissued through another publisher. The same book may books or suggestions for alternate modesbooks or suggestions for of study that may be useful in preparing tutorials, commercial review as flash cards, computer-based the examination, such sites. courses, and Internet Web We actively encourage medical students and faculty to submit their opinions and and faculty to submit their opinions actively encourage medical students We so that we may update our science review books listed here ratings of the clinical we ask that publishers and to Contribute,” p. xvii). In addition, database (see “How including new editions copies of clinical science review books, authors submit review new also solicit reviews of in our database, for evaluation. We and books not included None of the material in this book, including the ratings, reflects the opinion or influ- None of the material in this book, including omissions will gladly be corrected if brought to All errors and ence of the publisher. publisher. the attention of the authors through the Disclaimer/Conflict of Interest Statement Disclaimer/Conflict of Interest Please note that the data listed are subject to change because: are subject to change that the data listed Please note REVIEW RESOURCES 544 B B A + Cons: pedics, OB/GYN,thenervoussystem,pediatricandphonemedicine, andtraumamedicine. features 91reviewcasescoveringinternalmedicineandfamilypractice,surgeryortho- note writingcoverOB/GYN,neurology, psychology, pediatrics,andtraumamedicine.Also cludes awealthofbasicscienceinformation.Chaptersdevotedtothephysicalexamand Pros: A sleekreviewbookpresentedinanon-workbookformat. Lippincott Williams &Wilkins, 2007,296pages,2ndedition,ISBN9780781766937 A ClinicalSkillsExam ReviewfortheUSMLE NMS Summary: recent changestotheexamandpatientnoteformat. tice casesdoesnotfacilitatepracticewithapartner. Contenthasnotbeenupdatedtoreflect Cons: plus 100full-colorimagesthathelpclarifyandorganizeinformation. simulates appropriatedoctor-patient communication.Includesintegratedtablesandfigures and isthensummarizedby“Take HomePoints.”Casesarewrittenwithsampledialoguethat Information isorganizedlinearlyinto“Symptoms,”“Diagnosis,”and“Treatment” sections mon complaintsanddiagnoses,includingtelephoneencountersdifficultconversations. Pros: An organizedcase-by-casepresentationofpatientencounters. Oxford UniversityPress,2010,268pages,1stedition,ISBN9780195398236 S Step2ClinicalSkillsTriageUSMLE about notewriting. Summary: diagnoses. Cons: edition alsooffersinformationandtipsregardingnotewriting. best tointeractwithpatients.Alsoincludes43casesthatcoveravarietyofspecialties.This Pros: A reviewbookofcase-by-casepresentations. Kaplan, 2013,408pages,3rdedition,ISBN9781609788896 B Step2CSCoreCases USMLE patient examornote. number ofconcisecases.However, thetextdoesnotprovideastep-by-stepapproachto Summary: and patientnoteformat. is neededfortheexam.Contenthasnotbeenupdatedtoreflect recentchangestotheexam find thattheinformationprovidedistoogeneralandlacking inspecificdetailsaboutwhat a diagnosticworkupplan,andshortparagraphonclinical correlation.Somereadersmay are compressedintoaone-pageformatwithwrittenpatient note,adifferentialdiagnosis, CHWECHTEN RIAS ROTTMAN Providesextensivecoverageofphysicalexamtechniquesandsigns/symptoms,in- Offers40casesthatsimulateactualexaminationscenarioscoveringthemostcom- Someofthematerialmaynotbesufficientlyhighyieldfor Step2CSexam.Cases Somereadersmayfindthatcasesarenotsufficientlydetailed.Theformatoftheprac- Features an introductory sectiononappropriatephraseologyfortheexamandhow Features anintroductory Some reviewersfeltthatcasesweretoosimpleandlackingindetaileddifferential Asuccinctbutthoroughreviewofthemostcommonclinicalconditions. A good bookforbothStep2CS andStep2CKpreparationthatprovidesalarge Agood A greatreviewbookwithavarietyofclinicalcasesandupdatedinformation $44.95 $39.95 $40.00 Review Review Review REVIEW RESOURCES 545 Review Review $44.95 $39.95 May be best suited to examinees who seek practice documenting checklists and May be best suited to examinees who seek A clear and thorough concept-oriented review book that is useful for all Step 2 A clear and thorough concept-oriented Because the workbook format takes up a significant amount of space, only 30 review Because the workbook format takes up a If you are not used to the integrated format of the content, you may not find this text the integrated format of the content, you If you are not used to Easy to read and printed in a large point size. Includes a detailed chapter outlining the Easy to read and printed in a large point Features 61 common clinical cases described in detail. Incorporates solid clinical skills clinical cases described in detail. Incorporates Features 61 common WARTZ HARMA checklist is included with each case. Cons: updated to reflect recent changes to the exam and cases are provided. Content has not been patient note format. Summary: skills and U.S. culture make it more patient notes. The lengthy sections on communication concise suitable for IMGs. Those who seek a quick review might be better served by a more text. mat. Summary: CS test takers. the USMLEThe Ultimate Guide and Review for Step 2 Clinical Skills Exam S ISBN 9781416037279 2006, 402 pages, 1st edition, Elsevier, A large workbook-style review text. Pros: are devoted to communication skills, and an basics of the exam. Approximately 20 pages to U.S. culture. A standardized patient grading additional 10 pages are devoted to a guide S 380 pages, 1st edition, ISBN 9781453608043 CreateSpace, 2010, of the Step 2 CS exam. reviews core clinical concepts in the context A thorough text that Pros: The text thoroughly ex- in preparation for the Step 2 CS exam. for evaluating a patient so that test takers are better of standardized patients and examiners plains the expectations specific exam elements such as useful for IMGs. Also includes prepared, which is especially and phone interviews. ethical dilemmas, counseling, Cons: the differential diagnoses in- the suggested interview questions and appropriate. Some found to practice with a partner. of the practice cases makes it challenging adequate. The format recent changes to the exam and patient note for- Content has not been updated to reflect Core Concepts for USMLECore Concepts Step 2 CS: and Goal-OrientedA Focused Approach B B REVIEW RESOURCES 546 http://usmlecsprep.com http://www.kaptest.com/Medical-Licensing/Step2cs.html http://www.usmleworld.com exam: The followingisacollectionofonlineresourcesandWeb sitesthat mayalsohelpyoupreparefortheStep2CS B USEFUL WEBSITES − needed onthephysicalexam. Overall, ahigh-yieldbookthatmayrequireadditionaltextsifmorein-depthinformationis However, thecontentmaynotbeasaccurateordetailedother, morethoroughsources. Cases arestreamlinedtoallowforpracticewithouttakingupcopiousamountsofspace. Summary: has notbeenupdatedtoreflectrecentchangestheexamandpatientnoteformat. discussion includesreferrals,whicharenotrequiredontheactualStep2CSexam.Content interpret examfindings,andtheformattingmaybesomewhatconfusing.Themanagement Cons: noses andcorrespondingpagenumbers. workup, andabriefparagraphonclinicalcorrelation.Alsofeaturescaseindexwithdiag- page “answers”listedattheendofbookincludeadifferentialdiagnosis,anappropriate checklist, aphysicalexamandfindingsareprovidedaswell.One- organized bychiefcomplaintalongwithabriefparagraphonhistory. Apatienthistory cal exam,andpatientnotewrite-up,presentedinbullet-pointformat.Alsofeatures55cases Pros: A pocket-sizedtextpresentedinworkbookformat. McGraw-Hill, 2007,348pages,2ndedition,ISBN9780071488235 R Step2CS CS Checklists:PortableReviewfortheUSMLE OONEY Providesdetaileddescriptionsofthecomponentsacompletehealthhistory, physi- Offerslimitedgeneralinformationonthephysicalexamandhowtoperform AconcisetextthatcontainsalltheinformationneededforStep2CSreview. $34.95 Review APPENDIX ACRONYMS AND ABBREVIATIONS

Abbreviation Meaning Abbreviation Meaning AAMC Association of American Medical CPK-MB creatine phosphokinase, MB fraction Colleges Cr ABG arterial blood gas CRP C-reactive protein ACE angiotensin-converting enzyme CS Clinical Skills [exam] ADH antidiuretic hormone CSA Clinical Skills Assessment ADHD attention-deficit hyperactivity disorder CSEC Clinical Skills Evaluation ADLs activities of daily living Collaboration [testing center] AFB acid-fast bacillus CSF cerebrospinal fluid AIDS acquired immunodeficiency syndrome CT computed tomography ALT alanine aminotransferase CTA computed tomography angiogram ANA antinuclear antibody CV cardiovascular ANCA antineutrophil cytoplasmic antibody CVA costovertebral angle AP anteroposterior CXR chest x-ray AST aspartate aminotransferase D&C dilatation and curettage AXR abdominal x-ray DDAVP 1-deamino (8-D-arginine) vasopressin BNP B-type natriuretic peptide DEXA dual-energy x-ray absorptiometry BP blood pressure DFA direct fluorescent antibody [test] BPH benign prostatic hypertrophy DHEAS dehydroepiandrosterone sulfate BPPV benign paroxysmal positional vertigo DI diabetes insipidus BS bowel sounds DIC disseminated intravascular coagulation BUN DM diabetes mellitus CBC complete blood count DNA deoxyribonucleic acid CC chief complaint DSD dementia syndrome of depression CCP cyclic citrullinated peptide dsDNA double-stranded deoxyribonucleic acid CD cluster of differentiation DSM Diagnostic and Statistical Manual [of CEA carcinoembryonic antigen Mental Disorders] CHF congestive heart failure DTR deep tendon reflex CIS Communication/Interpersonal Skills DVT deep venous thrombosis [CS score] EBNA Epstein-Barr nuclear antigen CK Clinical Knowledge [exam] EBV Epstein-Barr virus CMV cytomegalovirus ECFMG Educational Commission for Foreign CNS central nervous system Medical Graduates c/o complains of ECG electrocardiogram COPD chronic obstructive pulmonary disease ED emergency department, erectile CPK creatine phosphokinase dysfunction

547 ACRONYMS AND ABBREVIATIONS 548 A lymphadenopathy kidney, ureter, bladder[imaging] potassiumhydroxide jugularvenousdistention LAD Interactive Web Application intravenous pyelography KUB intravenous KOH JVD internationalmedicalgraduate IWA IVP immunoglobulin InternationalMedicalEducation IV IMG Integrated Clinical Encounter[CS IMED instrumentalactivitiesofdailyliving Ig herpessimplexvirus hormone replacement therapy heartrate ICE IADLs humanpapillomavirus historyofpresent illness HSV virus humanimmunodeficiency HRT HR acid[scan] hepatobiliaryiminodiacetic HPV HPI 5-hydroxyindoleaceticacid HIV head,eyes,ears,nose,andthroat hydrochlorothiazide HIDA humanchorionicgonadotropin 5-HIAA hepatitisBvirus HEENT hepatitis Bsurfaceantigen HCTZ hCG HBV HBsAg FederationofStateMedicalBoards follicle-stimulatinghormone familyhistory FT FSMB focusedassessmentwithsonographyfor FSH ethylalcohol essentialtremor FH erythrocytesedimentationrate FAST EtOH endoscopicretrograde ET extraocularmovementsintact ESR ear, nose,andthroat electromyogram ERCP enzyme-linkedimmunosorbentassay electroencephalogram EOMI ENT EMG ELISA EEG Meaning Abbreviation Igastrointestinal growthhormone gastroesophagealrefluxdisease HbA glucose-6-phosphate dehydrogenase GI GH GERD G6PD FT 4 3 1c hemoglobin A free thyroxine free triiodothyronine Directory score] trauma [scan] cholangiopancreatography 1c V motorvehicleaccident metatarsophalangeal[joint] magnetic resonance imaging MVA magneticresonance MTP magneticresonanceangiography myocardial infarction MRI MRCP metered-doseinhaler methylenedioxymethamphetamine MRA MI majordepressivedisorder metacarpophalangeal [joint] MDMA leftventricularhypertrophy MDI leftupperquadrant lumbarpuncture MDD MCP lossofconsciousness LVH lastmenstrualperiod leftlowerquadrant LUQ luteinizinghormone LP LOC lactatedehydrogenase LMP LLQ LH LDH Meaning Abbreviation S pastsurgical history prostate-specific antigen prorenata [as needed] packperday; purifiedprotein PSH patientnote PSA pointofmaximalimpulse prn pastmedical history pelvicinflammatory disease PPD pulmonaryfunctiontest PN PMI PMH Parkinson’s disease PID pupils equal,round,andreactiveto polymerasechainreaction PFT phencyclidine(“angeldust”) PERRLA polycysticovarysyndrome posteroanterior PD PCR overthecounter PCP obstructivesleepapnea PCOS oralcontraceptivepill PA OTC OnlineApplicantStatusand nauseaor vomiting OSA OCP nonsteroidalanti-inflammatorydrug NationalResidencyMatchingProgram OASIS noknowndrugallergies N/V normocephalic/atraumatic NSAID NationalBoardofMedicalExaminers NRMP mitralvalvestenosis NKDA NC/AT NBME MVS cholangiopancreatography (“Ecstasy”) derivative [tuberculinskintest] light andaccommodation Information System ACRONYMS AND ABBREVIATIONS 549 Examination VSWBCWNLXRyo vital signs white blood cell within normal limits x-ray year old Abbreviation Meaning TCATEETIATIBC tricyclic antidepressant TMTMJ transesophageal echocardiography TOEFL attack transient ischemic capacity total iron-binding TSETSH tympanic membrane TTE joint temporomandibular a Foreign Language of English as Test UAURI test of spoken English thyroid-stimulating hormone U/SUSMLE transthoracic echocardiography urinalysis UTD respiratory infection upper United States Medical Licensing UTI ultrasound VCAVDRL up to date [vaccinations] V/Q urinary tract infection viral capsid antigen Disease Research Laboratory Venereal ventilation-perfusion [scan] of] antidiuretic hormone tomography triiodothyronine thyroxine 3 4 T TB tuberculosis SOBSPSPECTSTD shortness of breath T single-photon emission computed standardized patient disease sexually transmitted RUQSEPSHSIADH right upper quadrant SLE Spoken English Proficiency [CS score] syndrome of inappropriate [secretion social history systemic lupus erythematosus RFRLQROSRPRRR rheumatoid factor right lower quadrant RRR review of systems rapid plasma reagin respiratory rate regular rate and rhythm Abbreviation Meaning PTPTSDPTTRARBC time prothrombin posttraumatic stress disorder partial thromboplastin time arthritis rheumatoid red blood cell ACRONYMS AND ABBREVIATIONS 550 NOTES INDEX

A Achilles tendinitis, 357, 358 air travel. See travel information for abbreviations, using in patient acoustic neuroma students note, 82 dizziness and, 94, 95, 339 albuterol, tremor caused by, 488 abdominal exam hearing loss and, 295 alcoholic hepatitis bruits, 62 acromegaly, with abdominal pain and, 112 checklist for, 58 hyperglycemia, 196 jaundice and, 331 signs simulated by SPs, 61 action tremor, 488 alcohol use abdominal pain active listening, 9 acetaminophen toxicity in children, 135 acute alcoholic hepatitis, 112 and, 330, 331 diagnostic workup for, 257–259 acute appendicitis, 115 asking patient about, 51 jaundice and, 324 acute bronchitis, 321, 322 aspiration and, 322 key history/physical exam, 112– acute cholecystitis, 113, 115 blood in stool and, 119 116 acute gallstone cholangitis, 113 confusion/memory loss motor vehicle accident and, 172, acute glomerulonephritis and, 89 176 abdominal pain and, 113 cough and, 322 practice case for (21-year-old hematuria and, 120 counseling patient on, 78 female), 416–425 acute otitis media, 133, 365, 367 depression and, 92 practice case for (48-year-old acute pancreatitis, 112, 114 erectile dysfunction and, 122, female), 251–259 acute pyelonephritis, 121 214, 396 sore throat and, 524 acute renal failure, 94 fatigue and, 98 abdominal x-ray, for motor vehicle acute stress disorder, 108 gout and, 348 accident, 179 acute subhepatic appendicitis, 113 insomnia and, 100 ABO incompatibility, 301, 303 acute viral hepatitis, 112 laryngeal cancer and, 240, 242 abortion. See miscarriage adjustment disorder loss of consciousness and, 95 abscess, with abdominal pain, 115 in children, 138 numbness/weakness and, 96 abuse. See also sexual assault depression and, 92 pancreatitis and, 386, 387 challenging questions on, 67 fatigue and, 98 peripheral neuropathy and, 97, child abuse, 135 insomnia and, 405 214 diagnosing, 127–128, 452, 453 adnexal torsion, 424 psychosis and, 93 dyspareunia and, 127, 286 adrenal disease, 462 tremor and, 488 elder abuse, 205 adrenal hyperplasia, 522 weight change and, 108, 109 joint/limb pain and, 128 adrenal malignancy, 123 alopecia practice case for, 445–453 Aeromonas, 506 hypertension and, 389 questions to ask patient agenda, setting with patient, 43 hypogonadism and, 395, 396 about, 54 agoraphobia, 108 hypothyroidism and, 443, 444 ACE inhibitors, with cough, 377 AIDS. See also HIV lithium and, 498 acetaminophen toxicity, 330, 331 challenging questions on, 71 weight gain and, 491 achalasia, with dysphagia, 110 cough/shortness of breath alternative medicines, challenging Achilles reflex, testing, 59 and, 104 questions on, 68

551 Alzheimer’s disease, 89, 90, 432, antihypertensive medication ascariasis, 116, 117 434 dizziness and, 338 ascending cholangitis, 113 challenging questions on, 75 erectile dysfunction and, 396 Asherman’s syndrome, 124, 277 amebiasis syncope and, 540 aspirin, and hearing loss, 295 abdominal pain and, 115 anxiety. See also depression assault. See also abuse diarrhea and, 117 amenorrhea and, 124 practice case for, 216–224 amenorrhea dysphagia and, 110 asthma, 103, 104, 377, 471 as chief complaint, 123–124 hyperthyroidism and, 404 ataxia diagnostic workup for, 276 insomnia and, 403, 405 headache and, 87 practice case for, 269–277 palpitations and, 108 simulated by SPs, 62 ampullary carcinoma, 329 tremor and, 488 atenolol, and syncope, 539 amyotrophic lateral sclerosis (ALS) weight loss and, 108 Atlanta travel information, 24–27 dysphagia and, 110 anxious patients, 65 atrial myxoma, 540 numbness/weakness and, 98 aortic aneurysm, 112 atrophic endometrium, 125 anal fissure, 119 aortic dissection, 105, 106, 107, 150 atrophic vaginitis analgesic withdrawal, 88 aortic stenosis, 96, 540 dyspareunia and, 127 anal stenosis, 135 apnea, 98, 99 vaginal bleeding and, 125 anemia apology, in patient attention-deficit hyperactivity diabetes and, 215 communication, 43 disorder (ADHD), 137, 138 diagnosing, 452 appearance, professional, 9 atypical depression fatigue and, 98, 99 appendicitis fatigue and, 99 iron deficiency and, 453 abdominal pain and, 112, 113, weight gain and, 109 multiple myeloma and, 250 115, 422, 424 atypical patients, 65–67 palpitations and, 107 challenging questions on, 77 atypical pneumonia, 103, 377 angina in children, 135 auditory hallucinations. arm pain and, 131 appetite, asking patient about, 49 See hallucinations chest pain and, 105, 106 arms auscultating abdominal sounds, 58 confusion/memory loss and, 91 examining flexion and auscultating heart sounds in female palpitations and, 107 extension, 59 patients, 61 angiodysplasia examining joints, 60 auscultating lung sounds, 58 blood in stool and, 119 pain in, 197–205 avoidant personality disorder, 108 GI bleeding and, 480 arrhythmia angioedema, 414 confusion/memory loss and, 91 B angry patients, 65–66, 72 syncope and, 539, 540 Babinski’s sign anhedonia arsenic exposure, tremor and, 488 back pain and, 163 depression and, 92, 443 arthritis memory loss and, 433 posttraumatic stress disorder back pain and, 168 simulated by SPs, 62 and, 443 gonococcal septic, 348 testing, 59 ankle sprain, 358 gout, 346, 348 back, examining, 60 ankylosing spondylitis, 133 insomnia and, 100 back pain anorexia nervosa joint/limb pain and, 129, 130, diagnostic workup for, 168–169 amenorrhea and, 123, 124 131 key history and physical night sweats and, 100 nongonococcal septic, 348 exam, 132–133 weight loss and, 109 osteoarthritis, 348 practice case for, 161–169 antibiotics pseudogout, 348 bacteremia colitis and, 479, 480 rheumatoid, 347, 348 diarrhea and, 506 hearing loss and, 295 symmetric, 348 occult, 312, 522

552 bacterial conjunctivitis, 136 body language, 9 bruising bacterial diarrhea, 504, 506 Boerhaave syndrome, 112 domestic violence and, 452 bacterial vaginosis, 126, 286 bones examining, 61 Baker’s cyst rupture, 131, 232, 233 arm pain, practice case for, 197– bruits, 62–63 baldness. See alopecia 205 buffalo hump, in Cushing’s bathroom breaks requested by broken, 197–204, 205, 222 syndrome, 109 patient, 74 bowel infarction/perforation, 115 bursitis, retrocalcaneal, 358 bed-wetting, in children, 507–514 bowel symptoms behavioral issues in patient, 68–74 blood in stool, 119 C behavioral problems in constipation/diarrhea, 116– caffeine use childhood, 137–138 118 fatigue and, 98 benign paroxysmal positional infarction/perforation, 115 insomnia and, 100, 101, 403, vertigo (BPPV), 94, 95, questions to ask patient 404, 405 338, 339 about, 49 tremor and, 488 benign prostatic hypertrophy (BPH) brachioradialis reflex, 59 withdrawal from, 88 hematuria and, 158, 159 bradykinetic gait, 486 CAGE questionnaire, 78 urinary symptoms, 121 brain tumor calcaneal stress fracture, 357, 358 bereavement, 92 headache and, 267, 268 calf pain, 225–234 beta-agonists, and tremor, 488 hearing loss and, 295 Campylobacter jejuni, 506 beta-blockers tremor and, 488 cancer bradyarrhythmia and, 540 breast-feeding jaundice, 303 abdominal pain and, 112, 114, erectile dysfunction and, 396 breast milk jaundice, 303 258 syncope and, 539 breathing problems in challenging questions on, 70, biceps reflex, 59 children, 407–416 75, 76 biliary atresia, 303 breath, shortness of constipation/diarrhea and, 116 biliary colic, 258 key history/physical exam, 102– cough and, 104, 376 biliary obstruction, 329, 331 104 dysphagia and, 110 biphasic stridor, 414 neck mass and, 111 hematuria and, 119 bipolar disorder, 92 questions to ask patient hoarseness and, 240 birth control pills. about, 48 jaundice and, 329 See contraception simulation of by SPs, 61 neck pain and, 250 bladder cancer, 120, 157, 159 brief psychotic disorder, 462 urinary symptoms and, 121 bladder stones, 121 broken bones weight loss and, 109 bleeding assault and, 222, 224 candidal vaginitis, 126, 453 in stool, 119, 472–480 osteoporosis and, 205 car accident, practice case for, 170– upper GI, 118 practice case for, 197–205 178 in urine (hematuria), 119, broken neck. See cervical fracture carcinoid syndrome 152–160 bronchiectasis, 103 diarrhea and, 118 vaginal, 124–125 bronchitis night sweats and, 100 blood alcohol level test, 179 chronic, 469, 471 palpitations and, 107 blood pressure. See hypertension; cough and, 103, 104, 321, 322 cardiac arrhythmia hypotension lung cancer and, 376 confusion/memory loss and, 91 blood thinners pneumonia and, 376 cough/shortness of breath blood in stool and, 119 Brudzinski’s sign and, 104 upper GI bleeding and, 118 neck pain and, 245 palpitations and, 107, 108 blood transfusions, challenging in neurologic exam, 59 syncope and, 95, 96, 539, 540 questions on, 73 simulated by SPs, 62 cardiac enzyme tests, 150

553 cardiac thrills, checking for, 57 chickenpox. See varicella clerkships covered in exam, 4 cardiac valvular disease, 104 chief complaint, summarizing, 64 clinical cases, 86 cardiovascular exam, 57 child abuse, 135. See also abuse clinical encounter. See also patient- carotid artery dissection, 87 child care concerns of patient, 71 centered interviews (PCIs) carotid bruit, 63 children. See pediatric patients closure, 63–65 carpal tunnel syndrome chlamydia. See also sexually counseling patients, 77–79 joint/limb pain and, 129 transmitted diseases doorway information, 4, 40, numbness/weakness and, 98 assault and, 224 42–44 celiac disease sore throat and, 531 entering the examination abdominal pain and, 115 vaginal discharge and, 126 room, 42 diarrhea/constipation and, 117, chocolate cyst, 424 overview of, 40–41, 80–81 118 cholangiocarcinoma patient note, 79–84 cellulitis, 131, 232, 233 abdominal pain and, 112 physical exam, 54–63 cephalohematoma, 303 jaundice and, 329 clinical rotations for IMGs, 17–19 cerebellum cholangitis, 331 Clostridium difficile, 117, 506 testing, 59 cholecystitis, 112, 113, 114, 115, closure, 63–65 tumors of, 94 256, 258 cluster headaches, 87, 88, 89, 268 cervical cancer, 125 choledocholithiasis, 112, 113, 258 CNS tumor, 97 cervical fracture, 249, 250 cholesterol panel, 151 CNS vasculitis, 87, 97 cervical polyp, 125 chorea, simulated by SPs, 62 coagulation disorder cervical spondylosis, 250 chronic bronchitis, 469, 471. See hematuria and, 120 cervicitis also bronchitis vaginal bleeding and, 125 dyspareunia and, 127, 284, 286 chronic fatigue syndrome, 99 coagulopathy, 105 vaginal bleeding and, 125 chronic obstructive pulmonary coal mining, cough and, 469, 470, vaginal discharge and, 126 disease (COPD), 377, 469, 471 challenging questions, 64–65, 471 cocaine use. See substance use 67–77 chronic pancreatitis cochlear nerve damage, 293, 295 behavioral issues, 68–74 abdominal pain and, 112, 113, cognitive impairment. confidentiality/ethical issues, 67– 115 See confusion; memory loss 68 diarrhea/constipation and, 117 cold intolerance disease-related, 75–77 symptoms of, 386, 387 hypothyroidism and, 443, 444, chest pain chronic paroxysmal 498 challenging questions on, 70 hemicrania, 268 lithium and, 498 as chief complaint, 105–107, chronic renal insufficiency, 137 weight gain and, 496 142–145 chronic subdural hematoma, 89, colic differential diagnosis for, 148– 90 GI symptoms and, 136 151 circadian rhythm sleep urinary tract infections motor vehicle accident and, 171, disorder, 101 and, 312 176 cirrhosis colitis palpitations and, 107 acetaminophen toxicity and, 331 bloody stool and, 479, 480 postprandial, 149 hyperprolactinemia and, 277 ulcerative, 480 practice case for, 142–151 primary biliary, 331 colon cancer chest x-ray CIS (Communication and abdominal pain and, 114 for chest pain, 151 Interpersonal Skills) score, 5 bloody stool and, 119 for motor vehicle accident, 178 elements of, 9–10 fatigue and, 99 CHF. See heart failure (CHF) medical history, taking, 10–11 colonoscopy, challenging questions Chicago travel information, 27–29 patient counseling, 11 on, 75

554 colorectal cancer contraception currant jelly stools, 506 bloody stool and, 119, 478, 480 asking patient about, 51 Cushing’s syndrome constipation/diarrhea and, 116 challenging questions on, 76 diabetes and, 195, 196 Communication and Interpersonal counseling patients on, 78–79, weight gain and, 109, 498 Skills (CIS) score, 5 317, 343, 419, 526 cyanosis, with chest pain, 144 elements of, 9–10 emergency, post-assault, 224 cyclothymic disorder, 92 medical history, taking, 10–11 conversion disorder, 97 cystic fibrosis communication skills convulsive syncope, 95, 538, 540 hyperglycemia and, 196 empathy, 66 Coombs test, 304 short stature and, 137 honesty, 65 COPD (chronic obstructive cystitis interruptions, avoiding, 45 pulmonary disease), 103, abdominal pain and, 116 jargon, avoiding, 10, 11, 44, 65 104 urinary symptoms and, 121 PEARLS elements, 43–44 coronary artery disease, 150 cystoscopy, 159 in physical exam, 55 corticosteroids, and upper GI cytomegalovirus (CMV) redirecting conversation, 45 bleeding, 118 neonatal jaundice and, 303 reflective listening, 43 costochondritis, 105, 106, 150 sore throat and, 531 summary technique, 45 costs. See financial concerns of transitioning, 46 patient D compassion for patient, 10 cough data-gathering score, 5 concentration, testing, 59 as chief complaint, 102–104 data interpretation score, 5 condoms. See contraception croup and, 414 D-dimer test, 233 conduct disorder, 137, 138 diagnostic workup for, 321–322 deafness. See hearing loss confidentiality agreement, 12 practice case for (26-year-old deep venous thrombosis. See DVT confidentiality issues, 67–68 male), 314–322 (deep venous thrombosis) confusion. See also memory loss practice case for (32-year-old degenerative arthritis, 168 challenging questions on, 75 male), 463–471 dehydration examining patient practice case for (54-year-old dizziness and, 94 experiencing, 66 female), 369–378 in infant, 504 key history/physical exam, 89–91 questions to ask patient orthostatic hypotension and, 339 lithium and, 498 about, 47 syncope and, 95 practice case for, 426–435 counseling patients, 11, 77–79 delirium, 90, 91 simulated by SPs, 62 coxsackievirus, 322 delusional disorder, 99 congenital adrenal hyperplasia, 522 cranial nerves, testing, 59 delusions, in psychosis, 93 congestive heart failure. See heart crepitus, 60 dementia failure (CHF) Creutzfeldt-Jakob disease, 90, 91 Alzheimer’s disease, 432, 434 conjunctival pallor, 453 Crohn’s disease asking patients about, 53–54 conjunctivitis, 136 abdominal pain and, 115 depression and, 433, 434 consciousness, loss of bloody stool and, 119 practice case for, 426–435 as chief complaint, 95–96 diarrhea and, 118 simulated by SPs, 62 palpitations and, 107 croup, 412, 414 vascular, 89, 90, 433, 434 practice case for, 532–540 crying patients, 66 depression constipation cryptosporidiosis, 117 amenorrhea and, 124 bed-wetting and, 514 CSEC centers, 22–37 atypical, 99 as chief complaint, 116–118 CT scan confusion/memory loss and, 88, in children, 135 for chest pain, 151 89, 90 hypothyroidism and, 498 for hematuria, 159 constipation and, 116 constitutional short stature, 137 for motor vehicle accident, 179 counseling patients on, 79, 382

555 depression (Continued) diagnoses, communicating diaphoresis, 95 dementia and, 433, 434 to patient, 64. See diarrhea diagnosis of, 443 also differential diagnosis bacterial, 506 dyspareunia and, 127 diagnostic workup, 84, 86 colitis and, 479 erectile dysfunction and, 396 for abdominal pain, 257–259, dizziness and, 333, 338, 339 fatigue and, 98, 99, 385, 386, 387 423–425 gastroenteritis and, 504 headaches and, 268 for amenorrhea, 276 key history/physical exam, 116– hypothyroidism and, 498 for arm pain, 205 118 insomnia and, 100, 101, 405 for back pain, 168–169 night sweats and, 100 key history/physical exam, 92 for bed-wetting, 513–514 palpitations and, 107 major depressive disorder, 443, for bloody stool, 479–480 pancreatic cancer and, 387 444, 453 for chest pain, 150–151 practice case for (6-month-old nausea/vomiting and, 111 for cough, 321–322, 376–378 infant), 499–506 pancreatic cancer and, 387 for cough, chronic, 470–471 diet practice case for, 436–445 for diabetes, 195, 196, 214–215 asking patient about, 49 psychosis and, 93 for diarrhea in infant, 505–506 low-fiber, constipation caused thyroid disease and, 387 for dizziness, 338–339 by, 116 weight change and, 108, 109 for fatigue, 443–444 differential diagnosis, 82–83, 86 de Quervain’s tenosynovitis, 129 for fever in child, 311–313, discoid rash, 348 diabetes insipidus, 99, 453 366–368 disease-related questions, 75–77 diabetes mellitus for leg pain, 232–234 disk herniation counseling patient on, 78 for hallucinations, 461–462 back pain and, 132, 133, 166, diabetic peripheral for headaches, 267 168 neuropathy, 97 for hearing loss, 294–295 neck pain and, 248, 250 diagnosing, 453 for heel pain, 357–359 dislocated shoulder, 131, 204, 205 diagnostic workup for, 195, 196, for hematuria, 159–160 disseminated gonorrhea, 131 214–215 for hypertension, 395–396 diverticulitis, 114, 115 erectile dysfunction and, 122 for insomnia, 404–406 diverticulosis fatigue and, 99 for jaundice in adult, 330 bloody stool and, 119 hearing loss and, 295 for jaundice in infant, 302, insomnia and, 100 304 constipation/diarrhea and, 116 low back pain and, 132 for knee pain, 347–349 GI bleeding and, 480 nausea/vomiting and, 111 for memory loss in geriatric Dix-Hallpike maneuver, 94, 334, numbness/weakness and, 96 patient, 433–435 338, 339 obesity and, 498 for motor vehicle accident, 178 dizziness. See also vertigo palpitations and, 107 for neck pain, 249 key history/physical exam, 94–95 pancreatic cancer and, 387 for noisy breathing in lithium and, 498 peripheral neuropathy, 213, child, 413–415 practice case for, 332–339 214 for painful sexual questions to ask patient polyuria/polydipsia and, 452 intercourse, 285–286 about, 50 practice case for adult for pneumonia, 322 domestic violence. See abuse; patient, 206–215 for pregnancy, 187, 188 assault practice case for pediatric for seizures, 521–522 doorway information, 4, 42–44 patient, 189–196 for sexual assault, 224 incorrect, handling, 68 urinary incontinence and, 122 for sore throat, 530–531 time allotment for, 40 vaginal bleeding and, 125 for throat hoarseness, 241–242 draping manners, 9 weight gain and, 109 for weight gain, 497–498 drinking alcohol. See alcohol use

556 drug use. See alcohol use; substance amenorrhea and, 186, 188 Escherichia coli, 506 use vaginal bleeding and, 125 esophageal cancer, 110 dry skin edema esophageal rupture, 106, 107 hypothyroidism and, 498 chest pain and, 144 esophageal spasm, 105, 106, 107 lithium and, 498 pitting, 231 esophageal varices, 118 Dubin-Johnson syndrome, 331 weight gain and, 109 esophagitis DVT (deep venous thrombosis), elbows chest pain and, 105, 106 131, 231, 233 examining, 60 dysphagia and, 110 dysentery, 119 pain in, 130, 131 essential tremor, 486, 488 dysfunctional uterine bleeding, 125 elder abuse, 205. See also abuse; ethical issues, 67–68 dysmenorrhea. See also menstrual geriatric patients examination dates, canceling/ problems emergency contraception, 224. See rescheduling, 7 abdominal pain and, 116 also contraception executive function, 434 endometriosis and, 285, 424 empathy, 10, 43, 66 expiratory stridor, 414 dyspareunia employment concerns of patient externships, for IMGs, 18 endometriosis and, 285, 424 job loss, 71 extrahepatic biliary key history/physical exam, 126– preemployment medical obstruction, 331 127 checkup, 463–471 eye exam practice case for, 278–286 returning to work, 70 checklist for, 56–57 dyspepsia, 258 encephalitis, 88 signs simulated by SPs in, 62 dysphagia encephalopathy, 90 key history/physical exam endometrial cancer, 125 F for, 110 endometrial hyperplasia, 125 facial nerve palsy, 96 neck mass and, 111 endometriosis facial paralysis, 62 dysphoria, 443 abdominal pain and, 115, 116, factitious disorder, 128 dyspnea 424 failing the USMLE Step 2 CS chest pain and, 105, 143 dysmenorrhea and, 285, 286 exam, 8 cough/shortness of breath dyspareunia and, 127 fainting. See consciousness, loss of and, 102 English proficiency, 3, 6, 16–17 familial neonatal in infants, 306 entering the examination room, 42 hyperbilirubinemia, 303 motor vehicle accident and, 176 enterocolitis, 135 familial obesity, 498. See palpitations and, 107 enuresis, 507–514 also obesity; weight upper respiratory infection epidural abscess, 88 family history, asking patient and, 312 epidural hematoma, 97 about, 50 dysthymic disorder, 92 epigastric pain. See abdominal pain fat embolism, 107 dysuria, 121 epiglottitis, 412, 414 fatigue epilepsy, 462 anemia and, 452 E erectile dysfunction (ED) anxiety and, 405 ears, examining, 57 alcohol use and, 214 hyperthyroidism and, 404 eating disorders. See also anorexia challenging questions on, 68 hypothyroidism and, 498 nervosa diabetes and, 208, 213, 214 key history/physical exam, 98–99 challenging questions on, 77 hypertension and, 389, 395 lithium and, 498 eating habits. See diet hypogonadism and, 395 lupus and, 347 ECFMG certification, 14 key history/physical exam, 122 practice case for (32-year-old ectopic pregnancy medication-induced, 394, 396 female), 445–453 abdominal pain and, 115, 423, practice case for, 388–397 practice case for (46-year-old 424 vascular disease and, 396 male), 436–444

557 fatigue (Continued) fremitus, 62 chest pain and, 105, 106, 107, practice case for (61-year-old functional bladder disorder, 514 149–150 male), 379–388 functional incontinence, 122 in children, 135, 136 tremor and, 488 funduscopic exam, 57 cough and, 103, 104, 377, 471 fear of surgery, 69 dysphagia and, 110 G febrile seizures, 520, 522 hoarseness and, 242 Federation of State Medical Boards G6PD deficiency, 303 laryngitis and, 241, 242 (FSMB), 2 gait abnormalities geriatric patients feeding history, asking patients back pain and, 163 abuse of, 205 about, 52 confusion/memory loss and, 90 aspiration in, 322 fever simulated by SPs, 62 confusion/memory loss in, 426– appendicitis and, 422 galactorrhea 435 bacteremia and, 506 amenorrhea and, 123 hearing loss in, 293, 295 in children, 133–134, 305–313, hyperprolactinemia and, 277 laryngeal cancer and, 240 360–368 galactosemia, 303 osteoporosis in, 205 gastroenteritis and, 504 gallstones pseudogout in, 348 intussussception and, 506 abdominal pain and, 113, 114, giardiasis abdominal pain and, 115 lung cancer and, 376 256, 258 pancreatitis and, 387 diarrhea/constipation and, 116, otitis media and, 365, 367 gastric cancer 117 pneumonia and, 320, 376 abdominal pain and, 258 glomerulonephritis questions to ask patient neck mass and, 111 abdominal pain and, 113 about, 48 upper GI bleeding and, 118 hematuria and, 159 streptococcal pharyngitis gastritis gonococcal septic arthritis, 348 and, 530 abdominal pain and, 112, 113, gonorrhea. See also sexually tremor and, 488 257 transmitted diseases tuberculosis and, 375 nausea/vomiting and, 111 assault and, 224 upper respiratory infection upper GI bleeding and, 118 HIV and, 529 and, 321 gastrocnemius muscle rupture, 233 joint/limb pain and, 129, 131 fifth disease, 134, 367, 368 gastroenteritis pelvic inflammatory disease financial concerns of patient, 66, abdominal pain and, 114, 115, (PID) and, 286 67 424 sore throat and, 531 challenging questions on, 70 childhood fever and, 134 vaginal discharge and, 126 Fitz-Hugh–Curtis syndrome, 113 in children, 135, 136, 312, 504, gout, 129, 130, 346, 348 food poisoning 506 group B streptococcus, 303 abdominal pain and, 114 diarrhea/constipation and, 117, growth and development, asking childhood fever and, 134 118 patient about, 52 diarrhea and, 117 gastrointestinal bleeding, 118 growth hormone (GH) foot pain, 130 gastrointestinal parasitic infections deficiency, 137 diabetes and, 207, 214 abdominal pain and, 115 Guillain-Barré syndrome, 97 in heel, 350–359 constipation/diarrhea and, 116, gynecologic history, asking patients foreign body aspiration, 412, 414 117 about, 52 forgetfulness. See confusion; generalized anxiety disorder memory loss fatigue and, 99 H fractured bone palpitations and, 108 Haglund’s deformity, 358 assault and, 222, 224 GERD (gastroesophageal reflux hair loss. See alopecia osteoporosis and, 205 disease) hallucinations practice case for, 197–205 abdominal pain and, 113, 258 challenging questions on, 74

558 practice case for, 454–462 hemiparesis, 62 night sweats and, 100 psychosis and, 93, 461 hemochromatosis, 196 pregnancy and, 188 substance use and, 462 hemophilia, 125 sore throat and, 101, 102, 529, hands, testing motor control of, 59 hemoptysis, 102 531 hand washing, 9 hemorrhage, subarachnoid, 88 weight loss and, 108, 109 headache hemorrhoids, 119, 478, 480 hoarseness key history/physical exam, 87–89 hemothorax epiglottitis and, 414 lithium and, 498 assault and, 224 laryngitis and, 414 meningitis and, 367 motor vehicle accident and, 177, practice case for, 235–242 practice case for, 260–268 178 stridor and, 414 questions to ask patient hepatitis Hodgkin’s/non-Hodgkin’s about, 48 abdominal pain and, 112, 113, lymphoma, 111 hearing loss 114 Homans’ sign dizziness and, 94, 333 alcohol use and, 331 DVT and, 233 examining patient with, 66 in children, 135 leg pain and, 231 labyrinthitis and, 339 diarrhea and, 117 honesty with patient, 65 Ménière’s disease and, 337 jaundice and, 330, 331 Horner’s syndrome perilymphatic fistula and, 339 sore throat and, 102 headaches and, 268 practice case for, 287–295 hepatitis B. See also sexually numbness/weakness and, 98 simulated by SPs, 62 transmitted diseases hotels. See travel information for heart attacks. See also myocardial assault and, 224 students infarction (MI) hepatomegaly, 331 Houston travel information, 29–32 responding to patient fears of, 70 herbal medicines, challenging humeral fracture, 131, 203, 205 heartburn, 102 questions on, 68 hydrocephalus, 89, 90 heart disease, 95 hernia hyperbilirubinemia heart failure (CHF) abdominal pain and, 114 familial, 303 chest pain and, 106 in children, 136 jaundice and, 331 cough/shortness of breath herniated disk. See disk hypercalcemia and, 103, 104 herniation fatigue and, 99 heart murmurs, 242 herpes simplex virus (HSV), 303 multiple myeloma and, 250 heart palpitations. See palpitations hips nausea/vomiting and, 111 heart sounds dislocated/fractured, 129, 130 hypercoagulability testing, 233 examining in female patients, 61 examining, 60 hypercortisolism, 498 murmurs simulated by SPs, 63 Hirschsprung’s disease, 135 hyperglycemia. See also diabetes heel pain, 350–359 hirsutism mellitus HEENT exam, 56–57 amenorrhea and, 123 causes of, 196 hematochezia, 478 polycystic ovary syndrome yeast infections and, 453 hematomas and, 277 hyperlipidemia epidural, 97 weight gain and, 109 chest pain and, 105 leg pain and, 233 history. See patient history erectile dysfunction and, 395 subdural, 91, 97 HIV. See also AIDS; sexually obesity and, 498 hematuria transmitted diseases hyperprolactinemia, 123, 276, 277 differential diagnosis for, 157– assault and, 224 hypersomnia, 101 160, 159–160 challenging questions on, 68 depression and, 405 key history/physical exam, 119– cough and, 104, 322 hypertension 120 depression and, 444 chest pain and, 105 practice case for, 152–160 dysphagia and, 110 Cushing’s syndrome and, 498

559 hypertension (Continued) hypotension infectious mononucleosis, 77, 102, erectile dysfunction and, 122, dizziness and, 94, 338, 339 329, 531 395 loss of consciousness and, 96 infertility headache and, 88 vasovagal syncope and, 540 endometriosis and, 424 hearing loss and, 295 hypothyroidism polycystic ovary syndrome memory loss and, 433 amenorrhea and, 123 and, 277 palpitations and, 107 in children, 135, 137 inflammatory bowel disease PCP intoxication and, 462 confusion/memory loss and, 89, arthritis and, 129 practice case for, 388–396 90 bloody stool and, 119 substance use and, 460 diarrhea/constipation and, 116 diarrhea/constipation and, 116, hyperthyroidism fatigue and, 98, 99, 387, 443, 117 amenorrhea and, 124 444, 453 insomnia in children, 137 hoarseness and, 242 caffeine and, 404, 405 diarrhea and, 117, 118 hyperprolactinemia and, 277 key history/physical exam, 100– insomnia and, 404 neonatal jaundice and, 303 101 night sweats and, 100 neuropsychiatric symptoms, 434 posttraumatic stress disorder palpitations and, 107, 108 peripheral neuropathy and, 214 and, 443 pregnancy and, 188 pregnancy and, 188 practice case for, 397–406 symptoms of, 405 primary biliary cirrhosis and, 331 inspiratory stridor, 414 tremor and, 488 symptoms of, 498 insulin-induced hypoglycemia, 213, weight loss and, 109 vaginal bleeding and, 125 214 hypertrophic obstructive weight gain and, 109, 496 insulinoma, 109 cardiomyopathy, 540 Integrated Clinical Encounter hyperventilation I (ICE) score, 5 anxiety and, 405 ICE (integrated clinical encounter) international medical graduates numbness/weakness and, 97 score, 5 (IMGs), 13–19 palpitations and, 107 ileus, and abdominal pain, 114 application tips, 15–16 hypnagogic hallucinations, 462 IMGs (international medical clinical rotations and hypnopompic hallucinations, 462 graduates), 13–19 observerships, 17–19 hypocalcemia, 97 application tips, 15–16 eligibility, determining, 13–15 hypoglycemia clinical rotations and English proficiency, 16–17 confusion/memory loss and, 91 observerships, 17–19 failure rate of, 6 insulin-induced, 213, 214 eligibility, determining, 13–15 registering for exam, 6 loss of consciousness and, 95, 96 English proficiency, 16–17 scheduling USMLE Step 2 CS, numbness/weakness and, 96 failure rate, 6 14 palpitations and, 107 registering for exam, 6 visa, obtaining, 15 tremor and, 488 scheduling USMLE Step 2 CS, interpersonal skills, 5 weight gain and, 109 14 interviewing patients. See patient- hypogonadism visa, obtaining, 15 centered interviews (PCIs) hypertension and, 396 impotence. See erectile dysfunction intestinal obstruction, 114 sexual dysfunction and, 395 (ED) intracranial abscess, 88 hypomenorrhea inclusion body myositis, 132 intracranial hemorrhage, 88 pregnancy and, 275 incontinence. See urinary intracranial mass lesion weight gain and, 496 incontinence headache and, 267, 268 hyponatremia infants. See pediatric patients hearing loss and, 339 diarrhea and, 506 infectious diseases, challenging intracranial neoplasms, 87, 88, 89, seizures and, 521, 522 questions on, 77 90, 91, 97, 98

560 intracranial venous thrombosis, 88 knees lung disease, and coal mines, 471 introductions to patient, 9 examining, 60 lungs, percussing, 58 intussusception, 134, 135, 506 fracture in, 133 lupus, 129, 131, 347, 348 irritable bowel syndrome pain in, 130, 340–349 Lyme arthritis, 130 abdominal pain and, 115 lymphadenopathy in children, 135 L laryngeal cancer and, 240 diarrhea/constipation and, 116, labral disease, 205 mononucleosis and, 531 117 labyrinthitis, 94, 95, 339 night sweats and, 100 ischemic bowel disease, 119 lactose intolerance sore throat and, 101 itching. See pruritus in children, 135, 136 streptococcal pharyngitis diarrhea/constipation and, 117 and, 530 J laryngeal cancer, 240, 242 lymphoma jargon, avoiding, 10, 11, 44, 65 laryngeal papilloma, 414 cough/shortness of breath jaundice laryngitis, 241, 242, 414 and, 104 acetaminophen and, 330 lateral epicondylitis, 130 night sweats and, 100 biliary obstruction and, 329 lead poisoning in children, 135 diagnostic workup for, 330 legitimization, in patient M hepatitis and, 330 communication, 44 major depressive disorder (MDD), pancreatic cancer and, 387 legs. See also joint/limb pain 92. See also depression practice case for, 296–304 pain in, 225–234 diagnostic criteria for, 444 joint exam, 60 testing flexion and extension, 59 domestic violence and, 453 signs simulated by SPs in, 62 Leriche syndrome, 131 insomnia and, 101 joint/limb pain lesions malabsorption in heel, practice case for, 350– examining, 61 in infant, 504, 506 359 simulated by SPs, 63 weight loss and, 109 in knees, practice case for, 340– leukemia, 100 malar rash, 348 349 Lewy body dementia, 90 malingering key history/physical exam, 128– Lhermitte’s sign, 245, 248 back pain as, 168 132 lid lag, 62 challenging questions on, 72 in legs, practice case for, 225– ligaments, torn/sprained, 233 loss of consciousness as, 95 234 limb pain. See joint/limb pain low back pain as, 133 practice case for, 197–205 lithium numbness/weakness as, 97, 98 questions to ask patient tremor and, 488 in pediatric patients, 135 about, 50 weight gain and, 497, 498 Mallory-Weiss tear, 118 jugular venous distention (JVD) liver disease manic episodes, 138 checking for, 57 bloody stool and, 119 the Match, 13–14 chest pain and, 144 upper GI bleeding and, 118 measles, 134 Los Angeles travel Meckel’s diverticulum, 135 K information, 32–34 medial epicondylitis, 98 keratitis, 136 low back pain, 132–133 median nerve Kernig’s sign luggage, storing during exam, 12 compression of, 129 neck pain and, 245 lumbar muscle strain, 132, 133, overuse injury of, 98 in neurologic exam, 59 168 medical costs. See financial simulated by SPs, 62 lumbar spinal stenosis, 131, 132, concerns of patient kidney stones 167, 168 medical history. See patient history abdominal pain and, 114 lung abscess, 103, 104, 377 medications, asking patient low back pain and, 132 lung cancer, 103, 104, 376, 377 about, 66

561 memory loss. See also confusion mitral valve prolapse, 108 National Board of Medical Alzheimer’s disease and, 89, 90, mitral valve stenosis (MVS) Examiners (NBME), 2 432, 434 cough/shortness of breath nausea key history/physical exam, 89–91 and, 104 abdominal pain and, 252, 256 practice case for, 426–435 dysphagia and, 110 appendicitis and, 422 testing for, 58–59 hoarseness and, 242 chest pain and, 105, 143 Ménière’s disease mixed incontinence, 122 dizziness and, 94, 333 diagnosing, 295 molar pregnancy, 125, 187, 188 fatigue and, 380, 385 dizziness and, 94, 95, 337, 339 moles, examining, 61, 63 intracranial mass lesion and, 267 meningeal signs, testing for, 59 mononucleosis key history/physical exam, 111 meningitis challenging questions on, 77 loss of consciousness and, 95 childhood fever and, 133, 134 sore throat and, 102, 529, 531 migraine and, 266, 268 fever and, 312, 366, 367 monosymptomatic primary night sweats and, 100 headache and, 87, 88 nocturnal enuresis, 512, 514 questions to ask patient seizures and, 520, 522 moon facies, in Cushing’s about, 47 meningococcal meningitis, 366, 367 syndrome, 109, 498 upper GI bleeding and, 118 menopause, 124 motor system, testing, 59 neck mass, 111 menstrual problems motor vehicle accident neck pain, 243–250 abdominal pain and, 116 diagnostic workup for, 178 neonatal sepsis, 133, 301, 303 amenorrhea, 123–124, 269–277 practice case for, 170–179 nephrolithiasis dysmenorrhea, 285 mouth, examining, 57 abdominal pain and, 112, 115 endometriosis and, 424 multiple myeloma hematuria and, 120 heavy flow, 452 back pain and, 168 urinary symptoms and, 121 hyperthyroidism and, 405 neck pain and, 250 nervous system signs simulated by hypothyroidism and, 498 peripheral neuropathy and, 214 SPs, 62 weight gain and, 109, 491, 496 multiple sclerosis, 97, 98 neuroleptics, tremor and, 488 mental status exam. See neurologic murmurs, 62–63 neurologic exam, 58–59 exam Murphy’s sign, 58, 112, 113, 256, neuropathy, peripheral, 97 mercury exposure, and tremor, 488 258, 326, 382 neurosyphilis, 89, 90, 97 mesenteric ischemia, 112, 113, 114, myasthenia gravis nevi, examining, 61, 63 258 fatigue and, 453 nightmares, 100 metabolic acidosis, 506 numbness/weakness and, 97, 98 night sweats metastatic cancer Mycoplasma, 102, 103, 531 key history/physical exam, 100 back pain and, 168 myeloma, 97 lung cancer and, 376 cough/shortness of breath and, 104 myocardial infarction (MI) tuberculosis and, 375 neck pain and, 250 arm pain and, 131 nocturia, 121 metoclopramide, and Parkinson’s chest pain and, 105, 106, 107, hematuria and, 157 disease, 488 148, 150 nocturnal enuresis, 512 midbrain lesion, 488 dementia and, 433, 434 noisy breathing in child, 407–416 migraine, 87, 88, 89, 266, 268. See loss of consciousness and, 96 nongonococcal septic arthritis, 348 also headache myositis, 131 norovirus, 506 numbness/weakness and, 98 myositis ossificans, 233 nose, examining, 57 mini-mental status exam, 61 note taking, 46 miscarriage N NSAIDs (nonsteroidal anti- abdominal pain and, 116, 424 narcolepsy inflammatory drugs) challenging questions on, 76 fatigue and, 99 abdominal pain and, 256 vaginal bleeding and, 125 hallucinations and, 462 gastritis and, 258

562 peptic ulcer disease and, 258 osteoarthritis fatigue and, 386, 387 upper GI bleeding and, 118 diagnosing, 348 gallstones and, 387 nuchal rigidity joint/limb pain and, 130, 131 hyperglycemia and, 196 headaches and, 88 neck pain and, 250 panic attacks, 107, 108 simulated by SPs, 62 osteogenesis imperfecta, 127 Pap smear, 188 numbness osteopenia, 358 parainfluenza, 414 headache and, 87 osteoporosis, 204, 205 paraproteinemia, 97 key history/physical exam, 96–98 otitis media, 133, 135, 311, 312, parkinsonism, simulated by SPs, 62 nutrition. See diet 365, 367 Parkinson’s disease nystagmus otosclerosis, 294, 295 practice case for, 481–489 PCP intoxication and, 462 ototoxicity, 295 tremor and, 486, 488 simulated by SPs, 62 ovarian cysts partial duodenal atresia, 135 substance use and, 460 abdominal pain and, 115, 424 partnership, in patient vaginal bleeding and, 125 communication, 43 O ovarian failure, 277 parvovirus B19 infection (fifth obesity. See also weight ovarian torsion disease), 129, 134, 367 challenging questions on, 76 abdominal pain and, 115 passing out. See consciousness, loss of diabetes and, 452 vaginal bleeding and, 125 patellar reflex, 59 DVT and, 233 overflow incontinence, 122 patient-centered interviews (PCIs), polycystic ovary syndrome 5–6, 42–44 and, 277 P challenging questions, 64–65 observerships for IMGs, 17–19 pain interruptions, avoiding, 45 obstetric history, asking patient challenging questions on, 72 questions to ask, 46–54 about, 52 examining patient summary technique, 45 obstructive sleep apnea (OSA), 99, experiencing, 61, 66 taking notes, 46 101, 405 questions to ask patient transitioning into closure, 64 occult bacteremia, 312, 522 about, 47, 50 patient encounter. See clinical odynophagia, 101 painful intercourse. See dyspareunia encounter oligomenorrhea pain medication, challenging patient history, 10–11 amenorrhea and, 275, 277 questions on, 72, 75 questions to ask patient weight gain and, 496 palpitations about, 50 On-line Applicant Status and anxiety and, 405 summarizing, 82 Information System caffeine and, 404 taking, 10–11, 44–54 (OASIS) account, 7 diabetes and, 207 time allotment for, 40 oppositional defiant disorder, 137, hyperthyroidism and, 404 patient note, 79–84 138 key history/physical exam, 107– character limits for, 80 oral contraceptive pills. See 108 physical exam, summarizing, 82 also contraception pancreatic cancer preparing for, 84 headache and, 88 abdominal pain and, 112 scoring, 84 oral ulcers, 347, 348 diagnosing, 387 summarizing history, 82 ordering tests, 84 fatigue and, 385 time allotment for, 80 orientation videos, 6–7 jaundice and, 329, 331 writing differential, 82–83 orthostatic hypotension pancreatitis patients. See also standardized dehydration-induced, 94, 339 abdominal pain and, 112, 113, patients (SPs) diarrhea and, 338 114, 115, 258 atypical, guidelines for, 65–67 drug-induced, 96 chest pain and, 107 counseling and delivering syncope and, 540 diarrhea/constipation and, 117 information to, 11

563 patients (Continued) percussing lungs, 58 handwashing before, 54 names, memorizing, 42 perforated ulcer, 258 for headaches, 87 professional interactions with, 9 pericarditis of HEENT, 56 trusting relationship with, 42–43 chest pain and, 105, 106, 107, of joints, 60 payment concerns, alleviating, 66 150 of neurologic system, 58–59 PCIs. See patient-centered cough/shortness of breath privacy considerations, 54 interviews (PCIs) and, 103 of pulmonary system, 58 PCP intoxication, 460, 462. See perilymphatic fistula, 339 scoring of, 55 also substance use peripheral neuropathy, 97 SP simulations in, 61–63 PEARLS elements, 43 alcoholic, 214 summarizing, 64, 82 pedal edema, checking for, 57 diabetic, 213, 214 time allotment for, 40, 54 pediatric patients, 4 peripheral vascular disease physiologic jaundice, 301, 303 bed-wetting, 507–514 joint/limb pain and, 131 physiologic tremor, 487, 488 behavioral problems in, 137–138 low back pain and, 132 pink eye in children, 136 breathing difficulty, 407–416 peritonsillar abscess, 414 pitting edema, 231 diabetes in, 189–196 personal safety, asking patient pituitary tumor, 123, 124 diarrhea in, 499–506 about, 54 plantar fasciitis, 130, 353, 356, 358 fever in, 133–134, 305–313, petechial rash, 367 pleurisy, 106 360–368 Peyronie’s disease, 122, 396 pleuritis, 178 foreign body aspiration, 412, 414 Phalen’s sign, 98 pleurodynia, 322 gastrointestinal symptoms simulated by SPs, 62 Plummer-Vinson syndrome, 110 in, 134–136 pharyngitis pneumoconiosis, 470, 471 history, taking, 52 practice case for, 523–531 pneumonia jaundice in, 296–304 sore throat and, 102 abdominal pain and, 114 meningitis in, 366, 367 streptococcal, 530, 531 atypical, 377 otitis media in, 311, 312 pheochromocytoma chest pain and, 105 questions to ask about, 52 night sweats and, 100 in children, 310, 312 red eye in, 136 palpitations and, 107, 108 cough and, 103, 104, 320, 322, seizures in, 515–522 tremor and, 488 376 short stature in, 136–137 Philadelphia travel diagnostic workup for, 322 urinary tract infection in, 312, information, 34–37 fever and, 133 506 philosophical challenging motor vehicle accident and, 177, pelvic fracture, 129 questions, 71 178 pelvic inflammatory disease (PID) phobias, 108 sore throat and, 102 abdominal pain and, 115, 116 phone encounters, 66 typical, 377 diagnosing, 423, 424 photophobia pneumonitis, 103 dyspareunia and, 286 headaches and, 87, 88 pneumothorax vaginal bleeding and, 125 meningitis and, 367 assault and, 224 pelvic tumors, 286 migraine and, 266, 268 chest pain and, 105, 150 peptic ulcer disease simulated by SPs, 62 motor vehicle accident and, 176, abdominal pain and, 112, 113, photosensitivity, 348 178 114 phototherapy, for jaundice, 304 podagra, 348 chest pain and, 105 physical abuse. See abuse polyarteritis nodosa, 214 diagnosing, 256, 258 physical exam, 54–63 polycystic kidney disease, 120 epigastric pain and, 387 of abdominal area, 58 polycystic ovary syndrome upper GI bleeding and, 118 of cardiovascular system, 57 amenorrhea and, 123, 276, 277 percussing abdomen, 58 general inspection during, 56 weight gain and, 109

564 polycythemia, 303 prognosis, challenging questions Q polydipsia on, 71 QuantiFERON Gold test, 377, 471 diabetes insipidus and, 453 prolactinoma, 123 questions to ask patient, 46–54 diabetes mellitus and, 194, 452 propranolol, and erectile primary, 99 dysfunction, 394, 396 R polymyositis, 97, 132 prostate cancer radiculopathy, 248, 250 polyuria back pain and, 167, 168 rape. See sexual assault diabetes insipidus and, 453 hematuria and, 120, 159 rashes diabetes mellitus and, 194, 452 urinary symptoms and, 121 in children, 367 hematuria and, 153 prostatitis, 121 meningitis and, 366 popliteal cyst. See Baker’s cyst pruritus scarlet fever and, 366, 367 rupture biliary obstruction and, 329 Raynaud’s phenomenon posttraumatic stress disorder jaundice and, 324 dysphagia and, 110 (PTSD) night sweats and, 100 lupus and, 346–347, 348 amenorrhea and, 124 pseudogout, 130, 348 reactive airway disease, 103 diagnosing, 443, 444 pseudomembranous colitis, 117, reactive arthritis, 130, 131 fatigue and, 99 118, 479, 480 reactive postprandial postural tremor, 488 pseudotumor cerebri, 87, 88, 268 hypoglycemia, 109 posture, while interviewing psoriatic arthritis, 129 rectal exam, for back pain, 168 patient, 44 psoriatic lesions, examining, 61 red eye, in children, 136 PPD (tuberculin skin test), 377, psychiatric history, asking patients redirecting patient 471 about, 53 conversations, 45 preemployment medical psychogenic tremor, 488 checkup, 463–471 psychosis refilling prescriptions, challenging pregnancy brief psychotic disorder, 462 questions on, 70 abdominal pain and, 116 key history/physical exam, 93 reflective listening, 43 amenorrhea and, 123, 124, 275, substance-induced, 461, 462 reflexes 277 psychotic disorder, 99 simulated by SPs, 62 assault and, 223, 224 pulmonary edema, 104, 322 testing, 59, 60 challenging questions on, 76 pulmonary embolism registering for USMLE Step 2 CS, 6 diagnostic workup for, 187, 188 chest pain and, 105, 106, 150 Reiter’s syndrome, 130, 131 DVT and, 233 cough and, 322 renal artery stenosis, 62 ectopic, 115, 125, 186, 188, 423, loss of consciousness and, 96 renal cell carcinoma 424 pulmonary exam abdominal pain and, 112 molar, 125, 187, 188 checklist for, 58 hematuria and, 120, 159 nausea/vomiting and, 111 signs simulated by SPs, 61 urinary symptoms and, 121 practice case for, 180–188 pulmonary fibrosis, 104 renal failure questions to ask patient pulmonary tuberculosis, 375, 377, in children, 137 about, 52 470, 471 dizziness and, 94 vaginal bleeding and, 125 pulse, measuring, 57, 60 fatigue and, 99 weight gain and, 498 pyelonephritis hyperprolactinemia and, 277 premature ovarian failure, 123, 277 abdominal pain and, 112, 116 multiple myeloma and, 250 presbycusis, 293, 295 diarrhea and, 506 peripheral neuropathy and, 214 prescription refills, challenging fever and, 133, 312 renting a car, 24 questions on, 70 hematuria and, 120 respect, in patient primary biliary cirrhosis, 331 pyloric stenosis communication, 43 proctitis, 119 in children, 135 resting tremor, 488

565 restless leg syndrome, 100 scarlet fever sexual dysfunction retaking USMLE Step 2 CS, 8 childhood fever and, 134 challenging questions on, 68 retinal artery occlusion, 91 diagnosing, 367 diabetes and, 208, 213, 214 retinal vein occlusion, 91 facial rash of, 366 hypertension and, 389 retrocalcaneal bursitis, 358 sore throat and, 102 medication-induced, 394 retropharyngeal abscess, 414 scars, examining, 61, 63 sexual history retrosternal heave, checking for, 57 scheduling permit for exam, 6, 22 asking patient about, 51 rhabdomyolysis scheduling USMLE Step 2 CS, 14 high-risk behavior, PCP intoxication and, 462 schizoaffective disorder, 92 communicating to statins and, 132 schizoid/schizotypal personality patient, 71 rheumatic fever, 242 disorder, 93 sexually transmitted diseases rheumatic heart disease, 367 schizophrenia, 93 assault and, 222, 224 rheumatoid arthritis schizophreniform disorder, 93 challenging questions on, 67, joint/limb pain and, 129, 131, scleral icterus 72, 76 347, 348 acetaminophen and, 330 counseling patients on, 78–79 peripheral neuropathy and, 214 biliary obstruction and, 329 diagnosing in pregnancy, 188 Rh incompatibility, 301, 303 hepatitis and, 330 diagnostic workup for, 286 rhinorrhea sclerosing cholangitis, 113 sexual orientation, challenging cough and, 320 score reports, 7 questions on, 74 headaches and, 268 scoring of USMLE Step 2 CS, 5–6 sexual pain. See dyspareunia upper respiratory infection seasonal allergies, 136 Sheehan’s syndrome, 124 and, 312 secondary enuresis, 512, 514 Shigella, 506 rib fracture second opinions, challenging shortness of breath abdominal pain and, 114 questions on, 74 questions to ask patient assault and, 222, 224 seizures about, 48 chest pain and, 178 febrile, 520, 522 simulation of by SPs, 61 Rinne test hallucinations and, 462 shoulders dizziness and, 94, 334, 337 headache and, 87 dislocation of, 131, 204, 205 hearing loss and, 293 in infants, 515–522 examining, 60 presbycusis and, 293 meningitis and, 367 rotator cuff tear, 131, 205 Romberg’s sign, 59, 334 numbness/weakness and, 96, 97 SIADH, 91 roseola, 134 practice case for, 515–522 sickle cell anemia, 75 rotator cuff injury, 131, 205 syncope and, 96, 533, 539 acute chest syndrome and, 105 rotavirus, 506 tonic-clonic, 95 sight loss. See vision loss Rovsing’s sign, 424 sensory system, testing, 59 sightseeing. See travel information RPR test, 188 sepsis for students rubella jaundice and, 331 sinusitis, 88, 268 fever and, 134 neonatal, 301, 303 skin lesions neonatal jaundice and, 303 occult bacteremia and, 312 examining, 61 pregnancy and, 188 SEP (Spoken English Proficiency) simulated by SPs, 63 sore throat and, 531 score, 6 sleep apnea ruptured ectopic pregnancy, 423 septic abortion, 424 bed-wetting and, 514 ruptured ovarian cyst, 424 septic arthritis, 130 fatigue and, 99 serositis, 348 insomnia and, 100, 101 S sexual assault obstructive sleep apnea (OSA), Salmonella, 506 diagnostic workup for, 128 405 sarcoidosis, 104 practice case for, 216–224 sleep, asking patient about, 49–50

566 sleep complaints special patients, 65–67 subarachnoid hemorrhage, 88 anxiety and, 405 spherocytosis, 303 subdural hematoma depression and, 443 sphincter of Oddi, in jaundice, 329 confusion/memory loss and, 91, fatigue, 98–99 spinal fracture, 249 434 insomnia, 100–101, 397–406 spinal stenosis, 132 numbness/weakness and, 97 key history/physical exam, 98– spinal x-ray, 168 substance use 99 splenic infarct, 114 asking patient about, 51 narcolepsy, 462 splenic rupture chest pain and, 148, 150 nightmares, 443 abdominal pain and, 114 in children, 138 night sweats, 100 motor vehicle accident and, 178 constipation and, 116 obstructive sleep apnea (OSA), splenomegaly depression and, 92 405 mononucleosis and, 531 erectile dysfunction and, 122 tremor and, 488 sore throat and, 101 fatigue and, 98 small bowel cancer Spoken English Proficiency (SEP) hallucinations and, 460 abdominal pain and, 114 score, 6 hypotension and, 96 diarrhea and, 118 spontaneous abortion. See insomnia and, 100, 405 smoking also pregnancy joint/limb pain and, 128 asking patient about, 51 abdominal pain and, 116, 424 loss of consciousness and, 95 bronchitis and, 469, 471 challenging questions on, 76 low back pain and, 132 challenging questions on, 71, 76 vaginal bleeding and, 125 nongonococcal septic arthritis counseling patients on, 77, 466, sprained ankle, 358 and, 348 493, 526 SPs. See standardized patients (SPs) palpitations and, 108 dysphagia and, 110 Spurling’s sign, 245, 248 PCP intoxication, 460, 462 erectile dysfunction and, 122 sputum, examining, 46, 315 psychosis and, 93, 461, 462 hearing loss and, 295 standardized patients (SPs), 4, 40. tremor and, 488 hematuria and, 119 See also patients weight change and, 108, 109 laryngeal cancer and, 240, 242 scoring done by, 5 suicidal ideation, 92, 443, 444 laryngitis and, 241, 242 simulations of physical exam summary technique, 11, 45 low back pain and, 132 findings, 61–63 superficial venous thrombosis, 131 lung cancer and, 376 stomach cancer. See gastric cancer support, in patient pancreatic cancer and, 385 stool communication, 44 pneumonia and, 376 bloody, 119, 472–480 surgery tremor and, 488 currant jelly appearance, 506 challenging questions on, 72, weight gain after quitting, 493, greasy, 385, 386, 387, 403 77 496, 498 strangulated hernia, 136 patient fears of, 69 Snellen eye chart, 57 strength, joint, 60 swallowing. See dysphagia snoring streptococcal pharyngitis, 530, 531 symmetric arthritis, 348 fatigue and, 98 streptococcal tonsillitis. syncope. See consciousness, loss of insomnia and, 100 See tonsillitis syphilis social history, asking patient stress fracture, 130, 357, 358 Ménière’s disease and, 295, 339 about, 51 stress incontinence, 122 sore throat and, 102 social phobia, 108 stridor, 414 syringomyelia, 97 somatoform disorder, 135 stroke systemic lupus erythematosus (SLE) sore throat challenging questions on, 75 diagnostic criteria, 348 key history/physical exam, 101– dementia and, 433, 434 joint/limb pain and, 129, 131, 102 numbness/weakness and, 96, 97, 347, 348 practice case for, 523–531 98 systemic sclerosis, 110

567 T tonsillitis, 102 tumors, challenging questions TORCH infections, 303 tachycardia on, 70 toxoplasmosis, 303 tympanic membrane in otitis anxiety and, 403, 405 transesophageal echocardiography media, 312 caffeine and, 404 (TEE), 150 mitral valve stenosis and, 242 transient ischemic attack (TIA) U PCP intoxication and, 462 confusion/memory loss and, 91 ulcerative colitis substance use and, 460 numbness/weakness and, 96, 97, abdominal pain and, 115 syncope and, 540 98 bloody stool and, 119, 480 tachypnea, 312 transportation to CSEC center diarrhea and, 118 tarsal tunnel syndrome, 358 in Atlanta, 25 ultrasound exam for pregnancy, 188 telephone encounters, 66 in Chicago, 27–28 uncooperative patients, 66 temporal arteritis, 87 in Houston, 30 United States, traveling to, 22–24 temporomandibular joint (TMJ) in Los Angeles, 32–33 upper endoscopy for chest disorder, 87, 89 in Philadelphia, 35 pain, 150 tendinitis transthoracic echocardiography upper GI bleeding arm pain and, 131 (TTE), 150 blood in stool and, 119 heel pain and, 357, 358 traveler’s diarrhea, 117 key history/physical exam, 118 tennis elbow, 130 travel history, asking patient upper respiratory infection (URI) tension headaches, 87, 88, 89, 267, about, 50 childhood fever and, 133, 134 268 travel information for students, 22– in children, 310, 312 test-day tips for the USMLE 37 cough and, 102, 104, 321, 322 Step 2 CS, 11–13 for Atlanta, 24–27 otitis media and, 367 testicular torsion, 136 for Chicago, 27–29 urethritis, 121 Test of English as a Foreign for Houston, 29–32 urge incontinence, 122 Language (TOEFL), 17 for Los Angeles, 32–34 urinanalysis Test of Spoken English (TSE), 17 for Philadelphia, 34–37 for diabetes, 196 throat for United States, 22 in pregnancy, 188 examining, 57 treatment plans, challenging urinary incontinence, 90, 122 hoarseness in, 235–242, 414 questions on, 72 in children, 507–514 thrombophilia, 233 tremors urinary symptoms thyroid bruit, 63 practice case for, 481–489 back pain and, 166 thyroid disease simulated by SPs, 62 dysuria, 121 amenorrhea and, 123, 124, 277 triceps reflex, 59 hematuria, 119–120, 152–160 depression and, 387 trichomonal vaginitis, 126 incontinence, 90, 122, 507–514 psychosis and, 462 trichomoniasis. See also sexually key history/physical exam, 120 thyrotoxicosis, 488 transmitted diseases nausea/vomiting and, 111 time management during assault and, 224 questions to ask patient exam, 40–41 counseling patients on, 79 about, 48 Tinel’s sign, 62, 98 tricyclic antidepressants (TCAs), urinary tract infection (UTI) tinnitus 488 amenorrhea and, 123 dizziness and, 94 trigeminal neuralgia, 87, 89 bed-wetting and, 512, 514 labyrinthitis and, 339 tuberculosis in children, 135 Ménière’s disease and, 295 cough and, 103, 104, 371–372, fever and, 134, 522 Todd’s paralysis, 98 375, 377, 470, 471 hematuria and, 120, 159 tonic-clonic seizures, 95, 520. See neck mass and, 111 in infants, 312, 506 also seizures night sweats and, 100 low back pain and, 132

568 nausea/vomiting and, 111 vertebral compression fracture, 132 Wegener’s granulomatosis, 103 urinary symptoms and, 121 vertebral tumor, 132 weight urine hCG test, 188 vertebrobasilar insufficiency, 94, 95 challenging questions on, 76, 77 urine toxicology after motor vertigo, 95, 338, 339. See familial obesity, 498 vehicle accident, 179 also dizziness questions to ask patient urolithiasis, 157, 159 Ménière’s disease and, 295 about, 49 USMLE Step 2 CS vestibular disease, 339 weight change breaks during, 12 vestibular neuronitis, 94, 95 amenorrhea and, 275 common preparation mistakes, 2 videotaping of exam, 4, 12 colon cancer and, 478 duration of, 11 viral conjunctivitis, 136 depression and, 92, 386 exam locations, 7 viral gastroenteritis, 506 diabetes and, 194 failing, 8 vision loss. See also eye exam diagnostic workup for, 497–498 overview of, 3–4 key history/physical exam, 91 dysphagia and, 110 philosophy of, 2–3 simulated by SPs, 62 fatigue and, 98, 380, 442 preparing for, 9–11 visual hallucinations. hyperthyroidism and, 404 registering for, 6–7 See hallucinations hypothyroidism and, 443, 444 rescheduling/canceling vital signs, analyzing, 42 insomnia and, 403 examination date, 7 vitamin deficiency laryngeal cancer and, 240 score reports, 7 confusion/memory loss and, 89, lung cancer and, 376 scoring of, 5–6 90, 434 lupus and, 347 structure of, 4–5, 40 numbness/weakness and, 97 night sweats and, 100 test-day tips, 11–13 vocal cord polyps/nodules, 241, 242 pancreatic cancer and, 385 uveitis, 136 vocal problems. See hoarseness tuberculosis and, 375 volvulus weight gain, as chief V abdominal pain and, 114, 115 complaint, 109–110, vaginal bleeding, 124–125 in children, 135 490–498 vaginal discharge fever and, 134 weight loss, as chief diagnostic workup for, 286 vomiting complaint, 108–109 key history/physical exam, 126 abdominal pain and, 252, 256 Wernicke’s encephalopathy, 90 painful sexual intercourse appendicitis and, 422 and, 284 intracranial mass lesion and, 267 Wilson’s disease, 488 vaginal yeast infections, 126, 453 key history/physical exam, 111 workplace concerns of patient, 70, vaginismus, 286 meningitis and, 367 71 vaginitis migraine and, 266, 268 preemployment medical abdominal pain and, 116 questions to ask patient checkup, 463–471 dyspareunia and, 127 about, 47 workup. See diagnostic workup vaginal discharge and, 126 von Willebrand’s disease, 125 wrists varicella, 134, 367 vulvodynia, 286 examining, 60 vascular dementia, 90, 433, 434 vulvovaginitis, 284, 286 pain in, 131 vasculitides, 214 Y vasculitis, 103, 104 W vasovagal syncope, 540 washing hands, 9 yeast infections, 126, 453 vehicular accident. See motor weakness, key history/physical yellow skin/eyes. See jaundice vehicle accident exam, 96–98 Yersinia enterocolitica, 506 ventricular tachycardia, 540 Weber test vertebral artery dissection, 88 for dizziness, 94, 334 Z vertebral canal tumor, 97 hearing loss and, 293 Zenker’s diverticulum, 110

569 NOTES

570 NOTES

571 NOTES

572 ABOUT THE AUTHORS

Tao Le, MD, MHS

Tao developed a passion for medical education as a medical student. He currently edits more than 15 titles in the First Aid series. In addition, he is the founder and editor of the USMLE-Rx test bank and online video series as well as a cofounder of the Underground Clinical Vignettes series. As a medical student, he was editor-in-chief of the University of California, San Francisco (UCSF) Synapse, a university newspaper with a weekly circulation of 9000. Tao earned his medical degree from UCSF in 1996 and completed his residency training in internal medicine at Yale University and fellowship training at Johns Hopkins University. He subsequently went on to cofound Medsn, a medical education technology venture, and served as its chief medical officer. He is currently conducting research in asthma education at the University of Louisville.

Vikas Bhushan, MD

Vikas is a writer, editor, entrepreneur, and teleradiologist on sabbatical. In 1990 he conceived and authored the original First Aid for the USMLE Step 1. His entrepreneurial endeavors include a student-focused medical publishing enterprise (S2S), an e-learning company (medschool.com/Medsn), and an ER teleradiology practice (24/7 Radiology). Firmly anchored to the West Coast, Vikas completed a bachelor’s degree at the University of California, Berkeley; an MD with thesis at UCSF; and a diagnostic radiology residency at the University of Cali- fornia, Los Angeles. His eclectic interests include technology, information design, photography, South Asian diasporic culture, and avoiding a day job. Always finding the long shortcut, Vikas is an adventurer, a knowledge seeker, and an occasional innovator. He enjoys novice status as a kiteboarder and single father and strives to raise his children as global citizens.

Mae Sheikh-Ali, MD

Mae is currently an associate professor of medicine and an associate program director for the Endocrinology Fel- lowship Program at the University of Florida College of Medicine, Jacksonville. She earned her medical degree from Damascus University School of Medicine in Syria. She completed her residency training in internal medi- cine at Drexel University College of Medicine in Philadelphia and endocrinology fellowship training at Mayo Clinic College of Medicine, Jacksonville, Florida. She is an editor and contributing author of several editions of First Aid for the USMLE Step 2 CS and the Underground Clinical Vignettes series. Mae is passionate about medi- cal education and is taking an active role in teaching and empowering medical students, residents, fellows, and patients. She is currently pursuing an academic career in endocrinology with a focus on vitamin D, obesity, and diabetes research projects.

Kachiu Cecilia Lee, MD, MPH

Kachiu received her medical degree and master’s degree in public health from Northwestern University and then served as chief resident in her dermatology residency at Brown University. After completing her residency, she will begin fellowship training in laser surgery at Massachusetts General Hospital’s Wellman Center for Photo- medicine (Harvard Medical School). Kachiu intends to pursue an academic career with a research emphasis on epidemiology, health services, and medical education. As the first physician in her family, she hopes to play a critical role in helping medical students and residents fulfill their career aspirations. She has contributed to several projects in the First Aid series, including the USMLE-Rx test bank, First Aid for the USMLE Step 1, and First Aid for the USMLE Step 2 CS. In her leisure time, she enjoys photography, playing piano, and exploring New England with her husband and daughter.

573 See Web site for terms and conditions